237
http://vk.com/my.printing 1 Біологія 1 У життєвому циклі клітини і в процесі мітозу відбувається закономірна зміна кількості спадкового матеріалу. На якому етапі кількість ДНК подвоюється? A * Інтерфаза B Профаза C Метафаза D Анафаза E Телофаза 2 Поліпептид, синтезований в рибосомі, складається з 54 амінокислот. Яку кількість кодонів мала і-РНК, що слугувала матрицею для даного синтезу? A * 54 B 27 C 108 D 162 E 44 3 Мукополісахаридоз відноситься до хвороб накопичення. Через відсутність ферментів порушується розщеплення полісахаридів. У хворих спостерігається підвищення виділення їх з сечею і нагромадження. В яких органелах відбувається накопичення мукополісахаридів? A * Лізосоми B Комплекс Гольджі C Ендоплазматичний ретикулум D Мітохондрії E Клітинний центр 4 В наслідок порушення розходження хромосом при мейозі утворилися: яйцеклітина тільки з 22 аутосомами і полярне тільце з 24 хромосомами. Який синдром можливий у дитини при заплідненні такої яйцеклітини нормальним сперматозооном (22+Х)? A *Синдром Шерешевського-Тернера B Синдром Клайнфельтера C Трисомія Х D Синдром Дауна E Синдром Едвардса 5 Відомо, що старіючі епітеліальні клітини відмирають. Процес перетравлення та виділення рештків забезпечують органели: A *Лізосоми B Рибосоми C Мітохондрії D Клітинний центр E Комплекс Гольджі 6 У людини встановлено попередній діагноз токсоплазмоз. Якій матеріал використовано для діагностики цієї хвороби? A *Кров B Фекалії C Сеча D Дуодентальний вміст E Харкотиння 7 У жінки народилася мертва дитина з багатьма вадами розвитку. Яке протозойне захворювання могло спричинити внутрішньоутробну загибель плода. A *Токсоплазмоз B Амебіаз C Малярія D Лейшманіоз E Лямбліоз 8 При мікроскопії зіскобу з переанальних складок виявлені безбарвні яйця, що мають форму несиметричних овалів, розміром 50x23 мкм. Про який вид гельмінту йде мова? A *Гострик B Аскарида C Кривоголовка D Волосоголовець E Карликовий ціп’як 9 У червоподібному відростку виявлено гельмінта білого кольору, завдовжки 40 мм з тонким ниткоподібним переднім кінцем. У фекаліях знайдені яйця овальної форми з пробками на полюсах. Визначте вид гельмінта. A *Волосоголовець B Гострик C Аскарида D Кривоголовка E Вугриця кишкова 10 До лікарні потрапив хворий із скаргами на головний біль, біль у м’язах під час руху, слабкість, температуру, набряк повік і

крок 1 база 2014р_11шр-240ст (для брошури)_35грн

  • Upload
    -1-2-3-

  • View
    411

  • Download
    3

Embed Size (px)

Citation preview

Page 1: крок 1 база 2014р_11шр-240ст (для брошури)_35грн

http://vk.com/my.printing

1

Біологія 1 У життєвому циклі клітини і в процесі мітозу відбувається закономірна зміна кількості спадкового матеріалу. На якому етапі кількість ДНК подвоюється? A * Інтерфаза B Профаза C Метафаза D Анафаза E Телофаза

2 Поліпептид, синтезований в рибосомі, складається з 54 амінокислот. Яку кількість кодонів мала і-РНК, що слугувала матрицею для даного синтезу? A * 54 B 27 C 108 D 162 E 44

3 Мукополісахаридоз відноситься до хвороб накопичення. Через відсутність ферментів порушується розщеплення полісахаридів. У хворих спостерігається підвищення виділення їх з сечею і нагромадження. В яких органелах відбувається накопичення мукополісахаридів? A * Лізосоми B Комплекс Гольджі C Ендоплазматичний ретикулум D Мітохондрії E Клітинний центр

4 В наслідок порушення розходження хромосом при мейозі утворилися: яйцеклітина тільки з 22 аутосомами і полярне тільце з 24 хромосомами. Який синдром можливий у дитини при заплідненні такої яйцеклітини нормальним сперматозооном (22+Х)? A *Синдром Шерешевського-Тернера B Синдром Клайнфельтера C Трисомія Х D Синдром Дауна E Синдром Едвардса

5 Відомо, що старіючі епітеліальні клітини відмирають. Процес перетравлення та виділення

рештків забезпечують органели: A *Лізосоми B Рибосоми C Мітохондрії D Клітинний центр E Комплекс Гольджі

6 У людини встановлено попередній діагноз токсоплазмоз. Якій матеріал використовано для діагностики цієї хвороби? A *Кров B Фекалії C Сеча D Дуодентальний вміст E Харкотиння

7 У жінки народилася мертва дитина з багатьма вадами розвитку. Яке протозойне захворювання могло спричинити внутрішньоутробну загибель плода. A *Токсоплазмоз B Амебіаз C Малярія D Лейшманіоз E Лямбліоз

8 При мікроскопії зіскобу з переанальних складок виявлені безбарвні яйця, що мають форму несиметричних овалів, розміром 50x23 мкм. Про який вид гельмінту йде мова? A *Гострик B Аскарида C Кривоголовка D Волосоголовець E Карликовий ціп’як

9 У червоподібному відростку виявлено гельмінта білого кольору, завдовжки 40 мм з тонким ниткоподібним переднім кінцем. У фекаліях знайдені яйця овальної форми з пробками на полюсах. Визначте вид гельмінта. A *Волосоголовець B Гострик C Аскарида D Кривоголовка E Вугриця кишкова

10 До лікарні потрапив хворий із скаргами на головний біль, біль у м’язах під час руху, слабкість, температуру, набряк повік і

Page 2: крок 1 база 2014р_11шр-240ст (для брошури)_35грн

2

обличчя. Лікар пов’язує цей стан із вживанням свинини, купленої у приватних осіб. Який попередній діагноз може поставити лікар? A *Трихінельоз B Теніоз C Теніарінхоз D Опісторхоз E Фасциольоз

11 При дегельмінтезації з фекаліями виділився гельмінт довжиною до 2 м. Тіло сегментоване, з маленькою голівкою, на якій є чотири присоски і гачки. Визначте вид гельмінта. A *Озброєний ціп’як B Неозброєний ціп’як C Карликовий ціп’як D Ехінокок E Стьожак широкий

12 Схильність до цукрового діабету обумовлює аутосомний рецесивний ген. Цей ген проявляється лише у 30% гомозиготних особин . Цей частковий прояв ознаки є прикладом слідуючої властивості гена: A *Пенетрантність B Домінантність. C Експресивність D Дискретність. E Рецесивність.

13 У букальних мазках епітелію жінки виявлено в ядрі клітини 2 тільця Барра. Це характерно для синдрому: A *Трисомія статевих хромосом B Трисомія 21-ї хромосоми C Трисомія 13-ї хромосоми D Трисомія по У-хромосомі E Моносомія статевих хромосом

14 У більшості клітинах епітелію слизової щоки мужчини виявлено глибку статевого Х-хроматину. Це характерно для синдрому: A * Клайнфельтера B Шерешевського -Тернера C Трипло-Х D Дауна E Трипло-У

15 У певних клітинах дорослої людини на

протязі життя не спостерігається мітоз і кількісний вміст ДНК залишається постійним. Ці клітини: A * Нейрони B Ендотелію C М’язові (гладкі ) D Епідерміс E Кровотворні

16 Під час опитування студентів за темою: “Молекулярня біологія” викладачем було задане запитання: “Чому генетичний код є універсальним?” Правильною повинна бути відповідь: “Тому що він…”: A * єдиний для більшості організмів B містить інформацію про будову білка C є триплетним D кодує амінокислоти E колінеарний

17 У студента 18 років виявлено збільшення щитовидної залози. При цьому був підвищений обмін речовин, збільшення частоти пульсу. Ці ознаки спостерігаються при гіперсекреції гормону тироксину. Які органели клітин щитовидної залози найбільш відповідні за секрецію і виділення гормонів: A * комплекс Гольджі B мітохондрії C рибосоми D центросоми E лізосоми

18 При вивченні під електронним мікроскопом клітин підшлункової залози були знайдені структури, які поділяють клітину на велику кількість комірок, каналів, цистерн та поєднані з плазмолемою. Вкажіть ці органели: A * ендоплазматична сітка B мітохондрії C центросоми D рибосоми E комплекс Гольджі

19 Під дією різних фізичних і хімічних агентів при біосинтезі ДНК у клітині можуть виникати пошкодження. Здатність клітин до виправлення пошкоджень у молекулах ДНК

Page 3: крок 1 база 2014р_11шр-240ст (для брошури)_35грн

http://vk.com/my.printing

3

називається: A * репарація B траскрипція C реплікація D трансдукція E трансформація

20 У людини один і той же генотип може спричинити розвиток захворювання з різними ступенями прояву фенотипів. Ступінь прояви ознаки при реалізації генотипу у різних умовах середовища – це: A * експресивність B пенетрантність C спадковість D мутація E полімерія

21 У клітину потрапив вірус грипу. Трансляція при біосинтезі вірусного білка в клітині буде здійснюватися: A *На полірибосомах B У ядрі C У лізосомах D На каналах гладкої ендоплазматичної сітки E У клітинному центрі

22 Експериментально була встановлена кількість та послідовність амінокислот у молекулі гормона інсуліна. Ця послідовність кодується: A *Кількістю та послідовністю нуклеотидів у екзонних частинах гена B Послідовністю структурних генів C Кількістю та послідовністю азотистих основ ДНК D Певним чергуванням екзонних та інтронних ділянок. E Кількістю та послідовністю нуклеотидів у інтронних ділянках гена

23 При електронно-мікроскопічному вивченні клітини виявлені кулясті пухирці, які обмежені мембраною і місять безліч різноманітних гідролітичних ферментів Відомо, що ці органели забезпечують внутрішньоклітинне травлення, захисні реакції клітини і являють

собою: A * Лізосоми B Центросоми C Ендоплазматичну сітку D Рибосоми: E Мітохондрії

24 У людини діагностовано галактоземію – хворбу накопичення. Цю хворобу можливо діагностувати при допопмозі слідуючого методу: A *Біохімічного B Цитогенетичного C Популяційно-статистичного D Близнюкового E Генеалогічного

25 Було доведено, що молекула незрілої і-РНК (про-і-РНК) містить більше триплетів, чим знайдено амінокислот у синтезованому білку. Це пояснюється тим, що транслції у нормі передує: A *Процесінг B Ініціація C Репарація D Мутація E Реплікація

26 Біля ядра виявлена органела. Вона складається з двох циліндрів, розташованих перпендикулярно один до одного. Циліндри утворені мікротрубочками. Було з’ясовано, що ця органела забезпечує формування мітотичного апарата і являє собою: A * Центросому B Рибосому C Ендоплазматичну сітку D Мітохондрію E Лізосому

27 У хворого виявлено зниження іонів магнію, які потрібні для прикріплення рибосом до гранулярної ендоплазматичної сітки. Відомо, що це призводить до порушення біосинтезу білка. Порушення відбувається на етапі: A * трансляція B транскрипція C реплікація D активація амінокислот E термінація

28

Page 4: крок 1 база 2014р_11шр-240ст (для брошури)_35грн

4

У людини часто зустрічаються хвороби, пов язані з накопиченням у клітинах вуглеводів, ліпідів та ін. Причиною виникнення цих спадкових хвороб є відсутність відповідних ферментів у: A * лізосомах B мітохондріях C ендоплазматичній сітці D мікротрубочках E ядрі

29 При амавротичній ідіотії Тея - Сакса, яка успадковується аутосомно-рецесивно, розвиваються незворотні важкі порушення центральної нервової системи, що приводять до смерті в ранньому дитячому віці. При цьому захворюванні спостерігається розлад обміну: A * ліпідів B вуглеводів C амінокислот D мінеральних речовин E нуклеїнових кислот

30 Одна з форм рахіту успадковується за домінантним типом. Хворіють і чоловіки і жінки. Це захворювання є наслідком мутації: A *Генної B Геномної C Хромосомної D Поліплоідії E Анеуплоідії

31 Підтримка життя на будь-якому рівні зв'язано з явищем репродукції . На якому рівні організації репродукція здійснюється на основі матричного синтезу? A * молекулярному B субклітинному C клітинному D тканинному E рівні організму

32 Під час операції в печінці хворого виявлені дрібні міхурці малих розмірів, з незначною кількістю рідини, які щільно прилягають один до одного. Який гельмінтоз виявився у хворого? A *Альвеококоз B Фасціольоз C Опісторхоз

D Клонорхоз E Дікроцеліоз

33 Людини з кариотипом 46, ХУ має жіночий фенотип з розвиненими зовнішніми вторинностатевими ознаками.. За цією інформацією лікар встановив попередній діагноз: A * Синдром Морріса B Синдром Дауна C Синдром суперчоловіка D Синдром Клайнфельтера E Синдром Тернера-Шерешевського

34 При яких групах крові батьків за системою резус -фактор можлива резус-конфліктна ситуація під час вагітності? A *Жінка Rh–, чоловік Rh+ (гомозигота) B Жінка Rh+, чоловік Rh+ (гомозигота) C Жінка Rh+, чоловік Rh+ (гетерозигота) D Жінка Rh–, чоловік Rh– E Жінка Rh+(гетерозигота), чоловік Rh+ (гомозигота)

35 У глухонімих батьків з генотипами DDee і ddEE народились діти з нормальним слухом. Яка форма взаємодії генів D i E? A *Комплементарність B Домінування C Епістаз D Полімерія E Наддомінування

36 Речовини виводяться з клітини в результаті з’єднання мембранної структури апарату Гольджі з цитолемою. Вміст такої структури викидається за межі клітини. Цей процес має назву: A * Екзоцитиоз B Осмос C Ендоцитоз D Активний транспорт E Полегшена дифузія

37 Мати і батько були фенотипово здоровими ігетерозиготними за генотипом. У них народилася хвора дитина, в сечі і крові якої знайдена фенілпіровиноградна кислота. З приводу цього і був встановлений

Page 5: крок 1 база 2014р_11шр-240ст (для брошури)_35грн

http://vk.com/my.printing

5

попередній діагноз – фенілкетонурія. Вкажіть тип успадкування цієі хвороби: A *Аутосомно-рецесивний B Зчеплений з Х-хромосомою рецесивний C Аутосомно-домінантний D Зчеплений з У-хромосомою E Зчеплений з Х-хромосомою домінантний

38 У пологовому будинку народилась дитина з численними порушеннями, як зовнішніх так і внутрішніх органів – серця, нирок, травної системи. Був встановлений попередній діагноз – синдром Дауна. Яким методом можливо підтвердити цей діагноз? A *Цитогенетичним B Популяційно-статистчним C Близнюковим D Генеалогічним E Біохімічним

39 При медичному огляді юнаків у деяких під пахвами були виявлені комахи розміром 1,0 – 1,5 мм сірого кольору, з коротким широким тілом, груди і черевце майже не відмежовані, тіло вкрите волосками. Цими ектопразитами є: A *Лобкова воша B Блоха C Головна воша D Блощиця E Коростяний свербун

40 Альбінізм спостерігається у всіх класів хребетних тварин. Ця спадкова патологія зустрічається також у людини і обумовлена геном, який має аутосомно рецесивне успадкування. Проявом якого закону є наявність альбінізму в людини та в представників класів хребетних тварин: A *Гомологічних рядів спадкової мінливості Вавілова; B Біогенетичного Геккеля-Мюллера; C Одноманітності гібридів 1 покоління Менделя; D Незалежного успадкування ознак Менделя; E Зчепленого успадкування Моргана.

41

Рибосоми являють собою органели, які здійснюють зв’язування амінокислот у поліпептидний ланцюг. Кількість рибосом в клітинах різних органів неоднакова і залежить від функції органу. Вкажіть, в клітинах якого органу кількість рибосом буде найбільшою: A *Секреторні клітини підшлунковоі залози; B Епітелію сечового міхура; C Епітелію канальців нирок; D Верхнього шару клітин епідермісу шкіри; E Епітелію тонкого кишечника.

42 Першим етапом діагностування хвороб, зумовлених порушенням обміну речовин, є скринінг-метод, після якого використовують більш точні методи дослідження ферментів, амінокислот. Яку назву має описаний метод: A *біохімічний ; B імунологічний; C цитогенетичний; D популяційно-статистичний; E гібридизації соматичних клітин.

43 В ядрі клітині з молекули незрілої і- РНК утворилася молекула зрілої і-РНК, яка має менший розмір, ніж незріла і-РНК. Сукупність єтапів цього перетворення має назву: A *Процесінг; B Реплікація; C Рекогніція; D Трансляція E Термінація.

44 Деякі триплети і-РНК (УАА, УАГ, УГА) не кодують амінокислоти, а є термінаторами в процесі зчитування інформації, тобто здатні припинити трансляцію. Ці триплети мають назву: A * Стоп-кодони B Оператори C Антикодони D Екзони E Інтрони

45 В пресинтетичному періоді мітотичного циклу синтез ДНК не відбувається, тому молекул ДНК стільки ж, скільки й хромосом. Скільки молекул ДНК має соматична клітина людини в пресинтетичному періоді?

Page 6: крок 1 база 2014р_11шр-240ст (для брошури)_35грн

6

A * 46 молекул ДНК B 92 молекули ДНК C 23 молекули ДНК D 69 молекул ДНК E 48 молекул ДНК

46 В анафазі мітозу до полюсів розходяться однохроматидні хромосоми. Скільки хромосом має клітина людини в анафазі мітозу? A * 92 хромосоми B 46 хромосом C 23 хромосоми D 69 хромосом E 96 хромосом

47 У чоловіка за системою АВ0 встановлена IV (АВ) група крові, а у жінки - III (В). У батька жінки I (0) група крові. В них народилося 5 дітей. Вкажіть генотип тієї дитини, яку можна вважати позашлюбною : A * i i B IA IB C IB IB D IA i E IB i

48 У хворого, який страждає вуграми і запальними змінами шкіри обличчя, при мікроскопії матеріалу з осередків ураження виявлені живі членистоногі, продовгуватої форми, з 4 парами дуже редукованих кінцівок. Встановіть попередній діагноз: A * Демодекоз B Ураження шкіри блохами C Алергія D Ураження шкіри коростяним свербуном E Педикульоз

49 У лікарню потрапив хворий з розчухами на голові. При огляді виявлені комахи, сірого кольору, довжиною 3 мм, із сплощеним у дорзовентральному напрямі тілом і трьома парами кінцівок. Вказана ситуація характерна для: A *Педикульозу B Скабієсу C Ураження шкіри клопами D Алергії E Демодекозу

50

У людини виявлено протозойне захворювання, при якому вражений головний мозок і спостерігається втрата зору. При аналізі крові знайдені одноклітинні півмісячної форми з загостреним кінцем. Збудником цього захворювання є: A *Токсоплазма B Лейшманія C Лямблія D Амеба E Трихомонада

51 У медико-генетичну консультацію звернувся юнак з приводу відхилень у фізичному і статевому розвитку. При мікроскопії клітин слизової оболонки рота виявлене одне тільце Барра. Вкажіть найбільш вірогідний каріотип юнака: A *47, ХХУ; B 45, Х0; C 47, 21+ D 47, 18+ E 47, ХУУ.

52 У фекаліях хворого з розладами травлення виявлені зрілі нерухомі членики ціп’яка; матка в них має 7-12 бічних відгалужень. Який це може бути вид гельмінта? A * Ціп’як озброєний B Ціп’як неозброєний C Ціп’як карликовий D Стьожак широкий E Ціп’як ехінокока

53 В експерименті на культуру тканин, що мітотично діляться, подіяли препаратом який руйнує веретено поділу. Це призвело до порушення: A * Розходження хромосом до полюсів клітини B Постсинтетичного періоду C Формування ядерної оболонки D Подвоєння хроматид E Деспіралізації хромосом

54 До лікаря звернулося кілька жителів одного села з однаковими симптомами: набряк повік та обличчя, сильний м"язевий біль, висока температура, головний болі. Усі хворі три

Page 7: крок 1 база 2014р_11шр-240ст (для брошури)_35грн

http://vk.com/my.printing

7

тижні тому були гостями на весіллі, де страви були приготовані із свинини. Лікар запідозрив трихінельоз. Який метод допоможе підтвердити діагноз: A *Імунологічний; B Овогельмінтоскопія; C Аналіз крові; D Аналіз сечі; E Анліз мокроти.

55 При обстеженні букального епітелію чоловіка з євнухоїдними ознаками у багатьох клітинах був виявлений статевий Х- хроматин. Для якої хромосомної хвороби це характерно? A *Синдром Клайнфельтера B Синром Дауна C Трисомія за Х-хромосомою D Синдром Шерешевського-Тернера E Синдром Марфана.

56 Після аналізу родоводу, лікар - генетик встановив: ознака проявляється у кожному поколінні, жінки та чоловіки спадкують ознаку однаково часто, батьки в однаковій мірі передають ознаки своїм дітям. Визначте, який тип успадкування має досліджувана ознака? A *Аутосомно- домінантний B Аутосомно- рецесивний C Полігенний D Х-зчеплений рецесивний E У- зчеплений

57 У медико- генетичну консультацію звернулося подружжя у зв"язку з народженням дитини з багатьма вадами розвиту (мікроцефалія, ідіотія тощо).дитини . Жінка під час вагітності хворіла, але мутагенів та тератогенів не вживала. Каріотип батьків і дитини нормальний. Як вияснив лікар, в квартирі сім"я утримує кота. Що може бути ймовірною причиною каліцтва новонародженої дитини. A *Під час вагітності жінка хворіла на токсоплазмоз B Під час вагітності жінка хворіла на лейшманіоз C Під час вагітності жінка хворіла на дизентерію

D Під час вагітності жінка хворіла на балантидіаз E Під час вагітності жінка хворіла на трихомоноз.

58 У новонародженого хлопчика спостерігається деформація мозкового та лицьового черепа, мікрофтальмія, деформація вушної раковини, вовча паща, і т.ін. Каріотип дитини виявився 47,ХУ,13+. Про яку хворобу йде мова: A *Синдром Патау B Синдром Клайнфельтера C Синдром Едвардса D Синдром Дауна E Синдром Шерешевського-Теренера

59 При розтині жінки в тканинах головного мозку були виявлені цистицерки. Причиною смерті було відмічено цистицеркоз мозку. Який паразит спричинив дане захворювання? A *Taenia solium. B Taeniarhynchus saginatus C Fasciola hepatica D Hymenolepis nana E Alveococcus multilocularis

60 До лікарні потрапили пацієнти зі скаргами: слабість, болі в кишечнику, розлади травлення. Після дослідження фекалій були виявлені цисти з чотирма ядрами. Для якого найпростішого характерні такі цисти? A *амеба дизентерійна B амеба кишкова C балантидій D амеба ротова E лямблія

61 У хворого спостерігається типова для нападу малярії клінічна картина: пропасниця, жар, проливний піт. Яка стадія малярійного плазмодію найвірогідніше буде виявлена в крові хворого в цей час? A *Мерозоїт B Спорозоїт C Оокінета D Спороциста

Page 8: крок 1 база 2014р_11шр-240ст (для брошури)_35грн

8

E Мікро- або макрогамети

62 За даними ВООЗ малярією щорічно на Землі хворіють приблизно 250 млн. чоловік. Ця хвороба зустрічається переважно у тропічних і субтропічних областях. Межі ії розповсюдження співпадають з ареалами комарів роду: A * Анофелес B Кулекс C Аедес D Мансоніа E Кулізета

63 У малярійного плазмодія - збудника триденної малярії розрізняють два штами: південний та північний. Вони відрізняються тривалістю інкубаційного періода: у південного він короткий, а у північного - довгий. В цьому проявляється виражена дія добору: A *Дизруптивного B Стабілізуючого C Штучного D Статевого E Рушійного

64 Гризуни є резервуаром збудників лейшманіозів - природно-осередкових захворювань, які переносяться трансмісивно. Якщо людина потрапила в осередок лейшманіозу, то їй необхідно уникати укусів: A * Москітів: B Бліх C Кліщів D Комарів E Кровосисних мух

65 В медико - генетичному центрі проведено каріотипування дитини з такими ознаками: вкорочення кінцівок, маленький череп, аномалії будови обличчя, вузькі очні щілини, епікант, розумова відсталість, порушення будови внутрішніх органів. Вкажіть найбільш вірогідний каріотип: A *47, 21+ B 47, 13+ C 47, 18+ D 47, ХХУ E 47, ХХХ

66 Згідно правила сталості числа хромосом кожний вид більшості тварин має певне і стале число хромосом. Механізмом, що підтримує цю сталість при статевому розмноженні організмів є : A *Мейоз B Шизогонія C Амітоз D Регенерація E Брунькування

67 Для вивчення локалізації біосинтезу білка в клітини мишей ввели мічені амінокислоти аланін і триптофан. Біля яких органел спостерігається накопичення мічених амінокислот: A * Рибосоми B Гладенька ЕПС C Клітинний центр D Лізосоми E Апарат Гольджі

68 Надмірна волосатість вушних раковин (гіпертрихоз) визначається геном, локалізованим у Y-хромосомі. Цю ознаку має батько. Імовірність народження хлопчика з такою аномалією буде: A * 100% B 0% C 25% D 35% E 75%

69 До гінеколога звернулася 28-річна жінка з приводу безпліддя. Під час обстеження виявлено: недорозвинені яєчники та матка, нерегулярний менструальний цикл. При дослідженні статевого хроматину в більшості соматичних клітин знайдені 2 тільця Барра. Яка хромосомна хвороба найбільш вірогідна в цієї жінки? A * Трисомія Х B Клайнфельтера C Патау D Шрешевського-Тернера E Едвардса

Page 9: крок 1 база 2014р_11шр-240ст (для брошури)_35грн

http://vk.com/my.printing

9

70 Під час мітотичного поділу диплоідної соматичної клітини на неї подіяли колхіцином. Хід мітозу порушився і утворилась одноядерна поліплоїдна клітина. Мітоз було призупинено на стадії: A *Анафаза B Профаза C Метафаза D Телофаза E Цитокінез

71 У дівчини виявлена диспропорція тіла, крилоподібні складки шкіри на шиї. При цитогенетичному дослідженні у ядрах лейкоцитів не виявлені “барабанні палички”, а у ядрах букального епітелію відсутні тільця Барра. Попередній діагноз буде: A *Синдром Шерешевського-Тернера B Синдром Клайнфельтера C Синдром Дауна D Синдром Патау E Синдром Едвардса

72 На певному етапі онтогенезу людини між кровоносними системами матері і плоду встановлюється фізіологічний зв’язок. Цю функцію виконує провізорний орган: A *Плацента B Жовтковий мішок C Амніон D Серозна оболонка E Алантоіс

73 У людини після укусу москітом виникли виразки шкіри. Аналіз вмісту виразки виявив внутрі клітин людини безджгутикові одноклітинні організми. Вкажіть попередній діагноз протозойного захворювання:: A *Лейшманіоз дерматотропний. B Лейшманіоз вісцеральний. C Трипаносомоз D Токсоплазмоз. E Балантідіоз.

74 Хворому 7 років. Спостерігається кишкове захворювання, що супроводжується загальною слабкістю, поганим апетитом, проносом, надчеревним болем, нудотою.

При обстеженні дуоденального вмісту виявлені вегетативні джгутикові форми грушоподібної форми з 4 джгутиками та 2 ядрами. Яке захворювання може бути у хворого? A *Лямбліоз. B Лейшманіоз вісцеральний. C Трихомоноз D Токсоплазмоз. E Малярія

75 До лікаря потрапив чоловік 35 років зі скаргами на біль в області печінки. З'ясовано, що хворий часто вживає недосмажену рибуу. У фекаліях виявлені дуже маленькі яйця гельмінту коричневого кольору, з кришечкою овальної форми. Який гельмінтоз найбільш вірогідний? A *Опісторхоз B Парагонімоз C Фасціольоз D Шистосомоз E Дікроцеліоз.

76 В медико-генетичну консультацію звернулось подружжя з питанням про ймовірність народження у них дітей хворих на гемофілію. Подружжя здорове, але батько дружини хворий на гемофілію. На гемофілію можуть захворіти: A *Половина синів B Сини і дочки. C Тільки дочки. D Половина дочок. E Всі діти

77 У вівчара, який пас отару овець під охороною собак, через деякий час з’явився біль у грудях, кровохаркання. Рентгенологічно у легенях виявлено кулясте утворення. Імунологічна реакція Касоні позитивна. Вкажіть гельмінта, який міг спричинити це захворювання: A * Ехінокок B Ціп’як карликовий C Лентець широкий D Печінковий сисун E Ціп'як озброєний

Page 10: крок 1 база 2014р_11шр-240ст (для брошури)_35грн

10

78 Хворій під час пологів перелили кров донора, який прибув із Анголи. Через два тижні у рецепієнтки виникла пропасниця. Яке лабораторне дослідження необхідно використати для підтвердження діагнозу малярії? A *Вивчення мазка товстої краплі крові для знаходження еритроцитарних стадій збудника. B Вивчення лейкоцитарної формули. C Визначення збудника методом посіву крові на поживне середовище. D Проведення серологічних досліджень. E Вивчення пунктату лімфатичних вузлів.

79 До лікаря звернувся пацієнт з приводу сильної сверблячки шкіри, особливо між пальцями рук, у пахових западинах. на нижній частині живота. При огляді шкіри хворого помічені звивисті ходи білувато-брудного кольору з крапинками на кінцях. Який діагноз міг передбачити лікар? A *Скабієс. B Педикульоз. C Дерматотропний лейшманіоз. D Демодекоз (вугрова залозниця). E Міаз.

80 При вивченні фаз мітотичного циклу знайдено клітину, в якій хромосоми лежать в екваторіальній площині, створюючи зірку. На якій стадії мітозу перебуває клітина? A *Метафази. B Профази. C Анафази. D Телофази. E Інтерфази.

81 У районах Південної Африки у людей розповсюджена серпоподібно-клітинна анемія, при якій еритроцити мають форму серпа внаслідок зміни в молекулі гемоглобіну амінокислоти глутаміну на валін. Чим викликана ця хвороба? A *Генною мутацією. B Порушенням механізмів реалізації генетичної інформації. C Кросинговером. D Геномними мутаціями. E Трансдукцією.

82 У здорових батьків народився син з фенілкетонурією, але завдяки спеціальній дієті розвивався нормально. З якою формою мінливості пов’язано його одужання? A * Модифікаційна B Комбінативна C Соматична D Генотипова E Співвідносна

83 При обстеженні дівчини 18 років знайдені ознаки: недорозвинення яєчників, широкі плечі, вузький таз, вкорочення нижніх кінцівок, “шия сфінкса”, розумовий розвиток не порушено. Встановлено діагноз – синдром Шерешевского-Тернера. Які порушення хромосом у хворої? A *моносомія Х B трисомія Х C трисомія 13 D трисомія 18 E нульосомія Х

84 При обстеженні хворого встановлено діагноз – кліщовий поворотний тиф. Яким шляхом міг заразитися хворий? A *Через укус селищного кліща B Через укус кліща роду Sarcoptes C Через укус тайгового кліща D Через укус гамазового кліща E Через укус собачого кліща

85 У деяких регіонах України поширилися місцеві випадки малярії. З якими комахами це пов'язано? A * комарі роду Anoрheles B москіти роду Phlebotomus C мошки роду Simullium D мокреці родини Ceratopogonidae E гедзі родини Tabanidae

86 У жінки народилася мертва дитина з багатьма видами аномалій розвитку. Яке протозойне захворювання могло спричинити внутрішньоутробну загибель плоду? A *Токсоплазмоз

Page 11: крок 1 база 2014р_11шр-240ст (для брошури)_35грн

http://vk.com/my.printing

11

B Амебіаз C Трихомоніаз D Лямбліоз E Балантідіаз

87 У немовля присутня мікроцефалія. Лікарі вважають, що це зв’язано з застосуванням жінкою під час вагітності актіноміцину D. На які зародкові листки подіяв цей тератоген? A *Ектодерма B Ентодерма C Мезодерма D Ентодерма та мезодерма E Усі листки

88 У медико-генетичну консультацію звернувся чоловік з приводу безпліддя. В ядрах більшості клітин епітелію слизової оболонки щоки було виявлено одне тільце Барра. Причиною такого стану може бути: A *Синдром Клайнфельтера B Синдром Шерешевського -Тернера C Трипло-Х D Синдром Дауна E Трипло-У

89 У генетично здорової жінки, яка під час вагітності перенесла вірусну кореву краснуху, народилася глуха дитина із розщілиною верхньої губи і піднебіння. Це є проявом: A *фенокопії B генних мутацій C генокопії D комбінативної мінливості E хромосомної аберації

90 До гастро-ентерологічного відділення поступив хворий із запаленням жовчних шляхів. У порціях жовчі виявлено рухомі найпростіші грушоподібної форми, двоядерні, з опорним стрижнем-аксостилем. Яке протозойне захворювання діагностується у хворого? A *Лямбліоз B Амебіаз кишковий C Балантидіаз кишковий D Трихомоноз E Амебна дизентерія

91

Хворий із скаргами на головний біль, біль у лівому підребер’ї. Захворювання почалось гостро з підвищенням температури до 40oС, лімфатичні вузли збільшені. Приступи повторювались ритмічно через 48 год. Визначте ймовірного збудника захворювання. A *Збудник 3-денної малярії B Збудник тропічної малярії C Збудник 4-денної малярії D токсоплазма E трипаносома

92 Дитина неспокійно спить, уві сні скрегоче зубами, часто розчухує область анального отвору. При огляді виявлені гельмінти довжиною до 1 см, ниткоподібної форми, білого кольору. Визначте вид гельмінта. A *Гострик B Аскарида C Вугриця кишкова D Трихінела E Волосоголовець

93 При дегельмінтизації у хворого виявлені довгі фрагменти гельмінта, що має членисту будову. Ширина члеників перевищує довжину, в центрі членика виявлено розеткоподібної форми утвір. Визначте вид гельмінта. A *Стьожак широкий B Ціп’як озброєний C Ціп’як неозброєний D Альвеокок E Карликовий ціп’як

94 У хворої дитини періодично з’являється рідке випорожнення, іноді біль у ділянці живота, нудота, блювання. За розповіддю матері, одного разу у дитини з блювотною масою виділився гельмінт веретеноподібної форми, розміром 20 см. Причиною такого стану може бути: A *Аскаридоз B Трихоцефальоз C Анкілостомоз D Дракункульоз E Трихінельоз

95 Мати і батько здорові. Методом

Page 12: крок 1 база 2014р_11шр-240ст (для брошури)_35грн

12

амніоцентезу визначено каріотип плода: 45 ХО. Діагноз A *Синдром Шершевського-Тернера B Синдром Едвардса C Синдром Патау D Синдром котячого крику E Синдром "супержінка"

96 Мати і батько здорові. Методом амніоцентезу визначено каріотип плода: 47, XX + 21. Діагноз: A *Синдром Дауна B Синдром Шерешевського-Тернера C Синдром Едвардса D Синдром котячого крику E Синдром "супержінка"

97 Юнак був обстежений у медико-генетичній консультації. Виявлений каріотип 47 ХУУ. Вкажіть найбільш ймовірний діагноз. A *Синдром "суперчоловік" B Синдром Патау C Синдром Кляйнфельтера D Синдром Шерешевського-Тернера E Синдром Едвардса

98 Під час дослідження клітин букального епітелію слизової оболонки щоки у пацієнта чоловічої статі виявлені 2 тільця Барра. Можливий діагноз: A *Синдром Кляйнфельтера B Синдром Шерешевського-Тернера C Синдром Патау D Синдром "супержінки" E Синдром "суперчоловіка"

99 При мікроскопії мазка фекалій школяра виявлені жовто-коричневого кольору яйця з горбкуватою оболонкою. Якому гельмінту вони належать? A *Аскарида B Гострик C Волосоголовець D Ціп’як карликовий E Стьожак широкий

100 У медико-генетичну консультацію звернулася хвора дівчина з попереднім діагнозом “синдром Шерешевського-Тернера”. Яким генетичним методом можна уточнити діагноз?

A * Цитогенетичний B Генеалогічний C Гібридологічний D Біохімічний E Дерматогліфіки

101 У генетично здорових батьків народилася дитина, хвора на фенілкетонурію (аутосомно-рецесивне спадкове захворювання). Які генотипи батьків? A *Аа х Аа B АА х АА C АА х Аа D Аа х аа E аа х аа

102 У хворого кров’яні випорожнення, 3-10 і більше разів на добу. Яке протозойне захворювання це може бути? A *Амебіаз B Лейшманіоз C Трипаносомоз D Трихомоноз E Малярія

103 При мікроскопії мазка фекалій виявлені чотирьохядерні цисти. Якому паразиту із Найпростіших вони належать? A * Дизентерійна амеба B Балантидій C Лямблія D Трихомонада E Токсоплазма

104 У пацієнта, який приїхав з Африки з’явилась кров у сечі. При мікроскопії осаду сечі виявлені яйця овальної форми, жовтого кольору, з шипом на одному з полюсів. Якому гельмінту вони належать? A *Шистосома B Опісторхіс C Клонорхіс D Парагонімус E Фасціола

105 Шахтар 48 років скаржиться на слабкість, головний біль, запаморочення, почуття важкості у шлунку. Раніше у нього була сильна сверблячка шкіри ніг, кропивниця. При

Page 13: крок 1 база 2014р_11шр-240ст (для брошури)_35грн

http://vk.com/my.printing

13

дослідженні виявлене недокрів'я. У своїх фекаліях хворий іноді бачив маленьких рухомих черв’яків червоного кольору, величиною приблизно 1 см. Яку найбільш імовірну хворобу може запідозрити лікар? A * анкілостомоз B Трихоцефальоз C Трихінельоз D Аскаридоз E Дракункульоз

106 На клітину подіяли колхіцином, що блокує “збирання” ахроматинового веретена. Які етапи мітотичного циклу будуть порушені? A *Анафаза B Профаза C Цитокінез D Передсинтетичний період інтерфази E Постсинтетичний період інтерфази

107 В органелі встановлена наявність власної білоксинтезуючої системи. Це органелла: A *Мітохондрії B Апарат Гольджі C Лізосоми D Вакуолі E Ендоплазматичний ретикулум

108 До приймальної медико-генетичної консультації звернулась пацієнтка. При огляді виявились слідуючи симптоми: трапецевидна шийна складка (шия “сфінкса”); широка грудна клітка, широко розставлені, слабо розвинені соски молочних залоз. Який найбільш ймовірний діагноз пацієнтки? A *Синдром Шерешевського-Тернера. B Синдром Патау. C Синдром Мориса D Синдром Клайнфельтера E Синдром “крику кішки”

109 Хворий звернувся із скаргою на загальну слабість, головний біль, нудоту, блювання, рідкі випорожнення із домішкою слизу і крові. При мікроскопії дуоденального вмісту і при дослідженні свіжих фекалій виявлено рухомі личинки. Поставте діагноз. A *Стронгілоїдоз

B Анкілостомоз C Ентеробіоз D Трихоцефальоз E Дракункульоз

110 Провідником наукової експедиції по Індії був місцевий житель, який ніколи не розлучався зі своєю улюбленою собакою. Якими інвазійними захворюваннями можуть бути заражені члени експедиції при контакті з цією собакою, якщо вона є джерелом інвазії? A *ехінококозом B теніозом C парагонімозом D дикроцеліозом E фасціольозом

111 При поділі клітини досліднику вдалося спостерігати фазу, при якій були відсутні мембрана ядра, ядерце, центріолі знаходились на полюсах клітини. Хромосоми мали вигляд клубка ниток, які вільно розташовані у цитоплазмі. Для якої фази це характерно? A * Профази B Метафази C Анафази D Інтерфази E Телофази

112 До медико-генетичної консультації звернулися батьки хворої дівчинки 5 років. Після дослідження каріотипу виявили 46 хромосом. Одна з хромосом 15-ї пари була довша від звичайної, тому що до неї приєдналася хромосома з 21-ї пари. Який вид мутації має місце в цієї дівчинки? A * Транслокація B Делеція C Інверсія D Нестача E Дуплікація

113 До лікаря звернувся юнак 16 років зі скаргами на свербіння між пальцями рук і на животі, яке посилювалося вночі. При огляді на шкірі були виявлені тоненькі смужки сірого кольору

Page 14: крок 1 база 2014р_11шр-240ст (для брошури)_35грн

14

і дрібненький висип. Який найбільш вірогідний збудник цієї хвороби? A *Sarcoptes scabiei B Ixodes ricinus C Ornitodorus papillipes D Dermacentor pictus E Ixodes persulcatus

114 При дослідженні каріотипу у паціента були виявлені два типи клітин в рівній частині з хромосомними наборами 46XY та 47XХY. Який діагноз поставить лікар? A *Синдром Клайнфельтера B Моносомія-Х C Синдром Патау D Синдром Дауна E Нормальний каріотип

115 У хворого виявлено розлади травлення, болі в животі, слинотеча. Схожі симптоми у нього проявлялися і раніше. При лабораторній діагностиці у фекаліях виявлені яйця овальної форми, вкриті бугристою оболонкою. Визначте можливу причину розладів здоров’я людини: A *аскаридоз B трихоцефальоз C діфілоботріоз D ентеробіоз E фасциольоз

116 До лікарні потрапив хворий з Східного Сибіру зі скаргою на біль у печінці. У фекаліях знайдені яйця до 30 мкм, які по формі нагадують насіння огірків. Який попередній діагноз можна поставити хворому? A *Опісторхоз. B Гіменолепідоз. C Дікроцеліоз. D Парагонімоз. E Теніарінхоз.

117 В родині зростає дочка 14 років у якої спостерігаються деякі відхилення від норми: зріст її нижче, ніж у однолітків, відсутні ознаки статевого дозрівання, шия дуже коротка, плечі широкі. Інтелект в нормі. Яке захворювання

можна припустити? A *Синдром Шерешевського-Тернера B Синдром Дауна C Синдром Едвардса D Синдром Патау E Синдром Клайнфельтера

118 При медичному огляді у військкоматі був виявлений хлопчик 15 років, високого зросту, з євнухоїдними пропорціями тіла, гінекомастією, волосся на лобку росте за жіночим типом. Відмічається відкладання жиру на стегнах, відсутність росту волосся на обличчі, високий голос, коефіцієнт інтелекту – знижений. Виберіть каріотип, що відповідає даному захворюванню. A *47, ХХУ B 45, ХО C 46, ХХ D 46, ХУ E 47, ХХХ

119 Фенілкетонурія успадковується як аутосомна рецесивна ознака. У родині, де обидвоє батьків здорові, народилася дитина, хвора на фенілкетонурію. Які генотипи батьків? A *Аа х Аа B АА х АА C АА х Аа D Аа х аа E аа х аа

120 Хвора звернулася до лікаря зі скаргами на появу в випорожненнях білих плоских рухливих утворів, які нагадують локшину. При лабораторному дослідженні виявлені членики з такою характеристикою: довгі, вузькі, з розміщеним поздовжньо каналом матки, яка має 17-35 бічних відгалужень з кожного боку. Який вид гельмінтів паразитує у кишечнику жінки? A *Taeniarhynchus saginatus B Taenia solium C Hymenolepis nana D Diphyllobothrium latum E Echinococcus granulosus

121

Page 15: крок 1 база 2014р_11шр-240ст (для брошури)_35грн

http://vk.com/my.printing

15

Человек длительное время проживал в условиях высокогорья. Какие изменения крови будут у него? A *Увеличение количества гемоглобина B Увеличение количества лейкоцитов C Снижение количества лейкоцитов D Урежение пульса E Увеличение диаметра кровеносных сосудов

122 В кариотипе матери 45 хромосом. Установлено, что это связано с транслокацией 21-й хромосомы на 15-ю. Какое заболевание вероятнее всего будет у ребенка, если кариотип отца нормальный? A *Синдром Дауна B Синдром Клайнфельтера C Синдром Патау D Синдром Эдвардса E Синдром Морриса

123 К врачу обратился больной по поводу округлых незаживающих изъязвлений на коже лица. Он недавно вернулся из Туркменистана. Врач заподозрил кожный лейшманиоз. Каким путем проник в организм человека возбудитель этого заболевания: A *Трансмиссивным. B Воздушно--капельным. C Контактно-бытовым. D Фекально-оральным. E Алиментарным.

124 З метою одержання каріотипу людини на клітини в культурі тканини подіяли колхіцином – речовиною, яка блокує скорочення ниток веретена поділу. На якій стадії припиняється мітоз? A *Метафаза B Інтерфаза C Анафаза D Профаза E Телофаза

125 У мазку дуоденального вмісту хворого з розладом травлення виявлено найпростіших

розміром 10-18 мкм. Тіло грушоподібної форми, 4 пари джгутиків, у розширеній передній частині тіла два ядра, які розміщені симетрично. Який вид найпростіших найбільш імовірний? A *Лямблія B Дизентерійна амеба C Трихомонада D Кишкова амеба E Балантидій

126 При обстеженні хворого була виявлена недостатня кількість імуноглобулінів. Порушена функція яких клітин імунної системи хворого може бути причиною цього? A *Плазматичні B Т-хелпери C Т-кілери D Т-супресори E Плазмобласти

127 У больного наблюдается типичная для приступа малярии клиническая картина: озноб, жар, проливной пот. Какая стадия малярийного плазмодия вероятнее всего будет обнаружена в крови больного в это время: A *Мерозоит B Шизонт C Спорозоит D Спороциста E Оокинета

128 РНК, содержащая вирус СПИДа, проникла внутрь лейкоцита и с помощью фермента ревертазы вынудила клетку синтезировать вирусную ДНК. В основе этого процесса лежит: A *Обратная транскрипция B Репрессия оперона C Обратная трансляция D Дерепрессия оперона E Конвариантная репликация

129 У дитини зі спадково обумовленими вадами зразу ж після народження спостерігався характерний синдром, який називають "крик кішки". При цьому у ранньому

Page 16: крок 1 база 2014р_11шр-240ст (для брошури)_35грн

16

дитинстві малюки мають "нявкаючий" тембр голосу. Під час дослідження каріотипу цієї дитини було виявлено: A *Делецію короткого плеча 5-ї хромосоми B Додаткову 21-у хромосому C Додаткову Х-хромосому D Нестачу Х-хромосоми E Додаткову Y-хромосому

130 Жінка з I (O) Rh- групою крові вийшла заміж за чоловіка з IV (AB) Rh+ групою крові. Який варіант групи крові і резус-фактора можна очікувати у дітей? A *III (B) Rh+ B I (O) Rh- C IV (AB) Rh+ D IV (AB) Rh- E I (O) Rh+

131 У новонародженої дитини виявлено наступну патологію: аномалія розвитку нижньої щелепи та гортані, що супроводжується характерними змінами голосу, а також мікроцефалія, вада серця, чотирьохпалість. Найбільш йморівною причиною таких аномалій є делеція: A * короткого плеча 5-ої хромосоми B короткого плеча 7-ої хромосоми C короткого плеча 9-ої хромосоми D короткого плеча 11-ої хромосоми E 21-ої хромосоми

132 У подружжя народився син, хворий на гемофілію. Батьки здорові, а дідусь за материнською лінією також хворий на гемофілію. Визначте тип успадкування ознаки. A *Рецесивний, зчеплений зі статтю B Аутосомно-ресесивний C Домінантний, зчеплений зі статтю D Неповне домінування E Аутосомно-домінантний

133 У немовляти виявлено мікроцефалію. Лікарі вважають, що це пов’язано з застосуванням жінкою під час вагітності актиноміцину Д. На які зародкові листки в першу чергу подіяв цей тератоген? A * Ектодерма B Усі листки

C Ентодерма D Мезодерма E Ентодерма та мезодерма

134 У юнака 18 років діагностовано хворобу Марфана. При дослідженні встановлено порушення розвитку сполучної тканини, будови кришталика ока, аномалії серцево-судинної системи, арахнодактилію. Яке генетичне явище зумовлює розвиток цієї хвороби? A * Плейотропія B Комплементарність C Кодомінування D Множинний алелізм E Неповне домінування

135 В одному з районів Полісся для боротьби з гельмінтозом, характерними ознаками якого є судоми, набряки обличчя, були розроблені профілактичні заходи. Серед них особлива увага зверталася на заборону вживання в їжу зараженого м’яса свинини навіть після термічної обробки. Про який гельмінтоз іде мова? A *Трихінельоз B Теніарінхоз C Аскаридоз D Ехінококоз E Альвеококоз

136 До медико-генетичної консультації звернулась жінка. При огляді у неї виявилися такі симптоми: крилоподібні шийні складки (шия „сфінкса”); широка грудна клітка, слабко розвинені молочні залози. Під час дослідження клітин букального епітелію в ядрах не було виявлено жодної грудочки Х-хроматину. Це вказує що у пацієнтки: A * Синдром Шерешевського-Тернера B Синдром Клайнфельтера C Синдром Патау D Синдром Дауна E Синдром Едвардса

137 Жінка під час вагітності хворіла на вірусну краснуху. Дитина у неї народилась з вадами розвитку – незрощення губи і піднебіння. Генотип у дитини нормальний. Ці аномалії

Page 17: крок 1 база 2014р_11шр-240ст (для брошури)_35грн

http://vk.com/my.printing

17

розвитку є проявом: A *Модифікаційної мінливості B Поліплоїдії C Комбінативної мінливості D Хромосомної мутації E Анеуплодії

138 Організми мають ядро, оточене ядерною мембраною. Генетичний матеріал зосереджений переважно в хромосомах, які складаються з ниток ДНК і білкових молекул. Діляться ці клітини мітотично. Це: A * Еукаріоти B Бактеріофаги C Прокаріоти D Віруси E Бактерії

139 У пацієнта, що прибув з ендемічного за малярією району, підвищилася температура тіла, відзначається головний біль, озноб, загальне нездужання – симптоми, що характерні й для звичайної застуди. Які лабораторні дослідження необхідно провести, щоб підтвердити або спростувати діагноз “малярія”? A * Мікроскопія мазків крові B Дослідження пунктату лімфовузлів C Аналіз сечі D Дослідження спинномозкової рідини E Мікроскопія пунктату червоного кісткового мозку

140 При дослідженні вмісту дванадцятипалої кишки людини знайдені найпростіші грушоподібної форми з парними ядрами, чотирма парами джгутиків. Між ядрами – дві опірні нитки, з вентрального боку розташований присмоктувальний диск. Який представник найпростіших виявлений у хворого? A * лямблія B токсоплазма C лейшманія D трихомонада кишкова E трипаносома

Біохімія 1 Метильні групи (-СН3) використовуються в організмі для синтезу таких важливих

сполук, як креатин, холін, адреналін, інші. Джерелом цих груп є одна з незамінних амінокислот, а саме: A *Метионін B Валін C Лейцин D Ізолейцин E Триптофан

2 Похідні птерину – аміноптерин і метотрексат – є конкурентними інгібіторами дігідрофолатредуктази, внаслідок чого вони пригнічують регенерацію тетрагідрофолієвої кислоти з дигідрофолату. Ці лікарські засоби приводять до гальмування міжмолекулярного транспорту одновуглецевих груп. Біосинтез якого полімеру при цьому пригнічується? A *ДНК B Білок C Гомополісахариди D Гангліозиди E Глікозаміноглікани

3 Характерною ознакою глікогенозу є біль у м”язах під час фізичної роботи. В крові реєструється гіпоглікемія. Вроджена недостатність якого фермента зумовлює цю патологію? A * Глікогенфосфорилази B Глюкозо-6-фосфатдегідрогенази C альфа-амілази D гама-амілази E Лізосомальної глікозидази

4 У пацієнта, що перебував у зоні радіаційного ураження, в крові збільшилась концентрація малонового діальдегіду, гідропероксидів. Причиною даних змін могло послужити: A * Збільшення в організмі кисневих радикалів і активація ПОЛ B Збільшення кетонових тіл C Збільшення молочної кислоти D Збільшення холестерину E Зменшення білків крові

5 Ціанід калію, що потрапив в організм пацієнта Б і викликав смерть через кілька хвилин на фоні явищ гіпоксії. Найбільш ймовірною причиною токсичної дії ціаніду було

Page 18: крок 1 база 2014р_11шр-240ст (для брошури)_35грн

18

гальмування активності: A * Цитохромоксидази B НАДН-дегідрогенази C АТФ-синтетази D НАДФН-дегідрогенази E АТФ-ази

6 У немовляти внаслідок неправильного годування виникла виражена діарея. Одним з основних наслідків діареї є екскреція великої кількості бікарбонату натрію. Яка форма порушення кислотно-лужного балансу має місце у цьому випадку? A * Метаболічний ацидоз B Метаболічний алкалоз C Респіраторний ацидоз D Респіраторний алкалоз E Не буде порушень кислотно-лужного балансу

7 Хвора Л., 46 років скаржиться на сухість в роті, спрагу, почащений сечоспуск, загальну слабкість. При біохімічному дослідженні крові виявлено гіперглікемію, гіперкетонемію. В сечі-глюкоза, кетонові тіла. На електрокардіограмі дифузні зміни в міокарді. У хворої вірогідно: A * Цукровий діабет B Аліментарна гіперглікемія C Гострий панкреатит D Нецукровий діабет E Ішемічна хвороба серця

8 На прийом до терапевта прийшов чоловік 37 років зі скаргами на періодичні інтенсивні больові приступи у суглобах великого пальця стопи та їх припухлість. При аналізі сечі встановлено її різко кислий характер і рожеве забарвлення. З наявністю яких речовин можуть бути пов’язані такі зміни сечі? A * Солі сечової кислоти B Хлориди C Амонієві солі D Фосфат кальцію E Сульфат магнію

9

У крові хворого на рак сечового міхура знайдено високий вміст серотоніну та оксиантранілової кислоти. З надлишком надходження в організм якої амінокислоти це пов’язано? A *Триптофану B Аланіну C Гістидину D Метіоніну E Тирозину

10 Після переводу на змішане харчування у новонародженої дитини виникла диспепсія з діареєю, метеоризмом, відставанням у розвитку. Біохімічна основа даної патології полягає у недостатності: A *Сахарази й ізомальтази B Лактази і целобіази C Трипсину і хімотрипсину D Ліпази і креатинкінази E Целюлази

11 Мати звернулася до лікаря: у дитини 5 років під дією сонячних променів на шкірі з’являються еритеми, везикулярний висип, дитина скаржиться на свербіж. Дослідження крові виявили зменшення заліза у сироватці крові, збільшення виділення з сечею уропорфіриногену І. Найбільш імовірною спадковою патологією у дитини є: A *Еритропоетична порфірія B Метгемоглобінемія C Печінкова порфірия D Копропорфірія E Інтермітуюча порфірия

12 У 3-річної дитини з підвищеною температурою тіла після прийому аспірину спостерігається посилений гемоліз еритроцитів. Вроджена недостатність якого фермента могла викликати у дитини гемолітичну анемію? A *Глюкозо-6-фосфатдегідрогенази B Глюкозо-6-фосфатази C Глікогенфоссфорилази D Гліцеролфосфатдегідрогенази E Гамма-глутамілтрансферази

13 Біохімічний аналіз сироватки крові пацієнта з гепатолентикулярною дегенерацією (хвороба Вільсона-Коновалова) виявив зниження вмісту церулоплазміну.

Page 19: крок 1 база 2014р_11шр-240ст (для брошури)_35грн

http://vk.com/my.printing

19

Концентрація яких іонів буде підвищена в сироватці крові цього пацієнта? A *Мідь B Кальцій C Фосфор D Калій E Натрій

14 Немовля відмовляється від годування груддю, збудливе, дихання неритмічне, сеча має специфічний запах “пивної закваски” або “кленового сиропу”. Вроджений дефект якого ферменту викликав дану патологію? A *Дегідрогеназа розгалужених альфа-кетокислот B Глюкозо-6-фосфатдегідрогеназа C Гліцеролкіназа D Аспартатамінотрансфераза E УДФ-глюкуронілтрансфераза

15 У немовляти на 6 день життя в сечі виявлено надлишок фенілпірувату та фенілацетату. Обмін якої амінокислоти порушено в організмі дитини? A *Фенілаланін B Триптофан C Метіонін D Гістидин E Аргінін

16 У крові 12-річного хлопчика виявлено зниження концентрації сечової кислоти і накопичення ксантину та гіпоксантину. Генетичний дефект якого ферменту має місце у дитини? A *Ксантиноксидаза B Аргіназа C Уреаза D Орнітинкарбамоїлтрансфераза E Гліцеролкіназа

17 У сироватці крові пацієнта встановлено підвищення активності гіалуронідази. Визначення якого біохімічного показника сироватки крові дозволить підтвердити припущення про патологію сполучної тканини?

A *Сіалові кислоти B Білірубін C Сечова кислота D Глюкоза E Галактоза

18 У регуляції активності ферментів важливе місце належить їхній постсинтетичній ковалентній модифікації. Яким із зазначених механізмів здійснюється регуляція активності глікогенфосфорилази і глікогенсинтетази? A *Фосфорилювання-дефосфорилювання B Метилювання C Аденілювання D Обмежений протеоліз E АДФ-рибозилювання

19 Відомо, що в деяких біогеохімічних зонах розповсюджене захворювання на ендемічний зоб. Недостача якого біоелемента викликає це захворювання? A *Йода B Заліза C Цинка D Міді E Кобальта

20 Хворий 45 скаржиться на невгамовну спрагу, споживання великої кількості рідини (до 5 л), виділення значної кількості сечі (до 6 л на добу). Концентрація глюкози крові становить 4,4 ммоль/л, рівень кетонових тіл не підвищений. Сеча незабарвлена, питома вага 1,002; цукор у сечі не визначається. Дефіцит якого гормону може призводити до таких змін? A *Вазопресину B Альдостерону C Інсуліну D Глюкагону E АКТГ

21 Хвора 46-ти років довгий час страждає прогресуючою м’язовою дистрофією (Дюшена). Зміни рівня якого ферменту крові є діагностичним тестом в даному випадку? A *Креатинфосфокінази B Лактатдегідрогенази

Page 20: крок 1 база 2014р_11шр-240ст (для брошури)_35грн

20

C Піруватдегідрогенази D Глутаматдегідрогенази E Аденілаткінази

22 Хворий хворіє на цукровий діабет, що супроводжується гіперглікемією натще понад 7,2 ммоль/л. Рівень якого білка плазми крові дозволяє ретроспективно (за попередні 4-8 тижні до обстеження) оцінити рівень глікемії A *Глікозильований гемоглобін B Альбумін C Фібріноген D С-реактивний білок E Церулоплазмін

23 Гідроксипролін є важливою амінокислотою у складі колагену. За участю якого вітаміну відбувається утворення цієї амінокислоти шляхом гідроксилювання проліну? A *C B D C B1 D B2 E B6

24 Захисна функція слини зумовлена декількома механізмами, в тому числі наявністю ферменту, який має бактерицидну дію, викликає лізис полісахаридного комплексу оболонки стафілококів, стрептококів. Укажіть цей фермент A *Лізоцим B альфа-амілаза C Оліго-1,6-глюкозидаза D Колагеназа E бета-глюкуронідаза

25 Альбіноси погано переносять сонячний загар, з'являються опіки. Порушення метаболізму якої амінокислоти лежить в основі цього явища? A *Фенілаланіну B Метіоніну C Триптофану D Глутамінової кислоти E Гістидину

26 В процесі катаболізму гемоглобіну звільняється залізо, яке в складі

спеціального транспортного білку надходить в кістковий мозок і знову використовується для синтезу гемоглобіну. Цим транспортним білком є: A *Трансферин (сидерофілін) B Транскобаламін C Гаптоглобін D Церулоплазмін E Альбумін

27 У хворої суглоби збільшені , болючі.У крові пацієнтки підвищений рівень уратів.Як називається така патологія? A *Подагра B Рахіт C Скорбут D Пелагра E Каріес

28 Після прийому жирної їжі хворий відчуває дискомфорт, а а у калі неперетравлені краплі жиру.Реакція сечі на жовчні кислоти позитивна.Причиною такого стану є нестача: A *Жовчних кислот B Жирних кислот C Хіломікронів D Тригліцеридів E Фосфоліпідів

29 В процесі метаболізму в організмі людини виникають активні форми кисню, у тому числі супероксидний аніон-радикал О2. Цей аніон інактивується за допомогою ферменту: A * Супероксиддисмутази B Каталази C Пероксидази D Глутатіонпероксидази E Глутатіонредуктази

30 Центральним проміжним подуктом всіх обмінів (білків, ліпідів, вуглеводів) є: A * Ацетил-КоА B Сукциніл-КоА C Щавелево-оцтова кислота D Лактат E Цитрат

31 Через деякий час після інтенсивного фізичного тренування у спортсмена активується глюконеогенез, основним субстратом якого

Page 21: крок 1 база 2014р_11шр-240ст (для брошури)_35грн

http://vk.com/my.printing

21

в цьому випадку є: A *Лактат B Аспарагінова кислота C Глутамінова кислота D Альфа-кетоглутарат E Серин

32 При визначенні залишкового азоту знайшли, що азот сечовини значно знижений. Для захворювання якого органа це характерно? A *Печінки B Мозку C Серця D Кишечнику E Шлунка

33 При малярії призначають препарати – структурні аналоги вітаміну В2 (рибофлавіну). Порушення синтезу яких ферментів у плазмодія викликають ці препарати? A *ФАД-залежних дегідрогеназ B цитохромоксидаз C пептидаз D НАД- залежних дегідрогеназ E амінотрансфераз

34 Дівчинка 10-ти років часто хворіє на гострі респіраторні інфекції, після яких спостерігаються множинні точкові крововиливі в місцях тертя одягу. Вкажіть, гіповітаміноз якого вітаміну має місце в дівчинки: A *С B В6 C В1 D А E В2

35 Для лікування урогенітальних інфекцій використовують хінолони - інгібітори ферменту ДНК-гірази. Укажіть, який процес порушується під дією хінолонів у першу чергу. A *реплікація ДНК B репарація ДНК C ампліфікація генів D рекомбінація генів E зворотна транскрипція

36

Вагітній жінці, що мала в анамнезі декілька викиднів, призначена терапія, яка містить вітамінні препарати. Укажіть вітамін, який сприяє виношуванню вагітності. A *Альфа-токоферол. B Фолієва кислота. C Цианкобаламін. D Піридоксальфосфат. E Рутін.

37 Хворий страждає на гіпертонію, атеросклеротичне ураження судин. Укажіть, вживання якого ліпіду йому необхідно знизити в добовому раціоні. A *Холестерин. B Олеїнову кислоту. C Лецитин. D Моноолеатгліцерид. E Фосфатиділсерин.

38 Хворий знаходиться у стані гіпоглікемічної коми. Укажіть передозування якого гормону може привести до такої ситуації. A *Інсулін. B Прогестерон. C Кортизол. D Соматотропін. E Кортікотропін.

39 У хворого з черепномозковою травмою спостерігаються епілептиформні судомні напади, що регулярно повторюються. Утворення якого біогенного аміну порушено при цьому стані. A *ГАМК. B Гістамін. C Адреналін. D Серотонін. E Дофамін.

40 У хворого явна прогресуюча м’язова дистрофія. Назвіть показник обміну азоту сечі, характерний для такого стану. A *Креатин. B Амонійні солі. C Креатинин. D Сечова кислота. E Сечовина.

41

Page 22: крок 1 база 2014р_11шр-240ст (для брошури)_35грн

22

У хворого спостерігається атонія м’язів. Назвіть фермент м’язової тканини, активність якого може бути знижена при такому стані: A *Креатинфосфокіназа B Амілаза C Транскетолаза D Глутамінтрансфераза E Каталаза

42 В печінці хворого порушена детоксикація природних метаболітів та ксенобіотиків. Назвіть цитохром, активність якого може бути знижена. A *Цитохром Р-450. B Цитохромоксидаза. C Гемоглобін. D Цитохром b. E Цитохром с1.

43 У дитини спостерігається затримка росту і розумового розвитку, з сечею виділяється велика кількість оротової кислоти. Ця спадкова хвороба розвиваєтья внаслідок порушення: A *Синтезу піримідинових нуклеотидів B Розпаду піримідинових нуклеотидів C Синтезу пуринових нуклеотидів D Розпаду пуринових нуклеотидів E Перетворення рибонуклеотидів у дезоксирибонуклеотиди

44 У крові хворого виявлено підвищення активності ЛДГ 4,5, Ал-АТ, карбамоїлорнітинтрансферази. В якому органі можна передбачити розвиток патологічного процесу? A *У печінці (можливий гепатит). B У серцевому м’язі (можливий інфаркт міокарду). C У скелетних м’язах. D У нирках. E У сполучній тканині.

45 Тестовим показником на розвиток пухлини мозкової частини наднирників є рівень гормонів: A *Катехоламінів. B Мінералокортікоїдів. C Глюкокортікоїдів. D Статевих гормонів. E Кортиколіберинів.

46 У хворого виявлено підвищення активності ЛДГ1,2, Ас-АТ, креатинфосфокінази. В якому органі (органах) ймовірний розвиток патологічного процесу? A *У серцевому м’язі (початкова стадія інфаркта міокарда). B У скелетних м’язах (дистрофія, атрофія). C У нирках та наднирниках. D У сполучній тканині. E У печінці та нирках.

47 У доношеного новонародженого спостерігається жовте забарвлення шкіри та слизових оболонок. Ймовірною причиною такого стану може бути тимчасова нестача ферменту: A * УДФ - глюкуронілтрансферази B Уридинтрансферази C Гемсинтетази D Гемоксигенази E Білівердинредуктази

48 При алкаптонурії у сечі хворого знайдено велику кількість гомогентизинової кислоти (сеча темніє на повітрі). Вроджений дефект якого ферменту має місце. A *Оксидази гомогентизинової кислоти. B Аланінамінотрансферази C Тирозинази D Фенілаланін-4-монооксигенази E Тирозинамінотрансферази

49 Концентрація глюкози в плазмі крові здорової людини знаходиться в таких межах: A *3,5-5,5 ммоль /л B 2-4 ммоль/л C 10-25 ммоль/л D 6-9,5 ммоль/л E 1-2 ммоль/л

50 Яка сполука є попередником в синтезі простагландинів в організмі людини? A * Арахідонова кислота B Пальмітинова кислота C Ліноленова кислота D Олеїнова кислота E Ліноленова кислота

51 Знешкодження хвороботворних бактерій та

Page 23: крок 1 база 2014р_11шр-240ст (для брошури)_35грн

http://vk.com/my.printing

23

розщеплення чужерідних тіл в лейкоцитах здійснюється за типом реакції окислення: A * пероксидазного B оксидазного C оксигеназного D перекисного E анаеробного

52 У жінки 62-х років розвинулася катаракта (замутненість кришталику) на фоні цукрового діабету. Вкажіть, який тип модифікації білків має місце при діабетичній катаракті A *Гликозилювання B Фосфорилювання C АДФ-рибозилювання D Метилювання E Обмежений протеоліз

53 У чоловіка 35 років феохромоцитома. В крові виявляється підвищений рівень адреналіну та норадреналіну, концентрація вільних жирних кислот зростає в 11 разів. Вкажіть, активація якого ферменту під впливом адреналіну підвищує ліполіз. A *ТАГ-ліпази B Ліпопротеїдліпази C Фосфоліпази А2 D Фосфоліпази С E Холестеролестерази

54 Для серцевого м"яза характерним є аеробний характер окислення субстратів. Основним з них є : A *Жирні кислоти B Триацилгліцероли C Гліцерол D Глюкоза E Амінокислоти

55 У немовляти спостерігаються епілептиформні судоми, викликані дефіцитом вітаміну В6. Це спричинено зменшенням у нервовій тканиніі гальмівного медіатора - гамма-аміномасляної кислоти. Активність якого ферменту знижена: A *Глутаматдекарбоксилази B Аланінамінотрансферази C Глутаматдегідрогенази D Піридоксалькінази

E Глутаматсинтетази

56 У чоловіка 53 років діагностована хвороба Педжета. В добовій сечі різко підвищений рівень оксипроліну, що свідчить передусім про посилення розпаду : A * Колагену B Кератину C Альбуміну D Гемоглобіну E Фібриногену

57 Хворому поставили попередній діагноз інфаркт міокарда. Характерною ознакою для даного захворювання є суттєве підвищення в крові активності: A *Кретинфосфокінази B Каталази C Г-6-ФДГ D a-амілази E Аргінази

58 Хворий звернувся до лікаря зі скаргами на часте та надмірне сечовиділення, спрагу. При аналізі сечі виявлено - добовий діурез –19 літрів, щільність сечі 1,001. Для якого захворювання ці показники є характерними? A * Нецукровий діабет B Стероїдний діабет C Цукровий діабет D Тіреотоксикоз E Хвороба Аддісона

59 У дитини з точковою мутацією генів виявлено відсутність глюкозо-6-фосфатази, гіпоглікемію та гепатомегалію. Визначте вид патології, для якої характерні ці ознаки? A *Хвороба Гірке B Хвороба Кори C Хвороба Аддісона D Хвороба Паркінсона E Хвороба Мак-Ардла

60 При операції на щитовидній залозі з приводу захворювання на Базедову хворобу, помилково були видалені паращитовидні залози. Виникли судоми, тетанія. Обмін якого біоелемента було порушено?

Page 24: крок 1 база 2014р_11шр-240ст (для брошури)_35грн

24

A * Кальція B Магнія C Калія D Заліза E Натрія

61 Хворого доставлено у медичний заклад в коматозному стані. Зі слів супроводжуючих вдалося з’ясувати, що хворий знепритомнів під час тренування на завершальному етапі марафонської дистанції. Яку кому найімовірніше запідозрити у даного пацієнта? A *Гіпоглікемічну. B Гіперглікемічну. C Ацидотичну. D Гіпотіреоїдну. E Печінкову.

62 При цукровому діабеті і голодуванні в крові збільшується вміст ацетонових тіл, що використовуються в якості енергетичного матеріалу. Назвіть речовину, з якої вони синтезуються: A *Ацетіл-КоА B Сукциніл-КоА C Цитрат D Малат E Кетоглутарат

63 Окуліст виявив у хворого збільшення часу адаптації ока до темряви. Неостаність якого вітаміну може бути причиною такого симптому? A *Вітаміну А B Вітаміну Е C Вітаміну С D Вітаміну К E Вітаміну D

64 Хворий скаржився на загальну слабкість та кровотечу з ясен. Недостатність якого вітаміну можна припустити? A *Вітамін С B Вітамін Е C Вітамін РР D Вітамін D E Вітамін В1

65 При обстеженні хворого виявлені дерматит, діарея, деменція. Вкажіть, відсутність якого вітаміну являється причиною цього стану. A *Нікотінаміду.

B Аскорбінова кислота. C Фолієва кислота. D Біотин. E Рутин.

66 У хворого діагностовано мегалобластичну анемію.Вкажіть сполуку, недостатня кількість якої може приводити до розвитку цієї хвороби. A *Ціанокобаламін. B Гліцин. C Мідь. D Холекальціферол. E Магній.

67 Яке похідне гемоглобіну виявляється в крові при отруєнні чадним газом (монооксидом вуглецю) A *Карбоксигемоглобін. B Метгемоглобін. C Оксигемоглобін. D Карбгемоглобін. E Вердогемоглобін.

68 При обстеженні хворого виявлена характерна клініка колагенозу. Вкажіть, збільшення якого показника сечі характерне для цієї патології. A *Гідроксипролін. B Аргінін. C Глюкоза. D Мінеральні солі. E Солі амонію.

69 При дослідженні крові хворого виявлено значне збільшення активності МВ-форм КФК (креатинфосфокінази ) та ЛДГ-1. Зробіть припущення можливої патології. A *Інфаркт міокарду. B Гепатит. C Ревматизм. D Панкреатит. E Холецистит.

70 У новонародженої дитини після годування молоком спостерігалися диспептичні розлади (диспепсія, блювота). При годуванні розчином глюкози ці явища зникали. Вкажіть фермент, що бере участь в перетравленні

Page 25: крок 1 база 2014р_11шр-240ст (для брошури)_35грн

http://vk.com/my.printing

25

вуглеводів, недостатня активність якого приводить до вказаних розладів. A *Лактаза. B Амілаза. C Сахараза. D Ізомальтаза. E Мальтаза.

71 Під час бігу на коротку дистанцію у нетренованих людей спостерігається м’язова крепатура внаслідок накопичення лактату. Вкажіть, з посиленням якого біохімічного процесу це може бути пов’язано. A *Гліколізу. B Глюконеогенезу. C Пентозофосфатного шляху. D Ліпогенезу. E Глікогенезу.

72 При патологічних процесах, які супроводжуються гіпоксією, відбувається неповне відновлення молекули кисню в дихальному ланцюзі і накопичення пероксиду водню. Вкажіть фермент, який забезпечує його руйнування. A *Каталаза. B Цитохромоксидаза. C Сукцинатдегідрогеназа. D Кетоглутаратдегідрогеназа. E Аконітаза.

73 Хворий напередодні операції знаходився в стані стресу. Збільшення концентрації якого гормону в крові супроводжує цей стан. A *Адреналін. B Інсулін. C Пролактин. D Прогестерон. E Глюкагон.

74 В психіатрії для лікування ряду захворювань ЦНС використовують біогенні аміни. Вкажіть препарат цієї групи, який являється медіатором гальмування. A *Гама-аміномасляна кислота. B Гістамін. C Серотонін. D Дофамін E Таурин..

75 При обстеженні хворого виявили застій жовчі в печінці та жовчні камені в жовчному міхурі. Вкажіть основний компонент жовчних каменів, які утворюються в цьому стані. A *Холестерин. B Тригліцериди. C Білірубінат кальцію. D Білок. E Мінеральні солі.

76 На судово-медичну експертизу надійшла кров дитини та передбачуваного батька для встановлення батьківства. Вкажіть ідентифікацію яких хімічних компонентів необхідно здійснити в дослідній крові. A *ДНК. B т-РНК. C р-РНК. D м-РНК. E мя-РНК.

77 На основі клінічних даних хворому поставлено попередній діагноз – гострий панкреатит. Вкажіть біохімічний тест, який підтверджує цей діагноз. A *Активність амілази кровіі. B Активність кислої фосфатази крові. C Активність лужної фосфатази крові. D Активність амінотрансфераз крові. E Рівень креатиніну в крові.

78 Електрофоретичне дослідження сироватки крові хворого пневмонією показало збільшення одної з білкових фракцій. Вкажіть її. A *Гама-глобуліни. B Альбуміни. C Альфа1-глобуліни. D Альфа2-глобуліни. E Бета-глобуліни.

79 Пацієнту 33 роки. Хворіє 10 років. Періодично звертається до лікаря зі скаргами на гострі болі в животі, судоми, порушення зору. У його родичів спостерігаються подібні симптоми. Сеча червоного кольору . Госпіталізований з діагнозом - гостра переміжна порфірія.

Page 26: крок 1 база 2014р_11шр-240ст (для брошури)_35грн

26

Причиною захворювання може бути порушення біосинтезу: A * Гему B Інсуліну C Жовчних кислот D Простагландинів E Колагену

80 У хворого з частими кровотечами у внутрішні органи і слизові оболонки у складі колагенових волокон виявили пролін і лізин. Відсутність якого вітаміну приводить до порушення їх гідроксилювання? A * С B Е C К D А E Д

81 У хворого 50 років діагностовано подагру, а в крові виявлено гіперурикемію. Обмін яких речовин порушений : A * Пуринів B Жирів C Амінокислот D Вуглеводів E Піримідину

82 У дитини грудного віку спостерігається потемніння склер, слизових оболонок, вушних раковин, виділена сеча темніє на повітрі. У крові та сечі виявлено гомогентизинову кислоту. Який найбільш імовірний симптом. A *Алкаптонурія B Альбінізм C Цистинурія D Порфірія E Гемолітична анемія

83 Хворий 20 років. Скаржиться на загальну слабість запаморочення, швидку стомлюваність. При обстеженні виявлено: гемоглобін крові 80 г/л, мікроскопічно виявлено еритроцити зміненої форми. Причиною може бути: A * Серповидноклітинна анемія B Паренхіматозна жовтяниця C Гостра переміжна порфірія D Обтураційна жовтяниця E Хвороба Аддісона

84

При хронічному панкреатиті спостерігається зменшення синтезу і секреції трипсину. Травлення яких речовин порушене? A * Розщеплення білків B Розщеплення полісахаридів C Розщеплення ліпідів D Розщеплення нуклеїнових кислот E Розщеплення жиророзчинних вітамінів

85 У дитини в крові підвищена кількість фенілпіровіноградної кислоти. Який вид лікування потрібен при фенілкетонемії? A *Дієтотерапія. B Вітамінотерапія. C Ферментотерапія. D Антібактеріальна терапія. E Гормонотерапія.

86 У хворого гострий приступ жовчно-кам'яної хвороби. Як це може бути відображено при лабораторному обстеженні? A *Негативна реакція на стеркобілін в калі. B Позитивна реакція на стеркобілін в калі. C Наявність сполучної тканини в калі. D Наявність перетравлюваної клітчатки в калі E Наявність крохмальних зерен в калі.

87 Для нормального метаболізму клітинам необхідні макроергічні сполуки. Що належить до макроергів? A *Креатинфосфат. B Креатин. C Креатинин. D Глюкозо-6-фосфат. E Аденозінмонофосфат.

88 Хворий 48 років звернувся до лікаря зі скаргами на сильні болі, припухлість, почервоніння в ділянках суглобів, підвищення температури до 38оС. В крові виявлено високий вміст уратів. Ймовірною причиною такого стану може бути порушення обміну: A * Пуринів B Колагену C Холестерину D Піримідинів E Вуглеводів

Page 27: крок 1 база 2014р_11шр-240ст (для брошури)_35грн

http://vk.com/my.printing

27

89 У чоловіка 53 років діагностовано сечокам’яну хворобу з утворенням уратів. Цьому пацієнту призначено аллопурінол, який є конкурентним інгібітором фермента: A * Ксантиноксидази B Уреази C Уратоксидази D Дигідроурацилдегідрогенази E Уридилілтрансферази

90 У пацієнта, що звернувся до лікаря спостерігається жовте забарвлення шкіри, сеча-темна, кал(темно-жовтого кольору. Підвищення концентрації якої речовини буде спостерігатися в сироватці крові? A * Вільного білірубіну B Кон’югованого білірубіну C Мезобілірубіну D Вердоглобіну E Білівердину

91 У юнака 18-и років діагностована м’язова дистрофія. Підвищення якої речовини в сироватці крові найбільш ймовірне при цій патології? A * Креатину B Міоглобіну C Міозину D Лактату E Аланіну

92 До лікарні поступив 9-ти річний хлопчик розумово і фізично відсталий. При біохімічному аналізі крові виявлено підвищену кількість фенілаланіну. Блокування якого фермента може призвести до такого стану? A * Фенілаланін-4-монооксигенази B Оксидази гомогентизинової кислоти C Глутамінтрансамінази D Аспартатамінотрансферази E Глутаматдекарбоксилази

93 Мати зауважила занадто темну сечу у її 5-річної дитини. Дитина скарг ніяких не висловлює. Жовчних пігментів у сечі не виявлено. Поставлено діагноз алкаптонурія. Дефіцит якого ферменту має місце? A * Оксидази гомогентизинової кислоти B Фенілаланінгідроксилази

C Тирозинази D Оксидази оксифенілпірувату E Декарбоксилази фенілпірувату

94 За клінічними показами хворому призначено приймання піридоксальфосфату. Для корекції яких процесів рекомендований цей препарат? A * Трансамінування і декарбоксилювання амінокислот B Окисного декарбоксилювання кетокислот C Дезамінування амінокислот D Синтезу пуринових і піримідинових основ E Синтезу білка

95 До лікаря звернувся хворий із скаргами на постійну спрагу. Виявлена гіперглікемія, поліурія та підвищений вміст 17-кетостероїдів у сечі. Яке захворювання ймовірне? A *Стероїдний діабет B Інсулінозалежний діабет C Мікседема D Глікогеноз I типу E Аддісонова хвороба

96 У пацієнта після вживання сирих яєць з?явилися дерматити. Який розвився авітаміноз? A *Авітаміноз біотіну B Авітаміноз фолієвої кислоти C Авітаміноз пантотенової кислоти D Авітаміноз параамінобензойної кислоти E Авітаміноз інозиту

97 У хворого в крові збільшена концентрація пірувата. Значна кількість екскретується з сечею. Авітаміноз якого вітаміну спостерігається у хворого? A *В1 B Е C В3 D В6 E В2

98 У клініку поступив хворий з підозрою на подагру. Який біохімічний аналіз слід назначити для уточнення діагнозу? A *Визначення сечової кислоти в крові та в сечі

Page 28: крок 1 база 2014р_11шр-240ст (для брошури)_35грн

28

B Визначення сечовини в крові та сечі C Визначення креатіну в крові D Визначення активності урікази в крові E Визначення амінокислот в крові

99 На основі лабораторного аналізу, у хворого підтверджено діагноз – подагра. Який аналіз був проведений для постановки діагноза? A *Визначення сечової кислоти в крові та сечі B Визначення креатинину в сечі C Визначення залишкового азоту в крові D Визначення сечовини в крові та сечі E Визначення аміаку в сечі

100 Аміак є дуже отруйною речовиною, особливо для нервової системи. Яка речовина приймає особливо активну участь у знешкодженні аміака у тканинах мозку? A *Глутамінова кислота B Лізин C Пролін D Гістидин E Аланін

101 Людина в стані спокою штучно примушує себе дихати часто і глибоко на протязі 3-4 хв. Як це відбивається на кислотно-лужній рівновазі організму? A *Виникає дихальний алкалоз B Виникає дихальний ацидоз C Виникає метаболічний алкалоз D Виникає метаболічний ацидоз E Кислотно-лужна рівновага не змінюється

102 Людина захворіла на пелагру. При опитуванні стало відомо, що напротязі тривалого часу вона харчувалась переважно кукурудзою, мало вживала м?яса. Що стало причиною виникнення пелагри? A *Дефіцит триптофану у кукурудзі B Дефіцит тирозину в кукурудзі. C Дефіцит проліну в кукурудзі D Дефіцит аланіну в кукурудзі E дефіцит гістидину в кукурудзі

103 У хворого виявлено різке похудання, підвищену подразливість, невелике підвищення температури тіла, екзофтальм,

гіперглікемію, азотемію. Яке це захворювання ? A * базедова хвороба B бронзова хвороба C невроз D туберкульоз наднирників E мікседерма

104 Ціаністий калій є отрутою, смерть організму наступає миттєво. Назвіть, на які ферменти в мітохондріях діє цианистий калій: A *цитохромоксидазу [аа3] B флавінові ферменти C цитохром В5 D НАД+ - залежні дегідрогенази E цитохром Р-450

105 Який з перелічених гормонів знижує швидкість ліполізу в жировій тканині? A * інсулін B адреналін C гідрокортизон D соматотропін E норадреналін

106 Перетравлення білків у шлунку є початковою стадією розщеплення білків у травному каналі людини. Назвіть ферменти, які беруть участь в перетравлені білків у шлунку: A * пепсин та гастриксин B трипсин та катепсини C хімотрипсин та лізоцим D ентеропептидаза та еластаза E карбоксипептидаза та амінопептидаза

107 У хворого на цукровий діабет після ін’єкції інсуліну настала втрата свідомості, судоми. Який результат дав біохімічний аналіз крові на вміст глюкози? A *2,5 ммоль/л B 3,3 ммоль/л C 8,0 ммоль/л D 10 ммоль/л E 5.5 ммоль/л

108 Біогенні аміни: гістамін, серотонін, ДОФамін та інші – дуже активні речовини, які впливають на різноманітні фізіологічні функції організму. В результаті якого процесу утворюються біогенні аміни в тканинах

Page 29: крок 1 база 2014р_11шр-240ст (для брошури)_35грн

http://vk.com/my.printing

29

організму ? A * декарбоксилювання амінокислот B дезамінування амінокислот C трансамінування амінокислот D окислення амінокислот E відновного реамінування

109 До лікаря звернулися батьки з 5-річною дитиною. При обстеженні виявлено: відставання розумового розвитку та росту, дитина малорухлива. Загальний обмін знижений. Яке захворювання у дитини? A * кретинізм B синдром Леша –Ніхана C фенілкетонурія D гіперпаратиреоз E ендемічний зоб

110 Універсальною біологічною системою окислення неполярних сполук [багато лікарських засобів, токсичних сполук], стероїдних гормонів, холестерину являється мікросомальне окислення. Назвіть, який цитохром входить до складу оксигеназного ланцюгу мікросом: A * цитохром Р 450 B цитохром а3 C цитохром в D цитохром c E цитохром а

111 У жінки 46 років, що страждає на жовчно-кам'яну хворобу, розвинулась жовтяниця. При цьому сеча стала темно-жовтого кольору, а кал - знебарвлений. Вкажіть, концентрація якої речовини в сироватці крові зросте в найбільшій мірі . A * Кон'югованого білірубіну B Вільного білірубіну C Білівердину D Мезобілірубіну E Уробіліногену

112 У малюка, що народився 2 дні тому недоношеним, спостерігається жовте забарвлення шкіри та слизових оболонок. Вкажіть, тимчасова нестача якого ферменту є причиною

цього стану малюка. A * УДФ - глюкуронілтрансферази B амінолевулінатсинтази C Гем- оксигенази D Гемсинтетази E Білівердинредуктази

113 Хворий 46 років звернувся до лікаря зі скаргою на біль в суглобах, яка посилюється напередодні зміни погоди. У крові виявлено підвищення концентрації сечової кислоти. Посилений розпад якої речовини є найімовірнішою причиною захворювання? A * АМФ B ЦМФ C УТФ D УМФ E ТМФ

114 Встановлено, що до складу пестициду входить арсенат натрію, який блокує ліпоєву кислоту. Вкажіть, активність яких ферментів порушується A * ПВК - дегідрогеназного комплексу B Мікросомального окислення C Метгемоглобінредуктази D Глутатіонпероксидази E Глутатіонредуктази

115 У відділення реанімації надійшов чоловік 47 років з діагнозом інфаркт міокарду. Яка з фракцій лактатдегідрогенази (ЛДГ) буде переважати в сироватці крові на протязі перших двох діб? A *ЛДГ1 B ЛДГ2 C ЛДГ3 D ЛДГ4 E ЛДГ5

116 У відділення інтенсивної терапії доставлено жінку 50 років з діагнозом інфаркт міокарду. Активність якого ферменту буде найбільш підвищена на протязі перших двох діб? A *Аспартатамінотрансферази B Аланінамінотрансферази C Аланінамінопептидази D ЛДГ4 E ЛДГ5

117 У хлопчика 2 років спостерігається

Page 30: крок 1 база 2014р_11шр-240ст (для брошури)_35грн

30

збільшення в розмірах печінки та селезінки, катаракта. В крові підвищена концентрація цукру, однак тест толерантності до глюкози в нормі. Вкажіть, спадкове порушення обміну якої речовини є причиною цього стану? A *Галактози B Фруктози C Глюкози D Мальтози E Сахарози

118 В сечі новонародженого визначається цитрулін та високий рівень аміаку. Вкажіть, утворення якої речовини наймовірніше порушене у цього малюка? A *Сечовини B Сечової кислоти C Аміаку D Креатиніну E Креатину

119 У лікарню поступила робітниця хімічного підприємства з ознаками отруєння. У волоссі цієї жінки знайдено підвищену концентрацію арсенату, який блокує ліпоєву кислоту. Вкажіть, порушення якого процесу є найімовірною причиною отруєння A *Окислювального декарбоксилювання ПВК B Мікросомального окислення C Відновлення метгемоглобіну D Відновлення органічних перекисей E Знешкодження супероксидних іонів

120 У чоловіка 42 років, який страждає на подагру в крові підвищена концентрація сечової кислоти. Для зниження рівню сечової кислоті йому призначено аллопуринол. Вкажіть, конкурентним інгібітором якого ферменту є аллопуринол. A *Ксантиноксидази B Аденозиндезамінази C Аденінфосфорибозилтрансферази D Гіпоксантинфосфорибозилтрансферази E Гуаніндезамінази

121 В організмі людини основним місцем депонування триацилгліцеролів (ТАГ) є жирова

тканина. Разом з тим їх синтез відбувається в гепатоцитах. У вигляді чого проходить транспорт ТАГ із печінки в жирову тканину? A *ЛПДНЩ B Хіломікронів C ЛПНЩ D ЛПВЩ E Комплексу з альбуміном

122 Вторинним посередником в механізмі дії адреналіну є: A *цАМФ B цГМФ C УМФ D ТМФ E ЦМФ

123 При різноманітних захворюваннях рівень активних форм кисню різко зростає, що призводить до руйнування клітинних мембран. Для запобігання цьому використовують антиоксиданти. Найпотужнішим природнім антиоксидантом є: A *Альфа-токоферол B Глюкола C Вітамін Д D Жирні кислоти E Гліцерол

124 У новонародженої дитини з’явились симптоми гемморагічної хвороби в зв’язку з гіповітамінозом К. Розвиток захворювання обумовлений особливою біологічною роллю вітаміну К, який: A *Є кофактором гама-глутамат--карбоксилази B Є кофактором протромбіну C Є специфічним інгібітором антитромбінів D Впливає на протеолітичну активність тромбіну E Інгібує синтез гепарину

125 Молекулярний аналіз гемоглобіну пацієнта, що страждає на анемію, виявив заміну 6Глу на 6Вал бета-ланцюга. Який молекулярний механізм патології? A *Генна мутація B Хромосомна мутація C Геномна мутація D Ампліфікація генів E Трансдукція генів

126

Page 31: крок 1 база 2014р_11шр-240ст (для брошури)_35грн

http://vk.com/my.printing

31

У хворої 38 років ревматизм в активній фазі. Визначення якого з наступних лабораторних показників сироватки крові має діагностичне значення при даній патології? A *С-реактивного білка B Сечової кислоти C Сечовини D Креатиніну E Трансферину

127 В організмі людини хімотрипсин секретується підшлунковою залозою і в порожнині кишечника піддається обмеженому протеолізу з перетворенням в активний хімотрипсин під дією: A *Трипсину B Ентерокінази C Пепсину D Амінопептидази E Карбоксипептидази

128 У хворого виявлена болючість по ходу крупних нервових стволів та підвищенний вміст пірувата в крові. Недостатність якого вітаміну може викликати такі зміни? A *В1 B В2 C РР D Пантотенова кислота E Біотин

129 У добовому раціоні дорослої здорової людини повинні бути жири, білки, вуглеводи, вітаміни, мінеральні солі та вода. Вкажіть кількість білку, яка забезпечує нормальну життєдіяльність організму. A *100 – 120. B 50 – 60. C 10 – 20. D 70 –80. E 40 – 50.

130 У відділення травматології надійшов хворий з розтрощенням м’язової тканини. Вкажіть, який біохімічний показник сечі при цьому буде збільшений: A *Креатинін B Загальні ліпіди

C Глюкоза D Мінеральні солі E Сечова кислота

131 З метою аналгезії можуть бути використані речовини, що імітують ефекти морфіну, але виробляються в ЦНС. Вкажіть їх. A *Бета-Ендорфін. B Окситоцин. C Вазопресин. D Кальцитонін. E Соматоліберин.

132 У хворого через 12 годин після гострого приступу загрудинного болю знайдено різке підвищення активності АсАТ в сироватці крові. Вкажіть патологію, для якої характерне це зміщення. A *Інфаркт міокарду. B Вірусний гепатит. C Колагеноз. D Цукровий діабет. E Нецукровий діабет.

133 Хвора 58 років. Стан важкий, свідомість затьмарена , шкіра суха , очі запалі, ціаноз, запах гнилих яблук з рота. Результати аналізів: глюкоза крові 15,1 ммоль/л, в сечі 3,5 \% глюкози. Причиною такого стану є: A * Гіперглікемічна кома B Гіпоглікемічна кома C Анафілактичний шок D Уремічна кома E Гіповалемічна кома

134 Хворий 13 років. Скаржиться на загальну слабість, запаморочення, втомлюваність. Спостерігається відставання у розумовому розвитку. При обстеженні виявлено високу концентрацію валіну, ізолейцину, лейцину в крові та сечі. Сеча специфічного запаху. Що може бути причиною такого стану: A * Хвороба кленового сиропу B Хвороба Аддісона C Тирозиноз D Гістидинемія E Базедова хвороба

135 Жінка 30 років хворіє близько року, коли вперше з’явились болі в ділянці суглобів, їх

Page 32: крок 1 база 2014р_11шр-240ст (для брошури)_35грн

32

припухлість, почервоніння шкіри над ними. Попередній діагноз ревматоїдний артрит. Однією з вірогідних причин цього захворювання є зміна в структурі білка сполучної тканини: A * Колагена B Муцина C Міозина D Овоальбуміна E Тропоніна

136 При обстеженні хворого виявлено підвищено вмісту в сироватці крові ліпопротеінів низької щільності. Яке захворювання можна передбачити у цього хворого ? A *атеросклероз; B ураження нирок C гострий панкреатит D гастрит E запалення легень

137 У хворого 27-ми років виявлено патологічні зміни печінки і головного мозку. У плазмі крові виявлено різке зниження, а в сечі підвищення вмісту міді. Поставлено діагноз - хвороба Вільсона. Активність якого ферменту в сироватці крові необхідно дослідити для підтвердження діагнозу? A *Церулоплазміну B Карбоангідрази C Ксантиноксидази D Лейцинамінопептидази E Алкогольдегідрогенази

138 Пацієнт звернувся до лікаря зі скаргами на задишку, що виникала після фізичного навантаження. Клінічне обстеження виявило анемію та наявність пара протеїну в зоні гамма-глобулінів. Який показник у сечі необхідно визначити для підтвердження діагнозу мієломи? A *Білок Бенс-Джонса B Білірубін C Гемоглобін D Церулоплазмін E Антитрипсин

139 У пацієнта, що проживає на специфічній

геохімічній території, поставлено діагноз ендемічний зоб. Який вид посттрансляційної модифікації тиреоглобуліну порушений в організмі хворого? A *Йодування B Метилування C Ацетилування D Фосфорилювання E Глікозилювання

140 У хворого встановлено підвищення у плазмі крові вмісту кон'югованого (прямого) білірубіну при одночасному підвищенні некон'югованого (непрямого) і різкому зниженні в калі і сечі вмісту стеркобіліногену. Про який вид жовтяниці можна стверджувати ? A *Обтураційну B Паренхіматозну (печінкову) C Гемолітичну D Жовяницю немовлят E Хворобу Жильбера

141 Більша частина учасників експедиції Магелана в Америку загинула від захворювання авітамінозу, що проявлялось загальною слабкістю, підшкірних крововиливах, випадінням зубів, кровотечею з ясен. Вкажіть назву цього авітамінозу. A * Скорбут (цинга) B Пелагра C Рахіт D Поліневрит (бері-бері) E Анемія Бірмера

142 Під час патронажу лікар виявив у дитини симетричну щерехатість щік, діарею,порушення нервової діяльності. Нестача яких харчових факторів є причиною такого стану? A *Нікотинова кислота, триптофан. B Лізин, аскорбінова кислота. C Треонін, пантотенова кислота. D Метіонін, ліпоєва кислота. E Фенілаланін, пангамова кислота..

143 Жінка 62 років скаржиться на часту біль в області грудної кроківки і хребта, переломи ребер Лікар припустив мієломну хворобу (плазмоцитому) Який з перерахованих нижче

Page 33: крок 1 база 2014р_11шр-240ст (для брошури)_35грн

http://vk.com/my.printing

33

лабораторних показників буде мати найбільш діагностичне значення? A *Парапротеїнемі B Гіперальбумінемія C Протеїнурия D Гіпоглобулінемія E Гіпопротеїнемія

144 У новонародженаго спостерігались судорми, які проходили після призначення вітаміну B6 Цей ефект найбільш ймовірно викликаний тим, що вітамін B6 входить до складу фермента: A *Глутаматдекарбоксилази B Піруватдегідростази C Нетоглубаратдегідромин D Амінолевулінатсинтази E Глікогенфосфорилази

145 У юнака 18 років з ураженням паренхіми печінки в сироватці крові найвірогідніше буде виявлено підвищений рівень A *Аланінамінотрансферази B Лактатдегідрогенази-1 C Креатинкінази D Кислої фосфатази E aльфа-амілази

146 Після обстеження хворому на сечокам'ну хворобу призначили алопурінол – конкурентний інгібітор ксантиноксидази. Підставою для цього був хімічний аналіз ниркових каменів, які складалися переважно з: A *Урату натрію B Дигідрату оксалату кальцію C Моногідрату оксалату кальцію D Фосфату кальцію E Сульфатикальцію

147 В сироватці крові хворого знайдено високу активність ізоферменту ЛДГ1. Патологічний процес в якому органі має місце? A *Серці B Печінці C Скелетних м'язах D Підшлунковій залозі E Нирках

148 [факт] Мікроелемент мідь є складовим компонентом білків (металопротеїнів). При порушенні обміну міді виникає хвороба Вільсона (гепатоцеребральна дистрофія). Концентрація якого білка зменшується в крові? A *Церулоплазміну. B Трансферину. C Феритину D Колагену. E Глобуліну.

149 Хворий 50-ти років звернувся до клініки зі скаргами на загальну слабість, втрату апетиту, аритмію. Спостерігається гіпотонія м'язів, мляві паралічі, послаблення перистальтики кишечника. Причиною такого стану може бути: A *Гіпокаліємія B Гіпопротеінемія C Гіперкаліємія D Гіпофосфатемія E Гіпонатріємія

150 У крові хворого виявлено підвищення активності ЛДГ1,2, АсАТ, креатинкінази. В якому органі найбільш ймовірний розвиток патологічног процесу? A *Серце B Підшлункова залоза C Печінка D Нирки E Скелетні м”язи

151 Споживання забруднених овочів і фруктів протягом тривалого часу призвело до отруєння пацієнта нітратами і утворення в крові похідног гемоглобіну… A *Hb-OH B Hb СО C Hb O2 D Hb CN E Hb NHCOOH

152 При глікогенозі – хворобі Гірке – порушується перетворення глюкозо-6-фосфату на глюкозу, що приводить до накопичення глікогену в печінці та нирках. Дефіцит якого

Page 34: крок 1 база 2014р_11шр-240ст (для брошури)_35грн

34

ферменту є причиною захворювання? A * Глюкозо-6-фосфатази B Глікогенсинтетази C Фосфорілази D Гексокінази E Альдолази

153 Підвищенну стійкість “моржів” до холодної води пояснюють тим, що у них синтезується у великих кількостях гормони що підсилюють процеси окислення і утворення тепла в мітохондріях шляхом роз’єднання. Які це гормони (гормон)? A * Йодвмісткі гормони щитовидної залози (йодтироніни) B Адреналін та норадреналін C Глюкагон D Інсулін E Кортикостероїди

154 В дитячу лікарню поступила дитина з ознаками рахіту (деформація кісток, пізне заростання тім’я та ін.) При біохімічному аналізі крові відмічені такі зміни: A *Зниження рівня Са++ B Зниження рівня К+ C Підвищення рівня фосфатів D Зниження рівня Mg+ E Підвищення рівня Na+

155 Робітник цеху по виробництву нітросполук звернувся до лікаря зі скаргами на задишку та швидку утомлюванність. При обстеженні хворого виявлено цианоз нижніх кінцівок. Яка причина цього стану? A *Посилене метгемоглобіноутворення. B Гіповітаміноз. C Гіпервітаміноз. D Жирова інфільтрація печінки. E Авітаміноз.

156 Після курсу терапії хворому на виразку дванадцятипалої кишки лікар пропонує вживання соків із капусти та картоплі. Вміст яких речовин в цих харчах сприяє профілактиці та заживленню виразок? A *Вітамін U. B Пантотенова кислота. C Вітамін С.

D Вітамін В1. E Вітамін К.

157 Кал хворого вміщує багато нерозщепленого жиру і має сірувато-білий колір. Укажіть причину цього явища. A *Обтурація жовчного протоку. B Недостатня активація пепсину соляною кислотою. C Гіповітаміноз. D Ентерит. E Подразнення епітелія кишечнику.

158 У 12-річного хлопчика в сечі виявлено високий вміст усіх амінокислот аліфатичного ряду. При цьому відмічена найбільш висока екскреція цистіну та цистеїну. Крім того, УЗД нирок показало наявність каменів у них. Виберіть можливу патологію. A *Цистинурія. B Алкаптонурія. C Цистіт. D Фенілкетонурія. E Хвороба Хартнупа.

159 Пацієнт відмічає часті проноси, особливо після вживання жирної їжі, схуднення. Лабораторні дослідження показали наявність стеатореї, кал гіпохолічний. Можлива причина такого стану: A * Обтурація жовчних шляхів B Запалення слизової тонкого кишечника C Нестача ліпази D Порушення активності фосфоліпаз E Незбалансована дієта

160 Дитина квола, апатична. Печінка збільшена і при біопсії печінки виявлено значний надлишок глікогену. Концентрація глюкози в крові нижче норми. У чому причина пониженої концентрації глюкози в крові цієї хворої? A * Понижена (відсутня) активність глікоген-фосфорилази в печінці. B Понижена (відсутня) активність гексокінази. C Підвищена активність глікогенсинтетази. D Понижена (відсутня) активність глюкозо-6-фосфатази. E Дефіцит гену, який відповідає за синтез глюкозо-1-фосфатуридинтрансферази.

Page 35: крок 1 база 2014р_11шр-240ст (для брошури)_35грн

http://vk.com/my.printing

35

161 У хворого 30-ти років із гострим запаленням підшлункової залози (панкреатитом) виявлено порушення порожнинного травлення білків. Це може бути пов’язано із недостатнім синтезом та виділенням залозою: A * Трипсину B Пепсину C Ліпаза D Дипептидаз E Амілази

162 У хворої жінки з низьким артеріальним тиском після парентерального введення гормону відбулось підвищення артеріального тиску і також підвищився рівень глюкози та ліпідів у крові. Який гормон було введено? A *Адреналін B Глюкагон C Інсулін D Прогестерон E Фолікулін

163 Жінка 43 років, робітниця лакофарбового підприємства, скаржиться на загальну слабкість, зниження ваги, апатію, сонливість. Хронічна свинцева інтоксикація підтверджена лабораторно - виявлена гіпохромна анемія. В крові підвищений рівень Zn-протопорфірину і знижений рівень альфа-амінолевулинової кислоти, що свідчить про порушення синтезу: A *Гему B ДНК C РНК D Білка E Мевалонової кислоти

164 Судово-медичний експерт при розтині трупу 20-річної дівчини встановив, що смерть настала внаслідок отруєння ціанідами. Порушення якого процесу найбільш вірогідно було причиною смерті дівчини? A *Тканинного дихання B Синтезу гемоглобіну C Транспорту кисню гемоглобіном

D Синтезу сечовини E Транспорту водню за допомогою малат-аспартатного механізму

165 В лікарню швидкої допомоги доставили дитину 7 років в стані алергічного шоку, який розвинувся після укусу оси. В крові підвищена концентрація гістаміну. В результаті якої реакції утворюється цей амін? A *Декарбоксилювання B Гідрооксилювання C Дегідрування D Дезамінування E Відновлення

166 Чоловік 65 років, який страждає на подагру, скаржиться на болі в області нирок. При ультразвуковому обстеженні встановлена наявність ниркових каменів. Підвищення концентрації якої речовини є найбільш імовірною причиною утворення каменів в даному випадку? A *Сечової кислоти B Холестерину C Білірубіну D Сечовини E Цистину

167 У дівчинки 7 років явні ознаки анемії. Лабораторно встановлений дефіцит піруваткінази в еритроцитах. Порушення якого процесу грає головну роль в розвитку анемії у дівчинки? A *Анаеробного гліколізу B Окислювального фосфорилювання C Тканинного дихання D Розкладу пероксидів E Дезамінування амінокислот

168 Для підвищення результатів спортсмену рекомендували застосовувати препарат, який містить у собі карнітин. Який процес в найбільшій ступені активується карнітином? A *Транспорт жирних кислот в мітохондрії B Синтез кетонових тіл C Синтез ліпідів D Тканинне дихання E Синтез стероїдних гормонів

Page 36: крок 1 база 2014р_11шр-240ст (для брошури)_35грн

36

169 При ненадходженні чи недостатньому утворенні в організмі людини ліпотропних факторів у неї розвивається жирове переродження печінки. Яку з наведених речовин можна віднести до ліпотропних ? A *Холін B Холестерин C Триацилгліцериди D Жирні кислоти E Рибофлавін

170 При осмотре больного врач заподозрил синдром Иценко-Кушинга. Определение какого вещества в крови больного подтвердит предположение врача? A *Кортизола B Токоферола C Ретинола D Адреналина E Холестерина

171 У больного цингой выявлено нарушение гидроксилирования пролина и лизина в составе коллагена. Торможение какого биохимического процесса приводит к этому нарушению? A *Микросомального окисления B Перекисного окисления липидов C Тканевого дыхания D Пероксидазного окисления E Окислительного фосфорилирования

172 Основна маса азоту з організму виводиться у вигляді сечовини. Зниження активності якого ферменту в печінці приводить до гальмування синтезу сечовини і нагромадження аміаку в крові і тканинах? A *Карбамоїлфосфатсинтази B Аспартатамінотрансферази C Уреази D Амілази E Пепсину

173 У дитини 6 місяців спостерігається різке відставання в психомоторному розвитку, бліда шкіра з екзематозними змінами, біляве волосся, блакитні очі, напади судом. Який із наступних лабораторних аналізів крові і

сечі найвірогідніше дозволить встановити діагноз? A *Визначення концентрації фенілпірувату B Визначення концентрації триптофану C Визначення концентрації гістидину D Визначення концентрації лейцину E Визначення концентрації валінду

174 Жінка 40 років звернулась до лікаря зі скаргами на болі в дрібних суглобах ніг і рук. Суглоби збільшені, мають вигляд потовщених вузлів. У сироватці крові виявлено підвищений вміст уратів. Причиною є порушення обміну: A *Пуринів B Амінокислот C Вуглеводів D Ліпідів E Піримідинів

175 У хворого 35 років, який часто вживає алкоголь, на фоні лікування сечогінними засобами виникла сильна м'язова і серцева слабість, блювота, діарея, АТ - 100/60 мм рт.ст., депресія. Причиною такого стану є посилене виділення з сечею: A *Калію B Натрію C Хлору D Кальцію E Фосфатів

176 Хворий після прийому жирної їжі відчуває нудоту, млявість, з часом з'явилися ознаки стеатореї. В крові холестерин - 9,2 ммоль/л. Причиною такого стану є нестача: A *Жовчних кислот B Тригліцеридів C Жирних кислот D Фосфоліпідів E Хіломікронів

177 Дитина 10-ти місячного віку, батьки якої брюнети, має світле волосся, дуже світлу шкіру і блакитні очі. Зовнішньо при народженні виглядала нормально, але протягом останніх 3 місяців спостерігалися порушення мозкового кровообігу, відставання у розумовому розвитку. Причиною такого стану може

Page 37: крок 1 база 2014р_11шр-240ст (для брошури)_35грн

http://vk.com/my.printing

37

бути: A *Фенілкетонурія B Галактоземія C Глікогеноз D Гостра порфірія E Гістидинемія

178 У дитини грудного віку спостерігається забарвлення склер, слизових оболонок. Виділяється сеча, яка темніє на повітрі. В крові та сечі виявлено гомогентизинову кислоту. Що може бути причиною даного стану? A *Алкаптонурія B Альбінізм C Галактоземія D Цистинурія E Гістидинемія

179 У людей, які тривалий час перебували у стані гіподинамії, після фізичного навантаження виникають інтенсивні болі в м'язах. Яка найбільш вірогідна причина цього? A *Накопичення в м'язах молочної кислоти B Посилений розпад м'язових білків C Накопичення креатиніну в м'язах D Зменшення вмісту ліпідів в м'язах E Підвищення вмісту АДФ в м'язах

180 Лікар, перш ніж призначити виснаженому хворому білкове парентеральне харчування, призначив в лабораторії визначити електрофоретичний спектр білків крові. На яких фізико-хімічних властивостях білків оснований цей метод? A *Наявність заряду B В'язкість C Нездатність до денатурації D Гідрофільність і здатність до набрякання E Оптична активність

181 У чоловіка, який тривалий час не вживав з їжею жирів, але отримував достатню кількість вуглеводів і білків, виявлено дерматит, погане загоювання ран, погіршення зору. Яка можлива причина порушення обміну речовин? A *Нестача лінолевої кислоти, вітамінів А, Д, Е, К

B Нестача пальмітинової кислоти C Нестача вітамінів РР, Н D Низька калорійність дієти E Нестача олеїнової кислоти

182 Жінка 33 років страждає на гепатоцеребральну дистрофію (хвороба Вільсона). В крові знижений вміст церулоплазміну. В сечі різко підвищений вміст амінокислот. Ці зміни в першу чергу обумовлені посиленням процесу: A *Комплексоутворення амінокислот з міддю B Синтезу сечовини C Переамінування амінокислот D Разпаду тканинних білків E Глюконеогенезу

183 У чоловіка 32 років діагностована гостра променева хвороба. Лабораторно встановлено різке зниження рівня серотоніну в тромбоцитах. Найбільш вірогідною причиною зниження тромбоцитарного серотоніну є порушення процесу декарбоксилювання: A *5-окситриптофану B Серину C Тирозину D Піровиноградної кислоти E Гістидину

184 У хворого з жовтяницею встановлено: підвищення у плазмі крові вмісту загального білірубіну за рахунок непрямого (вільного), в калі і сечі – високий вміст стеркобіліну, рівень прямого (зв'язаного) білірубіну в плазмі крові в межах норми. Про який вид жовтяниці можна стверджувати? A *Гемолітична B Паренхіматозна (печінкова) C Механічна D Жовтяниця немовлят E Хвороба Жильбера

185 Повар в результате неосмотрительности обжег руку паром. Повышение концентрации какого вещества вызвало покраснение, отечность и болезненность пораженного участка кожи?

Page 38: крок 1 база 2014р_11шр-240ст (для брошури)_35грн

38

A *Гистамина B Тиамина C Глутамина D Лизина E Галактозамина

186 Экспериментальному животному давали избыточное количество глюкозы, меченой по углероду, в течение недели. В каком соединении можно обнаружить метку? A *Пальмитиновой кислоте B Метионине C Витамине А D Холине E Арахидоновой кислоте

187 У ребенка 3 лет после перенесенной тяжелой вирусной инфекции отмечается повторяющаяся рвота, потеря сознания, судороги. При исследовании крови больного выявлена гипераммониемия. С чем может быть связано изменение биохимических показателей крови у данного ребенка? A *С нарушением обезвреживания аммиака в орнитиновом цикле B С активацией процессов декарбоксилирования аминокислот C С нарушением обезвреживания биогенных аминов D С усилением гниения белков в кишечнике E С угнетением активности ферментов трансаминирования

188 У больного с частыми кровотечениями из внутренних органов и слизистых оболочек выявлены пролин и лизин в составе коллагеновых волокон. Из-за отсутствия какого витамина нарушено их гидроксилирование? A *Витамина С B Витамина К C Витамина А D Тиамина E Витамина Е

189 К косметологу обратился пациент с просьбой избавить его от татуировки на плече. Какое вещество, содержащееся в соединительной ткани, ограничивает распространение красителя и делает возможным такой вид “живописи”?

A *Гиалуроновая кислота B Гамма-глобулин C Фибронектин D Гепарин E Эластин

190 Хворий 23 років скаржиться на головний біль, зміну зовнішнього вигляду (збільшення розмірів ніг, кистей, рис обличчя), огрубіння голосу, погіршення пам'яті. Захворювання почалося приблизно 3 роки тому без видимих причин. При огляді - збільшення надбрівних дуг, носа, язика. Аналіз сечі без особливих змін. Причиною такого стану може бути: A *Гіперпродукція соматотропіну B Нестача глюкагону C Нестача тироксину D Нестача альдостерону E Гіперпродукція кортикостероїдів

191 Хвора 36-ти років страждає на колагеноз. Збільшення вмісту якого метаболіту найбільш вірогідно буде встановлено у сечі? A *Оксипроліну B Iндикану C Креатиніну D Сечовини E Уробіліногену

192 Альбіноси погано переносять вплив сонця - загар не розвивається, а з'являються опіки. Порушення метаболізму якої амінокислоти лежить в основі цього явища? A *Фенілаланіну B Метіоніну C Триптофану D Глутамінової E Гістидину

193 Хворі на алкоголізм отримують основну масу калорій із спиртними напоями. У них може виникнути характерна недостатність тіаміну (синдром Верніке-Корсакова), при якій спостерігаються порушення функцій нервової системи, психози, втрата пам'яті. Зі зниженням активності якого ферменту пов'язаний цей процес? A *Піруватдегідрогеназа B Алкогольдегідрогеназа C Трансаміназа D Альдолаза

Page 39: крок 1 база 2014р_11шр-240ст (для брошури)_35грн

http://vk.com/my.printing

39

E Гексокіназа

194 У жінки 45 років відсутні симптоми діабету, але визначається натщесерце підвищений вміст глюкози в крові (7,5 ммоль/л). Який наступний тест необхідно провести? A *Визначення толерантності до глюкози B Визначення ацетонових тіл в сечі C Визначення залишкового азоту в крові D Визначення глюкози крові натщесерце E Визначення гліколізованого гемоглобіну

195 У крові пацієнта вміст глюкози натщесерце був 5,65 ммоль/л, через 1 годину після цукрового навантаження становив 8,55 ммоль/л, а через 2 години - 4,95 ммоль/л. Такі показники характерні для: A *Здорової людини B Хворого з прихованим цукровим діабетом C Хворого з інсулінозалежним цукровим діабетом D Хворого з інсулінонезалежним цукровим діабетом E Хворого з тиреотоксикозом

196 Лікування дитини, хворої на рахіт, за допомогою вітаміну $D_3$ не дало позитивного результату. Яка найбільш імовірна причина неефективності лікування? A *Порушення гідроксилювання вітаміну $D_3$ B Недостатність ліпідів у їжі C Порушення включення вітаміну $D_3$ до ферменту D Підвищене використання вітаміну $D_3$ мікрофлорою кишечника E Порушення транспорту вітаміну $D_3$ білками крові

197 Хворому з прогресуючою м'язовою дистрофією було проведено біохімічне дослідження сечі. Поява якої речовини у великій кількості в сечі може підтвердити захворювання м'язів у даного хворого? A *Креатину B Порфиринів C Сечовини D Гіпурової кислоти E Креатиніну

198 Хворому на подагру лікар призначив алопуринол, що привело до зниження концентрації сечової кислоти. Яка властивість алопуринолу забезпечує терапевтичний ефект у даному випадку? A *Конкурентне інгібування ксантиноксидази B Збільшення швидкості виведення азотвмісних речовин C Прискорення катаболізму піримідинових нуклеотидів D Уповільнення реутилізації піримідинових нуклеотидів E Прискорення синтезу нуклеїнових кислот

199 Пацієнт звернувся до клініки зі скаргами на загальну слабість, ниючі болі в животі, поганий апетит, з підозрою на жовтяницю. У сироватці крові знайдено 77,3 мкмоль/л загального білірубіну і 70,76 мкмоль/л кон'югованого білірубіну. Який найбільш імовірний вид жовтяниці? A *Механічна жовтяниця B Гострий гепатит C Цироз печінки D Обтураційна жовтяниця E Гемолітична жовтяниця

200 У пацієнта цироз печінки. Дослідження якої з перелічених речовин, що екскретуються з сечею, може характеризувати стан антитоксичної функції печінки? A *Гіпурової кислоти B Амонійних солей C Креатиніну D Сечової кислоти E Амінокислот

201 При жировій інфільтрації печінки порушується синтез фосфоліпідів. Вкажіть, яка з перелічених речовин може посилювати процеси метилювання в синтезі фосфоліпідів? A *Метіонін B Аскорбінова кислота C Глюкоза D Гліцерин E Цитрат

Page 40: крок 1 база 2014р_11шр-240ст (для брошури)_35грн

40

202 Після лікування хворого антибіотиками внаслідок подавлення мікрофлори кишечника можливий гіповітаміноз вітамінів: A *В12 B С C А D Р E Д

203 Для лікування злоякісних пухлин призначають метотрексат-структурний аналог Фолієвої кислоти, який є конкурентним інгібітором дігідрофолатредуктази і тому подавляє синтез: A *Нуклеотидів ДНК B Моносахаридів C Жирних кислот D Гліцерофосфатидів E Глікогену

204 Для утворення транспортної форми амінокислот для синтезу білка необхідно: A *Аміноацил-тРНК синтетаза B ГТФ C мРНК D Рибосома E Ревертаза

205 Для утворення транспортної форми амінокислот для синтезу білка на рибосомах необхідно: A *тРНК B Ревертаза C ГТФ D мРНК E Рибосома

206 При отруєнні аманітином – отрутою блідої поганки блокується РНК-полімераза В(ІІ). При цьому припиняється: A *Синтез мРНК B Синтез тРНК C Зворотня транскрипція D Синтез праймерів E Дозрівання мРНК

207 У хворого , виснаженого голодуванням, в печінці та нирках підсилюється процес: A *Глюконеогенезу

B Синтезу сечовини C Синтезу білірубіна D Утворення гіпурової кислоти E Синтезу сечової кислоти

208 У крові хворих на цукровий діабет спостерігається підвищення вмісту вільних жирних кислот (НЕЖК). Причиною цього може бути: A *Підвищення активності тригліцеридліпази адипоцитів B Накопичення в цитозолі пальмітоїл-КоА C Активація утилізації кетонових тіл D Активація синтезу аполіпопротеїнів А-1, А-2, А-4. E Зниження активності фосфатидилхолін-холестеин-ацилтрансферази плазми крові

209 Посилення пероксидного окиснення ліпідів та біополімерів є одним із основних механізмів пошкодження структури та функції клітинних мембран і загибелі клітини. Причиною цього є: A *Посилене утворення вільних радикалів кисню та пригнічення антиоксидантних систем B Гіповітаміноз В1 C Гіпервітаміноз В1 D Гіповітаміноз В12 E Гіпервітаміноз В12

210 Знешкодження ксенобіотиків (лікарських засобів, епоксидів, ареноксидів, альдегідів, нітропохідних тощо) та ендогенних метаболітів (естрадіолу, простагландинів, лейкотрієнів) проходить в печінці шляхом їх кон’югації з: A *Глутатіоном B Аспарагіновою кислотою C Гліцином D S-Аденозилметіоніном E Фосфоаденозином

211 Вітамін А у комплексі зі специфічними циторецепторами проникає через ядерні мембрани, індукує процеси транскрипції, що стимулює ріст та диференціювання клітин. Ця біологічна функція реалізується наступною формою вітаміну А: A *Транс-ретиноєва кислота B Транс-ретиналь C Цис-ретиналь

Page 41: крок 1 база 2014р_11шр-240ст (для брошури)_35грн

http://vk.com/my.printing

41

D Ретинол E Каротин

212 Тривале вживання великих доз аспірину викликає пригнічення синтезу простагландинів в результаті зниження активності фермента: A *Циклооксигенази B Пероксидази C 5-ліпоксигенази D Фосфоліпази А2 E Фосфодіестерази

213 Єритроцит для своєї життедіяльності потребує енергію у вигляді АТФ. Який процес забезпечує цю клітину необхідною кількістю АТФ? A *Анаєробний гліколіз B Аєробне окислення глюкози C Пентозний цикл D Бета-окислення жирних кислот E Цикл трикарбонових кислот

214 У хворого на гострий панкреатит при аналізі крові та сечі різко підвищена активність одного з вказаних ферментів, що підтверджує діагноз захворювання: A *Альфа-амілаза B Пепсин C Дипептидаза D Сахароза E Лактаза

215 При недостатності тіаміну - вітаміну В1 виникає хвороба бері-бері (поліневрит) та порушується вуглеводний обмін. Який метаболіт при цьому накопичується в крові? A *Піруват B Лактат C Сукцинат D Цитрат E Малат

216 При декарбоксилюванні амінокислоти гістидину утворюється надзвичайно активний амін-медіатор запалення та алергії, а саме: A *Гістамін B Серотонін C Дофамін

D (-аміномасляна кислота E Триптамін

217 Злоякісна гіперхромна анемія – хвороба Бірмера – виникає внаслідок нестачі вітаміну В12. Який біоелемент входить до складу цього вітаміну? A *Кобальт B Молібден C Цинк D Залізо E Магній

218 Виродженість генетичного коду – здатність декількох триплетів кодувати 1 амінокислоту. А яка аімнокислота кодується 1 триплетом? A *Метионін B Серин C Аланін D Лейцин E Лізин

219 Продуктами гідролізу та модифікації деяких білків є біологічно активні речовини-гормони. Вкажіть, з якого із приведених білків в гіпофізі утворюються ліпотропін, кортикотропін, меланотропін та ендорфіни? A * Проопіомеланокортин (ПОМК) B Нейроальбумін C Нейростромін D Нейроглобулін E Тиреоглобулін

220 При хворобі Іценко-Кушинга (гіперфункція кори наднирників з підвищеною продукцією кортикостероїдів) виникає гіперглікемія. Який процес при цьому стимулюється? A * Глюконеогенез B Фосфороліз глікогену C Цикл Кребса D Пентозофосфатний шлях окислення глюкози E Гліколіз

221 Використання гюкози відбувається шляхом її транспорту з екстрацелюлярного простору через плазматичну мембрану в середину клітини. Цей процес стимулюється

Page 42: крок 1 база 2014р_11шр-240ст (для брошури)_35грн

42

гормоном: A *Інсуліном B Глюкагоном C Тироксином D Альдостероном E Адреналіном

222 Назвіть фермент, визначення якого в крові є найбільш інформативним в перші години після виникнення інфаркту міокарда: A *Креатинфосфокіназа B Аспартатамінотрансфераза C Аланінамінотрансфераза D Лактатдегідрогеназа E Глутаматдегідрогеназа

223 В слині міститься фермент, який володіє сильною бактерицидною дією завдяки здатності руйнувати пептидоглікани бактеріальної стінки. Вкажіть на цей фермент: A *Лізоцим (мурамідаза) B α-амілаза C Трипсин D Фосфатаза E Рибонуклеаза

224 В процесі лікування парадонтозу застосовують антиоксидант природного та штучного походження. Вкажіть, яка з наведених природних сполук використовується в якості антиоксидантного засобу? A *Токоферол B Тіамін C Глюконат D Піридоксин E Холін

225 При парадонтозі відбувається деструкція білкових та полісахаридних компонентів сполучної тканини. Який з наведених білків входить до складу сполучної тканини: A *Колаген B Альбумін C Трансферин D Церулоплазмін E Антитрипсин

226 В якості антикоагулянтів використовують різноманітні речовини, в тому числі полісахарид природного походження, а саме:

A *Гепарин B Гіалуронова кислота C Дерматансульфат D Хондроітинсульфат E Декстран

227 У хворих з непрохідністью жовчевивідних шляхів пригнічується зсідання крові, виникають кровотечі, що є наслідком недостатнього засвоєння вітаміну: A *К B А C D D Е E С

228 Генний апарат людини містить біля 30 тисяч генів, а кількість варіантів антитіл сягає мільйонів. Який механізм використовується для утворення нових генів, що відповідають за синтез такої кількості антитіл? A *Рекомбінація генів B Ампліфікація генів C Реплікація ДНК D Репарація ДНК E Утворення фрагментів Оказакі

229 Одна з форм вродженої патології супроводжується гальмуванням перетворення фенілаланіну в тирозин. Біохімічною ознакою хвороби є накопичення в організмі деяких органічних кислот, у тому числі кислоти: A *Фенілпіровіноградної B Лимонної C Піровиноградної D Молочної E Глутамінової

230 Анаєробне розщеплення глюкози до молочної кислоти регулюється відповідними ферментами. Вкажіть, який фермент є головним регулятором цього процесу? A *Фосфофруктокіназа B Глюкозл-6-фосфат ізомераза C Альдолаза D Енолаза E Лактатдегідрогеназа

231 З метою ранньої діагностики вагітності

Page 43: крок 1 база 2014р_11шр-240ст (для брошури)_35грн

http://vk.com/my.printing

43

досліджується сеча жінки. З'явлення яких гормонів в сечі вірогідно свідчить про вагітність ? A *Хоріонічний гонадотропін. B Естріол. C 17-бета-естрадіол. D Тестостерон. E Прогестерон.

232 При тіреотоксикозі підвищується продукція тіреоїдних гормонів Т3 та Т4, розвивається похудіння, тахікардія, психічна збудженість та інше. Як саме впливають тіреоїдні гормони на енергетичний обмін в мітохондріях клітин? A *Роз`єднують окислення та окисне фосфорилювання B Активують субстратне фосфорилювання C Блокують субстратне фосфорилювання D Блокують дихальний ланцюг E Активують окисне фосфорилювання.

233 При отруєнні цианідами настає миттєва смерть. В чому полягає механізм дії цианідів на молекулярному рівні? A *Інгібують цитохромоксидазу B Зв,язують субстрати ЦТК C Блокують сукцинатдегідрогеназу D Інактивують кисень E Інгібують цитохром в.

234 В сечі хворого Б. виявлено цукор, кетонові тіла, вміст глюкози в крові становить 10,1 ммоль/л. Наявність якого захворювання Ви можете припустити у хворого Б? A *Цукровий діабет B Атеросклероз C Токсичний гепатит D Панкреатит E Інфаркт міокарду.

235 У хворого К. в сечі підвищена амілазна активність і виявлено наявність трипсину, в крові підвищена амілазна активність. Про патологію якого органу це свідчить? A *Підшлункової залози B Печінки C Шлунку D Нирок E Кишечника

236 У хворого виявлено зниження рН крові та вмісту бікарбонатних іонів (падіння лужного резерву крові), зростання вмісту молочної, піровиноградної кислот в крові та сечі. Який тип порушення кислотно-основної рівноваги спостерігається? A *Метаболічний ацидоз B Респіраторний ацидоз C Метаболічний алкалоз D Респіраторний алкалоз E Дихальний алкалоз

237 Карбомоїлфосфатсинтетаза каталізує реакцію утворення карбомоїлфосфату з вільного аміаку. Цей фермент постачає карбомоїлфосфат для синтезу: A *сечовини B пуринів C креатину D ліпідів E амінокислот

238 Для синтезу полісахаридних ланцюгів глікогену використовується попередник - активна форма глюкози. Безпосереднім донором залишків глюкози в процесі синтезу глікогену є: A *УДФ-глюкоза B глюкозо-1-фостат C АДФ-глюкоза D глюкозо – 6-фосфат E глюкозо –3 фосфат

239 Хворому 65 років з ознаками загального ожиріння, небезпекою жирової дистрофії печінки рекомендована дієта, збагачена ліпотропними речовинами, серед яких важливе значення має вміст у продуктах: A *Метіоніну B Холестерину C Глюкози D Вітаміну С E Гліцину

240 Біосинтез пуринового кільця відбувається на рибозо-5-фосфаті шляхом поступового нарощення атомів азоту і вуглецю та

Page 44: крок 1 база 2014р_11шр-240ст (для брошури)_35грн

44

замикання кілець. Джерелом рибозофосфату служить процес: A *пентозофосфатний цикл B гліколіз C гліконеогенез D глюконеогенез E глікогеноліз

241 У хворого відмічені такі зміни: і порушення зору в сутінках, підсихання кон'юнктиви та рогової оболонки. Такі порушення можуть бути при недостачі: A *Вітаміну А B Вітаміну В C Вітаміну С D Вітаміну D E Вітаміну В12

242 Після видалення 2/3 шлунка у крові зменшилась кількість еритроцитів, зріс їх об'єм, знизився рівень гемоглобіну. Дефіцит якого вітаміну приводить до таких змін картини крові? A *В12 B С C Р D В6 E РР

243 Арахідонова кислота як незамінимий компонент їжі є попередником біологічно активних речовин. Вкажіть які сполуки синтезуються з неї? A *Простагландин Е1 B Холін C Норадреналін D Етаноламін E Трийодтиронін

244 При якому гіповітамінозі спостерігається одночасне порушення репродуктивної функції і дистрофія скелетної мускулатури? A *Вітамін Е B Вітамін А C Вітамін К D Вітамін Д E Вітамін В1

245

У хворого збільшені і болючі суглоби, а у сироватці крові підвищений вміст уратів. Обмін яких речовин порушений? A *Пуринів B Піримідинів C Холестерину D Фенілаланіну E Гліцерину

246 Жирні кислоти, як висококалорійні сполуки зазнають перетворень у мітохондріях у результаті яких утворюється велика кількість енергії. Якими шляхами проходять ці процеси ? A *Бета - окиснення B Декарбоксилювання C Трансамінування D Дезамінування E Відновлення

247 При підвищенні концентрації чадного газу в повітрі може наступити отруєння. При цьому порушується транспортування гемоглобіном кисню від легень до тканин. Яке похідне гемоглобіну при цьому утворюється? A *Карбоксигемоглобін B Оксигемоглобін C Метгемоглобін D Карбгемоглобін E Гемохромоген

248 Відомо, що синовіальна рідина зменшує тертя суглобових поверхонь. При ревматизмі чи артриті її в’язкість знижується внаслідок деполімеризації (руйнування) такої речовини : A *Гіалуронової кислоти B Глікогену C Колагену D Гепарину E Альбуміну

249 У новонародженої дитини у шлунку відбувається “згурджування” молока, тобто перетворення розчинних білків молока казеїнів у нерозчинні – параказеїни за участю іонів кальцію і ферменту. Який фермент приймає участь у цьому процесі ? A *Ренін

Page 45: крок 1 база 2014р_11шр-240ст (для брошури)_35грн

http://vk.com/my.printing

45

B Пепсин C Гастрин D Секретин E Ліпаза

250 У хворого встановлено зниження синтезу вазопресину, що призводить до поліурії і , як наслідок, до вираженої дегідратації організму. Що з переліченого є найбільш ймовірним механізмом поліурії? A *Зниження канальцієвої реабсорбції води B Порушення канальцієвої реабсорбції іонів Nа C Зниження канальцієвої реабсорбції белка D Порушення реабсорбції глюкози E Підвищення гідростатичного тиску

251 При нанесенні стоматологом пероксиду водню на слизову оболонки порожнини роту з'явилась інтенсивна піна. Наявність якого ферменту розкладає пероксиду водню? A *Каталаза. B Холінестераза C Ацетілтрасфераза D Глюкозо-6-фосфатдегідрогеназа E Метгемоглобінредуктаза

252 Після ремонту автомобіля в гаражному приміщенні водій потрапив в лікарню з симптомами отруєння вихлопними газами. Концентрація якого гемоглобіну в крові буде підвищена? A *Карбоксигемоглобіну B Метгемоглобіну C Карбгемоглобіну D Оксигемоглобіну E Глюкозильованного гемоглобину

253 Еритроцити людини не містять мітохондрій. Який основний шлях утворення АТФ в цих клітинах? A *Анаеробний гліколіз B Аеробний гліколіз C Окиснювальне фосфорилювання D Креатинкіназна реакція E Аденілаткіназна реакція

254 В організмі людини є пептид, в утворенні якого бере участь гамма-карбоксильна група глутамінової кислоти. Цей пептид називається:

A *Глутатіоном B Карнозином C Ансерином D Окситоцином E Вазопресином

255 При інтенсивній роботі в м’язах утворюється значна кількість аміаку. Яка амінокислота відіграє основну роль в транспортуванні його в печінку та використовується в реакціях глюконеогенезу? A *Аланін B Аргінін C Лізин D Орнітин E Аспартат

256 Цианіди є надзвичайно потужними клітинними отрутами, які при надходженні в організм людини можуть спричинити смерть. Блокування якого ферменту тканинного дихання лежить в основі такої їх дії: A *Цитохромоксидази B Ферохелатази C Каталази D Гемоглобінредуктази E Глюкозо-6-фосфатдегідрогенази

257 Центральну роль в обміні амінокислот у нервовій тканині відіграє глутамінова кислота. Це пов’язано з тим, що дана амінокислота: A *Зв’язує аміак з утворенням глутаміну B Використовується для синтезу ліків C Використовується для синтезу глюкози D Використовується для синтезу нейроспецифічних білків E Використовується для синтезу ацетонових тіл

258 У хворого 37 років на фоні тривалого застосування антибіотиків спостерігається підвищена кровоточивість при невеликих пошкодженнях. У крові – зниження активності факторів згортання крові ІІ, VII, X, подовження часу згортання крові. Недостатністю якого вітаміну обумовлені зазначені зміни?

Page 46: крок 1 база 2014р_11шр-240ст (для брошури)_35грн

46

A *Вітамін К B Вітамін А C Вітамін С D Вітамін D E Вітамін Е

259 У хворого 43 років спостерігається хронічний атрофічний гастрит, мегалобластна злоякісна гіперхромна анемія. Підвищується виділення метилмалонової кислоти з сечею. Недостатністю якого вітаміну обумовлене виникнення зазначеного симптомокомплексу? A *Вітамін В12 B Вітамін В2 C Вітамін В3 D Вітамін В5 E Вітамін В1

260 Внаслідок тривалого голодування в організмі людини швидко зникають резерви вуглеводів. Який з процесів метаболізму поновлює вміст глюкози в крові? A * Глюконеогенез B Анаеробний гліколіз C Аеробний гліколіз D Глікогеноліз E Пентоофосфатний шлях

261 У хлопчика 9 років, що знаходиться на стаціонарному лікуванні виявлено ураження нирок та підвищенний артеріальний тиск. З підвищенням якого біологічно активного пептиду пов`язаний цей стан? A * Ангіотензину ІІ B Антидіуретичного гормону C Глюкагон D Калідину E Інсуліну

262 У хворого гострий панкреатит. Які препарати повинен призначити лікар, щоб уникнути аутолізу підшлункової залози? A * Інгибітори протеаз B Активатори протеаз C Трипсин D Хімотрипсин E Амілазу

263 Під час харчування новонародженої дитини молоком матері з`явилися блювання, метеорізм, пронос. Про спадкову недостатність якого ферменту слід думати? A * Лактази B Мальтази C Ізомерази D Оліго-1,6-глюкозидази E Пепсину

264 Під час аналізу крові виявлено високий вміст холестерину в ліпопротеїнах низької щільності (ЛПНЩ). Які можливі наслідки для організму цього явища? A * Виникнення атеросклерозу B Цукрового діабету C Ожиріння D Гіпертонія E Жовтяниця

265 У хворого вкрові та сечі виявлено високий вміст індикану – показника активації процесів гниття білків в кишечнику. Яка амінокислота є джерелом індикану? A * Триптофан B Тирозин C Пролін D Фенілаланін E Гістидин

266 Для діагностики ряду захворювань визначають активність трансаміназ крові. Який вітамін входить до складу кофакторів цих ферментів? A *В6 B В2 C В1 D В8 E В5

267 Хворий поступив в реанімаційне відділення з підозрою на отруєння чадним газом (монооксидом вуглецю). Яка сполука гемоглобіну буде виявлена при спектральному аналізі? A *Карбоксигемоглобін B Карбгемоглобін C Метгемоглобін D Оксигемоглобін

Page 47: крок 1 база 2014р_11шр-240ст (для брошури)_35грн

http://vk.com/my.printing

47

E Дезоксигемоглобін

268 Біологічне окислення та знешкодження ксенобіотиків відбувається за рахунок гемвмісних ферментів. Який метал є обов`язковою складовою цих ферментів? A *Fe B Zn C Co D Mg E Mn

269 У новонародженого фізіологічна жовтяниця. Рівень вільного білірубіну в крові значно перевищує норму. Нестачею якого ферменту це обумовлено? A *УДФ-глюкуронілтрансферази B Трансамінази C Ксантиноксидази D Аденозиндезамінази E Гем-оксигенази

270 У юнака 19 років явні ознаки депігментації шкіри, обумовленої порушенням синтезу меланіну. Вкажіть порушенням обміну якої амінокислоти це викликано? A *Тирозина. B Триптофана. C Гистидина. D Проліна. E Гліцина.

271 В лікарню поступив хворий з підозрою на гострий панкреатит. Підвищення активності якого ферменту в сечі або крові слід очікувати при цьому? A *Амілази. B Пепсину. C Гастриксину. D Креатинкінази. E Аспартаттрансамінази.

272 Для визначення антитоксичної функції печінки хворому призначено бензонат натрію, який в печінці перетворюється в гіппурову кислоту. Яка сполука використовується для цього процесу? A *Гліцин.

B Цистеїн. C Метіонін. D ФАФС. E УДФ – Глюкуронова к-та.

273 Для лікування жовтяниць показано призначення барбітуратів, які індукують синтез УДФ-глюкуронілтрансферази. Лікувальний ефект при цьому обумовлений утворенням: A *Прямого (кон'югованого) білірубіна. B Непрямого (некон'юговано) білірубіна. C Білівердина. D Протопорфирина. E Гема.

274 Визначення активності трансаміназ широко застосовується з метою діагностики пошкоджень внутрішніх органів. Кофактором цих ферментів є активна форма вітаміну A *В6 B В1 C В12 D В2 E РР

275 Лікар призначив аспірин хворому на ревматизм як протизапальний засіб. Синтез яких речовин, пов’язаних з запаленням блокує аспірин? A *Простагландинів. B Гліцерину. C Треоніну. D Глюкагону. E Дофаміну.

276 Хвороба Гірке - це захворювання, при якому спостерігається накопичення глікогену в печінці та нирках. Дефіцит якого ферменту є причиною цього захворювання? A *Глюкозо-6-фосфатази B Глікогенфосфорилаза C Кінази фосфорилази D Фосфоглюкомутаза E Глюкокіназа

277 При цукровому діабеті збільшується вміст кетонових тіл у крові, що приводить до метаболічного ацидозу. З якої речовини

Page 48: крок 1 база 2014р_11шр-240ст (для брошури)_35грн

48

синтезуються кетонові тіла? A *Ацетил-КоА B Сукциніл-КоА C Пропіонил-КоА D Малонил-КоА E Метилмалонил-КоА

278 У хворого спостерігається кетонурія. При якому захворюванні в сечі з’являються кетонові тіла? A *Цукровий діабет B Гострий гломелуронефрит C Сечокам’янна хвороба D Туберкульоз нирки E Інфаркт нирки

279 У пацієнта 40 років непереносимість вуглеводів молока. Недостатністю якого ферменту травлення можна пояснити це явище? A *Лактази B Лактатдегідрогенази C Мальтази D Ліпази E Амілази

280 У юнака 20 років діагностовано спадковий дефіцит УДФ-глюкуронілтрансферази. Підвищення якого показника крові підтверджує діагноз? A *Непрямого (некон'югованого) білірубіну B Прямого (кон'югованого) білірубіну C Уробіліну D Стеркобіліногену E Тваринного індикану

281 Для запобігання післяопераційної кровотечі 6 - річній дитині рекомендовано приймати вікасол, який є синтетичним аналогом вітаміну К. Вкажіть, які посттрансляційні зміні факторів згортання крові активується під впливом вікасолу. A *Карбоксилювання глутамінової кислоти B Фосфорилювання радикалів серину C Частковий протеоліз D Полімеризація E Глікозилювання

282 Чоловік 58 років звернувся до лікаря зі скаргою на біль в суглобах. При обстеженні виявлено підвищення концентрації сечової

кислоти в крові та сечі. Вкажіть, при розпаді яких речовин утворюється сечова кислота A *Пуринових нуклеотидів B Піримідинових нуклеотидів C Амінокислот D Білків E Хромопротеїнів

283 У чоловіка 58 років клінічна картина гострого панкреатиту. Підвищення в сечі якої з перерахованих нижче речовин буде підтвердженням діагнозу? A *Амілази B Залишкового азоту C сечовини D Альбуміну E Сечової кислоти

284 60-літній чоловік звернувся до лікаря після появи болю в грудній клітці.В сироватці крові виявлено значне зростання активності ферментів: креатинфосфокінази та її МВ-ізоформи, аспартатамінотрансферази. Про розвиток патологічного процесу в якій тканині свідчать ці зміни? A *В серцевому м’язі. B В тканині легень. C В скелетних м’язах. D В тканині печінки. E В гладеньких м’язах.

285 У хворого на цукровий діабет після введення інсуліну настала втрата свідомості, спостерігаються судоми. Який результат дав біохімічний аналіз крові на вміст цукру? A *1,5 ммоль/л B 3,3 ммоль/л C 8 ммоль/л D 10 ммоль/л E 5,5 ммоль/л

286 В ендокринологічному відділенні з діагнозом цукровий діабет лікується жінка 40 років зі скаргами на спрагу, підвищений апетит. Які патологічні компоненти виявлені при лабораторному дослідженні сечі пацієнтки? A *Глюкоза, кетонові тіла B Білок, амінокислоти C Білок, креатин D Білірубін, уробілін E Кров

Page 49: крок 1 база 2014р_11шр-240ст (для брошури)_35грн

http://vk.com/my.printing

49

287 Іони Са2+ - один з еволюційно найдревніших вторинних месенджерів в клітинах. Вони є активаторами глікогенолізу, якщо взаємодіють з: A *Кальмодуліном. B Кальцитоніном. C Кальциферолом. D Кіназою легких ланцюгів міозину. E Фосфорилазою С.

288 У хворого спостерігаються часті кровотечі з внутрішніх органів, слизових оболонок. Аналіз виявив недостатність гідрооксипроліну та гідроксилізину в складі колагенових волокон. Через нестачу якого вітаміну порушено в організмі пацієнта процеси гідроксилювання названих амінокислот? A *С B А C Н D К E РР

289 Недостатня секреція якого ферменту зумовлює неповне перетравлювання жирів в кишково-шлунковому тракті та появу великої кількості нейтральних жирів в калових масах? A *Панкреатичної ліпази B Фосфоліпази C Ентерокінази D Амілази E Пепсину

290 Чоловік 55 років, що страждає на болі в області нирок, надійшов в лікарню. При ультразвуковому обстеженні пацієнта виявлено наявність ниркових каменів. Наявність якої речовини в сечі є найбільш вірогідною причиною утворення каменів в даного пацієнта? A *Сечової кислоти. B Білірубіну. C Білівердину. D Уробіліну. E Креатиніну.

291

У хлопчика 4 років після перенесеного важкого вірусного гепатиту спостерігається блювання, втрати свідомості, судоми. У крові - гіперамоніємія. Порушення якого біохімічного процесу викликало подібний патологічний стан хворого? A *Порушення знешкодження аміаку в печінці. B Порушення знешкодження біогенних амінів. C Посилення гниття білків у кишечнику. D Активація декарбоксилування амінокислот. E Пригнічення ферментів транс амінування.

292 Ріст дорослого чоловіка становить 112 см при пропорційному складові тіла та нормальному розумовому розвитку. Недостатність вироблення якого гормону спричинила такі симптоми? A *Соматотропного гормону. B Гонадотропного гормону. C Антидіуретичного гормону. D Тиреотропного гормону. E Тироксину.

293 Ті організми, які в процесі еволюції не створили захисту від Н2О2, можуть жити лише в анаеробних умовах. Які з перелічених ферментів можуть руйнувати пероксид водню? A * Пероксидаза та каталаза B Оксигенази та гідроксилази C Цитохромоксидаза, цитохром в5 D Оксигеназа та каталаза E Флавінзалежні оксидази

294 У клініку доставлена хвора з цукровим діабетом у прекоматозному стані кетоацидотичного типу. Збільшення змісту якого метаболіту до цього привело? A *Ацетоацетату B Цитрату C Альфа-кетоглутарату D Малонату E Аспартату

295 У хворого гострим панкреатитом виникає загроза некрозу підшлункової залози, що супроводжується надходженням у кров і тканини активних панкреатичних протеїназ і

Page 50: крок 1 база 2014р_11шр-240ст (для брошури)_35грн

50

розщеплення тканинних білків. Які захисні фактори організму можуть інгібувати ці процеси? A *Альфа2 – макроглобулін, альфа1- антитрипсин B Імуноглобуліни C Кріоглобулін, інтерферон D Церулоплазмін, трансферин E Гемопексин, гаптоглобін

296 Для профілактики атеросклерозу, ішемічної хвороби серця, порушень мозкового кровообігу рекомендується споживання жирів із високим вмістом поліненасичених жирних кислот. Однією з таких жирних кислот є: A * Лінолева B Олеїнова C Лауринова D Пальмітоолеїнова E Стеаринова

297 Стеатоз виникає внаслідок накопичення триацилгліцеролів у гепатоцитах. Одним з механізмів розвитку цього захворювання є зменшення утилізації нейтрального жиру ЛПДНЩ. Які ліпотропні речовини попереджують розвиток стеатозу? A *Метіонін, В6, В12 B Аргінін, В2, В3 C Аланін, В1, РР D Валін, В3, В2 E Ізолейцин, В1, В2

298 У пацієнта М., 32 р. має місце гіповітаміноз В2. Причиною виникнення специфічних симптомів (ураження епітелію, слизових, шкіри, рогівки ока) ймовірно є дефіцит A * Флавінових коферментів B Цитохрома а1 C Цитохромоксидази D Цитохрома в E Цитохрома с

299 Порушення процесів мієлінізації нервових волокон призводить до неврологічних розладів і розумової відсталості. Такі симптоми характерні для спадкових і набутих порушень обміну: A *Сфінголіпідів B Нейтральних жирів C Вищих жирних кислот

D Холестерину E Фосфатидної кислоти

300 Підшлункова залоза - орган змішаної секреції. Ендокринно продукує бета-клітинами гормон інсулін, який впливає на обмін вуглеводів. Вкажіть, як він впливає на активність глікогенфосфорилази (ГФ) і глікогенсинтетази (ГС)? A *Пригнічує ГФ, активує ГС B Активує ГФ і ГС C Пригнічує ГФ і ГС D Активує ГФ, пригнічує ГС E Не впливає на активність ГФ і ГС

Гістологія 1 При гетеротрансплантації органу виявлено відторгнення трансплантату. Які клітини крові забезпечують цей процес? A *Т-лімфоцити - кілери. B Т-лімфоцити-хелпери C Т-лімфоцит-супресор D Т-лімфоцит-О E Т-лімфоцити-пам’яті

2 При ендоскопічному дослідженні шлунка виявлено пошкодження епітеліального покриву слизової оболонки. За рахунок яких гландулоцитів можлива репаративна його регенерація A *Малодиференційованих шийкових мукоцитів. B Додаткових мукоцитів C Головних екзокриноцитів D Парієтальних екзокриноцитів E Покривного залозистого епітелію

3 Після перенесеного хімічного опіку стравоходу наступило локальне його звуження внаслідок утворення рубця. Які клітини пухкої сполучної тканини беруть участь в утворенні рубців? A *Зрілі спеціалізовані фібробласти B Юні малоспеціалізовані фібробласти C Фіброцити D Міофібробласти E Фіброкласти

Page 51: крок 1 база 2014р_11шр-240ст (для брошури)_35грн

http://vk.com/my.printing

51

4 Відомо, що в периферичній крові людини можуть з'являтися мегалоцити. Коли в нормі є ці клітини в крові? A *В ембріональному періоді B У віці до 1 року C У віці від 1 до 30 років D У старому віці E Під час вагітності

5 У хворого взята кров для аналізу, ЇЇ дані показують, що 30% еритроцитів мають неправильну форму. Як називається цей стан? A *Патологічний пойкілоцитоз B Анізоцитоз C Фізіологічний пойкілоцитоз D Макроцитоз E Мікроцитоз

6 В мазку периферійної крові видно велику клітину із слабобазофільною цитоплазмою і бобовидним ядром. Клітина є найбільшою серед видимих в полі зору. Яка це клітина? A *Моноцит B Макрофаг C Плазмоцит D Середній лімфоцит E Малий лімфоцит

7 В мазку периферійної крові серед лейкоцитів переважають округлі клітини з посегментованими ядрами. Дрібна зернистість в їх цитоплазмі фарбується як кислими, так і основними барвниками. Як називаються ці клітини? A *Сегментоядерні нейтрофіли B Базофіли C Еозинофіли D Юні нейтрофіли E Моноцити

8 На гістологічному зрізі бачимо орган, який ззовні вкритий серозною та білочною оболонками. Строму органа складає пухка сполучна тканина, в якій містяться клітини Лейдіга, паренхіма представлена канальцями, внутрішню поверхню канальців вистеляє сперматогенний епітелій. Що це за орган? A *Сім’яник.

B Придаток сім’яника. C Простата. D Молочна залоза. E Яєчник.

9 У жінки спостерігається гіперемія яєчника, підвищення проникливості гемато-фолікулярного бар’єру з послідовним розвитком набряку, інфільтрація стінки фолікула сегментоядерними лейкоцитами. Об’єм фолікула великий. Стінка його потоншена. Якому періоду статевого циклу відповідає описана картина? A *Предовуляторна стадія. B Овуляція. C Менструальний період. D Постменструальний період. E Період відносного спокою.

10 На гістологічному препараті нирки в дистальному звилистому канальці виявляються клітини, які щільно прилягають до ниркового тільця. Базальна мембрана їх дуже тонка і не утворює складок. Ці клітини відчувають зміни вмісту натрію у сечі та впливають на секрецію реніна юкстагломерулярними клітинами. Які це клітини? A *Клітини щільної плями. B Юкстагломерулярні клітини. C Мезангіальні клітини. D Подоцити. E Ендотелій капілярів клубочка.

11 Під час клінічного обстеження у 35-річної жінки із захворюванням нирок в сечі виявлені клітини крові, фібриноген, що вірогідно пов’язано з порушенням ниркового фільтру. З яких структур складається цей фільтр? A *Ендотелій капілярів клубочка, тришарова базальна мембрана, подоцити. B Тришарова базальна мембрана. C Ендотелій капілярів, базальна мембрана D Подоцити, базальна мембрана. E Ендотелій, подоцити.

12 При електронній мікроскопії нирки виявлені канальці, які вистелені кубічним епітелієм. В епітелії розрізняють світлі та темні клітини.

Page 52: крок 1 база 2014р_11шр-240ст (для брошури)_35грн

52

В світлих клітинах мало органел. Цитоплазма утворює складки. Ці клітини забезпечують реабсорбцію води з первинної сечі у кров. Темні клітини за будовою і функцією нагадують парієтальні клітини шлунку. Які канальці представлені на електроннограмі? A *Збірні ниркові трубочки. B Проксимальні канальці. C Дистальні канальці. D Висхідні канальці петлі Генле. E Нисхідні канальці петлі Генле.

13 В гістологічному препараті представлений поперечний зріз стінки порожнистого органу, слизова оболонка якого вкрита багатоша-ровим плоским незроговілим епітелієм. Який це орган ? A *Стравохід. B 12-пала кишка. C Товста кишка. D Матка. E Апендикс.

14 Після гастректомії у хворого розвивається злоякісна анемія. Відсутність яких клітин залоз шлунку викликає дану патологію? A *Парієтальних. B Головних. C Шийкових мукоцитів. D Ендокриноцитів. E Келихоподібних.

15 На гістологічному препараті підслизова основа тонкої кишки заповнена кінцевими секреторними відділами білкових залоз. Який відділ кишки представлений на препараті ? A *12-пала кишка. B Товста кишка. C Голодна кишка. D Клубова кишка. E Апендикс.

16 На гістологічному препараті стінки тонкої кишки на дні крипт знайдено розташовані групами клітини, в апікальній частині яких містяться великі ацидофільні секреторні гранули; цитоплазма забарвлена базофільно. Які це клітини? A * Клітини Панета.

B Клітини без облямівки. C Ендокринні клітини. D Келихоподібні клітини. E Стовпчасті з облямівкою.

17 В гістологічному препараті паренхіма органа представлена час-точками, які мають форму шестигранних призм і складаються з анастомозуючих пластинок, між якими лежать синусоїдні капі-ляри, які радіально сходяться до центральної вени. Який ана-томічний орган має дану морфологічну будову? A *Печінка. B Підшлункова залоза. C Тимус. D Селезінка. E Лімфатичний вузол.

18 Мозкова речовина часточки кровотворного органа на гісто-логічному препараті має світліше забарвлення і містить епі-теліальні тільця. Якому органу належать дані морфологічні оз-наки? A *Тимусу. B Лімфатичному вузлу. C Селезінці. D Печінці. E Нирці.

19 Стінки судин мають досить значні морфологічні розбіжності у будові середньої оболонки. Чим зумовлена поява специфічних особливостей будови цієї оболонки у різних судинах? A *Гемодинамічними умовами. B Впливом органів ендокринної системи. C Регуляцією з боку центральної нервової системи. D Індуктивним впливом нейронів вегетативних гангліїв. E Високим вмістом катехоламінів у крові.

20 Артерії великого калібру під час систоли розтягуються і поверта-ються у вихідний стан під час діастоли, забезпечуючи стабільність кровотоку. Наявністю яких елементів стінки судини це можна пояснити? A *Еластичних волокон. B М’язових волокон.

Page 53: крок 1 база 2014р_11шр-240ст (для брошури)_35грн

http://vk.com/my.printing

53

C Ретикулярних волокон. D Колагенових волокон. E Великою кількістю фібробластів.

21 Внутрішню оболонку судини (інтиму) зсередини вистеляє епі-телій. Назвіть його. A *Ендотелій. B Мезотелій. C Епідерміс. D Перехідний епітелій. E Багаторядний епітелій.

22 На електронній мікрофотографії представлені структури у вигляді відкритих міхурців, внутрішня поверхня яких вистелена одно-шаровим епітелієм, який утворений респіраторними та секре-торними клітинами. Які це структури? A *Альвеоли. B Бронхіоли. C Ацинуси. D Альвеолярні ходи. E Термінальні бронхи.

23 В епітелії повітроносних шляхів є клітини з куполоподібною апі-кальную частиною, на поверхні якої розміщуються мікровор-синки. В клітині виявляється добре розвинений синтетичний апарат, а в апікальній частині – секреторні гранули. Назвіть цю клітину. A *Клітина Клара. B Келихоподібна. C Ендокринна. D Клітина без облямівки. E Камбіальна

24 У недоношених дітей розвивається синдром дихальної недос-татності. Недостатність якого компоненту аерогематичного бар'єру лежить в основі цієї патології? A *Сурфактант . B Ендотелій капілярів. C Базальна мембрана ендотелію. D Базальна мембрана альвеолоцитів. E Альвеолоцити.

25 Кінцеві відділи апокринових потових залоз містять міоепі-теліальні клітини.Яка функція цих клітин? A *Скоротлива.

B Секреторна. C Захисна. D Регенераторна. E Підтримуюча.

26 В гістологічному препараті представлена тканина, основною структурною одиницею якої є волокно, яке складається із сим-пласта і сателітоцитів, вкритих спільною базальною мембраною. Для якої тканини характерна дана структура? A *Скелетної поперечно-посмугованої м`язової тканини. B Гладкої м`язової тканини. C Серцевої м'язової тканини. D Пухкої сполучної тканини. E Ретикулярної тканини.

27 На гістологічному препараті хрящової тканини виявляються ізогенні групи клітин. Які клітини є початковими в утворенні цих груп? A *Хондроцити I типу. B Хондробласти. C Прехондробласти. D Хондроцити II типу. E Хондроцити III типу.

28 З ектодермального епітелію вистилки верхньої частини ротової ямки зародка людини формується кишеня Ратке, яка направ-ляється до основи майбутнього головного мозку. Що розвивається з даного ембріонального зачатка? A *Аденогіпофіз. B Нейрогіпофіз. C Медіальна еміненція. D Гіпофізарна ніжка. E Передній гіпоталамус.

29 Паренхіма аденогіпофіза представлена трабекулами, утвореними залозистими клітинами. Серед аденоцитів є клітини з гранулами, які забарвлюються основними барвниками і містять глікопротеїди. Які це клітини? A *Гонадотропоцити, тиротропоцити. B Соматотропоцити. C Меланотропоцити.

Page 54: крок 1 база 2014р_11шр-240ст (для брошури)_35грн

54

D Мамотропоцити. E Хромофобні

30 В стінці фолікулів та в міжфолікулярних прошарках сполучної тканини на території щитовидної залози розміщуються великі ендокриноцити, секреторні гранули яких осміо- і аргірофільні. Назвіть ці клітини. A *Кальцитоніноцити. B Тироцити. C Паратироцити. D Пінеалоцити. E Пітуіцити.

31 В гістопрепараті представлений паренхіматозний орган, повер-хневий шар кіркової речовини якого формують клубочки, утворені ендокриноцитами. Якому органу належить дана морфологічна ознака? A *Наднирнику. B Лімфатичному вузлу. C Селезінці. D Щитовидній залозі. E Яєчнику.

32 При повторному попаданні антигена в організм виділяються антитіла. З функцією яких імунокомпетентних клітин пов’язане це явище? A *Лімфоцитами пам’яті B Т - кіллерами C Т - супресорами D Макрофагами E Дендритними клітинами

33 У клітині штучно блоковано синтез гістонових білків. Яка структура клітини буде пошкоджена? A * Ядерний хроматин B Ядерце C Комплекс Гольджі D Клітинна оболонка E Ядерна оболонка

34 У розвитку клінічних проявів алергії провідну роль відіграє гістамін. Якими клітинами він виробляється? A * тучними клітинами

B Т- лімфоцитами C макрофагами D В- лімфоцитами E плазмоцитами

35 У експиременті вибірково стимулювали одну з популяцій клітин крові. В резутьтаті цього значно підвищилась проникливість судин, що виявилось у формі набряку пеиваскулярної тканини та сповільнення процесу згортання крові. Які клітини крові підлягли стимуляції? A *Базофіли B Еритроцити C Тромбоцити D Еозинофіли E Лімфоцити

36 У експерименті помітили міткою В-лімфоцити крові. Тварині введено під шкіру чужорідний білок. Які клітини у сполучній тканині будуть містити цю мітку? A * плазмоцити B Т-лімфоцити C макрофаги D тканинні базофіли E фібробласти

37 У хворої на аденому гіпофіза (новоутворення в передній частці гіпофіза) спостерігається збільшення тривалості фази великого росту фолікулів. Яка тривалість періоду великого росту овоцитів в процесі овогенезу в нормі? A *12-14 днів B Декілька десятків років (від 10-13 до 40-50) після народження C Після народження і до наступу статевої зрілосиі D З 3 місяця пренатального розвитку і до народження E 28 днів

38 При механічній травмі калитки у хворого виявлено порушення епітеліального вистелення сітки сім(яника. Який епітелій зазнав ушкодження? A *Одношаровий кубічний B Миготливий C Одношаровий призматичний D Дворядний

Page 55: крок 1 база 2014р_11шр-240ст (для брошури)_35грн

http://vk.com/my.printing

55

E Перехідній

39 У крові дівчини 16 років, котра страждає аутоімунним запаленням щитовидної залози, виявлено численні плазматичні клітини. З проліферацією та диференціюванням яких клітин крові пов’язано збільшення кількості плазмоцитів? A *В-лімфоцитів B Т-хелперів C Тканинних базофілів D Т-кілерів E Т-супресорів

40 При дослідженні поперечно-смугастого м(язового волокна після дії гідролітичних ферментів спостерігається руйнування тонких міофіламентів. Які саме структури зазнали ушкодження? A *Актинові міофіламенти B Тонофібрили C Т - системи D Саркоплазматична сітка E Міозинові міофіламенти

41 Жінка 25 років через місяць після пологів звернулась до лікаря зі скаргою на зменшення кількості молока. Недолік якого гормону призвів до такого стану? A *Пролактину B Адренокортикотропного гормону C Соматостатіну D Інсуліну E Глюкагону

42 Відомо, що альдостерон регулює вміст натрію в організмі. Які клітини наднирників виробляють цей гормон? A *Клітини клубочкової зони B Епінефроцити C Клітини сітчастої зони D Клітини пучкової зони E Норепінефроцити

43 У чоловіка 53 років діагностована злоякісна епітеліальна пухлина перикарду. Який епітелій є джерелом розвитку пухлини? A *Одношаровий плоский B Одношаровий багаторядний війчастий C Перехідний

D Багатошаровий зроговілий E Багатошаровий незроговілий

44 При обстеженні окуліст з'ясував, що пацієнт не розрізняє синій та зелений колір, при нормальному сприйнятті іншої кольорової гами. З порушенням функції яких структур сітківки це пов'язано? A *колбочкові нейрони B паличкові нейрони C біполярні нейрони D амакринні нейрони E горизонтальні нейрони

45 В експерименті на зародку жаби зруйновано зовнішній зародковий листок – ектодерму. Яка морфологічна структура з перелічених не буде в подальшому розвиватись у даного зародка ? A * Епідерміс. B Соміти. C Нефротом. D Спланхнотом. E Кісткова тканина

46 На мікропрепараті тонкої кишки у власній пластинці слизової оболонки виявили скупчення клітин кулястої форми з великими базофільними ядрами, які оточені вузьким ободком цитоплазми. У більшості таких скупчень центральна частина світла і містить менше клітин, ніж периферійна. До якої морфологічної структури належать такі скупчення? A * Лімфатичний вузлик. B Нервовий вузлик. C Жирові клітини. D Кровоносні судини. E Лімфатичні судини.

47 На мікропрепараті з контурами бобоподібного органу спостерігається кіркова та мозкова речовина. Кіркова речовина представлена окремими кулястими вузликами діаметром 0,5 …1 мм, а мозкова – мозковими тяжами. З якого органа зроблено гістологічний зріз ? A * Лімфатичного вузла. B Нирки.

Page 56: крок 1 база 2014р_11шр-240ст (для брошури)_35грн

56

C Тимуса. D Наднирника. E Селезінки.

48 Зроблено гістологічний зріз через лімфатичний вузол. На мікропрепараті спостерігається розширення його паракортикальної зони. Проліферація якого виду клітин лімфатичного вузла обумовила цей процес? A * Т-лімфоцитів. B Береговіх макрофагів. C Плазмоцитів. D Макрофагів. E Ретикулоцитів.

49 На мікропрепараті виявлено кулясті утворення з лімфоцитів. В середині утворень - центральна артерія. Який орган досліджується ? A * Селезінка. B Нирка. C Тимус. D Кістковий мозок. E Лімфатичний вузол.

50 Після тривалого запалення слизової оболонки носової порожнини у хворого спостерігаються зміни епітелію. Який епітелій зазнав змін? A *Одношаровий багаторядний B Одношаровий плоский C Багатошаровий плоский D Багатошаровий кубічний E Багатошаровий циліндричний

51 Під час тренування у спортсмена була травмована нижня кінцівка. Лікар травматолог встановив діагноз: розрив сухожилка. До якого типу сполучної тканини належить сухожилок? A *Щільної оформленої волокнистої тканини B Щільної неоформленої волокнистої тканини C Пухкої волокнистої сполучної тканини D Ретикулярної тканини E Хрящової тканини

52 З віком шкіра людини зазнає змін, що можуть проявлятися зменьшенням її

пружності. Які елементи сполучної тканини найбільше забезпечують її пружність? A * Колагенові та еластичні волокна B Основна речовина C Клітини епідерміса D Клітини сполучної тканини E Ретикулярні волокна

53 Хворий, 55 років, наглядається у ендокринолога з приводу порушення ендокринної функції підшлункової залози, що проявляється зменьшенням кількості гормону глюкагону в крові.. Функція яких клітин цієї залози порушена в цьому випадку? A *А-клітини острівців Лангерганса B В-клітини острівців Лангерганса C Д-клітини острівців Лангерганса D Д1-клітини острівців Лангерганса E РР-клітини острівців Лангенганса

54 У ендокринолога наглядається хворий,40 років, у якого спостерігається недостатість функції кіркової речовини надниркових залоз, що проявляється зменьшенням кількості гормону альдостерону в крові. Функція яких клітин кори порушена? A *Клітини клубочкової зони B Клітини пучкової зони C Клітини сітчастої зони D Клітини суданофобної зони E Клітини Х-зони

55 При дослідженні мазка крові хворого А. Виявлені клітини, які складають 0,5\% від загального числа лейкоцитів, та мають S-образно зігнуте ядро, метахроматично пофарбовані гранули в цитоплазмі. Назвіть ці клітини. A * Базофіли B Нейтрофіли C Еозінофіли D Моноцити E Лімфоцити

56 У хворого сухим плевритом вислуховується шум тертя плеври. Який епітелій при цьому пошкоджується ? A * Одношаровий плоский епітелій B Одношаровий кубічний епітелій C Одношаровий призматичний епітелій D Перехідний епітелій

Page 57: крок 1 база 2014р_11шр-240ст (для брошури)_35грн

http://vk.com/my.printing

57

E Багатошаровий епітелій

57 При обстеженні хворого 35 років проведено гістологічне дослідження пунктату червоного кісткового мозку і виявлено значне зменшення кількості мегакаріоцитів. Якими змінами периферичної крові це супроводжується? A *Зменшення кількості тробоцитів B Збільшення кількості лейкоцитів C Збільшення кількості тробоцитів D Зменшення кількості гранулоцитів E Зменшення кількості лейкоцитів

58 В результаті травми носа у чоловіка 30 років пошкоджена слизова оболонка, що вкриває верхню частину верхньої раковини. До яких наслідків це призвело? A * Порушення сприйняття пахучих речовин B Порушення зволоження повітря C Порушення секреторної активності келихоподібних клітин D Порушення зігрівання повітря E Порушення зігрівання і зволоження повітря

59 Хвора з 14 років хворіє на цукровий діабет. Які клітини підшлункової залози не функціонують? A * В - клітини B А - клітини C Д - клітини D Д1- клітини E РР - клітини

60 У дитини першого року життя спостерігається порушення створожування материнського молока. З порушенням діяльності яких клітин власних залоз шлунку це пов’язано? A *Головні екзокриноцити B Парієтальні екзокриноцити C Шийкові мукоцити D Додаткові мукоцити E Екзокриноцити

61 У хворого 14 років, спостерігається порушення сутінкового бачення. Якого вітаміну недостатньо в організмі? A * А B В1

C В6 D С E В12

62 Студентові дано препарати двох мазків. На одному- все поле зору вкрите еритроцитами, на другому визначаються формені елементи крові різного ступеня зрілості. Що це за мазки? A *Кров і червоний кістковий мозок людини B Кров і лімфа C Кров жаби і кров людини D Кров і мазок жовтого кісткового мозку E Мазок жовтого і червоного кісткового мозку

63 Студентові видано два гістологічні препарати. На обох- органи, які мають лімфатичні вузлики. На першому препараті- тільки фолікули, а на другому- фолікули ексцентрично містять судину. Визначте що це за органи? A *Перший-лімфатичний вузол, другий-селезінка B Перший-червоний кістковий мозок, другий-селезінка C Перший-тимус, другий-селезінка D Перший-печінка, другий- лімфатичний вузол E Перший-печінка, другий-селезінка

64 Суглобові хрящі, як відомо, не мають охрястя. Який ріст цих хрящів відбуваеться в процесах регенерації? A *Інтерстиційний B Апозиційний C Шляхом накладання D Апозиційний і інтерстиційний E Не реагує

65 Один з критичних періодів ембріогенезу людини є імплантація зародка в стінку матки на протязі 7-ої доби. Який процес гаструляції відбувається в ембріобласті в цей період? A *Делямінація. B Міграція. C Епіболія. D Інвагінація.

Page 58: крок 1 база 2014р_11шр-240ст (для брошури)_35грн

58

E Нейруляція.

66 При гистохимическом исследовании лейкоцитов мазка крови определяются клетки, в цитоплазме которых находятся гранулы, содержащие гистамин и гепарин. Какие это клетки ? A *Базофилы. B Нейтрофилы. C Эозинофилы. D Моноциты. E Эритроциты.

67 Тривалий вплив на організм токсичних речовин призвів до значного скорочення синтезу білків у гепатоцитах. Які органели найбільше постраждали від інтоксикації? A *Гранулярна ендоплазматична сітка B Мітохондрії C Мікротрубочки D Лізосоми E Комплекс Гольджі

68 Під час гаструляції у зародку недостатньо сформувався первинний Гензеновський вузлик. Розвиток якого осьового органу загальмується? A *Хорди B Нервових гребінців C Нервового жолобка D Нервової трубки E Мантійного шару нервової трубки

69 Під впливом радіації постраждали клітини базального шару епідермісу. Яка функція останнього послабиться, або загальмується перш за все? A *Регенеративна B Захисна C Бар’єрна D Всмоктувальна E Діелектрична

70 За результатами вивчення плям крові на місці злочину судово-медичний експерт визначив, що це кров жінки. За якими ознаками? A *Наявність сателітів ядер в нейтрофілах B Наявність мікроцитів і макроцитів C Явища пойкілоцитозу D Наявність специфічних гранул в

еозинофілах. E За кількістю еритроцитів.

71 Під час гетеротрансплантації органів виявлено відтторження трансплантту. Які клітини головним чином забеспечують цей процес? A * Т-кілери B Макрофаги C В-лімфоцити D Т-хелпери E Т-супресори

72 У цітоплазмі клітин підшлункової залози в процесі секреторного циклу в апiкальній частині з’являються і зникають гранули секрету. До яких структурних елементів можна віднести ці гранули? A * До включень B До мікрофіламентів C До лізосом D До екзоцитозних вакуолей E До гранулярної ендоплазматичної сітки

73 Під дією шкідливих факторів сталося вогнещеве пошкодження епітелію шлунка. За рахунок яких клітин сталося його регенерація? A *Шиєчні мукоцити B Паріетальні екзокриноцити C Головні екзокриноцити D Ендокриноцити E Мукоцити тіла залоз

74 У хворого значно підвищено добове виділення сечі. Недоліком секреції якого гормону гіпотоламусу можна з’яснити це явище? A *Вазопресин B Оксiтоцин C Ліберіни D Статіни E Тіріоідний

75 У хворого пошкоджено целіарне тіло. Функція якого апарату ока при цьому страждає? A *Акомадацiйний B Світлопровідний C Світловідчуваючий D Захисний

Page 59: крок 1 база 2014р_11шр-240ст (для брошури)_35грн

http://vk.com/my.printing

59

E Трофічний

76 Сталася травма шкіри з пошкодженням сітчатого шару дерми. За рахунок діяльності яких клітин станеться регенерація цього шару? A *Фібробластів B Макрофагів C Лімфобластів D Тканинних базофілів E Плазматичних клітин

77 У біопсійному матеріалі шкіри в епідермісі виявлені клітини з відростками які мають гранули темно-коричневого кольору у цитоплазмі. Що це за клітини? A *Меланоцити. B Внутрішньоепідермальні макрофаги. C Кератиноцити. D Клітини Меркеля. E Лімфоцити.

78 У дитини навколо подряпини на шкірі виникли ознаки запалення: біль, почервоніння, набряк як ознаки негайної гіперчутливості. Які клітини крові обумовлюють ці зміни? A * Базофіли B Еозинофіли C Нейтрофіли D Лімфоцити E Моноцити

79 В судово-медичній експертизі широко використовується метод дактилоскопії, який оснований на тому, що сосочковий шар дерми визначає строго індивідуальний малюнок на поверхні шкіри. Яка тканина утворює цей шар дерми? A * Пухка волокниста неоформлена сполучна частина B Щільна оформлена сполучна тканина C Щільна неоформлена сполучна тканина D Ретикулярна тканина E Жирова тканина

80 На одній з фаз сперматогенезу спостерігаються зміни ядра і цитоплазми сперматид, які призводять до утворення зрілих статевих клітин. Назвіть азу гаметогенеза.

A * Формування. B Дозрівання. C Росту D Розмноження E Проліферація

81 Важливою складовою частиною ниркового фільтраційного бар’єру є тришарова базальна мембрана, яка має спеціальну сітчасту будову її середнього електроннощільного шару. Де міститься ця базальна мембрана? A *В нирковому тільці B в капілярах перитубулярної капілярної сітки C в проксимальному канальці D в тонкому канальці E в дистальному прямому канальці

82 У гістологічному препараті виявляються судини, що починаються сліпо, мають вигляд сплющених ендотеліальних трубок, не містять базальної мембрани і перицитів, ендотелій цих судин фіксований стропними філаментами до колагенових волокон сполучної тканини. Які це судини? A *Лімфокапіляри B гемокапіляри C артеріоли D венули E артеріо-венозні анастомози

83 При мікроскопічному дослідженні внутрішніх статевих жіночих органів, що видалені під час операції був знайдений ембріон побудований з двох бластомерів. Назвати місце його локалізації при умові нормального розвитку. A * Маткова труба, близько ампульної частини B маткова труба, близько маткової частини C порожнина матки D черевна порожнина E яєчник

84 На гістологічному зрізі однієї з ендокринних залоз видно округлі структури різних розмірів, стінка яких утворена одним шаром

Page 60: крок 1 база 2014р_11шр-240ст (для брошури)_35грн

60

епітеліальних клітин на базальній мембрані, всередині ці структури містять гомогенну неклітинну масу. Яка це залоза? A * Щитовидна залоза B Надниркова залоза, кіркова речовина C Прищитовидна залоза D передня частка гіпофізу E задня частка гіпофізу

85 Гістологічна картина ендометрію має наступні характерні ознаки: потовщення, набряк, наявність звивистих залоз із розширеним просвітом, які секретують велику кількість слизу, мітози в клітинах не спостерігаються, у стромі наявні децидуальні клітини. Яка стадія менструального циклу відповідає описаній картині? A *Секреторна (пременструальна). B менструальна C регенераторна D проліферативна E відносного спокою

86 На гістологічному зрізі дна шлунка у складі залоз видно порівняно великі клітини з ацидофільною цитоплазмою, електронномікроскопічно в цих клітинах є наявною складна система внутрішньоклітинних канальців. Який компонент шлункового соку утворюється в наслідок діяльності цих клітин? A * Соляна кислота B Пепсиноген C Слиз D Серотонін E Гастрин

87 В гістологічному препараті відділ нервової системи, в якому визначається пошарове розташування нейроцитів, серед яких є клітини таких форм: зірчасті, веретеноподібні, горизонтальні, пірамідні. Який це відділ нервової системи? A * Кора великих півкуль головного мозку. B Кора мозочка C спинномозковий вузол D вегетативний вузол E спинний мозок

88 На електронній мікрофотограммі виявляються клітини альвеол, які входять

до складу аерогематичного бар’єру. Що це за клітини? A * Респіраторні епітеліоцити альвеол B секреторні епітеліоцити альвеол C альвеолярні макрофаги D клітини Клара E мікроворсинчасті епітеліоцити

89 При аналізі рентгенограми хворого 57 років лікар звернув увагу на локальне розсмоктування твердих тканин окремих кісток. З підвищеною активністю яких клітин можуть бути пов'язані ці зміни? A *Остеокластів B Хондробластів C Остеоцитів D Остеобластів E Хондроцитів

90 В експерименті у тварини шляхом звуження ниркової артерії отримано стійке підвищення кров"яного тиску. Функція яких клітин нирок обумовлює цей ефект? A *ЮГА-клітини B Подоцити C Ендотеліоцити D Інтерстиційні клітини E Клітини щільної плями

91 При розтині померлого чоловіка 65 років, який страждав захворюванням легень, патологічний процес переважно був локалізований у бронхах, де при гістологічному дослідженні були чітко видні залози, хрящові острівці та багаторядний циліндричний миготливий епітелій. В яких бронхах зміни? A *Середні бронхи B Головні бронхи C Великі бронхи D Малі бронхи E Термінальні бронхіоли

92 У стінці бронху при гістологічному дослідженні чітко визначаються залози, хрящові острівці та багаторядний циліндричний миготливий епітелій. В яких бронхах зміни? A *Середні бронхи B Головні бронхи C Великі бронхи D Малі бронхи

Page 61: крок 1 база 2014р_11шр-240ст (для брошури)_35грн

http://vk.com/my.printing

61

E Термінальні бронхіоли

93 При клінічному обстеженні пацієнта 70 років виявлено порушення рухових функцій, що пов'язано з віковими змінами у гіаліновому хрящу. Які вікові зміни викликали обмеження рухів у суглобах ? A * Відкладання солей кальцію в міжклітинній речовині B Збільшення кількості ізогенних груп C Збільшення кількості хрящових клітин D Потовщення охрястя E Збільшення гідрофільності основної речовини

94 Під час судово-медичної експертизи жінки , яка загинула у автокатастрофі, знайдено ембріон на стадії ранньої гаструли. Назвіть місце його локалізації при умові його нормального розвитку. A * стінка матки B ампульна частина яйцепроводу C маткова частина яйцепроводу D яєчник E черевна порожнина

95 Хворий,30 років, звернувся до лікаря із скаргами на підвищення температури тіла до тридцяти вісьми градусів,слабкість,болі у горлі. При обстеженні з’ясувалося, що язик хворого вкритий білим нальотом. Які гістологічні структури язика беруть участь в утворенні цього нальоту? A *Епітелій ниткоподібних сосочків B Епітеліц листоподібних сосочків C Епітелій грибоподібних сосочків D Епітелій жолобкуватих сосочків E Сполучнотканинна основа всіх сосочків язика

96 У мікропрепараті виявлена залоза, яка складається з кількох секреторних відділів у формі мішечків, які відкриваються в одну загальну вивідну протоку. Яка це залоза? A *проста розгалужена альвеолярна B складна розгалужена альвеолярна C проста нерозгалужена альвеолярна D складна нерозгалужена альвеолярна E проста розгалужена трубчаста

97

В результаті травми порушено цілісність переднього корінця спинного мозку. Визначте, відростки яких нейронів при цьому пошкоджено? A *Аксони рухових нейронів B Дендрити рухових нейронів C Аксони чутливих нейронів D Дендрити чутливих нейронів E Дендрити вставних нейронів

98 На електронній мікрофотографії представлений міжклітинний контакт, основою якого на кожній клітині є пластинки прикріплення; міжклітинний проміжок заповнений електронно-щільною речовиною, в якій розрізняються трансмембранні фібрилярні структури. Назвіть цей контакт: A * Десмосома B Синапс C Щільний замикальний D Нексус E За типом замка

99 При гістологічному дослідженні діафізу трубчастої кістки на іі поверхні під шаром волокон можна бачити базофільні клітини з розвиненими органелами синтезу. Ці клітини беруть участь у регенерації кісткової тканини. В якому шарі діафізу вони розташовані? A *Окістя B Власне кістка C Остеонний шар D Шар зовнішніх генеральних пластинок E Шар внутрішніх генеральних пластинок

100 На електронній мікрофотографії представлена клітина нейрального походження. Термінальна частина дендрита клітини має циліндричну форму і складається з 1000 замкнутих мембранних дисків. Яка клітина зображена на мікрофотографії? A *Паличкова нейросенсорна. B Колбочкова нейросенсорна C Нейрон спинномозкового вузла. D Нейрон кори великих півкуль. E Нейрон передніх рогів спинного мозку.

Page 62: крок 1 база 2014р_11шр-240ст (для брошури)_35грн

62

Мікробіологія 1 Досить часто причиною набутих імунодефіцитів є інфекційне ураження організму, при якому збудники безпосередньо розмножуються в клітинах імунної системи і руйнують їх. Виберіть серед перерахованих ті захворювання, при яких має місце вищезгадане: A * Інфекційний мононуклеоз, СНІД B Туберкульоз, мікобактеріоз C Поліоміеліт, гепатит А D Дизентерія, холера E Ку-гарячка, висипний тиф

2 При мікроскопії мікропрепарату з виділень хворої хронічним кольпо-вагінітом лікар виявив округлої форми та еліпсоподібні, що брунькуються клітини, розміром 3-6 мкм. Про збудника якої грибкової хвороби може йти мова в даному випадку? A *Кандидозу B Кокцидіозу C Епідермофітії D Мікроспорії E Криптококозу

3 У мікропрепараті виготовленому з пунктату регіонарного лімфовузла хворого, зафарбованому за Романовським-Гімза лікар виявив тонкі мікроорганізми з 12-14 рівномірними завитками з гострими кінцями довжиною 10-13 мкм блідо-рожевого кольору. Про збудника якої інфекційної хвороби може йти мова в даному випадку? A *Сифілісу B Трипаносомозу C Лептоспірозу D Поворотного тифу E Лейшманіозу

4 У хворої М. спостерігається локальна реакція на укус бджоли, що виникла в перші хвилини після укусу. За яким типом реакції гіперчутливості вона проходить? A *Анафілактичний B Цитотоксичний C Імунокомплексний D Уповільненого типу E Ідіотип-антиідіотип

5 У лабораторію поступило харкотиння хворого на туберкульоз. Який метод зафарбовування слід використати для виявлення збудників туберкульозу? A *Циля-Нельсена B Грам-Синьова C Гімзе-Романовського D Буррі-Гінса E Нейссера

6 При санітарно-бактеріологічному дослідженні водопровідної води отримані наступні результати: загальна кількість бактерій в 1,0 мл – 80, колі-індекс – 3. Як розцінити результат дослідження? A *Вода придатна для споживання B Вода є сумнівною C Вода є дуже сумнівною D Вода є забрудненою E Вода є дуже забрудненою

7 При санітарно-бактеріологічному дослідженні води методом мембранних фільтрів виявлено дві червоні колонії на мембранному фільтрі (середовище Ендо), через який пропустили 500 мл досліджуваної води. Розрахуйте колі-індекс та колі-титр досліджуваної води: A *4 та 250 B 2 та 500 C 250 та 4 D 500 та 2 E 250 та 2

8 У харкотинні хворого з підозрою на пневмонію виявлено грам позитивні диплококи, трохи подовжені з дещо загостреними протилежними кінцями. Які мікроорганізми виявлені у харкотинні? A *Streptococcus pneumoniae B Staphylococcus aureus C Klebsiella pneumoniae D Neisseria meningitidis E Neisseria gonorrhoeae

9 Хворому з підозрою на черевний тиф лікар-інфекціоніст призначив бактеріологічне дослідження крові. Доцільність цього

Page 63: крок 1 база 2014р_11шр-240ст (для брошури)_35грн

http://vk.com/my.printing

63

призначення пояснюється тим, що на першому тижні захворювання тифо-паратифами в хворого спостерігається: A *Бактеремія B Токсинемія C Септицемія D Септикопіємія E Вірусемія

10 З випорожнень хворого на гострий гастроентерит виділена чиста культура рухливих дрібних, дещо зігнутих грам негативних паличок, які впродовж 6 годин дають ріст на лужній 1% пептонній воді у вигляді ніжної голубуватої плівки. Яким мікроорганізмам притаманні такі властивості? A *Вібріонам B Спірохетам C Клостридіям D Бацилам E Спірилам

11 При плановому обстеженні доярок поставлено шкірно-алергічну пробу Бюрне. Вказана проба використовується для виявлення гіперчутливості до A *Бруцеліну B Туберкульну C Альттуберкуліну D Тулярину E Антраксину

12 У дитячому садку через кілька годин після вживання сирної маси майже у всіх дітей раптово з’явилися симптоми гастроентериту. При бактеріологічному дослідженні блювотних мас та залишків сирної маси було виділено золотистий стафілокок. Як доцільно продовжити дослідження для уточнення джерела інфекції? A *Провести фаготипування виділених штамів B Визначити здатність штамів до токсиноутворення C Провести дослідження обладнання харчоблоку D Вивчити наявність антитіл у хворих дітей E Поставити алергічну пробу

13

При мікроскопії харкотиння хворого на крупозну пневмонію виявлена значна кількість грампозитивних ланцетоподібних диплококів, оточених капсулою. Виявлення якого збудника слід очікувати? A *Streptococcus pneumoniae B Klebsiella pneumoniae C Chlamidia pneumoniae D Staphylococcus aureus E Escherichia coli

14 У бактеріологічній лабораторії підготували до стерилізації м’ясо-пептонний бульйон. Який із способів стерилізації слід застосувати? A *Автоклавування при 121oС 30 хв. B Сухим жаром 160oС 2 год. C Кип’ятінням 1 год. D Фільтрування E -

15 Лікар отоларинголог при огляді хворого відмітив гіперемію, значний набряк мигдаликів з сірим нальотом на них. При мікроскопії нальоту було виявлено грампозитивні палички розташовані під кутом одна до одної. Про яке захворювання слід думати? A *Дифтерія B Ангіна C Скарлатина D Менінгоназофарингіт E Епідемічний паротит

16 Хворий Н. поступив у лікарню зі скаргами на блювоту, запаморочення, двоїння в очах, важкість при ковтанні. Лікар запідозрив ботулізм. Які методи діагностики доцільно використати для підтвердження діагнозу? A *Біологічна проба, бактеріологічний B Алергічна проба, серологічний C Бактеріологічний, мікологічний D Протозоологічний, мікроскопічний E -

17 В інфекційне відділення лікарні госпіталізовано хворого зі скаргами на нудоту, рідкі випорож-нення зі слизом і прожилками крові, підвищення температури, слабкість.

Page 64: крок 1 база 2014р_11шр-240ст (для брошури)_35грн

64

Лікар запідозрив дизентерію. Який метод лабораторної діагностики найдоцільніше призначити для підтвердження діагнозу? A *Бактеріологічний B Серологічний C Мікологічний D Мікроскопічний E Протозоологічний

18 Людина, яка проживала в ендемічному вогнищі перехворіла 3-денною малярією. Після переїзду в іншу місцевість, через 1,5 року після переїзду захворла малярією знову. Яка найбільш вірогідна форма цього захворювання? A *Рецидив B Реінфекція C Суперінфекція D Персистуюча інфекція E Вторинна інфекція

19 У клініку доставлено хворого з важким загаль-ним станом, високою температурою, затрудне-ним диханням. Бактеріоскопічне дослідження матеріалу із зіву та дихальних шляхів дозволило попередньо діагностувати дифтерійний круп. Який метод фарбування при цьому був застосований? A *Нейссера B Циль-Нільсена C Гнса-Буррі D Пешкова E Ожешки

20 Хворий поступив в інфекційну лікарню на 8-й день зі скаргами на головний біль, недомагання, слабкість. Для серологічного дослідження взято кров. При проведенні реакція аглютинації Відаля встановлено, що вона позитивна в розведенні 1:200 з О-діагностикумом черевного тифу. Який діагноз можна встановити на підставі цього дослідження? A *Черевний тиф B Дизентерія C Холера D Лептоспіроз E Туберкульоз

21

При перевірці стану повітря в операційній перед операцією седиментаційним методом виявлено 5 дрібних округлих, навколо яких чітко було видно зону гемолізу. На яке середовище були зроблені посіви? A *Кров’яний МПА B МПА C Ендо D ЖСА E Левіна

22 В селищі К. у декількох господарствах було виявлено масову загибель щурів. Виникла підозра, що причиною може бути чума. Які постмортальні дослідження тварин слід провести з метою екстренного встановлення збудника інфекції? A * Реакція кільцепреципітації B Реакція аглютинації C Реакція пасивної аглютинації D Реакція зв”язування комплементу E Реакція нейтралізації

23 У пацієнта з ознаками коліту виділена чиста культура бактерій, яка за морфологічними, культуральними та біохімічними властивостями віднесена до роду шигел. Яку з названих реакцій доцільно застосувати для серологічної ідентифікації культури? A *Аглютинації з діагностичними сироватками B Зв’язування комплементу C Непрямої гемаглютинації D Преципітації E Затримки гемаглютинації

24 З метою перевірки крові донорів на наявність антигенів гепатиту В необхідно застосувати високочутливі методи .Яку з названих реакцій слід застосувати з вказаною метою? A *Твердофазний імуноферментний аналіз B Імуноелектрофорез C Непрямої гемаглютинації D Зв’язування комплементу E Непрямої імунофлуоресценції

25 Випорожнення хворого з підозрою на кишкову вірусну інфекцію обробили антибіотиками протягом доби при 40С. Потім суспензією заразили первинні та перещеплювані клітинні

Page 65: крок 1 база 2014р_11шр-240ст (для брошури)_35грн

http://vk.com/my.printing

65

культури. Через 2-3 дні в заражених клітинах культур виявлено цитопатичну дію. Як проводиться ідентифікація ентеровірусів? A * За допомогою реакції нейтралізації цитопатичної дії типоспецифічними енетровірусними сироватками. B За реакцією імунофлюоресценції. C За допомогою реакції гальмування гемаглютинації D За допомогою реакції аглютинації. E За допомогою реакції преципітації.

26 Пацієнт звернувся до лікаря на другому тижні хвороби, яка за клініко-епідеміологічними даними нагадувала тифо-паратифозне захворювання. Лікар вирішив підтвердити діагноз шляхом виявлення специфічних антитіл. Які препарати слід використовувати для цієї мети? A *Діагностикуми B Діагностичні сироватки C Мічені сироватки D Моноклональні антитіла E Адсорбовані монорецепторні сироватки

27 У лабораторію поступив матеріал /витяжка тваринницької сировини/ з району, де відмічені випадки на сибірку серед тварин. Яку серологічну реакцію необхідно застосувати для виявлення антигенів збудника в досліджуваному матеріалі? A *Реакцію термопреципітації B Реакцію зв’язування комплементу C Реакцію непрямої гемаглютинації D Радіоімунний аналіз E Реакцію преципітації в агарі

28 У зв’язку з випадком дифтерії виникла необхідність провести запобіжні щеплення в студентській групі. Який препарат слід використати для створення штучного активного імунітету? A *Дифтерійний анатоксин B Антидифтерійну сироватку C Специфічний імуноглобулін D Вакцина АКДП E Вакцина з вбитих бактерій

29 Реакція імунофлюоресценції широко

використовується для експрес-діагностики багатьох бактеріальних та вірусних інфекцій. Виберіть умову, без дотримання якої неможливо визначити результат реакції. A *Наявність люмінесцентного мікроскопа B Наявність електронного мікроскопа C Наявності імерсійного мікроскопа D Виділеної чистої культури збудника E Сироватки хворого

30 Від хворого з підозрою на черевний тиф виділено чисту культуру збудника, яку ідентифіковано за морфологічними, культуральними та біохімічними властивостями як сальмонелла тифу.Яке дослідження слід застосувати для остаточної ідентифікації збудника? A *Сероідентифікацію B Серодіагностику C Алергодіагностику D Антибіотикограму E Фаготипування

31 В хірургічний кабінет звернулась людина, яку покусала невідома собака. Широкі рвані рани локалізовані на обличчі. Яку лікувально-профілактичну допомогу потрівбно надати для профілактики сказу? A * Розпочати імунізацію антирабічною вакциною B Призначити комбіновану антибіотикотерапію C Терміново ввести вакцину АКДП D Госпіталізувати хворого і тримати під наглядом лікаря E Терміново ввести нормальний гама-глобулін

32 При спалаху гострої респіраторної інфекції з метою встановлення діагнозу грип проводиться експрес-діагностика, яка грунтується на виявленні специфічного вірусного антигену в досліджуваному матеріалі (змив носоглотки). Яку серологічну реакцію використовують для цього? A * Реакція імунофлюоресценції B Реакція зв’язування комплементу C Реакція аглютинації D Реакція преципітації

Page 66: крок 1 база 2014р_11шр-240ст (для брошури)_35грн

66

E Реакція опсонізації

33 Серологічна діагностика грипу передбачає виявлення наростання титру антитіл до збудника в сироватці крові хворого. В скільки разів повинен зрости титр антитіл з парною сироваткою, щоб результат вважався достовірним? A * В 4 рази і більше B В 2 рази C В один раз D В 3 рази E В пів-титру

34 Вірус грипу містить внутрішні антигени - нуклеопротеїдні (NP), полімеразні (P1, P2, P3), матриксний білок (М) та зовнішні антигени - гемаглютинін (Н) і нейрамінізазу (N). Яким з них належить основна роль у створені імунітету до грипозної інфекції? A * Гемаглютинін та нейрамінідаза B Нуклеопротеїдні антигени C Матриксний білок D Полімеразні білки E Нейрамінідаза

35 У пацієнтки 20 років встановлено діагноз - СНІД. Які популяції клітин найбільш чутливі до вірусу імунодефіциту людини? A * Т-хелпери B Гепатоцити C Ендотеліоцити D Епітеліоцити E В-лімфоцити

36 Відомо, що вірус імунодефіциту людини належить до родини Ретровірусів. Вкажіть основну ознаку, що характеризує дану родину. A * Наявність ферменту зворотньої транскриптази B Містять мінус-РНК C Прості віруси, що вражають тільки людину D Нуклеїнова кислота не інтегрує в геном хазяїна E Реакція імуноферментного аналізу для виявлення антигенів

37 У зв’язку з підозрою на внутрішньолікарняну інфекцію, проведено

обстеження у відділенні новонароджених пологового будинку. У кількох дітей, а також на деяких предметах догляду виявлено золотистий стафілокок. Які властивості виділених культур дають можливість встановити їхнє походження з одного джерела? A *Фаготип B Пігментоутворення C Антигенна структура D Біохімічна активність E Антибіотикограма

38 В закритому колективі виникла необхідність перевірити стан імунітету проти дифтерії, щоб обгрунтувати необхідність вакцинації. Які дослідження слід провести з такою метою? A *Встановити титр антитоксинів в РНГА B Перевірити членів колективу на носійство палички дифтерії C Встановити рівень антитіл проти дифтерійної палички D Перевірити медичну документацію щодо вакцинації E Перевірити стан імунітету щодо дифтерійної палички

39 Хлопчик 12 років знаходиться у лікарні з підозрою на харчову токсикоінфекцію. При посіві фекалій хворого на середовище Ендо виросла велика кількість безбарвних колоній. Який мікроорганізм можна з найбільшою імовірністю ВИКЛЮЧИТИ з числа можливих збудників захворювання? A * Escherichia coli B Salmonella enteritidis C Proteus vulgaris D Pseudomonas aeruginosa E Yersinia enterocolitica

40 Хлопчик 1,5 років, який не отримував планові щеплення, контактував з хворим на кір. З метою екстреної специфічної профілактики дитині було введено донорський гамаглобулін. Який вид імунітету було створено при цьому? A * Пасивний B Природний C Антитоксичний D Поствакцинальний E Місцевий

Page 67: крок 1 база 2014р_11шр-240ст (для брошури)_35грн

http://vk.com/my.printing

67

41 Лікар-педіатр, проводячи з батьками бесіду про профілактику кору, зауважив, що певна категорія дітей має природний пасивний імунітет до цього захворювання. Яких саме дітей мав на увазі лікар? A * Новонароджені B Старші 14 років C Ті, що перенесли кір на першому році життя D Ті, що отримали планові щеплення E Ті, чиї батьки не хворіли на кір

42 З метою діагностики тифо-паратифозного захворювання проведено реакцію аглютинації Відаля. Вона виявилася позитивною з черевнотифозним О-антигеном у розведенні 1:1600, з черевнотифозним Н-антигеном – у розведенні 1:200. Про що це свідчить? A * Про захворювання на черевний тиф. B Відсутність тифо-паратифозного захворювання. C Черевнотифозне бактеріоносійство. D Інкубаційний період черевного тифу. E Перенесений черевний тиф в анамнезі.

43 Істотним недоліком мікроскопічного методу діагностики інфекцій є його недостатня інформативність у зв’язку з морфологічною подібністю багатьох видів мікроорганізмів. Яка імунологічна реакція дозволяє значно підвищити інформативність цього методу? A * Реакція імунофлюоресценції. B Реакція Кумбса. C Імуноферментний аналіз. D Реакція опсонізації. E Радіофмунний аналіз.

44 У пацієнта з бронхіальною астмою за дпомогою шкірних алергічних проб встановлено сенсибілізацію алергеном тополиного пуху. Який фактор імунної системи відіграє вирішальну роль в розвитку цього імунопатологічного стану? A * IgE. B IgD. C IgM. D Сенсибілізовані Т-лімфоцити. E IgG.

45 Дитині віком 6 років, у якої запідозрено активний туберкульозний процес, проведено діагностичну реакцію Манту. Який імунобіологічний препарат при цьому було введено? A * Туберкулін. B Вакцину БЦЖ. C Вакцину АКДП. D Тулярін. E Вакцину АДП.

46 У хворого після вжитку в їжу консервованих грибів з’явилися симптоми бульбарного паралічу: птоз, двоїння в очах, афонія, порушення ковтання. Попередній діагноз: ботулізм. За допомогою якої реакції можна визначити тип токсину? A * Реакція нейтралізації B Реакція аглютинації C Реакція преципітації D Реакція зв’язування комплементу E Реакція імунно-флюорестенції

47 Хворому з великими опіками зробили пересадку донорської шкіри. Через 4-5 днів шкірний трансплантат прижився. Але на 8-му добу трансплантат набряк, змінився його колір і на 11 добу почав відторгатися. Які клітини приймають у цьому участь? A *Т-лімфоцити B Еритроцити C Базофіли D Еозинофіли E В-лімфоцити

48 Хворий 5 років скаржиться на сильний головний біль, блювоту. Об’єктивно: ригідність м’язів потилиці, блювота без попередження нудотою, герпетичні висипи на обличчі, лихоманка. На підставі бактеріологічних досліджень якого патологічного матеріалу можливе підтвердження попереднього діагнозу – цереброспінальний менінгіт ? A * Пункція спинно- мозгової рідини, яка витікає під тиском і має неприємний запах B Виділення урінокультур N.meningitidis C Виділення копрокультур N. Meningitidis D Дослідження блювоти E Виділення бактерій N. Meningitidis з

Page 68: крок 1 база 2014р_11шр-240ст (для брошури)_35грн

68

слизової оболонки сечостатевої системи

49 Бактеріологічне дослідження гнійних виділень з уретри з’ясувало присутність бактерій, які за Грамом фарбуваалися негативно, нагадували кавові зернини, розкладали глюкозу і мальтозу до кислоти. Розташовувалися в лейкоцитах. До збудників якої хвороби їх віднести? A * Гонореї B Сифілісу C Венеричного лімфогранулематозу D Мякого шанкру E Меліоідозу

50 Хворий 34 роки звернувся з приводу карбункулу на обличчі. Під час огляду : нещільний, без болю набряк підшкірної клітковини, у центрі карбункулу чорний струп, по периферії везикулярні висипання навколо карбункулу. Мікробіологічне дослідження з’ясувало наявність нерухомих стрептобацил, які здатні утворювати капсули. Які мікроорганізми є збудниками даної хвороби. A * Bacillus antracis B Staptylococcus aureus C Bacillus anthracoides D Bacillus subtilis E Bacillus megaterium

51 У хворого з жовтяницею одержано такі дані лабораторного обстеження: HbsAg-, НВеАg-, анти НВsAg+, анти НВsМ-, НСАg+. Який діагноз підтверджується на основі цих даних? A * Гепатит С, в анамнезі - гепатит В. B Гепатит С. C Хронічний гепатит В з низькою репликативною активністю. D Рецидив гепатиту В. E Повторне зараження гепатитом В.

52 Новонароджений, 20 днів, помер від правцю. Де з найбільшою імовірністю може бути знайдений збудник? A * Пупкова ранка B Спинний мозок

C Кров D Шлунково-кишковий тракт E М‘язи

53 Хворий поступив до iнфекцiйного вiддiлення з пiдозрою на холеру.Який основний метод дослiдження необхідно використати для пiдтвердження дiагнозу? A *Бактерiологiчний B lмунологiчний C Біологічний D Серологiчний E Алергiчний

54 З фекалій хворого виділені шигели Зонне. Які потрібно провести додаткові дослідження для встановлення джерела інфекції? A *Провести фаготипування виділеної чистої культури B Зробити антибіотикограму C Поставити реакцію преципітації D За допомогою реакції зв”язування комплементу E Поставити реакцію нейтралізації

55 На спеціальному живильному середовищі, після посіву виділення гною з уретри, виросли ніжні голубуваті колонії. При мікроскопії препаратів з них виявленні грамнегативні бобовидні диплококи. Збудником якої хвороби вони є? A *Гонореї B Хламідіозу C Сифілісу D Туляремії E Меліоїдозу

56 Під час експертизи шкір тварин була використана реакція преципітації по Асколі. При обліку результатів через декілька хвилин після поєднання імунної сироватки та екстракту шкіри було відмічено утворення білуватого кільця. Про що свідчить даний результат? A *Наявність антигенів сибірки B Наявність токсину анаеробної інфекції C Наявність збудника бруцельозу D Поверхневого антигену ешеріхій E Вірулентного антигену сальмонел

Page 69: крок 1 база 2014р_11шр-240ст (для брошури)_35грн

http://vk.com/my.printing

69

57 Серед дітей школи-інтернату мають місце випадки захворювання ангіною. При мікроскопії мазків з мигдаликів, зафарбованих по методу Нейсера знайдені тонкі палички жовтого кольору з темно-коричневими зернами на кінцях, що розміщуються в вигляді римської цифри п’ять. Яку інфекцію можна запідозрити в цьому випадку? A *Дифтерію B Інфекційний мононуклеоз C Лістеріоз D Тонзиліт E Скарлатину

58 В бактеріологічну лабораторію направлено харкотиння хворого на туберкульоз, Для бактеріоскопічного дослідження препаратів – мазків і виявлення туберкульозної палички потрібно використати один із вказаних методів фарбування: A *Ціля - Нільсена B Буррі - Гінса C Здрадовського D Грама E Романовського

59 В бактеріологічній лабораторії досліджувалась в’ялена риба домашнього виготовлення, яка стала причиною важкого харчового отруєння, При мікроскопії виділеної на середовищі Кітта-Тароці культури виявлені мікроорганізми схожі на тенісну ракетку. Який діагноз встановить лікар? A *Ботулізм B Сальмонельоз C Холера D Дизентерія E Черевний тиф

60 Для серодіагностики черевного тифу ставлять реакцію, при якій до різних розведень сироватки хворого добавляють діагностикуми трьох видів мікроорганізмів і результат якої оцінюють по наявності осаду із склеєних бактерій. Ця реакція відома під назвою: A *Відаля B Борде - Жангу

C Вассермана D Райта E Асколі

61 Фекалії дитини, що хворіє на ентерит емульгують в фізіологічному розчині і краплю емульсії наносять на елективне середовище: 10\% молочно – сольовий, або жовточно – сольовий агар. Які мікроорганізми передбачається виділити? A *Стафілококи B Кишкову паличку C Стрептококи D Клебсієли E Ентерококи

62 Для прискорення заживлення рани слизової оболонки в ротовій порожнині хворому призначено препарат, який являє собою термостабільний білок, що міститься у людини в сльозах, слині, грудному молоці матері, а також його можна виявити в свіжознесеному курячому яйці. Відомо, що він являє собою фактор природної резистентності організму і має назву: A *Лізоцим B Комплемент C Інтерферон D Інтерлейкін E Іманін

63 Лікар - педіатр при огляді трьохмісячнї дитини відмітив, що слизова оболонка порожнини рота, а також язик вкриті щільним білим нальотом. В матеріалі, взятому з місця ураження, бактеріолог виявив наявність грибів дріжжеподібної форми, в зв’язку з цим запідозрили мікоз: A *Кандідоз B Фавус C Епідермофітія D Актиномікоз E Трихофітія

64 До лікарні надійшла дитина з діагнозом “стафілококовий сепсис”. На яке живильне середовище потрібно посіяти кров хворого

Page 70: крок 1 база 2014р_11шр-240ст (для брошури)_35грн

70

з метою виділення збудника? A * Цукрово-пептонний бульйон B М’ясо-пептонний агар C Середовище Плоскірьова D Середовище Бучіна E Жовчно-сольовий агар

65 Доярка у розпал епідемії грипу звернулась до лікаря зі скаргами на високу температуру, загальну слабкість, відсутність апетиту, біль в суглобах. Протягом 10 діб вона лікувалась з діагнозом “Грип”. Але інфекціоніст запідозрив в неї бруцельоз. Якою реакцією можна поставити остаточний діагноз бруцельоз? A * Райта B Хедльсона C Імунофлюорестенції D Відаля E Оухтерлоні

66 У посіві гною з фурункулу знайдено кулястої форми мікроби, розташовані як “гроно” винограду. Які мікроби виявлено? A * Стафілококи B Диплококи C Мікрококи D Стрептококи E Тетракоки

67 У дитячому садку здійснено обстеження дітей і персоналу з метою виявлення менінгококового носійства. Підберіть метод мікробіологічного дослідження: A * Бактеріологічний B Алергічний C Бактеріоскопічний D Біологічний E Серологічний

68 У студента після вживання м(яса, консервованного в домашніх умовах, з(явились симптоми харчового отруєння Clostridium botulinum: диплопія, порушення мови та параліч дихання. Чим обумовлені такі симптоми ботулізму? A * Дією нейротоксину B Інвазією Cl. вotulinum в епітелій кишківника C Секрецією ентеротоксина

D Ендотоксичним шоком E Активацією аденілатциклази

69 З центрифугату порції сечі, одержаної від хворого з підозрою на туберкульоз нирок, приготували препарат для мікроскопії. Який метод фарбування препарату використовують для виявлення збудника? A * За Цілем-Нильсеном B За Бурі C За Грамом D За Леффлером E За Ожешко

70 В лабораторію особливо небезпечних інфекцій поступили випорожнення хворого з діагнозом “холера”. Який метод мікробіологічної діагностики потрібно використати, щоб підтвердити або відхилити діагноз? A * Бактеріологічний B Алергічний C Бактеріоскопічний D Біологічний E Вірусологічний

71 Збудник гепатиту Д (Дельта-агент) є дефектним вірусом, який може реплікуватись лише в клітинах, уже інфікованих одним з перерахованих вірусів: A * Вірусом гепатиту В B Вірусом гепатиту А C Вірусом гепатиту Е D Вірусом Епстайна-Барр E Вірусом імунодефіциту людини

72 При перевірці крові донорів на станції переливання крові, в сироватці одного з них виявлені антитіла до вірусу імунодефіциту людини. Який метод рекомендується для підтвердження діагнозу ВІЛ-інфекції? A *Вестернблоту (імуноблотингу) B Електронної мікроскопії C Імуноферментного аналізу D Імунофлюоресценції E Радіоімунного аналізу

73 Хворого 55-ти років госпіталізовано у хірургічну клініку з підозрою на сепсис. Який матеріал для дослідження необхідно взяти від хворого і на яке середовище його слід засіяти?

Page 71: крок 1 база 2014р_11шр-240ст (для брошури)_35грн

http://vk.com/my.printing

71

A *Кров, цукровий бульон B Ліквор, сироватковий агар C Сечу, м’ясо-пептонний бульон D Гній, жовточно-сольовий агар E Пунктат лімфовузла, цистеїновий агар.

74 Хворий поступив в інфекційну клініку з попереднім діагнозом “Черевний тиф?”. Відчуває себе хворим протягом трьох днів. Використання, якого метода, дасть змогу підтвердити діагноз? A *Виділення гемокультури B Виділення копрокультури C Виділення уринокультури D Виділення білікультури E Виділення розеолокультури

75 При забарвленні бакпрепаратів, виготовлених з мокротиння, методом Ціля-Нільсена виявлено наявність яскравочервоних паличок, які розміщувалися поодиноко або групами, не чутливі до дії кислот. На живильних середовищах перші ознаки росту з'являються на 10-15-ту добу. До якої родини відносяться бактерії? A *Micobacterium tuberculosis B Versinia pseudotuberculosis C Histoplasma dubrosii D Klebsiella rhinoscleromanis E Coxiella burnettii

76 В лабораторію направлено матеріал білуватого нашарування із слизових оболонок ротової порожнини. Висів матеріалу зроблено на середовище Сабуро, відмічено ріст сметаноподібних колоній, бактеріоскопія виявила короткі бруньковані нитки, до збудників якої інфекції відносять ізольовані мікроорганізми ? A * Мікози B Спірохетози C Рікетсіози D Мікоплазмози E Хламідіози

77

Бактеріолог при дослідженні слизу з носоглотки, дотримувася певних заходів щодо збереження збудників у матеріалі. Під час бактереоскопічного дослідження встановлено наявність грамнегативних коків, які нагадують кавові зерна та розташовані парами або тетрадами. Назвіть збудника, який був ізольований бактеріологом: A *Neisseria meningitidis B Staphilococcus aureus C Neisseria gonorrhoeae D Moraxella lacunata E Acinetobacter calcoaceticus

78 При обстеженні хворого чоловiка, госпіталызованого на 5-й день хвороби з проявами жовтяниці, болі в м’язах, ознобом,. носовими кровотечами. При проведені лабораторної діагностики бактеріолог виконав темнопольну мікроскопію краплини крові хворого. Назвіть збудників хвороби: A *Leptospira interrogans B Borrelia dutlonii C Calymmatobacterium granulomatis D Bartonella bacilloformis E Rickettsia mooseri

79 ВІЛ-інфікований пацієнт періодично обстежується з метою виявлення ознак активізації процесу. Назвіть найбільш суттєву ознаку, що вказує на перехід ВІЛ-інфекції в СНІД: A * Саркома Капоші, кількіть Т-хелперів нижче 200 кл/мкл B Зниження кількості нейтрофілів C Зниження кількості Т-хелперів D Кількість Т-хелперів нижче критичного рівня E Виявлення антитіл до gp 41

80 В інфекційну лікарню поступив хворий з ознаками пневмонії, яка розвинулась на 6-й день захворювання грипом.Який метод найдостовірніше підтверджує грипозну етіологію пневмонії? A *Виявлення антигенів віруса грипу в

Page 72: крок 1 база 2014р_11шр-240ст (для брошури)_35грн

72

харкотинні методом ІФА. B Дослідження парних сироваток. C Зараження курячих ембріонів. D Імунолюмінісцентне дослідження мазків-відбитків з носових ходів. E Виявлення антитіл проти гемаглютинінів вірусу грипу.

81 У зв'язку з важким перебігом гепатиту В, пацієнту призначено обстеження з метою виявлення можливого агента-супутника, що ускладнює перебіг основного захворювання. Вкажіть цей агент. A * Дельта-вірус. B Вірус гепатиту С. C Вірус гепатиту G. D Вірус гепатиту Е.. E НВs- антиген.

82 При бактериологічному дослідженні фекалій жінки 38 років, яка 1,5 років тому перенесла черевний тиф, було виявлено Salmonella typhi у кількості 102/г. Як найбільш точно охарактеризувати стан обстеженої жінки? A * Бактерионосійство B Дисбактериоз C Реінфекція D Суперінфекція E Рецидив

83 Після введення вакцини БЦЖ немовлятам імунітет до туберкульозу триває доти, доки в організмі є живі бактерії вакцинного штаму. Як найбільш правильно назвати такий вид імунітету? A * Нестерильний B Гуморальний C Типоспецифічний D Природжений E Перехресний

84 У чоловіка 36 років з підозрою на туберкульоз нирок було проведене дослідження осаду сечі. При мікроскопії знайдено кислотостійкі бактерії, але за методом Прайса корд-фактор не виявлений. Яке дослідження дозволить найбільш надійно підтвердити або спростувати попередній діагноз? A * Зараження лабораторних тварин B Вивчення токсигенності C Фаготипування виділеної культури

D Серологічна ідентифікація збудника E Шкірна алергічна проба

85 Оглядаючи дитину 6-ти років лікар помітив на глоткових мигдалинах сірувату плівку, при спробі видалення якої виникла помірна кровотеча. Бактеріоскопія мазків з мигдалин показала наявність грампозитивних бактерій булавоподібної форми. Які симптоми можуть виникнути у дитини у найближчі дні, якщо не буде проведене специфічне лікування? A * Токсичні ураження серцевої м‘язи, печінки, нирок B Набряк легенів C Дуже сильний нападоподібний кашель D Папульозні висипи на шкірі E Хвильоподібна лихоманка

86 У хлопчика 12 років після перенесеної ангіни розвинулося ревматичне ураження серця. Кожна наступна стрептококова інфекція погіршує стан хворого. Який препарат доцільно використати для профілактики ускладнень? A * Пеніцилін B Стрептококовий анатоксин C Стрептококовий бактеріофаг D Донорський гама-глобулін E Аутовакцина

87 У дитини 2-х років з катаральними явищами та висипом на шкірі лікар запідозрив скарлатину. Внутришкірно дитині було введено невелику кількість сироватки до еритрогенного токсину стрептокока, на місці ін'єкції висип зник. Що означають результати реакції? A * Клінічний діагноз підтвердився B У дитини підвищена чутливість до еритрогенного токсину C Захворювання викликав негемолітичний стрептокок D Всю дозу сироватки можна вводити внутривенно E Імунна система дитини дуже ослаблена

88 Для серологічної діагностики сифілісу в реакції Вассермана лікар-лаборант підготував

Page 73: крок 1 база 2014р_11шр-240ст (для брошури)_35грн

http://vk.com/my.printing

73

такі реактиви: кардіоліпіновий антиген, спиртовий екстракт ліпоідів із серцевої м‘язи бика з холестерином, антиген з трепонем, зруйнованих ультразвуком, гемолітична система, фізіологічний розчин, досліджувані сироватки. Який ще компонент необхідний для постановки реакції? A * Комплемент B Живі трепонеми C Еритроцити барана D Діагностична преципітуюча сироватка E Антиглобулінова сироватка

89 У чоловіка 40 років ушкоджені міжпальцеві проміжки на ногах: шкіра мокне, відшаровується, з‘явилися тріщини. При посіві зішкрябу шкіри на середовище Сабуро виросли пухнасті колонії, білі зверху та зеленувато-жовті внизу. У мазках з верхньої частини колоній видно конідії у вигляді “дубинок” з 1-5 клітинами. Які ще органи найбільш імовірно може уразити цей збудник? A * Нігті B Волосся C Підшкірна клітковина D Лімфатичні судини E Слизова статевих шляхів

90 У зв‘язку з тим, що наближається епідемія грипу, районний епідеміолог складає заявку на профілактичні препарати. Який з них сприятиме формуванню активного специфічного імунітету та є найменш реактогенним? A * Субодинична вакцина B Жива вакцина C Вбита вакцина D Донорський гама- глобулін E Лейкоцитарний інтерферон

91 У хлопчика 6 років помірне помірне підвищення температури, привушні залози збільшені. Зі слини хворого було виділено вірус, який розмножується у курячих ембріонах та тканинних культурах, має гемаглютинуючі властивості та викликає утворення симпластів у культурі клітин. Які ще органи найбільш

імовірно можуть бути уражені в наслідок інфекції, викликаної цим вірусом? A * Статеві залози B Печінка C Легені D Глоткові мигдалини E Головний мозок

92 У хворої дитини з явищами гнійного керато-кон'юнктивіту лікар-офтальмолог запідозрив бленорею (гонорейний кон'юнктивіт). Якими методами лабораторної діагностики слід скористатися для підтвердження діагнозу? A * Мікроскопічним та бактеріологічним B Серологічним та алергічним C Біологічним та методом фагодіагностики D Біологічним та алергічним E Мікроскопічним та серологічним

93 Хворій назо-фарингітом дитині лікар поставив діагноз "менінгококовий назо-фарингіт". Який метод лабораторної діагностики найбільш раціональний для підтвердження діагнозу? A * Бактеріологічний B Біологічний C Серологічний D Мікроскопічний E Алергічний

94 Від хворої дитини на церебро-спінальний менінгіт отримано спинномозкову рідину мутного характеру, яка вміщує велику кількість лейкоцитів. Якою із серологічних реакцій слід скористатися для експрес-діагностики захворювання? A * Реакцією преципітації B Реакцією аглютинації C Реакцією зв'язування коиплементу D Реакцією гемаглютинації E Реакцією нейтралізації

95 Із рото-глотки хлопчика, який хворіє на хронічний тонзиліт виділили культуру кокових бактерій. У мазках вони розташовувалися у вигляді ланцюжків. Які це можуть бути бактерії? A * Стрептококи B Стафілококи C Ешерихії

Page 74: крок 1 база 2014р_11шр-240ст (для брошури)_35грн

74

D Клостридії E Вібріони

96 Від хворого з діагнозом дизентерія було виділено шигелу зі здатністю продукувати екзотоксин. Про який вид шигел йде мова? A * Шигела дизентерії B Шигкла Зоне C Шигела Флекснера D Шигела Бойда E Шигела Нью-Кастла

97 У мазках, як. були виготовлені з харкотиня хворого на туберкульоз легень мікобактерій не виявлено. Яким методом можна підвищити імовірність виявлення мікобактерій в харкотині? A * Гомогенізації і флотації B Прайса і Школьнікової C Темнопільна мікроскопія D Мікроскопія препаратів, пофарбованих за Циль-Нільсеном E Мікроскопія нативних мікропрепартів

98 При мікроскопічному дослідженні нативного препарату з випорожнень хворого, які мають кров'яно-слизистий характер виявлені мікроорганізми округлої форми, в цитоплазмі яких містяться еритроцити, а також цисти дрібних розмірів з 4 ядрами. Про який збудник йде мова? A * Entamoeba hystolitica B Entamoeba coli C Lamblia intestinalis D Trichomonas intestinalis E Leichmania donovani

99 З випорожнень хворої дитини 6-місячного віку, яка знаходилась на штучному годуванні, виділена культура кишкової палички з антигенною структурою 0-111. Який діагноз можна поставити? A *Колі-ентерит B Гастро-ентерит C Холероподібне захворювання D Харчове отруєння E Дизентерієподібне захворювання

100

Хворому на туберкульоз, в акнамнезі якого була відкрита легенева форма захворювання, проведено мікроскопічне дослідження мокротиння, з метою виявлення збудника. Який метод забарвлення доцільно використати при цьому? A *Метод Ціля-Нільсена B Метод Грама C Метод Буррі-Гінса D Метод Романовського-Гімза E Метод Нейссера

101 Хворому з великими опіками зробили пересадку донорської шкіри. На 8-му добу трансплантат набряк, змінився його колір; на 11 добу почав відторгатися. Які клітини приймають у цьому участь? A *Т-лімфоцити B Ерітроцити C Базофіли D Єозінофіли E В-лімфоцити

102 Держсанінспекцією вилучена на базарі партія ковбаси з назвою "свиняча домашня" в зв'язку з підозрою щодо її фальсифікації. За допомогою якої серологічної реакції імунітету можна ідентифікувати харчовий продукт? A *Преципітації B РЗК C Аглютинації D Імунофлюорестенції E РНГА

103 При дослідженні гнійних виділень з шийки матки бактеріоскопічно виявлено присутність грамнегативних бобовидних диплококків, які знаходилися як в середині, так і поза лейкоцитами. Назвіть чинника гнійного запалення шийки матки ? A *Neisseria gonorroeae B Chlamidia trachomatis C Haemophilus vaginalis D Trichomonas vaginalis E Calymmatobacterium granulomatis

104 У дитини 4 років спостерігаються клінічні ознаки коклюшу. З метою серологічної діагностики була поставлена розгорнута

Page 75: крок 1 база 2014р_11шр-240ст (для брошури)_35грн

http://vk.com/my.printing

75

реакція з коклюшним та паракоклюшним діагностикумами. На дні пробірок, до яких було внесено діагностикум з Bordetella parapertussis, утворився зернистий осад. Які антитіла виявила ця реакція? A * Аглютиніни B Преципітини C Опсоніни D Бактеріолізіни E Антитоксини

105 До інфекційного відділення госпіталізовано хворого зі скаргами на багаторазовий пронос та блювоту, біль у м'язах ніг, слабкість, запаморочення. Після огляду лікар виставив попередній діагноз "холера". Як необхідно досліджувати матеріал від хворого для експрес діагнозу? A * Пряма і непряма РІФ B РА C Бактеріологічним методом D Серологічним методом E Біологчним методом

106 Хворий тривалий час лікується з приводу хронічної пневмонії. При мікроскопічному дослідженні мокротиння в мазках, зафарбованих за методом Ціля - Нільсена, виявлено червоні палички розмірами 0,25ґ4 мкм, розташован поодиноко, іноді невеликими скупченнями. Яке захворювання у пацієнта? A *Туберкульоз легень B Пневмококова пневмонія C Актиномікоз легень D Грипозна пневмонія E Кандідоз легень

107 Під час хірургічної операції пацієнту проведено переливання крові. На антигени якого збудника необхідно перевірити цю кров ? A *Віруса гепатиту В. B Віруса гепатиту А. C Вірус гепатиту Е. D Ентеровірусів. E Аденовірусів.

108 Через декілька хвилин після введення препарату хворому правцем з'явилася

задишка, частий пульс, впав артеріальний тиск. Який препарат міг бути найбільш імовірною причиною виниклого ускладнення? A *Антитоксична сироватка B Сульфаниламид C Антибіотик D Анатоксин E Донорський гама- глобулін

109 В інфекційне відділення поступив хворий з підозрою на дизентерію. Який основний із наведених методів лабораторної діагностики необхідно назначити? A *Бактеріологічний. B Серологічний. C Алергічний. D Біологічний. E Мікроскопічний.

110 До лабораторії доставлена кров хворого на черевний тиф для виявлення антитіл. Яку із перелічених серологічних реакцій слід для цього застосувати? A *Реакцію аглютинації B Реакцію преципітації C Реакцію зв'язування комплементу D Реакцію гемаглютинації E Реакцію гальмування гемаглютинації

111 У хворого з підозрою на дифтерію під час бактеріоскопічного дослідження мазка з зіву виявлені паличкоподібні бактерії з зернами волютину. Який етіотропний препарат препаратом вибору в даному випадку? A *Протидифтерійна антитоксична сироватка B Бактеріофаг C Дифтерійний анатоксин D Еубіотик E Інтерферон

112 У хворого з типовою клінічною картиною дизентерії, внаслідок раннього застосування антибіотиків під час бактеріологічного дослідження випорожнення шигелли не виявлені. Тітр антишигелльозних антитіл в РПГА у парних сироватках у даного хворого виріс в 4 рази. Про що це свідчить? A *Підтверджує діагноз дизентерії. B Виключає діагноз дизентерії.

Page 76: крок 1 база 2014р_11шр-240ст (для брошури)_35грн

76

C Переніс дизентерію раніше. D Неспецифічна реакція. E Вакцинальнареакція.

113 У померлого від гострого інфекційного захворювання, яке супроводжувалось гарячкою, жовтяницею, геморагічною висипкою на шкірі і слизових оболонках, а також гострою нирковою недостатністю, при гістологічному дослідженні тканини нирки (забарвлення за Романовськом-Гімзою) виявлені звивисті бактерії, які мають вигляд букв С та S. Які бактерії були виявлені? A *Лептоспіри B Трепонеми C Спіролли D Боррелії E Кампілобактерії

114 При бактерiологiчному дослiдженнi сечi хворого пiєлонефритом видiленi мiкроорганiзми, що утворюють на мясо-пептонному агарi жовто-зелений пiгмент i характерний запах. Як вони називаються? A *Псевдомонади B Ешерiхiї C Протеї D Клебсiєли E Азотобактерiї

115 В бактерiологiчну лабораторiю з вогнища харчового отруєння доставлена в'ялена риба, при дослiдженнi якої на середовищі Кiтта-Тароцi бактерiолог виявив мiкроорганiзми подiбнi до “тенiсної ракетки”. Збудником якоi хвороби вони могли бути? A *Ботулiзму B Сальмонельозу C Дизентерiї D Стафілококової токсикоінфекції E Черевного тифу

116 До ПТУ вступив юнак 16 років з сільської місцевості. При плановому проведенні реакції Манту виявилося, що у цього юнака вона негативна. Яка найбільш раціональна тактика лікаря? A *Зробити щеплення БЦЖ

B Повторити реакцію через 1 місяць C Провести серодіагностику туберкульозу D Терміново ізолювати юнака з навчального колективу E Провести прискорену діагностику туберкульозу методом Прайса

117 Бактеріологічне дослідження гнійних виділень з уретри з’ясувало присутність бактерій, які за Грамом фарбуваалися негативно, нагадували кавові зернини. Розташовувалися в лейкоцитах. До збудників якої хвороби їх віднести? A *Гонорея B Сифіліс C Венеричний лімфогранулематоз D М'який шанкр E Трихомонадоз

118 При бактеріологічному дослідженні взірців сметани ізольовані культури S. aureus. Як довести етіологічне значення ізольованої культури S. aureus збудником харчового отруєння, яке виникло серед групи споживачів сметани? A *Виявлення ентеротоксину. B Визначення палзмокоагулазної активності C Визначення гемотоксинів D Визначення цукролітичних властивостей E Визначення лецитіназної активності

119 При микроскопии соскоба с языка, окрашенного по Граму, обнаружены овальные, круглые, удлиненные цепочки почкующихся клеток темно-фиолетового цвета. О возбудителе какого заболевания может идти речь? A *Кандидоза. B Актиномикоза. C Стрептококковой инфекции. D Стафилококковой инфекции. E Дифтерии

120 После лабораторного обследования больного с часто рецидивирующими вирусными, бактериальными и грибковыми оппортунистическими инфекциями поставлен диагноз "ВИЧ-инфекция". Результаты какого исследования позволили поставить такой

Page 77: крок 1 база 2014р_11шр-240ст (для брошури)_35грн

http://vk.com/my.printing

77

диагноз? A *Иммуноферментный анализ. B Реакция связывания комплемента. C Реакция преципитации в геле. D Реакция торможения гемагглютинации. E Реакция пассивной гемагглютинации.

121 У ребенка с подозрением на дифтерию из зева выделена чистая культура микроорганизмов и изучены их морфологические, тинкториальные, культуральные и биохимические свойства, оказавшиеся типичными для возбудителей дифтерии. Какое исследование необходимо еще провести для выдачи заключения о том, что выделена патогенная дифтерийная палочка? A *Определение токсигенных свойств. B Определение протеолитических свойств. C Определение уреазной активности. D Определение цистиназной активности. E Определение способности расщеплять крахмал

122 При бактериологическом исследовании рвотных масс больных выделены грам-отрицательные палочки средних размеров с закругленными концами, подвижные, агглютинирующие с сальмонеллезной О-сывороткой группы В. Идентичные микроорганизмы обнаружены в мясном салате, который накануне употребляли все заболевшие. О возбудителях какого заболевания можно думать в данном случае? A *Сальмонеллы - возбудители острого гастроэнтерита. B Сальмонеллы - возбудители брюшного тифа. C Сальмонеллы - возбудители паратифа А. D Эшерихии - возбудители пищевой токсикоинфекции. E Протеи - возбудители пищевой токсикоинфекции.

123 Из кала и рвотных масс от больного с подозрением на холеру были выделены культуры вибрионов. Проведение какой реакции позволит определить вид микроба, вызвавшего

это заболевание? A *Агглютинации с сыворотками, содержащими О-антитела. B Агглютинации с сыворотками, содержащими Н-антитела. C Пассивной гемагглютинации с эритроцитарным антигенным диагностикумом. D Агглютинации Видаля. E Преципитации.

124 Из гнойной раны больного выделен патогенный стафилококк и определена чувствительность его к антибиотикам: пенициллин - зона задержки роста 8 мм; оксациллин - 9 мм; ампициллин - 10 мм; гентамицин - 22 мм; линкомицин - 11 мм. Какой антибиотик необходимо выбрать для лечения больного? A *Гентамицин B Оксациллин C Ампициллин D Пенициллин E Линкомицин

125 У дитини на слизовiй оболонцi щiк та на язицi виявленi бiлуватi плями, якi нагадують молоко, що скипілося. У виготовлених препаратах-мазках знайденi Грам+ овальнi дрiжджоподiбнi клiтини. Якi це збудники? A *Гриби роду Кандiда B Стафiлококи C Дифтерiйна паличка D Актиноміцети E Фузобактерії

126 В лабораторiї була визначена чутливість стафiлокока до антибіотиків i отримані наступні результати дослідження - діаметр зон затримки росту дорівнює: пенiцилiн - 8 мм, оксацилiн - 8 мм, ампiцилiн - 25 мм, гентамiцин - 22 мм. Який метод дослідження був використаний? A *Метод паперових дисків B Метод серійних розведень C Біохімічний D Бактерiоскопiчний E Біометричний

127 При мікроскопії харкотиння хворого з

Page 78: крок 1 база 2014р_11шр-240ст (для брошури)_35грн

78

попереднім діагнозом "гостра пневмонія" виявлено хаотично розташовані мікроорганізми овоїдної форми довжиною до 2 мкм, інтенсивніше забарвлені на полюсах. Який найбільш імовірний діагноз можна встановити на підставі отриманих даних? A *Легенева форма чуми B Пневмококова пневмонія C Стафілококова пневмонія D Клебсієльозна пневмонія E Дифтерія

128 У приймальному відділенні лікарні відбирають матеріал для бактеріологічного дослідження. З якою метою слід взяти матеріал у хворого з гнійним ураженням глибоких тканин нижньої кінцівки? A *Для встановлення етіології гнійного процесу і визначення чутливості до антибіотиків B Для виявлення патогенного стафілокока і визначення антибіотикограми C Для виявлення збудника, щоб попередити внутрішньолікарняне інфікування D Для підтвердження анаеробної інфекції E Для виявлення токсичності збудника

129 Кровь, взятую у больного с подозрением на сепсис, посеяли на сахарный бульон. В сахарном бульоне образовался придонный осадок. При пересеве на кровяной агар выросли мелкие, прозрачные, круглые колонии, окруженные зоной гемолиза. В мазке, приготовленном из осадка, определялись грам-положительные кокки, располагающиеся в виде длинных цепочек. Какие микроорганизмы присутствуют в крови у этого больного? A *Стрептококки B Микрококки C Стафилококки D Тетракокки E Сарцины

130 В одному з гірських селищ мала місце масова загибель гризунiв. Одночасно хворіло населення цієї мiсцевостi. Хвороба супроводжувалася швидким підвищенням to

до 40oС, вираженою інтоксикацією, збільшенням пахових лiмфовузлiв. У препаратах-мазках з трупного матеріалу виявлені грам-негативні палички овоїдної форми з біполярним забарвленням. Якi мікроорганізми є збудниками цього інфекційного захворювання? A *Паличка чуми B Стафілокок C Збудник туляремії D Збудник сибірки E Клостридії

131 Для оцінки придатності води для пиття проведено бактеріологічне дослідження. Який показник характеризує кількість бактерій групи кишкових паличок, що знаходяться в 1 л води? A *Колі-індекс B Колі-титр C Титр колі-фага D Перфрінгенс-титр E Мікробне число

132 При вірусоскопії клітинного моношару, зараженого інфекційним матеріалом, лікар-лаборант поставив діагноз - респіраторно-синцитіальна вірусна інфекція. Які зміни викликає цей вірус у культурі клітин? A *Утворення багатоядерних клітин B Круглоклітинна дегенерація C Тотальна деструкція клітинного моношару D Наявність тілець Бабеша-Негрі E Відшарування моношару

133 У хворого 25 років з численних шкірних пустул висівається золотистий стафілокок в асоціації з епідермальним стафілококом, в аналізі харкотиння виявлена пневмоциста карінії, у випорожненнях - криптоспоридії, вульгарний протей та гриби роду кандіда. При якому захворюванні зустрічається таке множинне інфікування умовно-патогенними мікроорганізмами? A *СНIД B Цукровий діабет C Сепсис D Дисбактеріоз

Page 79: крок 1 база 2014р_11шр-240ст (для брошури)_35грн

http://vk.com/my.printing

79

E Медикаментозний агранулоцитоз

134 У хворого на виразкову хворобу шлунка при проведенні фіброгастроскопії взятий біоптат слизової оболонки в області виразки. З біоптату виготовлений мазок-відбиток, пофарбований за методом Грама; з рештою біоптату проведена проба на уреазну активність. Під час мікроскопії мазка-відбитка виявлені Грам-негативні спіралеподібні мікро-організми, тест на уреазну активність - позитивний. Які бактерії були виявлені? A *Helycobacter pylori B Spirilla minor C Shigella flexneri D Treponema pallidum E Campylobacter jeuni

135 При бактеріологічному дослідженні випорожнень чотиримісячної дитини з симптомами гострої кишкової інфекції на середовищі Ендо виросли у великій кількості червоні колонії. Які це можуть бути мікроорганізми? A *Ешерихії B Сальмонели C Стафілококи D Стрептококи E Шигели

136 Хворому 50 років з метою лікування черевного тифу призначений левоміцетин, але на наступний день стан хворого погіршився, температура підвищилася до 39,6oС. Чим пояснити погіршення стану хворого? A *Дією ендотоксинів збудника B Алергічною реакцією C Нечутливістю збудника до левоміцетину D Приєднанням вторинної інфекції E Реінфекцією

137 У дитини, що одужує після кору, развинулася пневмонія, викликана умовно-патогенними бактеріями. Яка найбільш імовірна форма цієї інфекції? A *Вторинна інфекція B Реінфекція C Суперінфекція

D Персистуюча інфекція E Госпітальна інфекція

138 До лікаря-інфекціоніста на прийом прийшов хворий зі скаргами на пропасницю, яка триває три дні, загальну слабість, безсоння, погіршення апетиту. Лікар запідозрив черевний тиф. Який метод лабораторної діагностики найдоцільніше призначити для підтвердження діагнозу? A *Виділення гемокультури B Виділення копрокультури C Виділення уринокультури D Виділення білікультури E Виділення мієлокультури

139 У хлопчика 7 років - холероподібне захворювання (блювота, профузна діарея). При посіві фекалій хворого на середовище Ендо виросли однотипні колонії: малинового кольору, з металевим блиском. Який мікроорганізм є найбільш імовірним збудником захворювання? A * ентеротоксигенна Escherichia coli B Salmonella enteritidis C Yersinia enterocolitica D Shigella sonnei E НАГ-вібріон

140 Територію старого худобомогильника, який не використовувався більше 50 років, планується відвести під житлове будівництво. Однак дослідження грунта показало наявність життєздатних спор збудника особливо небезпечного захворювання. Який із вказаних мікроорганізмів найбільш імовірно міг зберігатися у грунті протягом такого тривалого часу? A * Bacillus anthracis B Francisella tularensis C Brucella abortus D Yersinia pestis E Mycobacterium bovis

141 У лікарні вирішено проводити контроль якості стерилізації інструменту в автоклаві за допомогою біологічного методу. Які

Page 80: крок 1 база 2014р_11шр-240ст (для брошури)_35грн

80

мікроорганізми найбільш доцільно використати як тест - культури? A * Спорові B Капсульні C Кислотоупорні D Патогенні E Термофільні

142 З носоглотки дитини 5 років виділено мікроорганізм, який за морфологічними та біохімічними ознаками ідентичний Corynebacterium diphtheriae, але не утворює екзотоксин. У результаті якого процесу цей мікроорганізм може стати токсигенним? A Фагова конверсія B Культивування на телурітовому середовищі C Пасаж через організм чутливих тварин D Вирощування у присутності антитоксичної сироватки E Хромосомна мутація

143 Для серологічної діагностики черевнотифозного бактеріоносійства було використано діагностикум, що являє собою оброблені таніном еритроцити барана, на яких адсорбований Vi-антиген Salmonella typhi. В якій реакції буде застосовано цей діагностикум? A * РПГА B РГГА C РГА D РП E РЗК

144 У хворої з клінічними ознаками імунодефіциту, з незміненою кількістю та функціональною активністю Т- і В-лімфоцитів, при обстеженні виявлений дефект на молекулярному рівні, при якому порушена функція антигенпрезентації імунокомпетентним клітинам. Дефект структур яких клітин є можливим? A *Макрофаги, моноцити B Т-лімфоцити, В-лімфоцити C NK-клітини D Фібробласти, Т-лімфоцити, В-лімфоцити E 0-лімфоцити

145 У хворого діагностовано набутий дефект

імунної системи – порушення активації системи комплементу за класичним шляхом на фоні достатнього вмісту компонентів системи. Запідозрено наявність дефекту антитілоутворення. Зменшення вмісту в організмі яких антитіл можна очікувати в першу чергу? A *Ig M, IgG B Ig A, IgG C Ig D, IgG D Ig E, IgG E Ig M, Ig А

146 В лікарню поступив хворий з рваною раною гомілки, внаслідок укусу хворої на сказ тварини. Яку вакцину необхідно ввести для попередження сказу? A * Антирабічну вакцину B АКДП C АДП D БЦЖ E ТАВte

147 При бактеріологічному дослідженні промивних вод хворого на харчове отруєння висіяли чисту культуру бактерій з такими властивостями: грамнегативна рухлива паличка, на середовищі Ендо росте у вигляді безбарвних колоній. Представником якого роду було зумовлене захворювання? A * Salmonella B Shigella C Iersinia D Esherichia E Citrobacter

148 Мисливець звернувся за медичною допомогою з приводу укусів рук пораненою лисицею. Яку допомогу слід надати йому з метою специфічної профілактики сказу? A * Введення антирабічної вакцини B Хірургічна обробка рани C Введення антибіотиків D Промивання рани мильним розчином E Обробка рани спиртовим розчином йоду

149 В інфекційну клініку поступила дівчинка 7 років з високою температурою, скаргами на біль

Page 81: крок 1 база 2014р_11шр-240ст (для брошури)_35грн

http://vk.com/my.printing

81

у горлі, загальну слабість. Лікар запідозрив дифтерію і дав вказівку взяти матеріал із зіву і виділити чисту культуру збудника. Виберіть, що із перерахованого є вирішальним для підтвердження діагнозу “дифтерія” після виділення чистої культури збудника? A *Проба на токсигенність B Виявлення у збудника волютинових зерен C Проба на цистиназу D Гемолітична здатність збудника E Фаголізабельність

150 У місті епідемія грипу. Який препарат із перерахованих нижче можна порекомендувати людям для неспецифічної профілактики захворювання? A * Лейкоцитарний інтерферон. B Протигрипозну вакцину. C Пеніцилін. D Протигрипозний імуноглобулін. E Протигрипозну сироватку.

151 У пацієнта після тривалого вживання антибіотиків розвинувся дизбактеріоз кишечника. Які препарати слід призначити для відновлення нормальної мікрофлори? A * Еубіотики (пробіотики). B Сульфаніламіди. C Інтерферон. D Протигрибкові препарати. E Нітрофурани.

152 У інфекційну лікарню потрапив пацієнт з вірусним гепатитом А. Які антитіла будуть синтезуватися першими у відповідь на збудника? A * IgM. B IgG. C IgA. D IgD. E IgE.

153 З метою встановлення токсигенності виділених від пацієнтів збудників дифтерії культури висіяли на чашку Петрі з поживним агаром по обидва боки від розташованої в центрі смужки фільтрувального паперу, змоченого протидифтерійною антитоксичною

сироваткою. Після інкубації посівів в агарі між окремими культурами і смужкою фільтрувального паперу виявлено смужкоподібні ділянки помутніння середовища. Яку імунологічну реакцію було виконано? A * Реакцію преципітації в гелі. B Реакцію Кумбса. C Реакцію аглютинації. D Реакцію кільцепреципітації. E Реакцію опсонізації.

154 Ліквідатору наслідків аварії на Чорнобильській АЕС, що отримав велику дозу опромінення проведено трансплантацію кісткового мозку. Через деякий час після проведеної операції у пацієнта діагностовано розвиток реакції трансплантат проти хазяїна. Які антигени послужили пусковим механізмом виникнення цієї реакції? A * Антигени системи HLA клітин організму ліквідатора. B Антигени системи Rh еритроцитів ліквідатора. C Антигени HBs, HBc, HBe. D Антигенами системи ABO еритроцитів ліквідатора. E Антигени системи HLA клітин організму донора.

155 Серологічна діагностика інфекційних захворювань заснована на специфічній взаємодії антитіл з антигенами. Як називається серологічна реакція, яка полягає в склеюванні мікроорганізмів при дії на них специфічних антитіл при наявності електроліту? A *Реакція аглютинації B Реакція преципітації C Реакція зв'язування комплементу D Реакція гемадсорбції E Реакція нейтралізації

156 Серологічна діагностика інфекційних захворювань заснована на специфічній взаємодії антитіл з антигенами. Як називається серологічна реакція, при якій високодисперсні антигени адсорбовані на еритроцитах? A *Реакція непрямої (пасивної)

Page 82: крок 1 база 2014р_11шр-240ст (для брошури)_35грн

82

гемаглютинації B Реакція преципітації C Реакція зв'язування комплементу D Реакція гемадсорбції E Реакція нейтралізації

157 У мазку з нальоту на мигдалинах хворого з підозрою на дифтерію виявлено палички синього кольору з потовщеннями на полюсах. Який метод фарбування мазків було використано? A *Леффлера. B Буррі. C Гінса. D Грама. E Нейссера.

158 Пацієнт госпіталізований з попреднім діагнозом “гепатит В”. Для діагностики захворювання здійснено постановку серологічної реакції, яка базується на взаємодії антигену з антитілом, хімічно зв’язаним з пероксидазою або лужною фосфатазою. Яку назву має використана серологічна реакція? A * Імуноферментний аналіз B Радіоімунологічний метод C Реакція імунофлюоресценції D Реакція зв’язування комплементу E Реакція імобілізації

159 У дитячому дошкільному закладі напередодні Новорічних свят був зареєстрований спалах кишкової інфекції При бактеріологічному дослідженні випорожнень хворих, патогенних бактерій не було виділено. При електронній мікроскопії виявлено утворення округлої форми з чітким обідком і товстою втулкою, які нагадують колесо. Вкажіть найбільш імовірний збудник даної інфекції. A *Rotavirus B Adenovirus C Coxsacki-virus D E.coli E P.vulgaris

160 У хворого діагностовано ГРВІ. У сироватці крові знайдено имуноглобуліни класу М. Який період інфекційного процесу в даному

випадку A *Гострий період. B Продромальний період. C Інкубаційний період. D Реконвалесценція. E Мікробоносійство.

Нормальна анатомія 1 У хворого після перелому верхньої третини плечової кістки розвинувся параліч задньої групи м’язів плеча і передпліччя. Який нерв пошкоджено? A *Променевий B Ліктьовий C Серединний D М’язово-шкірний E Пахвовий

2 Хворий не може розігнути гомілку, у нього відсутня шкірна чутливість на передній поверхні стегна. Неврологічне обстеження виявило ураження нерва. Якого? A *Стегнового B Затуленого C Сідничного D Верхнього сідничного E Статево-стегновий

3 У хворого запалення середнього вуха ускладнилось мастоідітом. Надалі виникла загроза гнойного тромбозу найближчої венозної пазухи. Якої? A * Сигмоподібної B Поперечної C Верхньої сагітальної D Прямої E Нижньої кам’янистої

4 В хірургічне відділення доставлено чоловіка 35 років з гнійною раною на шиї попереду трахеї ( в ділянці передвісцерального простору). Куди може розповсюджуватись інфекція, якщо хворому терміново не зроблять операцію? A *В грудну порожнину – переднє середостіння B В грудну порожнину – в середнє середостіння C В грудну порожнину – в заднє

Page 83: крок 1 база 2014р_11шр-240ст (для брошури)_35грн

http://vk.com/my.printing

83

середостіння D В ретровісцеральний простір E В надгрудинний міжапоневротичний простір

5 Внаслідок інсульту (крововилив в головний мозок) у хворого відсутні вольові рухи м'язів голови і шиї. Обстеження головного мозку за допомогою ЯМР показало, що гематома знаходиться в коліні внутрішньої капсули. Який провідний шлях пошкоджено у хворого? A *Tr.cortico-nuclearis. B Tr.cortico-spinalis. C Тr.cortico-thalamicus. D Tr.cortico-fronto-pontinus. E Tr.thalamo-corticalis.

6 Після зіштовхнення двох автомобілів у одного з водіїв відзначається деформація у середній третині лівої гомілки, сильний біль, особливо при спробі рухати лівою гомілкою. З рани виступають кінці кістки тригранного січення, посилюється крововтрата. Яка кістка може бути пошкоджена? A *Великогомілкова кістка B Малогомілкова кістка C Стегнова кістка D Наколінок E Надп’яткова кістка

7 Після звільнення з під завалу у постраждалого відзначається затемнення свідомості, багато підшкірних крововиливів на голові та шиї, дрібні рани на обличчі. У задньо-верхніх відділах голови скальпована рана та різка деформація контурів голови. Які кістки можуть бути пошкоджені? A *Тім'яна та потилична кістка B Клиновидна кістка та нижня щелепа C Лобна та носові кістки D Скронева кістка та верхня щелепа E Вилична кістка та слізна кістка

8 Під час ультразвукового обстеження серця лікар спостерігає за стулками мітрального

клапана. Що відбувається з ними при систолі передсердь? A * Вивертаються в порожнину шлуночка B Притискаються до стінок передсердя C Вивертаються в порожнину передсердя D Стуляються, закриваючи просвіт отвору E Притискаються до стінки судин

9 Під час розтину порожнин серця на внутрішній стінці були виявлені гребінцеві м'язи. Які відділи порожнин серця розкриті ? A *Праве і ліве вушко B Правий і лівий шлуночок C Ліве вушко і лівий шлуночок D Ліве передсердя і лівий шлуночок E Праве передсердя і правий шлуночок

10 Звуження крупної судини спричинило погіршення відтоку крові з лівого шлуночка. Яка судина зазнала патологічних змін? A * Аорта B Легеневий стовбур C Легенева вена D Верхня порожниста вена E Нижня порожниста вена

11 Підвищення кров'яного тиску в аорті, спричинило навантаження на серцевий м'яз. М'язова стінка якої ділянки серця реагує на подразнення? A * Лівий шлуночок B Ліве передсердя C Правий шлуночок D Праве передсердя E Венозний синус

12 Підвищення кров'яного тиску в крупній судині, що несе кров до легень, спричинило навантаження на серцевий м'яз. М'язова стінка якої ділянки серця реагує на подразнення? A *Правого шлуночка B Лівого шлуночка C Правого передсердя D Лівого передсердя E Венозного синуса

13 Після травми ока виникло нагноєння м'яких тканин орбіти. Через який анатомічний утвір гнійний процес може поширитись у

Page 84: крок 1 база 2014р_11шр-240ст (для брошури)_35грн

84

середню черепну ямку? A * Через верхю очноямкову щілину B Через передній решітчастий отвір C Через задній решітчастий отвір D Через нижню очноямкову щілину E Через вилично-очноямковий отвір

14 Рентгенологічно у пацієнта діагностовано перелом плечової кістки в ділянці міжгорбкової борозни. Сухожилок якого м’яза може бути травмований уламками кістки в першу чергу? A *Двоголового м’яза плеча B Дельтовидного м’яза C Великого круглого м’яза D Найширшого м’яза спини E Великого грудного м’яза

15 При обстеженні пацієнта з ножовими ранами правої руки встановлено втрату чутливості шкіри бічної половини тильної поверхні кисті та проксимальних фаланг 1-го, 2-го і частково 3-го пальців. Який нерв пошкоджено? A * Променевий B Серединний C Ліктьовий D М’язово-шкірний E Бічний шкірний передпліччя

16 У постраждалого виявлено рану верхньої частини передньої ділянки плеча. При обстеженні встановлена втрата активного згинання в ліктьовому суглобі і чутливості шкіри передньо-бічної поверхні передпліччя. Порушення якого нерва має місце? A *м’язово-шкірного B Променевого C Серединного D Ліктьового E Пахвового

17 Хворий після поранення лівої гомілки втратив здатність утримувати стопу в пронованому положенні. Який нерв пошкоджено? A *поверхневий малогомілковий B глибокий малогомілковий C Великогомілковий D Сідничний

E загальний малогомілковий

18 Хвора К., 30 років, скаржиться на сильну спрагу, сухість у роті, які з’явилися після сильного нервового потрясіння. При лабораторному обстеженні виявлено збільшення цукру в крові до 10 ммоль/л. Захворювання якої ендокринної залози у хворої? A *Підшлункової B Щитоподібної C Статевих D Наднирникових E Епіфіза

19 Хвора Б., 39 років, яка протягом 8 років не може завагітніти порадили звернутись до ендокринолога. При обстеженні у хворої виявлено екзофтальм, тремор повік, тахікардію. Захворювання якої ендокринної залози супроводжується такими симптомами? A * щитоподібної B підшлункової C статевих D наднирникових E епіфіза

20 До лікаря звернулася мати син якої за літо виріс на 18см. При обстеженні хлопця 12 років: зріст – 180 см, вага 68 кг. З порушенням діяльності якої ендокринної залози це пов’язано? A *гіпофіза B щитоподібної C статевих D наднирникових E епіфіза

21 Хвора Б. 50 років скаржиться на те, що останнім часом вуха, ніс, кисті почали збільшуватись в розмірі. Гіперфункція якої залози дасть подібні симптоми? A *гіпофіза B щитоподібної C статевих D наднирникових E епіфіза

22 У хворого спостерігається втрата загальної чутливості на окремих ділянках тіла справа. Яка із звивин великих півкуль головного

Page 85: крок 1 база 2014р_11шр-240ст (для брошури)_35грн

http://vk.com/my.printing

85

мозку уражена ? A * Зацентральна. B Верхня скронева. C Середня скронева. D Нижня скронева. E Перед центральна

23 Внаслідок вивиху нижньої щелепи у пацієнта спостерігається відсутність відчуття смаку передньою частиною язика та сльозовиділення. Подразненням якого нерва це викликано? A * Лицевого B Під’язикового. C Нижньощелепного. D Блукаючого. E Барабанного.

24 У хворого спостерігається параліч м'язів верхньої і нижньої кінцівок зліва. Яка із звивин великих півкуль головного мозку уражена ? A * Передцентральна. B Середня лобова. C Нижня лобова. D Верхня лобова. E Зацентральна.

25 Хворий не розуміє змісту слів, а також нерозуміє власної мови (словесна глухота). Яка із звивин великих півкуль головного мозку уражена ? A *Верхня скронева. B Нижня лобова. C Зацентральна. D Верхня тім'яна долька. E Нижня тім'яна долька.

26 Потерпілий доставлений у лікарню з переломом нижньої щелепи та значною кровотечою у ділянці перелому. Пошкодження якої артерії вірогідніше всього мало місце? A *Нижньої коміркової артерії B Висхідної глоткової артерії C Язикової артерії D Висхідної піднебінної артерії E Верхньої коміркової артерії

27 У новонародженого під час сечовиділення з сечового міхура спостерігається виділення

сечі із пупка. Чим обумовлене це явище? A *незарощення сечової протоки B незарощення жовточної протоки C неповна нориця D ектопія устя сечовода E дивертикул сечового міхура

28 При цистоскопії у нормі слизова оболонка сечового міхура утворює складки за винятком однієї ділянки трикутної форми, де слизова оболонка гладка. В якій частині сечового міхура знаходиться цей трикутник? A *дно міхура B шийка міхура C верхівка м D тіло м E перешийок м

29 Жінка 25 років госпіталізована в гінекологічне відділення з метою операції з приводу пухлини яєчника. При здійсненні операції необхідно розсікати зв’язку, що з’єднує яєчник з маткою. Яку саме? A *lig. ovarii proprium B lig. cardinale C lig. latum uteri D lig. suspensorium ovarii E lig. umbilicale laterale

30 Після значного похудіння у чоловіка 70 років з’явились тупі болі в поперековій ділянці. При обстеженні був виставлений діагноз: "Блукаюча нирка". Яка частина фіксуючого апарату нирки вірогідніше всього була уражена? A *capsula adiposa B capsula fibrosa C m. iliopsoas D lig. hepatorenalis E m. quadratus lumborum

31 Чоловік 35 років звернувся до лікаря із скаргами на біль та припухлість правого яєчка. При обстеженні була виявлена пухлина, що потребує операції, при якій необхідно розсікати 7 шарів калитки до білкової оболонки. Яка оболонка буде розсічена останньою

Page 86: крок 1 база 2014р_11шр-240ст (для брошури)_35грн

86

перед білковою оболонкою? A *tunica vaginalis testis B tunica spermatica externa C tunica dartos D tunica spermatica interna E cutis orchis

32 Після травми обличчя у хворого гематома щічної ділянки. Відтік з якої слинної залози блоковано гематомою? A *привушна B під’язикова C піднижньощелепна D губна E щічна

33 Пацієнт помилково випив розчин оцтової кислоти. Яка оболонка стравоходу зазнала найбільших пошкоджень? A *слизова B м’язова C серозна D еластична мембрана E м’язова і серозна

34 В клініку звернувся чоловік 45 років із скаргами на втрату чутливості в ділянці задньої 1/3 язика. Функція якої пари черепно-мозкових нервів порушена? A *ІХ B Х C VIII D V E XII

35 До лікаря звернулась жінка 54 років із скаргами на паморочення, нудоту, порушення рівноваги після падіння і травми голови. Порушення функції якої структури внутрішнього вуха ймовірніше всього було? A *organum vestibulare B labyrinthus osseus C organum spirale D membrana tympani E canalis longitudinalis modioliI

36 У хворого фронтит. З анамнезу відомо, що в нього було запалення верхньощелепної пазухи. Через який відділ носової порожнини могла потрапити інфекція в

лобну пазуху? A * із середнього носового ходу B із верхнього носового ходу C із нижнього носового ходу D із присінка носової порожнини E із решітчасто-клиноподібної кишені

37 В клініку госпіталізована пацієнтка із пухлиною, розташованою у середній долі правої легені. Показана операція. Яку найбільшу кількість сегментів можна видалити у складі цієї долі? A *2 B 3 C 4 D 5 E 1

38 Дитина 3-х років поступила у лікарню з інорідним тілом у бронхах. В який бронх вірогідніше всього потрапило інорідне тіло? A *в правий головний бронх B в лівий головний бронх C в правий сегментарний бронх D в лівий сегментарний бронх E в дольковий бронх

39 Жінці 58-ми років проведено повне видалення матки з придатками, після чого виділення сечі припинилося. При цистоскопії: міхур сечі не містить, з устів сечоводів сеча не поступає. Який відділ сечовидільної системи було ушкоджено при операції? A *Ureter B Pelvis renalis C Uretra D Vesica urinaria E Ren

40 Хворий скаржиться на порушення чутливості шкіри в медіальній частині тильної та долонної поверхні кисті. Який з нервів ушкоджений? A * N. ulnaris B N. radialis C N. medianus D N. musculocutaneus E N. cutaneus antebrachii medialis

41 Юнака 18-ти років доставлено в лікарню з

Page 87: крок 1 база 2014р_11шр-240ст (для брошури)_35грн

http://vk.com/my.printing

87

ознаками внутрішньої кровотечі. Під час гри в футбол отримав удар в ділянці лівого підребер’я. Ушкодження якого з органів, що проектуются в дану ділянку, може спричинити сильну кровотечу. A *D. Lien B Cauda pancreatis C Fundus ventriculi D Flexura coli sinistra E Ren sinistra

42 У дітей часто можна спостерігати затруднене носове дихання, яке пов’язане з надмірним розвитком лімфоїдної тканини слизової оболонки глотки. Розростання яких мигдаликів може спричинити це явище? A * Tonsilla pharyngea B Tonsilla palatina C Tonsilla lingualis D Tonsilla tubaria E Усіх названих мигдаликів

43 До лікарні доставлено пораненого вогнепальною зброєю з сильною кровотечею. При огляді хірургом установлено, що кульовий канал пройшов через передню стінку живота, склепіння шлунка і вийшов на рівні ІХ ребра по лівій серединній пахвинній лінії. Який орган постраждав разом з пораненням шлунка? A *Селезінка B Ліва нирка C Підшлункова залоза D Поперечна ободова кишка E Ліва частка печінки

44 До лікаря звернувся пацієнт із скаргами на підвищену больову чутливість шкіри вушної раковини та зовнішнього слухового проходу. Пальпація позаду грудинно-ключично-сосковидного м'язу боляча. Подразнення якого з нервів може дати таку кліничну картину? A * N. auricularis magnus. B N. transversus colli. C N. occipitalis minor. D Nn. supraclaviculares. E N. vagus.

45 Випадково стукнувшись ліктем об стіл, хворий відчув пекучість і поколювання на внутрішній поверхні передпліччя. Який нерв був травмований в цьому випадку? A *N. ulnaris. B N. radialis. C N. medianus. D N. axillaris. E N. musculocutaneus.

46 При запалені глибоких лімфатичних вузлів пахвової області хірург повинен був розкрити глибоко розміщений гнійник. Після операції хворий втратив здатність згинати передпліччя в ліктьовому суглобі та порушилася шкірна чутливість бічної поверхні передпліччя. Якого нерва було ушкоджено при оперативному втручанні? A *N.musculocutaneus. B N.radialis. C N.ulnaris. D N.medianus. E N.axillaris.

47 Чоловік 42 років звернувся до медпункта з приводу різаної рани нижньої частини передньої поверхні плеча. Об’єктивно: затруднене згинання передпліччя. Які з названих м’язів ймовірно ушкоджені у хворого? A *M. brachialis, m. biceps brachii. B M.biceps brachii, m. anconeus. C M.coracobrachialis, m. supraspinatus. D M. deltoideus, m. infraspinatus. E M. deltoideus, m. biceps brachii.

48 Внаслідок пухлини гіпофіза при рентгенографії виявлено руйнування і збільшення ямки турецького сідла. Яка кісткова порожнина при цьому вражена? A *Пазуха клиновидної кістки B Сонний канал. C Зоровий канал. D Барабанна порожнина. E . Лицевий канал.

49 Який нерв вражено, якщо у хворого права носогубна складка розглажена, розширена права очноямкова щілина (її не вдається

Page 88: крок 1 база 2014р_11шр-240ст (для брошури)_35грн

88

закрити під час примружування, тому що повіки не змикаються), виникають затруднення під час розмови і приймання їжі (їжа застряє між щокою і зубами)? A *N. facialis dexter. B N. abduceus dexter. C N. glossopharyngeus sinister. D N. vagus dexter. E N. trigeminus dexter.

50 У потерпілого є травма м'яких тканин та тім'яних кісток в області стрілоподібного шва, яка супроводжується сильною кровотечею? Яке з утворень вірогідно ушкоджено? A * Sinus sagittalis superior. B Sinus petrosus superior. C Sinus rectus. D Sinus sagittalis inferior. E Sinus transversus.

51 Лікар прогнозує можливість запалення клітковини орбіти при гнійному запаленні в ділянці підочноямкового отвору. Через яке з утворень можливе поширення інфекції? A * Canalis infraorbitalis. B Canalis pterigoideus. C Canalis nosolacrimalis. D Canalis incisivus. E Fissura orbitalis inferior.

52 При огляді травмованого у дорожній пригоді лікар встановив ураження зовнішньої стінки очної ямки. Потерпілий втратив можливість відведення очного яблука на травмованій стороні. Який нерв міг бути ураженим у даному випадку? A *N.abducens. B N.trochlearis. C N.oculomotorius. D N.ophthalmicus. E N. infraorbitalis..

53 У хворого при висуванні язика спостерігається відхилення його верхівки вліво. Рухова інервація якого черепного нерву порушена у цьому випадку? A * N.hypoglossus dexter. B N.glossopharyngeus dexter.

C N. vagus dexter. D N. trigeminus sinister. E N.facialis sinister

54 В результаті травми черепа з пошкодженням верхньої стінки правої очної ямки потерпілий втратив можливість піднімати верхню повіку правого ока і дивитися угору. Який нерв вірогідно пошкоджений? A * R. superior n.oculumotorius. B R. inferior n.oculumotorius. C N. trochlearis D N. abducens. E N. ophthalmicus.

55 При обстеженні у хворого виявлено відсутність зору в присередніх половинах полів зору обох очей. Яка частина зорового шляху вірогідно ушкоджена? A *D. Chiasma opticus B N. opticus. C Tractus opticus. D Sulcus calcarinus. E Corpus geniculatum laterale

56 Під час операції закриття природженої щілини піднебіння (уранопластики), при збиванні долотом крилоподібного гачка, ушкоджено великий піднебінний канал. Виникла кровотеча яку можна зупинити лише шляхом тампонади каналу. Яку артерію ушкоджено? A *Низхідну піднебінну артерію. B Висхідну піднебінну артерію. C Висхідну глоткову артерію. D Клиноподібну артерію. E Задню верхню альвеолярну артерію.

57 При лапаротомії хірургом виявлено гангренозне ураження низхідної ободової кишки. Тромбоз якої артерії зумовив цей стан? A *Лівої ободової. B Серединної ободової. C Правої ободової. D Клубово-ободової. E Верхньої брижової.

58 У хворого, 26 років, виявлений великий

Page 89: крок 1 база 2014р_11шр-240ст (для брошури)_35грн

http://vk.com/my.printing

89

фурункул м’яких тканин обличчя біля кореню носу та нижньої повіки. Грізним ускладненням цього захворювання може бути розповсюдження інфекції по венозним сполученням цього регіону до пазух твердої мозкової оболонки. Яка з пазух найбільш вірогідно може бути уражена? A *Печеріста пазуха B Верхня сагітальна пазуха C Потилична пазуха D Сигмовидна пазуха E Верхня кам’яниста пазуха

59 До лікаря - ендокрінолога звернулася мати дівчинки 9 років зі скаргами на збільшення молочних залоз, кров’яністі виділення з піхви, посиленного росту волосся на тілі та навколо зовнішніх статевих органів. Яка з ендокрінних залоз вражена (що гальмує передчасне статеве визрівання? A *Epiphysis cerebri B Hypophysis cerebri C Gl. thyroidea D Gl. suprarenalis E Gl. parathyroidea

60 У хворого виявлене руйнування стінки барабанної порожнини з розповсюдженням гною до задньої черепної ямки. Яка із стінок зруйнована? A *Paries mastoideus B Paries membranaceus C Paries labyrinthicus D Paries tegmentalis E Paries jugularis

61 При обстеженні хворого, виявлене новоутворення в білій речовині півкуль великого мозку з локалізацією у коліні та передньому відділі задньої ніжки внутрішньої капсули. Волокна якого провідного шляху мозку будуть зруйновані? A * Tr. pyramidalis B Tr. frontothalamicus C Тr. thalamocorticalis D Tr. frontopontinus E Tr. parietooccipitopontinus

62 При обстеженні підлітка, лікарем виявлена вроджена вада серця - функційнування боталової протоки. Що з’єднує ця протока у внутрішньоутробному періоді розвитку? A * Легеневий стовбур та аорту B Правий та лівий шлуночок C Аорту та нижню порожисту вену D Праве та ліве передсердя E Легеневий стовбур та верхню порожисту вену

63 В хірургічне відділення поступив хворий з ножевим пораненням грудної клітини справа та пневмотораксом (проникнення повітря у плевральну порожнину). Перкуторно нижня межа правої легені по середньоключичній лінії піднялася на рівень III ребра. Де у нормі вона повинна обстежуватися? A * VI ребро B VII ребро C VIII ребро D IX ребро E V ребро

64 В реанімаційне відділення поступив хворий з тяжким отруєнням. Для проведення комплексу лікування необхідно виконати катетерізацію та введення лікарських речовин до подключичної вени. У якому топографічному утворенні вона знаходиться? A *Spatium antesсalenum B Spatium interscalenum C Spatium retrosternocleidomastoideus D Spatium interaрoneuroticum suprasternale E Trigonum omotrapezoideum

65 У хворого пухлиною пошкоджено піраміди довгастого мозку. У якому з провідних шляхів порушиться проведення нервових імпульсів? A *Tr. corticospinalis B Tr. corticonuclearis C Tr. corticopontinus D Tr. dentatorubralis E Tr. spinocerebellaris

66

Page 90: крок 1 база 2014р_11шр-240ст (для брошури)_35грн

90

У хворого спостерігається ішемія тканин нижче колінного суглоба, що супроводжується “переміжною кульгавістю”. Про оклюзію якої артерії можна думати? A *Підколінна артерія. B Глибока артерія стегна C Низхідна артерія коліна D Тильна артерія стопи E Проксимальна частина стегнової артерії.

67 В приймальній покій поступив хворий з переломом кісток кисті з сильною кровотечею в ділянці анатомічної табакерки. Яка судина пошкоджена? A *Променева артерія. B Ліктьова артерія. C Передня міжостиста артерія. D Задня міжостиста артерія. E Поверхнева долонна дуга.

68 У хворого виявлено порушення кровотоку у басейні правої передньої мозкової артерії. Які ділянки кори кінцевого мозку можуть постраждати за цих умов? A * медіальна поверхня правих лобної та тім’яної часток B медіальна поверхня правої потиличної частки C нижня поверхня правої лобної частки D нижня поверхня правої скроневої частки E права острівцева частка

69 У хворого на гайморит порушено сполучення верхньощелепової біляносової пазухи з середнім носовим ходом. Який утвір в нормі забезпечує надходження повітря з носового ходу до гайморової пазухи? A * hiatus semilunaris B infundibulum ethmoidale C foramina ethmoidalia D recessus sphenopalatinus E canalis nasolacrimalis

70 У хворого встановлено ураження внутрішнього вуха судинного генезу. Гілками якої з названих артерій, головним чином, здійснюється кровопостачання внутрішнього вуха?

A * a. basilaris B a. cerebri posterior C a. vertebralis D a. cerebri media E a. cerebri anterior

71 У хворого діагностовано злоякісну пухлину черевної частини стравоходу. Яка група лімфатичних вузлів є регіонарною для вказаного відділу стравоходу? A * anulus lymphaticus cardiae B nodi lymphatici paratrachealis C nodi lymphatici prevertebralis D nodi lymphatici pericardiales E _

72 У хворого з аневрізмою правої підключичної артерії спостерігається осиплість голосу. З подразненням якого нерву це може бути пов’язано? A * n.laringeus reccurens dexter B n.laringeus superior dexter C n.laringeus reccurens sinister D n.laringeus superior sinister E n.laringeus inferior sinister

73 Під час проведення комп’ютерної томографії головного мозку у хворого виявлено крововилив у ділянці коліна внутрішньої капсули. Який з вказаних провідникових шляхів постраждає в першу чергу? A * tractus corticonuclearis B radiationes thalamicae anteriores C tractus corticospinalis D radiatio acustica E radiatio optica

74 Хворому з порушенням функції зовнішнього дихання необхідно зробити трахеостомію. На рівні яких хрящових кілець трахеї частіше за все може знаходитися перешийок щитоподібної залози? A * II- ІV B III-IV C I-II D IV-V E V-VI

75 Після травми хворий не може утримувати

Page 91: крок 1 база 2014р_11шр-240ст (для брошури)_35грн

http://vk.com/my.printing

91

тіло у вертикальному положенні, не може піднятись навшпиньки. Функція якого м’яза порушена ? A *M. triceps surae B M. tibialis anterior C M. extensor digitorum longus D M. fibularis longus E M. extensor hallucis longus

76 Оглядаючи черевну порожнину потерпілого з проникаючим пораненням передньої черевної стінки, хірург помітив пошкодження малого сальника, з якого витікала ясно червона (артеріальна) кров. Яка артерія виявилась пошкодженою? A * A. hepatica propria B A. gastroepiploica dextra C A. hepatica communis D A. lienalis E A. gastroepiploica sinistra

77 Обстежуючи секційний матеріал, патанатом помітив на внутрішній поверхні одного з відрізків кишечника folliculi lymphatici aggregati (Пеєрові бляшки). Якій кишці належить цей відрізок? A *Ileum B Jejunum C Duodenum D Colon ascendens E Colon descendens

78 Жінку госпіталізовано в клініку з симптомами гострого живота. При обстеженні виникла підозра на позаматкову вагітність. Яке з анатомічних утворень таза необхідно пропунктувати для підтвердження вірного діагнозу? A *Excavatio rectouterina B Excavatio vesicouterina C Excavatio rectovesicalis D Fossa ischiorectalis E Processus vaginalis peritonei

79 Хворий 65 років звернувся в клініку з приводу розладів сечопуску. При обстеженні виявлено гіпертрофію простати. Збільшення яких з перечислених частин передміхурової залози є можливою причиною даних

порушень? A *Перешийок (середня частка) B Права частка C Ліва частка D Капсула простати E Проточка передміхурової залози

80 Параніхій мізинця ускладнився флегмоною кисті та передпліччя. Нагнійний процес поширився по: A *Vagina synovialis communis mm.flexorum B Vagina tendinis m.flexor pollicis longi C Canalis carpalis D Vagina tendinis m.flexor carpi radialis E Міжфасціальних просторах

81 Хірург, здійснюючи доступ до органів грудної порожнини, зробив розріз на передній грудній стінці по одному з міжребрових просторів. При цьому він особливо обережно розтинав скальпелем тканини в передній медіальній ділянці міжребрового простору, щоб не пошкодити артерію, яка розташована паралельно краю груднини на 1-1,5 см латерально від неї. Яку артерію остерігався пошкодити хірург? A *Внутрішню грудну артерію. B Міжреброву передню артерію. C Верхню діафрагмальну артерію D Реброво-шийний стовбур E Нижню діафрагмальну артерію

82 Обстежуючи хворого, хірург досліджує пульсацію артерії позаду медіальної кісточки. Пульсацію якої артерії досліджує хірург? A * Задньої великогомілкової артерії B Малогомілкової артерії C Передньої великогомілкової артерії D Задньої поворотної великогомілкової артерії E Передньої поворотної великогомілкової артерії

83 У жінки 40 років фіброміома матки. Виконана надпіхвова ампутація матки з видаленням маткових труб. Яєчники здорові, не видалені. Повністю збережена lig. suspensorium ovarii,

Page 92: крок 1 база 2014р_11шр-240ст (для брошури)_35грн

92

в якій проходить судина, яка кровопостачає яєчники . Назвіть цю судину. A *a. ovarіca B Ramus ovaricus a.uterina C a. pudenda interna D a. iliaca interna E a. iliaca externa

84 Хворий, 45 років, скаржиться на запаморочення, шаткість під час ходьби, порушення координації рухів. При обстеженні встановлено, що у хворого остеохондроз шийного відділу хребтового стовбура і стиснута судина, яка проходить через поперечні отвори шийних хребців. Яка це судина? A *a. verterbralis B a. subclavia C a. carotis externa D a. carotis interna E a. occipitalis

85 У потерпілого виявлено перелом тіла та верхньої гілки лобкової кістки, уламки пошкодили судини, що проходять в судинній лакуні. Які судини пошкоджені? A *a. et v. femoralis B a. et v. iliaca interna C a. et v. iliaca externa D a. et v. pudenda interna E a. et v. Epigastrica superficialis

86 При видаленні жовчного міхура в лігатуру замість однієї a. cystyca була втягнена артерія, перев’язка якої призвела до некрозу правої частки печінки і смерті. Яка артерія була помилково перев’язана разом з a. cystyca? A *Ramus dexter a. hepatica propria B a. hepatica communis C a. gasro-duodenalis D Ramus sinister a. hepatica propria E a. pancreato- duodenalis sup.

87 При позаматковій вагітності у жінки відбувся розрив маткової труби, що призвело до кровотечі. Пошкодження якої судини викликало кровотечу? A *Ramus tubaria a. uterina B Ramus ovaricus a. uterina

C a. ovarica D a. pudenda interna E a. iliaca interna

88 При гнійному запаленні середнього вуха в патологічний процес втягнена артерія, яка розташована на передній стінці барабанної порожнини. Яка судина втягнена в патологічний процес? A *a. carotis interna B a. carotis externa C a. meningea media D a. auricularis posterior E a. temporalis superficsalas

89 При видаленні пухлини підшлункової залози пошкоджена судина, яка проходить по верхньому краю підшлункової залози. Яка судина пошкоджена? A *a. lienalis B a. pancreato- duodenalis superior. C a. pancreato- duodenalis inferior. D a. gastro- epiploica dextra. E a. gastro- epiploica sinistra.

90 У чоловіка 35 років під час травми задньої поверхні плеча був пошкоджений променевий нерв і артеріальна судина, яка проходить поруч з ним в canalis humeromuscularis .Яка артерія була пошкоджена? A *a. profunda brachii B a. axillaris C a. radialis D a. ulnaris E a. brachialis

91 У хворого 26 років після оперативного втручання пропала здатність читати, складати з літер слова і фрази. В якій ділянці кори головного мозку не відновлена функція? A *Gyrus anqularis B Cuneus C Sulcus calcarinus D Gyrus parietalis superior E Gyrus supramarginalis

92 При рентгенологічному дослідженні кісток основи черепа виявлено збільшення порожнини турецького сідла витончення передніх нахилених відростків, руйнування

Page 93: крок 1 база 2014р_11шр-240ст (для брошури)_35грн

http://vk.com/my.printing

93

різних ділянок турецького сідла. Пухлина якої ендокринної залози може спричинити таке руйнування кісток? A *Гіпофіз B Вилочкова залоза C Епіфіз D Щитовидної залози E Наднирники

93 У хворого, який поступив у неврологічне відділення, при обстеженні виявлено відхилення язика вбік при висовуванні, атрофічні зміни половини язика, порушення мови. Який нерв пошкоджений? A *Під’язиковий B Язиковий C Барабанна струна D Язико-глотковий E Блукаючий

94 При обстеженні хворого, який звернувся у неврологічне відділення, виявлено зглаженість лобних складок, неможливість примружити очі, кут рота опущений, ”парусить” щоки. Який нерв пошкоджений? A *Лицевий B Окоруховий C Трійчастий D Блукаючий E Додатковий

95 При обстеженні хворого виявлено опущення верхньої повіки, розхідну косоокість, розширена зіниця, обмеженість рухомості очного яблука. Який нерв пошкоджений? A Окоруховий B Очний C Блоковий D Відвідний E Зоровий

96 В лікарню звернулася хвора К. 38 років із скаргами на те,що після перенесеного гострого вірусного респіраторного захворювання вона втратила відчуття дотику їжі до передніх 2/3 язика, а також відчуття болю та температури (попекла язик гарячим чаєм).

Вкажіть яка з гілок якого нерва при цьму була ушкоджена? A *Язиковий нерв нижньощелепного нерва трійчастого нерва B Язикові гілки язикогорлового нерва C Язикові гілки під’язикового нерва D Барабанна струна лицевого нерва E Верхній гортанний нерв блукаючого нерва

97 До лікаря-невропатолога звернулася хвора Л.52 років із скаргами на втрату чутливості шкіри правої половини обличчя в ділянці нижньої повіки, спинки носа та верхньої губи. Вкажіть яка гілка якого нерва при цьому ушкоджена: A *Верхньощелепний нерв трійчастого нерва B Великий кам’янистий нерв лицевого нерва C Очний нерв трійчастого нерва D Нижньощелепний нерв трійчастого нерва E Барабанна струна лицевого нерва

98 До лікаря звернувся хворий з периодонтом нижнього кутного зуба. Встановлено, що запальний процес поширився на лімфатичні вузли. Які лімфовузли були першими втягнуті в запальний процес? A *Піднижньощелепні B Бічні шийні C Передні шийні D Підборідні E Лицеві

99 Хворий 45 р. випадково випив оцтову кислоту. Опік яких відділів травної системи перш за все буде відбуватися? A *Порожнина рота, ротоглотки . B Порожнина рота та гортаноглотки. C Глотки, шлунку D Глотки та стравоходу. E Стравоходу та шлунку.

100 Під час операції, маніпулюючи в ділянці між шлунком та печінкою, хірург остерігався пошкодити печінково-дванадцятипалу зв’язку, тому що там знаходиться: A *Спільна жовчна протока, власна печінкова артерія, ворітна вена B Спільна жовчна протока, спільна печінква

Page 94: крок 1 база 2014р_11шр-240ст (для брошури)_35грн

94

артерія, ворітна вена печінки. C Власна печінкова артерія, шлунково-дванадцятипала артерія. D Ворітна вена печінки, печінкові вени E Нижня порожниста вена, міхурова протока.

101 При кесарському розтині у хворої Д., 32 років, внаслідок сильної кровотечі і неможливості її зупинити, оператор вимушений був видалити матку. За рахунок яких судин здійснюється кровопостачання матки? A *Arteria uterina. B Arteria obturatoria. C Arteria sacralis lateralis. D Arteria pudenda interna. E Arteria clitoridis.

102 В травматологічний пункт доставлений хворий з пораненням великого поперекового м’яза. Хворий втратив можливість розгинати гомілку в колінному суглобі. Який нерв ушкоджений в даному випадку? A * Стегновий нерв B Клубово-підчеревний C Клубово-пахвинний D Статево-стегновий E Затульнний нерв

103 Під час хірургічного втручання лікар маніпулює на внутрішній стінці барабанної порожнини. Руйнування якого каналу скроневої кістки можливе? A * Лицевого. B М'язово-трубного. C Барабанного. D Канальця барабанної струни. E Сонного.

104 В клініку госпіталізована пацієнтка із скаргами на кровохаркання, пітливість. Рентгенологічно виявлено вогнище туберкульозу у верхній частці правої легені . Показана операція. Яку кількість сегментів можна видалити в складі верхньої частки правої легені? A *3 B 5 C 4

D 2 E 1

105 У хворого 60-и років виявлено розширення вен стравоходу, прямої кишки та підшкірних вен передньої черевної стінки. Система якої вени ушкоджена? A *Ворітної. B Нижньої порожнистої вени. C Непарної вени. D Верхньої порожнистої вени. E Верхньої брижової.

106 У постраждалого в дорожній пригоді лікар виявив перелом лівої ключиці і порушення кровообігу в кінцівці (немає пульсації в променевій артерії. Яка причина порушення кровообігу в кінцівці? A * Здавлення підключичної артерії B Здавлення пахвової артерії C Здавлення підключичної вени D Здавлення хребтової артерії E Здавлення пахвової вени

107 Хворий страждає на цироз печінки. Варикозні розширення яких вен з системи порто-кавальних анастомозів можуть бути ? A * V. epigastrica superficialis B V. femoralis C V. subcostalis D V. circumflexa ilium profunda E Vv. intercostales posteriores

108 Після травми хворий не може розігнути руку у ліктьовому суглобові. Порушення функції якого основного м’язів може це визвати? A *Musculus triceps brachii B Musculus infraspinatus C Musculus levator scapule D Musculus teres major E Musculus subscapularis

109 Внаслідок травми у хворого спостерігається різний діаметр зіниць (анізокорія). Діяльність якого м_яза блоковано? A *Musculus sphincter pupillae B Musculus ciliaris C Musculus rectus lateralis D Musculus rectus superior E Musculus rectus inferior

110

Page 95: крок 1 база 2014р_11шр-240ст (для брошури)_35грн

http://vk.com/my.printing

95

Запаленя барабанної порожнини (гнойний отіт) у хворого ускладнилося запалення комірок соскоподібного відростка. Через яку стінку барабанної порожнини проникнув гній до комірок? A *Задню B Передню C Медіальну D Латеральну E Верхню

111 У хворого стався тромбоз нижньої брижової артерії. Які відділи кишківника можуть бути враженими? A *Сигмовидна кишка B Дванадцятипала кишка C Клубова кишка D Сліпа кишка E Голодна кишка

112 У хворого - коса пахвинна кила (грижа). Яке анатомічне утворення стало слабким місцем передньої черевної стінки? A * латеральна пахвинна ямка B надміхурова ямка C стегнова ямкаі D медіальна пахвинна ямка E м’язова затока

113 У хворого зі защемленою грижею справа у грижовому мішці знаходиться дуже запалена кишка, на якій можна розрізнити жирові привіски, ковбоподібні випини, які стягуються стрічками м’язів. Про ушкодження якого відділу кишки в першу чергу повинен подумати лікар? A * ободова кишка B дванадцятипала кишка C порожня кишка D клубова кишка E пряма кишка

114 У хворого запалення легенів ускладнилось ексудативним плевритом. В якому з перелічених анатомічних утворень переважним чином може накопичуватися рідина? A *sinus costodiaphragmaticus pleurae B sinus costomediastinalis pleurae C sinus phrenicomediastinalis pleurae

D sinus transversus pericardii E sinus obliquus pericardii

115 Хворий після порушення кровопостачання головного мозку втратив здатність до написання букв і цифр. В якій частці мозку виникла патологія? A *lobus frontalis B lobus occipitalis C lobus temporalis D lobus parietalis E insula

116 Хворий не може розмовляти, але розуміє звернену до нього мову. Про ушкодження якої зі структур головного мозку йде мова? A *Gyrus frontalis inferior B gyrus frontalis superior C gyrus precentralis D gyrus postcentralis E gyrus temporalis superior

117 У хлопчика спостерігається вроджена калиткова грижа. Вада розвитку якої з оболонок яєчка є причиною виникнення цієї грижі? A *tunica vaginalis testis B fascia spermatica externa C fascia spermatica interna D fascia cremasterica E tunica dartos

118 У хворого виявлений фурункул у зовнішньому слуховому проході. Які з перелічених лімфатичних вузлів у першу чергу можуть відреагувати на запальний процес? A *nodi lymphatici parotidei B nodi lymphatici retropharyngeales C nodi lymphatici mandibulares D nodi lymphatici cervicales superficiales E nodi lymphatici cervicales profundi

119 Водій автомобіля дістав травму грудної клітки внаслідок удару об рульове колесо. Яка з перелічених артерій скоріше за все може бути ушкодженою? A *a. thoracica interna B a. thyroidea superior C a. subscapularis

Page 96: крок 1 база 2014р_11шр-240ст (для брошури)_35грн

96

D a. vertebralis E a. suprascapularis

120 Хворий втратив здатність впізнавати предмети за характерними для них звуками (годинник, колокол, музика). Яка частка мозку ушкоджена? A *lobus temporalis B lobus occipitalis C lobus frontalis D lobus parietalis E insula

121 Хворий не може розігнути гомілку, у нього відсутня шкірна чутливість на передній поверхні стегна. Неврологічне обстеження виявило ураження нерва. Якого? A *Стегнового B Затульного C Сідничного D Верхнього сідничного E Статево-стегновий

122 Чоловіка 45 років доставлено в хірургічне відділення з різаною раною у медіальному краї передпліччя. Обстеження показало, що у хворого перерізано два м’яза передпліччя – ліктьовий згинач зап’ястка і ліктьовий розгинач зап’ястка. Які рухи не може виконувати хворий? A *Приведення кисті B Згинання кисті C Розгинання кисті D Відведення кисті E Розгинання і відведення кисті

123 При травмі в області тазу у хворого на ренгенологичному знімку виявлено некроз головки стегнової кістки.Яка з перерахованих зв'язок кульшового суглобу була пошкоджена під час травми.? A *Зв'язка головки стегнової кістки B Клубово-стегнова зв'язка C Лобкова-стегнова зв'язка D Сіднично-стегнова зв'язка E Коловий пояс

124 При обстеженні хворого встановлено порушення кровообігу підшлункової залози.

Яка з перелічених артерій може бути пошкоджена? A *А. Lienalis B А. hepatica propria C А. dastrica sinistra D А. gastroepiploica dextra E А. gastrica dextra

125 У прийомне відділення лікарні доставили дитину зі скаргами на біль за грудиною, яка з’являеться після ковтання та супроводжується кашлем. При рентгенологічному дослідженні виявлено інорідне тіло у товщі стінки стравохіду на рівні 5 грудного хребця. В області якого звуження стравохіду сталося пошкодження його стінки? A *Бронхіального B Аортального C Діафрагмального D Абдомінального E Фарінгеального

126 У новонародженної дитини на протязії першої добипедіатр помітив відсутність акту дефекації. Про яку ваду розвитку свідчить цей факт? A *Атрезія заднього проходу B Атрезія стравохіду C Заяча губа D Дивертікул стравохіду E Дивертікул порожньої кишки

127 У новонародженої дитини на протязі першої доби педіатр помітив, що при смоктанні молока воно потрапляє у носову порожнину. Про яку ваду розвитку свідчить цей факт? A *Незарощення піднебіння B Дивертікул стравоходу C Атрезія стравохіду D Заяча губа E Звуження стравохіду

128 У хворого звужена зіниця, при зменшенні освітлення не розширюється. В якому місці знаходиться ураження центральної нервової системи? A * Бічний ріг сірої речовини спинного мозку на рівні СVIII – ThI B Покрив ніжки середнього мозку на рівні

Page 97: крок 1 база 2014р_11шр-240ст (для брошури)_35грн

http://vk.com/my.printing

97

верхніх горбиків покришки C Основа ніжки середнього мозку на рівні нижніх горбиків покришки D Бічне колінчасте тіло проміжного мозку E Верхні горбики покришки середнього мозку

129 Хворий втратив здатність читати (алексія). У якій частині головного мозку ураження? A * Кутова закрутка тім’яної частки великого мозку B Середня лобова закрутка лобної частки великого мозку C Зацентральна закрутка тім’яної частки великого мозку D Прицентральна часточка лобної частки великого мозку E Верхня вискова закрутка вискової частки великого мозку

130 Хворий не може на одній половині лиця підняти брову, закрити повністю око, оголити зуби. Який нерв уражений? A * Лицевий нерв B Очний C Верхньощелепний D Нижньощелепний E Окоруховий

131 У хворого порушений зір у правих половинах полів зору обох очей. Яка нервова структура уражена? A * Лівий зоровий тракт B Зорове перехрестя C Правий зоровий тракт D Сітківка ока E Зорові нерви

132 У хворого знижений слух. При огляді виявлено сірчані корки. В якій частині органа слуху зміни? A * У зовнішньому вусі B У середньому вусі C У внутрішньому вусі D У барабанній перетинці E У слуховій трубі

133 У хворого після травми коліна гомілку у зігнутому під прямим кутом положенні можна

зміщувати вперед і назад подібно до висувної шухляди. Які зв’язки розірвані? A * Схрещені зв’язки коліна B Коса підколінна C Поперечна зв’язка коліна D Великогомілкова колатеральна E Малогомілкова колатеральна

134 У хворого інфаркт міокарда в ділянці передньої стінки лівого шлуночка. В басейні якої артерії виникло порушення кровообігу? A * Передньої міжшлуночкової гілки лівої вінцевої артерії B Передніх шлуночкових гілок правої вінцевої артерії C Огинаючої гілки лівої вінцевої артерії D Лівої крайової гілки лівої вінцевої артерії E Передсердно-шлуночкових гілок лівої вінцевої арте_рії

135 У хворого розширення вен і тромбофлебіт на медіальній поверхні голінки. Яка вена уражена? A *Велика підшкірна вена B Мала підшкірна вена C Задня великогомілкова вена D Малогомілкова вена E Передня великогомілкова вена

136 Поранення у праву половину живота. Яка частина товстої кишки найімовірніше може бути пошкоджена? A *Висхідна ободова кишка B Поперечна ободова кишка C Низхідна ободова кишка D Сигмовидна ободова кишка E Пряма кишка

137 У хворого судоми. Про гіпофункцію якої ендокринної залози можна думати? A * Прищитоподібної залози B Шишкоподібного тіла C Гіпофіза D Статевих залоз E Наднирникових залоз

138 В очне відділення доставлений 40 річний чоловік, який два тижні тому переніс опік очного яблука. Які з перелічених структур постраждали?

Page 98: крок 1 база 2014р_11шр-240ст (для брошури)_35грн

98

A * Рогівка B Війчасте тіло C Райдужка D Кришталик. E Склисте тіло.

139 Під час виконання фізичних вправ у 15 річного учня раптово виникла біль в зоні кульшового суглоба при обертанні нижньої кінцівки всередину. Який м’яз ушкоджений? A * Середній сідничний. B Грушоподібний. C Внутрішній затульний. D Зовнішній затульний. E Квадратний стегновий.

140 30 річний чоловік звернувся до стоматолога зі скаргою на розлади жування, у нього виникає біль при відтягуванні щелепи назад. Лікар встановив запалення одного з жувальних м’язів. Якого? A * Скроневого (задні волокна) B Скроневого (передні волокна) C Крилоподібного медіального D Крилоподібного латерального E Жувального

141 У хворого 30 років з різаною раною передпліччя з’явились порушення розгинання пальців кисті. Це свідчить про пошкодження: A * Променевого нерва B Серединного нерва C М’язовошкірного нерва D Ліктьового нерва E Присереднього шкірного нерва передпліччя

142 Хворий 37 років страждає туберкульозом легень з дитинства. Йому була призначена операція. Під час операції у хворого була видалена середня доля правої легені. Які сегменти були видалені? A * Латеральний і медіальний. B Верхній і передній. C Медіальний базальний і латеральний базальний. D Верхній язичковий і нижній язичковий. E Задній і передній.

143 Під час рентгенологічного дослідження у хворого у вертикальному положенні лікар констатує наявність повітря у шлунці. В якій

частині воно знаходиться? A * В дні B В тілі C В кардіальній D В пілоричній E В ділянці малої кривини

144 У хворого 27 років встановлено гнійне запалення жовчного міхура. Визначте в який відділ очеревинної порожнини попаде гній під час розриву жовчного міхура при його типовому положенні. A *В печінкову сумку. B В чепцеву сумку. C В лівий бічний канал. D В верхній дванадцятипалий закуток. E В передшлункову сумку.

145 З метою уточнення діагнозу у хворої 70 років стало необхідним пальпаторне дослідження органів тазу через передню стінку прямої кишки. Які органи при такій пальпації можна дослідити у жінки? A * Матку, вагіну. B Маткову трубу, вагіну. C Вагіну, яєчники. D Яєчники, матку. E Маткову трубу, матку.

146 Хворий 45 років після травми правого плеча (внаслідок падіння) не може відвести праву руку до горизонтального рівня. Пошкодження якого з м’язів викликало вказане обмеження руху ? A * Дельтоподібного. B Плечового. C Підосного. D Двоголового м’яза плеча. E Великого круглого.

147 Хвора 40 років звернулась до дільничого лікаря зі скаргої на те, що шкіра на медіальній поверхні правої голінки мало чутлива і мерзне більш, ніж на лівій нозі. При обстеженні встановлено ураження: A * Підшкірного нерва. B Великогомілкового нерва. C Малогомілкового загального нерва.

Page 99: крок 1 база 2014р_11шр-240ст (для брошури)_35грн

http://vk.com/my.printing

99

D Литкового нерва. E Поверхневої гілки малогомілкового нерва.

148 Хворий 63 років звернувся до невропатолога зі скаргою на те, що протягом трьох місяців не може виконувати столярні роботи, які потребують точності виконання тому що права рука робить багато неціленаправленних рухів. При дослідженні виявлено, що у хворого пошкоджена: A *Gyrus supramarginalis. B Gyrus precentralis. C Gyrus postcentralis. D Gyrus temporalis superior. E Gyrus angularis.

149 У хворого 52 років раптово з’явилась асиметрія лиця. Вся уражена половина лиця непорушна, носогубна складка згладжена, очна щілина розширена; око не закривається, кут рота опущений. Вказаний синдром викликаний ураженням якого нерва ? A * Лицевого. B Язикоглоткового. C Трійчастого. D Додаткового. E Під’язикового.

150 Хворому 60 років важко формувати і рухати харчову грудку, це порушує процес їжі. При цьому язик нерухомий, мова стала неможливою. Причиною цього могло стати порушення: A * Під’язикового нерва B Додаткового нерва C Язикоглоткового нерва. D Трійчастого нерва. E Лицевого нерва

151 У хворого 45-ти років з підозрою на запалення оболонок мозку потрібно було отримати спинно-мозкову рідину. Зроблено діагностичну пункцію між дугами поперекових хребців (L3-L4).Через яку зв’язку повинна проникнути голка під час пункції? A * Жовту зв’язку B Клубово-поперекову

C Передню поздовжню D Задню поздовжню E Міжпоперечну зв’язки

152 Дитина, 5 років, потрапила в тяжкому стані в інфекційний відділ клінічної лікарні з діагнозом дифтерії. Для запобігання ядухи дитині провели трахеостомію, В якому трикутнику шиї проведена дана операція ?В trigonum A *Omotracheale. B Caroticum. C Omoclaviculare. D Submandibulare. E Omotrapezoideum.

153 Хворий 45 років переніс тяжке порушення мозкового кровообігу.Після стабілізації загального стану, спостерігається втрата можливості чітко вимовляти слова. Ураження якої ділянки кори головного мозку викликало порушення мовно-рухового центру ? Gyrus: A *Frontalis inferior. B Angularis. C Supramarginalis. D Precentralis. E Temporalis superior.

154 Хворий 28 років на виробництві отримав хімічний опік обличчя і рідина потрапила в око. Хворий втратив зір. Яка структура очного яблука ушкоджена внаслідок хімічного опіку? A Рогівка B Кришталик C Склисте тіло D Сітківка E Райдужка

155 У хворого 40 років при інструментальному видаленні каменя сечовода сталось ускладнення – розрив стінки сечовода в черевному відділі. Куди потраплятиме сеча через отвір в стінці сечовода ? A * Заочеревинний простір. B Печінкову сумку C Порожнину очеревини. D Чепцеву сумку. E Хребтовий канал.

Page 100: крок 1 база 2014р_11шр-240ст (для брошури)_35грн

100

156 У дитини 5 років при гнійному запаленні внутрішнього вуха з’явились симптоми запалення твердої мозкової оболони. Шлях проходження інфекції ? Через: A * Водопровід присінка. B Каналець завитки. C Вікно завитки. D Вікно присінка. E Барабанний каналець.

157 Хворий скаржиться на головний біль, утруднене дихання. Рентген підтвердив діагноз – фронтіт (запалення лобової пазухи). В якому носовому ході при огляді порожнини носа можуть спостерігатись гнійні виділення? A *Середньому B Верхньому C Нижньому D Загальному E Над верхньою носовою раковиною

158 Після перенесеної черепно-мозкової травми і рентгенологічного обстеження виявлено перелом основи черепа. Лінія перелому проходить через остистий і круглий отвори. Яка кістка пошкоджена внаслідок травми? A *Клиноподібна B Вискова C Решітчаста D Лобова E Потилична

159 Після перенесеного запального захворювання у хворого виникло неповне відведення очного яблука в латеральну сторону. Який нерв у хворого пошкоджений? A *Відвідний B Окоруховий C Блоковий D Зоровий E Лицевий

160 Хворий втратив смак в передній частині двох-третіх язика. Порушення функції якого нерва можна припустити? A *Барабанної струни B Язикиглоткового нерва C Блукаючого нерва

D Під'язикового нерва E Додаткового нерва

161 У хворого порушена моторна функція язика. З патологією якого нерва це пов'язано? A *Під'язикового B Блукаючого C Язикоглоткового D Лицевого E Додаткового

162 Лікар-стоматолог при лікуванні зубів вводить в просвіт між щокою і альвеолярним відростком верхньої щелепи ватний тампон. Вивідний отвір протока якої залози він закриває? A *Привушної B Піднижньощелепної C Під'язикового D Щитоподібної E Прищитоподібної

163 При проведенні дуоденального зондування зонд не проходить зі шлунка в дванадцятипалу кишку. В якому відділі шлунка з перешкода (пухлина)? A *В воротарному B В кардіальному C В ділянці дна D В ділянці тіла E У ділянці малої кривизни

164 В який відділ дванадцятипалої кишки необхідно ввести фіброгастроскоп, щоб оглянути великий сосок дванадцятипалої кишки? A *Низхідний B Висхідний C Верхній D Нижній E В дванадцятипало-порожній згин

165 Хворий не може розвести пальці кисті. Функція яких м'язів порушена? A *Тильних міжкісткових B Черв'якоподібних C Долоних міжкісткових D Поверхневий згинач пальців E Глибокий згинач пальців

Page 101: крок 1 база 2014р_11шр-240ст (для брошури)_35грн

http://vk.com/my.printing

101

166 У хворого з ножовим пораненням стегна утруднене розгинання гомілки. Який із м'язів постраждав сильніше? A *Чотирьохголовий м'яз стегна B Ніжний C Кравецький D Двоголовий м'яз стегна E Напівсухожилковий

167 У хворого після травми помічено зниження больової і температурної чутливості в ділянці 1,5 пальців на долонній поверхні і 2,5 пальців на тильній поверхні зі сторони мізинця. Який нерв пошкоджено внаслідок травми? A *Ліктьовий B Променевий C Серединний D М'язово-шкірний E Присередній шкірний нерв передпліччя

168 Хворий з ножовим пораненням верхньолатеральноі ділянки плеча не може відвести руку до горизонтальної площини. Який із м'язів пошкоджено? A *Дельтоподібний B Двоголовий C Плечовий D Трьохголовий E Дзьобоподібно-плечовий

169 У хворого після простудної захворювання виникло порушення виділення сльози. Який вегетативний вузол найбільше при цьому постраждав? A *Крилопіднебінний B Війчастий C Вушний D Піднижиьощепепний E Під'язиковий

170 У хворого після простудного захворювання з'явилося порушення больової і температурної чутливості передніх 2/3 язика? Який із нервів при цьому постраждав? A *Трійчастий B Під'язиковий C Діафрагмальний D Блукаючий

E Барабанна струна

171 Хворий потрапив у клініку з травмою черепа. При рентгенологічному обстеженні виявлено перелом основи черепа в ділянці канала під'язикового нерва. Яку кістку черепа пошкоджено? A *Потиличну B Вискову C Тім'яну D Клиноподібну E Лобну

172 Хворий потрапив у клініку з пораненням в ділянці шиі. При обстеженні виявлено пошкоджений нерв, розташований попереду переднього драбинчастого м'яза. Який нерв ушкоджено? A *Діафрагмальний B Блукаючий C Язикоглотковий D Під'язиковий E Шийний відділ симпатичного стовбуру

173 Після крововиливу в мозок (геморагічний інсульт) хворий з великим зусиллям став вимовляти слова, тобто виникла моторна афазія. Яка звивина мозку при цьому ушкоджена? A *Нижня лобова B Верхня лобова. C Середня лобова D Верхня вискова E Нижня вискова

174 При ревізії черевної порожнини виявлено венозну кровотечу з печінково-дванадцятипалої зв'язки. Яку з вен пошкоджено? A *Ворітну B Нижню порожнисту C Селезінкову D Верхню брижову E Нижню брижову

175 Після перенесеного запального процесу хворий став помічати слабість при згинанні кисті в ділянці 1, 2, 3 і 4 пальців, зменшення об’єму м'яза, підвищення великого пальця.

Page 102: крок 1 база 2014р_11шр-240ст (для брошури)_35грн

102

При обстеженні виявлено порушення больової і температурної чутливості в ділянці поверхні долоні 1, 2, 3 і променевої поверхні четвертого пальців. Який із нервів уражений? A *Серединний B Променевий C Ліктьовий D М'язово-шкірний E Присередній шкірний нерв передпліччя

176 Після перенесеного простудного захворювання у хворого з'явилось оніміння в ділянці правої половини обличчя. При обстеженні виявлено порушення больової і температурної чутливості правої половини обличчя. Який нерв ушкоджено? A *Трійчастий B Лицевий C Язикоглотковий D Блукаючий E Під'язиковий

177 При обстеженні хворого невропатолог виявив наступний симптомокомплекс: згасання кремастерного рефлекса (скорочення m. cremaster), пошкодження чутливості шкіри на передній і внутрішній поверхні верхньої третини стегна та калитки. Про пошкодження якого нерва йде мова? A *Стегново-статевий нерв B Клубово-пахвинний нерв C Сідничний нерв D Стегновий нерв E Затульний нерв

178 Під час обстеження пацієнта ЛОР лікар діагностував запалення верхньощелепних пазух. В якому носовому ході під час риноскопії був виявлений гній? A *В середньому B В верхньому C В нижньому D У загальному E В найвищому

179 Під час катетеризації сечового міхура при

швидкому введенні катетера виникла кровотеча вгаслідок травми слизової оболонки сечівника в ділянці довільного сфінктера. В якому місці уретри лікар повинен бути обережним і відчувати опір м'яких тканин при проходжені катетера? A *B pars membranacea uretrae B В ділянці fossa navicularis uretrae C B bulbus uretrae D B pars spongiosa uretrae E B past prostatica uretrae

180 При ультразвуковому обстеженні серця лікар спостерігає за півмісяцевими заслонками. Що відбувається з ними при діастолі (розслабленні) шлуночків? A *Стуляються, закриваючи просвіт судин B Вивертаються в порожнини судин C Вивертаються в порожнини шлуночків D Притискаються до стінок судин E Притискаються до стінок шлуночка

181 Внаслідок росту пухлини в порожнину ІІІ шлуночка головного мозку у пацієнта розвивається вегетативні розлади у вигляді порушення сну, терморегуляції, всіх видів обміну, не цукровий діабет. Подразнення ядер якої ділянки головного мозку викликало ці симптоми? A *Гіпотапамуса B Ніжок мозку C Покришки середнього мозку D Моста E Довгастого мозку

182 У пацієнта при епідемічному енцефаліті спостерігаються одно- або двосторонній птоз (опущення повіки), розходяща косоокість, порушення акомодації. Зіниці розширені. Ядра якої пари черепно-мозкових нервів уражені? A *ІІІ B ІV C V D VI E VII

183 Після травми ока виникло нагноєння м'яких

Page 103: крок 1 база 2014р_11шр-240ст (для брошури)_35грн

http://vk.com/my.printing

103

тканин орбіти. Через який анатомічний утвір гнійний процес може поширитись у крилопіднебінну ямку? A *Через нижню очноямкову щілину B Через круглий отвір C Через клинопіднебінний отвір D Через верхю очноямкову щілину E Через вилично-очноямковий отвір

184 При блокаді лікворних шляхів на рівні серединного і бічних отворів IV шлуночка розвивається оклюзивний синдром. В яку порожнину затруднений відтік ліквора при цій патології через ці отвори? A *Субарахноідальний простір B Бічні шлуночки C Водопровід мозку D ІІІ шлуночок E ІV шлуночок

185 Під час гінекологічного обстеження пацієнтці був виставлений діагноз ендометриту (запалення ендометрію). Яка частина маткової стінки уражена запальним процесом? A *Слизова оболонка B Серозна оболонка C М'язова оболонка D Адвентиційна оболонка E Навколоматкова клітковина

186 Під час рентгенологічного обстеження черепа виявлено збільшення розміру турецького сідла внаслідок пухлиноподібного утвору головного мозку. Яка частина головного мозку зазнала патологічних змін? A *Hypothalamus B Thalamus dorsalis C Metathalamus D Epithalamus E Tectum mesencephali

187 При операції правосторонньої лобектомії хірург підійшов до кореня правої легені з метою окремого виділення і обробки його складових. Вкажіть порядок розміщення елементів кореня правої легені з якими лікар

зустрінеться при виділенні та обробці в напрямку зверху вниз ? A * Бронх, легенева артерія, легеневі вени B Легенева артерія, бронх, легеневі вени C Легенева вена, легеневі артерія, бронх D Бронх, легенева артерія, діафрагмальний нерв E Діафрагмальний нерв, бронх, бронхіальна артерія і вена

188 У хворого під час трахеотомії виникла виражена кровотеча. Яка артерія була травмована під час операції? A *A thyroidea ima B *A thyroidea superior C *A thyroidea inferior D *A laryngea inferior E *A laryngea superior

189 Хворому проведена субтотальна субфасціальна резекція щитовидної залози. У післяопераційному періоді тривалий час зберігається охриплість голосу. Який нерв ушкоджено в ході операції? A *Зворотній гортанний нерв B Верхній гортанний нерв C Під’язиковий нерв D Язиковий нерв E Нижньощелепний нерв

190 У пораненого перев’язали плечову артерію в нижній третині плеча. За рахунок яких артерій можливе відновлення кровопостачання передпліччя і кисті? A * Глибокої артерії плеча, ліктьових колатеральних артерій. B М’язових гілок плечової артерії. C Передньої і задньої огинаючих артерій плеча.. D Підлопаткової, передньої міжкісткової артерій. E Грудонадплечової, задньої огинаючої артерій плеча.

191 При обстеженні кровопостачання стопи, лікар обстежує пульсацію крупної артерії, яка проходить попереду articulatio talocruralis проміж сухожилками довгого розгинача великого пальця стопи та довгого

Page 104: крок 1 база 2014р_11шр-240ст (для брошури)_35грн

104

розгинача пальців в окремому фіброзному каналі. Яка це артерія? A * A. dorsalis pedis B A. tibialis anterior C A. tarsea medialis D A. tarsea lateralis E A. fibularis

192 Через травму в ділянці плечового суглоба у хворого рентгенологічно був виявлений осколковий перелом підсуглобового горбка. Сухожилок якого м’яза, що кріпиться в цьому місці, зазнав пошкодження? A *Довгої головки m. triceps brachii B Довгої головки m. biceps brachii C Медіальної головки m. triceps brachii D Латеральної головки m. triceps brachii E Короткої головки m.biceps brachii

193 Хворий 32 років з ураженням ліктьового нерва не може звести до серединної лінії ІІ і V пальці. Функція яких м’язів при цьому порушена? A *Долонні міжкісткові м’язи B Тильні міжкісткові м’язи C Червоподібні м’язи D Короткий долонний м’яз E Відвідний м’яз мізинця

194 У хворого, внаслідок тривалого хронічного захворювання головного мозку, виникли мимовільні рухи, порушився тонус м’язів тулуба. На порушення якого провідного шляху вказують ці симптоми? A *Tractus rubrospinalis. B Tractus corticospinalis. C Tractus corticonuclearis. D Tractus spinothalamicus. E Tractus tectospinalis.

195 У пацієнта внаслідок черепно-мозкової травми знижена шкірна чутливість. Яка ділянка кори великого мозку може бути ураженою ? A *Задня центральна звивина B Потилична ділянка C Поясна звивина D Лобна ділянка кори E Передня центральна звивина

196 У пацієна 40 років перфорація виразки задньої стінки шлунка. У який анатомічний утвір потрапить кров та вміст шлунка? A *Чіпцеву сумку B Передшлункову сумку C Правий бічний канал D Лівий бічний канал E Печінкову сумку

197 У хворого 60 років при ангіокардіографії встановлено звуження кровоносної судини, що розташована у вінцевій борозні серця ліворуч. В якій з кровоносних судин віявлені патологічні зміни? A * ramus circumflexus B ramus interventricularis posterior C a. coronaria dextra D v.cordis parva E ramus interventricularis anterior

198 Больная обратилась с жалобами на боли в правой латеральной области живота. При пальпации определяется плотное, неподвижное, опухолевидное образование. В области какого отдела пищеварительной трубки возможно наличие опухоли? A *Colon ascendens B Colon transversum C Colon descendens D Colon sigmoideum E Caecum

199 У травматологічне відділення поступив пацієнт з різаною раною трапецієподібного м’яза. Яка з фасцій шиї формує піхву для даного м’яза ? A * Поверхнева пластинка шийної фасції B М’язова частина передтрахейної пластинки шийної фасції C Вісцеральна частина передтрахейної пластинки шийної фасції D Передхребтова пластинка шийної фасції E Сонна піхва шийної фасції

200 У постраждалого з ножовим пораненням шиї визначається кровотеча. При первинній обробці рани встановлено, що пошкоджена судина яка розташована вздовж латерального краю груднинно-ключично-

Page 105: крок 1 база 2014р_11шр-240ст (для брошури)_35грн

http://vk.com/my.printing

105

соскоподібного м’яза. Визначте цю судину. A * V. jugularis externa B V. jugularis anterior C A. carotis externa D A. carotis interna E V. jugularis interna

Нормальна фізіологія 1 Хворому з гіперсекрецією шлункового соку лікар рекомендував виключити з дієти насичені бульйони і овочеві відвари, тому що вони стимулюють шлункову секрецію переважно через активацію: A * вироблення гастрину B смакових рецепторів C механорецепторів ротової порожнини D механорецепторів шлунку E вироблення секретіну

2 Тривале перебування в умовах спеки викликало у людини спрагу. Сигналізація від яких рецепторів, перш за все, зумовило її розвиток? A *Осморецептори гіпоталамусу B Натрієві рецептори гіпоталамусу C Осморецептори печінки D Глюкорецептори гіпоталамусу E Барорецептори дуги аорти

3 В гострому експерименті у тварини здійснювали електричне подразнення chorda tympani, внаслідок чого з протоки привушної слинної залози виділялося: A *Багато рідкої слини B Мало рідкої слини C Не виділялася слина D Мало в'язкої слини E Багато в'язкої слини

4 За обідом людина з'їла солоного оселедця і картоплю з солоним огірком. Через деякий час у неї виникла спрага. Імпульсація від яких рецепторів зумовила це відчуття? A *Осморецептори гіпоталамусу B Волюморецептори порожнистих вен і передсердь C Осморецептори печінки D Волюморецептори гіпоталамусу E Барорецептори дуги аорти

5 Хворому видалили частину підшлункової залози. Які продукти, перш за все, йому потрібно обмежити в харчовому раціоні? A *Жирне м'ясо, міцні бульйони B Відварені овочі C Кисломолочні продукти D Овочі, багаті білками (боби, соя) E Фрукти

6 Внаслідок введення жабі розчину хімічної речовини, у відповідь на всі подразнення вона відповідає генералізованими судомами. Що було введено жабі? A *стрихнін B Адреналін C Ацетилхолін D Серотонин E Дофамін

7 У здорового обстежуваного в стані спокою кількість еритроцитів становить 5,65*1012/л. Причиною цього може бути те, що обстежуваний: A *Житель високогір’я B Шахтар C Студент D Вагітна жінка E Відповідальний працівник міністерства

8 У жінки 40 років при обстеженні виявлений підвищений основний обмін. Надлишок якого з наведених гормонів зумовить цей стан? A *Трийодтиронін B Тиреокальцитонін C Глюкагон D Альдостерон E Соматостатин

9 Під час емоційного збудження частота серцевих скорочень (ЧСС) у людини 30-ти років досягла 112 на хвилину. Зміна стану якої структури провідної системи серця є причиною збільшення ЧСС? A *Синоатріальний вузол B Волокна Пуркіньє C Ніжки пучка Гіса D Атріовентрикулярний вузол E Пучок Гіса

Page 106: крок 1 база 2014р_11шр-240ст (для брошури)_35грн

106

10 При обстеженні дівчинки 16 років виявлено: відсутність оволосіння на лобку і під пахвами, нерозвиненість молочних залоз, відсутність менструацій. Результатом яких гормональних порушень це може бути? A *Недостатність гормональної функції яєчників. B Гіперфункція щитовидної залози. C Гіпофункція щитовидної залози. D Недостатність острівцевого апарату підшлункової залози E Гіперфункція мозкової речовини наднирників

11 Обстеження хворого в ендокринологічному диспансері виявило підвищення рівня глюкози в крові до 11 ммоль/л. З недостачою якого гормону пов'язані ці зміни? A *Інсуліну B Глюкагону C Естрадіолу D Тестостерону E Паратгормону

12 До ендокринолога звернувся хворий зі скаргами на схуднення на 10 кг за 2 місяці, серцебиття, витрішкуватість. Для гіперфункції якої ендокринної залози ці скарги найбільш характерні? A *Щитовидної залози B Прищитовидної залози C Підшлункової залози D Яєчників E Наднирків

13 Хлопець віком 12 років має зріст 1 м 80 см. Порушення секреції якого гормону це обумовило? A *Соматотропного B Тироксину C Тіреотропного D Гонадотропного E Інсуліну

14 У людини зі хворобою нирок виявлена анемія. Найбільш ймовірною причиною анемії є порушення секреції: A *еритропоетину.

B реніну. C альдостерону. D натрійуретичного гормону. E АДГ.

15 У хворого виявлено порушення пальценосової проби. Порушення функції якої структури головного мозку може бути причиною цього? A *Мозочка B Гіпокампу C Ретикулярної формації D Червоні ядра E Вестибулярні ядра

16 У хворого після захворювання печінки виявлено зниження вмісту протромбіну в крові. Це призведе, перш за все, до порушення: A *другої фази коагуляційного гемостазу. B першої фази коагуляційнго гемостазу. C судинно-тромбоцитарного гемостазу. D фібринолізу. E антикоагулярних властивостей крові.

17 Яким буде скорочення м’язів верхньої кінцівки при намаганні підняти непосильний вантаж? A *Ізометричним B Ізотонічним C Ауксотонічним D Фазичним E Одиночним

18 Якої сили подразнення треба нанести на нервове волокно, щоб викликати збудження у фазі відносної рефрактерності ? A * Надпорогове B Підпорогове C Порогове D Підпорогове тривале E Порогове тривале

19 У хворого 70 років діагностовано крововилив у стовбур мозку. Обслідування виявило підвищення тонусу м’язів згиначів на тлі зниження тонусу м’язів розгиначів. Подразненням яких структур мозку можна пояснити зміни у тонусі м’язів? A *Червоних ядер

Page 107: крок 1 база 2014р_11шр-240ст (для брошури)_35грн

http://vk.com/my.printing

107

B Вестибулярних ядер C Чотирибугір'я D Чорної речовини E Ретикулярної формації

20 Під час обертання на каруселі у жінки 25 років з’явилася нудота, блювання, посилення потовиделення. Активація яких рецепторів зумовила рефлекторний розвиток цих симптомів? A *Вестибулярних півколових каналів B Пропріорецепторів скелетних м’язів C Кортієвого органу D Зорових E Отолітових вестибулярних

21 До приймально-діагностичного відділення доставлено жінку 38-ми років з маточною кровотечею. Що з наведеного буде виявлено при аналізі крові хворої? A *Зменшення гематокритного числа B Еозинофілія. C Сповільнення ШЗЕ D Лейкоцитоз E Збільшення кольорового показника

22 У студента 18 років під час фізичного навантаження реографічно зареєсторовано перерозподіл кровотоку органів. У яких судинах кровотік підвищився найбільшою мірою? A *Скелетних м’язів B Печінки C Головного мозку D Нирки E Шлунково-кишкового тракту

23 У пацієнта віком 60 років виявлено погіршення сприйняття звуків високої частоти. Порушення стану яких структур слухового аналізатора зумовлює ці зміни? A *Основної мембрани завитки біля овального віконця B Основної мембрани завитки біля гелікотреми C Євстахієвої труби D М’язів середнього вуха E Барабанної перетинки

24 У лабораторному експерименті на собаці

вивчали будову центральних відділів слухової сенсорної системи. Була зруйнована одна з структур середнього мозку. Собака втратив орієнтувальний рефлекс на звукові сигнали. Яка структура була зруйнована? A * Нижні горбики чотирибугір'я B Верхні горбики чотирибугір'я C Чорна речовина D Ядра ретикулярної формації E Червоне ядро

25 Внаслідок отруєння чадним газом (СО) у людини виникли головний біль, задишка, запаморочення. Зниження вмісту якої сполуки у крові призвело до цього? A * Оксігемоглобін B Карбоксігемоглобін C Карбгемоглобін D Метгемоглобін E Дезоксігемоглобін

26 Піддослідній тварині через зонд у порожнину шлунку ввели 150 мл м'ясного бульйону. Вміст якої речовини швидко збільшиться у крові? A *Гастрину B Соматостатину C Інсуліну D Глюкагону E Нейротензину

27 При токсичному ушкодженні клітин печінки з порушенням її функцій у хворого з'явилися набряки. Які зміни складу плазми крові є провідною причиною розвитку набряків? A *Зниження вмісту альбумінів B Збільшення вмісту глобулінів C Зменьшення вмісту фібріногену D Збільшення вмісту альбумінів E Зменшення вмісту глобулінів

28 Зріст дорослої людини становить 120 см при пропорціональній будові тіла і нормальному розумовому розвитку. Для недостатнього вироблення якого гормону у дитячому віці характерні вказані ознаки? A *Соматотропіну B Гонадотропного C Адренокортикотропного D Тиреотропного

Page 108: крок 1 база 2014р_11шр-240ст (для брошури)_35грн

108

E Тироксину

29 В експерименті встановлено, що при збудженні мотонейронів м'язів-згиначів, гальмуються мотонейрони м'язів-розгиначів. Який вид гальмування лежить у основі цього явища? A *Реципрокне B Гальмування слідом за збудженням C Пессимальне D Зворотнє E Латеральне

30 У пацієнта при незначних механічних впливах виникають підшкірні крововиливи. Що може бути причиною такого явища? A *тромбоцитопенія B еритропенія C лейкопенія D лімфоцитоз E зменшення вмісту гемоглобіну

31 У пацієнта після травми виникли паралічі, розлади больової чутливості справа; зліва – паралічі відсутні, але порушена больова і температурна чутливість. Яка причина такого явища? A *одностороннє пораження спинного мозку з правої сторони B пошкодження стовбура мозку C пошкодження середнього мозку D пошкодження рухової зони кори головного мозку E пошкодження мозочка

32 При аналізі ЕКГ необхідно визначити, що є водієм ритму серця. Зробити це можна на підставі вимірювання: A *Тривалості інтервалу R-R B Амплітуди зубців C Напрямку зубців D Тривалості зубців E Тривалості комплексу QRST

33 У юнака енерговитрати збільшились з 500 до 2000 кДж за годину. Що з наведеного може бути причиною цього? A * Фізичне навантаження B Підвищення зовнішньої температури

C Розумова праця D Прийом їжі E Перехід від сну до бадьорості

34 У юнака під час фізичного навантаження хвилинне споживання кисню та хвилинне виділення вуглекислого газу дорівнюють 1000 мл. Які субстрати окислюються в клітинах його організму? A *Вуглеводи B Білки C Жири D Вуглеводи та жири E Вуглеводи та білки

35 Експериментатору необхідно якнайшвидше виробити умовний рефлекс у собаки. На базі якого безумовного рефлекса доцільно виробляти умовний? A *Захисного B Травного C Статевого D Орієнтувального E Міотатичного

36 Студент старанно конспектує лекцію. Якість конспектування значно погіршилась, коли сусіди стали розмовляти. Який вид гальмування умовних рефлексів є причиною цього? A *Зовнішнє B Позамежне C Згасаюче D Диференціювальне E Запізніле

37 При патологоанатомічному дослідженні спинного мозку чоловіка 70 років виявлені деструкція та зменшення кількості клітин ядер передніх рогів у шийному і грудному відділах. Які функції були порушені при житті? A * Моторні функції верхніх кінцівок B Моторні функції нижніх кінцівок C Чутливість і моторні функції верхніх кінцівок D Чутливість нижніх кінцівок E Чутливість верхніх кінцівок

38 В результаті травми у чоловіка 40 років зруйновані задні корінці спинного мозку. Які

Page 109: крок 1 база 2014р_11шр-240ст (для брошури)_35грн

http://vk.com/my.printing

109

розлади будуть спостерігатися в ділянці інервації цих коренців ? A * Втрата всіх видів чутливості B Порушення функції посмугованих скелетних м’язів C Порушення функції гладеньких м’язів D Втрата температурнаої і вібраційної чутливості E Втрата больової чутливості

39 У людини внаслідок довільної затримки дихання на 40с зросли частота серцевих скорочень та системний артеріальний тиск. Реалізація яких механізмів регуляції зумовлює зміни показників? A *Безумовні симпатичні рефлекси B Безумовні парасимпатичні рефлекси C Умовні симпатичні рефлекси D Умовні парасимпатичні рефлекси E Рефлекси

40 Подразнення правого блукаючого нерва спричинило різке сповільнення атріоаентрикулярного проведення. На ЕКГ про це буде свідчити подовження: A * інтервалу РQ. B комплексу QRST. C зубця Т. D зубця Р. E 2нтервалу RR.

41 У людини виник крововилив у клубочкову зону кори наднирника. Це спричинило зменшення виділення такого гормону: A *Альдостерону B Адреналіну C Прогестерону D Кортизолу E Норадреналіну

42 В результаті травми відбулося пошкодження спинного мозку (з повним переривом) на рівні першого шийного хребця. Що відбудеться з диханням ? A *Дихання припиняється B Дихання не змінюється C Зростає частота дихання D Зростає глибина дихання E Зменшиться частота дихання

43 У барокамері тзнизили тиск до 400 мм

рт.ст. Як зміниться зовнішнє дихання людини в цій камер? A *Збільшиться глибина і частота дихання B Зменшиться глибина і частота дихання C Зменшиться глибина і зросте частота дихання D Збільшиться глибина і зменшиться частота дихання E Залишиться без змін

44 У хворої людини має місце позаклітинний набряк тканин (збільшині розміри м"яких тканин кінцівок, печінки тощо). Зменшення якого параметру гомеостазу є найбільш ймовірною спричино розвиток набряку? A *Онкотичного тиску плазми крові B Осм отичного тиску плазми крові. C рН. D В"язкості. E Гематокриту.

45 У людини з хронічним захворюванням нирок порушена їх видільна функція. При аналізі крові з"ясовано, що рН венозної крові становить 7,33 .Для корекції кислотно-лужного стану пацієнту доцільно внутрішньовенно ввести розчин: A * бікарбонату натрію B хлориду натрію C глюкози D хлориду калію E хлориду кальцію

46 Під час складання іспиту у студентів "пересихає в роті". Механізмом, що зумовлює розвиток цього стану, є посилена реалізація таких рефлексів: A *Умовних симпатичних B Безумовних парасимпатичних. C Умовних парасимпатичних. D Безумовних симпатичних. E Безумовних периферичних

47 Малюк попросив Вас надути гумову кульку якомога більше за один видих. Яким з перелічених об"ємів повітря Ви скористуєтесь?

Page 110: крок 1 база 2014р_11шр-240ст (для брошури)_35грн

110

A *Життєва ємкість легень B Ємкість вдиху. C Функціональна залишкова ємкість. D Загальна ємкість легень. E Резервний об"єм вдиху.

48 Людина знаходиться в середовищі з темепературою 38oC, відносною вологістю повітря 50%. Які шляхи тепловіддачі зумовлюють підтримку постійної температури ядра тіла за цих умов? A *Випаровування B Радіація C Теплопроведення D Конвекція E Конвекція і теплопроведення

49 При переведенні погляду з близьких на далеко розташовані предмети відбувається: A *Розслаблення війчастого м’язу B Скорочення війчастого м’язу C Розслаблення цинової зв'язки D Збільшення кривизни кришталика E Збільшення заломної сили очей

50 В умовах експерименту у тварини вимірювали залежність артеріального тиску від величини судинного опору. Вкажіть судини, в яких він найбільший ? A *Артеріолиї B Артерії C Аорта D Вени E Капіляри

51 У людини, 40 років, після емоційного збудження виявили півищення артеріального тиску. Вкажіть можливу причину цього ефекту ? A *Підвищення тонусу симпатичної нервової системи B Розширення артеріол C Зменшення частоти серцевих скорочень D Гіперполяризація кардіоміоцитів. E Підвищення тонусу парасимпатичної нервової системи.

52 У хворого 30 років на електрокардіограмі відмічено зниження амплітуди зубця R. Що означає цей зубець на ЕКГ? A * Поширення збудження по шлуночкам

B Поширення збудження від передсердь до шлуночків C Електричну діастолу серця. D Реполяризацію шлуночків E Поширення збудження по передсердям

53 В експерименті при вивченні процесів збудження кардіоміоцитів встановлено, що у фазу їх швидкої деполярізації іони Nа+ можуть додатково рухатися крізь: A * Са++ -канали B К+ -канали. C Сl-- -канали D Мg++ - канали. E Li+ - канали

54 При обстеженні хворого з травматичним пошкодженням головного мозку виявлено, що він перестав розрізняти переміщення предмета по шкірі. Який відділ кори мозку пошкоджений? A *Задня центральна звивина B Потилична доля кори. C Тім'яна доля кори D Лобна доля кори E Передня центральна звивина

55 У людини добовий діурез 6 літрів, вміст глюкози в плазмі крові нормальний. Порушення секреції якого гормону є причиною цього? A *Вазопресин B Інсулін C Глюкагон D Кортизол E Окситоцин

56 Піся травми у людини були пошкоджені напівкружні канали внутрішнього вуха. На які подразники не зможе реагувати ця людина? A *Рух з кутовим прискоренням B Шкірні C Світлові D Звукові E Рух з лінійним прискоренням

57 Внаслідок травми у людини ушкоджений отолітовий апарат внутрішнього вуха. На які підразники не зможе реагувати ця людина? A *Рух з лінійним прискоренням B Рух з кутовим прискоренням

Page 111: крок 1 база 2014р_11шр-240ст (для брошури)_35грн

http://vk.com/my.printing

111

C Шкірні D Світлові E Звукові

58 У людини, що сидить з заплющеними очима, реєструють електроенцефалограму (ЕЕГ). Який ритм з"явиться на ЕЕГ, якщо подали звуковий сигнал? A *Бета B Тета C Дельта D Альфа E Гама

59 У хворого напад тахікардії. Які мембранні циторецептори кардіоміоцитів доцільно заблокувати, щоб припинити напад? A *Бета-адренорецептори B Альфа-адренорецептори C М-холінорецептори D Н-холінорецептори E М- та Н-холінорецептори

60 У людини з захворюванням нирок віявлено збільшення артеріального тиску, особливо діастолічного. Концентрація якої біологічно-активної речовини збільшена у крові хворого? A *Реніну B Адреналіну C Норадреналіну D Вазопресину E Катехоламінів

61 У хворого з пересадженим серцем при фізичному навантаженні збільшився хвилинний об"єм крові. Який механізм регуляції забезпечує ці зміни? A *Катехоламіни B Симпатичні безумовні рефлекси C Парасимпатичні безумовні рефлекси D Симпатичні умовні рефлекси E Парасимпатичні умовні рефлекси

62 У хворого на ЕКГ виявлено, що інтервал RR дорівнює 1,5 с, частота серцевих скорочень - 40 разів за хвилину. Що є водієм ритму серця? A *Атріовентрикулярний вузол

B Синусовий вузол C Пучок Гіса D Ліва ножка Гіса E Права ножка Гіса

63 У хворого після травми виявлено порушення короткочасної пам'яті. Який процес, що зумовлює механізми пам"яті, при цьому порушений? A *Реверберація збудження в ланцюгах нейронів B Структурно-функціональні зміни синапсів ЦНС C Рух іонів у мембранах рецепторів D Проведення в аферентних нейронах E Структурні зміни в нейронах ЦНС

64 Гучний звук під час умовно-рефлекторної діяльності призвів до її гальмування. Вкажіть вид гальмування, що мав місце. A *Зовнішне B Позамежне C Згасаюче D Диференціювальне E Запізніле

65 У хворого внаслідок інсульту пошкоджена задня частина першоі скроневої звивини лівої півкулі (центр Верніке). До яких наслідків це призведе? A *Порушення розуміння усної мови B Порушення рахування C Порушення відтворювання усної мови D Порушення відтворювання письмової мови E Порушення розуміння письмової мови

66 У хворого внаслідок інсульту була пошкоджена нижня частина третьої лобної звивини лівої півкулі(центр Брока). До яких наслідків це може привести? A *Порушення відтворення усної мови B Порушення розуміння усної мови C Порушення рахування D Порушення відтворювання письмової мови E Порушення розуміння письмової мови

67 Хворому видалили частину підшлункової

Page 112: крок 1 база 2014р_11шр-240ст (для брошури)_35грн

112

залози. Які продукти йому потрібно обмежити в своєму раціоні? A *Жирне та смажене м'ясо B Нежирне відварне м'ясо C Кисломолочні продукти D Овочі E Фрукти

68 Хворому з гіперсекрецією шлункового соку лікар рекомендував виключити з харчового раціону: A *М"ясні бульони B Молоко C Солодке D Солоне E Білий хліб

69 Хворому з гіперсекрецією шлункового соку лікар рекомендував виключити з харчового раціону насичені бульони і овочеві відвари, бо вони стимулюють виділення: A *Гастрину B Секретину C Холецистокініну D Соматостатину E Нейротензину

70 Хворому, у якого підвищена кислотність шлункового соку, лікар порекомендував їсти варене, а не смажене м'ясо, оскільки смажене містить речовини, які стимулюють виділення: A *Гастрину B Секретину C Соматостатину D Панкреозиміну E Нейротензину

71 У давній Індії підозрюваним у злочин пропонували проковтнути жменю сухого рису. Злочинці не могли проковтнути рис через зменшене слиновиділення внаслідок : A *Активацїя симпато-адреналової системи B Активації парасимпатичного ядра лицьового нерва C Зменшення кровопостачання слинних залоз D Активації парасимпатичного ядра язикоглоткового нерва E Гальмування симпато-адреналової системи

72 У експериментальної тварини подразнювали периферичний відрізок chorda tympani. У результаті з фістули привушної слинної залози виділялося: A *Багато рідкої слини B Мало рідкої сслини C Слина не виділяється D Мало в'язкої слини E Багато в'язкої слини

73 У експериментальної тварини подразнювали периферичний відрізок симпатичних волокон, що іннервують підязикову слинну залозу. У результаті з фістули протоки залози виділяється: A *Мало в’язкої слини B Мало рідкої слини C Слина не виділяється D Багато рідкої слини E -

74 Вміст яких продуктів доцільно збільшити у харчовому раціоні людини із зниженою секреторною функцією шлунку? A *Бульони B Солодке C Солоне D Молоко E Сало

75 Людині внутрішньовенно ввели 0,5 л ізотонічного розчину лікарської речовини. Які з рецепторів насамперед прореагують на зміни водно-сольового балансу організму? A *Волюморецептори порожнистих вен і передсердь B Осморецептори гіпоталамусу C Осморецептори печінки D Натрієві рецептори гіпоталамуса E Барорецептори дуги аорти

76 При травмі людина втратила 500 мл крові, що призвело до зменшення діурезу. Вплив якого гормону на нирки забезпечив, перш за все, цю пристосувальну реакцію? A *Вазопресин B Натрійуретичні фактори C Альдостерон D Кортизол

Page 113: крок 1 база 2014р_11шр-240ст (для брошури)_35грн

http://vk.com/my.printing

113

E Ренін

77 За обідом людина з'їла солоного оселедця і картоплю з солоним огірком. Через деякий час у неї виникло відчуття спраги. Збудження яких рецепторів викликало у неї це відчуття? A *Осморецептори гіпоталамусу B Волюморецептори порожнистих вен і передсердь C Барорецептори каротидних синусів D Волюморецептори гіпоталамусу E Барорецептори дуги аорти

78 Після здачі крові у студента виникло відчуття спраги Збільшення секреції якої біологічно активної речовини сприяє цьому? A *Ангіотензин B Альдостерон C Еритропоетини D Адреналін E Норадреналін

79 В досліді з ізольованою ниркою кроля в перфузійний розчин додали 40 % розчин глюкози. Кількість сечі збільшилась тому, що: A *не вся глюкоза реабсорується B Збільшується осмотичний тиск перфузата C Збільшується осмотичний тиск первинної сечі D Збільшиться гідростатичний тиск перфузата E Збільшилась проникність ниркового фільтру

80 В гострому досліді собаці, що знаходилась під наркозом, ввели вазопресин, внаслідок чого зменшилась кількість сечі тому, що він: A *Посилює реабсорбцію води B Посилює реабсорбцію натрію C Зменшує реабсорбцію води D Зменшує реабсорбцію кальцію E Збільшує реабсорбцію кальцію

81 Отруєння ботуліністичним токсином, який блокує вхід іонів кальцію до нервових закінчень аксонів мотонейронів, небезпечно для життя, бо загрожує:

A * Зупинкою дихання B Зупинкою серця C Розладом тонусу судин D Розвитком блювоти E Розвитком проносу

82 Трансплантована нирка реагує на больві подразнення з зупинкою сечовиділення. Чим зумовлена ця реакція? A *Збільшення секреції АДГ B Зниження секреції АДГ C Впливом парасимпаттичної нервової системи D Впливом симпатичної нервової системи E Зниженням секреції АКТГ

83 На ізольованому серці шляхом охолодження припиняють функціонування окремих структур. Яку структуру охолодили, якщо серце внаслідок цього спочатку припинило скорочення, а далі відновили її з частотою, у 2 рази меншою за вихідну? A *Синоатріальний вузол B Атріоветрикулярний вузол C Пучок Гіса D Ніжки пучка Гіса E Волокна Пуркін’є

84 При аналізі ЕКГ людини з’ясовано,що у другому стандартному відведенні від кінцівок зубці Т позитивні, їх амплітуда та тривалість нормальні. Вірним є висновок, що у шлуночках серця нормально відбувається процес: A *Реполяризації B Деполяризаці. C Збудження D Скорочення E Розслаблення

85 При обробці атипових кардіоміоцитів біологічно активною речовиною зареєстровано збільшення їх мембранного потенціалу через збільшену проникність для іонів калію. Що впливало на кардіоміоцити? A *Ацетилхолін B Адреналін C Норадреналін

Page 114: крок 1 база 2014р_11шр-240ст (для брошури)_35грн

114

D Тироксин E Атріопептид

86 У жінки 30 років хвилинний об’єм крові у стані спокою становить 5 л/хв. Який об’єм крові проходить у неї через судини легень за 1 хвилину? A *5 л B 3,75 л C 2,5 л D 2,0 л E 1,5 л

87 У кроля через місяць після хірургічного звуження ниркової артерії зареєстровано суттєве підвищення системного артеріального тиску. Який з наведених механізмів регуляції спричинив зміну тиску у тварини? A *Ангіотензин-11 B Вазопресин C Адреналін D Норадреналін E Серотонін

88 Безпосередньо після переходу з горизонтального положення у вертикальне у мужчини частота серцевих скорочень збільшилась на 15 скорочень за хвилину. Які механізми регуляції переважно зумовлюють цю зміну? A *Безумовні симпатичні рефлекси B Умовні симпатичні рефлекси C Умовні та безумовні симпатичні рефлекси D Катехоламіни E Симпатичні рефлекси і катехоламіни

89 Яка з сполук гемоглобіну утворюється у мешканців будівлі якщо зарано перекрити димохід? A *Карбоксигемоглобін B Карбгемоглобін C Дезоксигемоглобін D Метгемоглобін E Оксигемоглобін

90 У тварини заблокували діяльність підслизового нервового сплетіння тонкої кишки. На якому з зазначених процесів це позначиться найбільш негативно? A *Секреція кишкового соку

B Пристінкове травлення C Ритмічна сегментація D Маятникоподібні рухи E Всмоктування

91 У людини внаслідок тривалого голодування швидкість клубочкової фільтрації зросла на 20%. Найбільш ймовірною причиною змін фільтрації в зазначених умовах є: A *Зменшення онкотичного тиску плазми крові B Збільшення системного артеріального тиску C Збільшення проникності ниркового фільтру D Збільшення коефіцієнта фільтрацїі. E Збільшення ниркового плазмотоку

92 У людини з масою 80 кг після тривалого фізичного навантаження об’єм циркулюючої крові зменшився, гематокрит -50%, загальний білок крові - 80 г/л. Такі показники крові є наслідком, перш за все: A *Втрати води з потом B Збільшення кількості еритроцитів C Збільшення вмісту білків в плазмі D Збільшення онкотичного тиску плазми E Збільшення діурезу

93 У вагітної жінки визначили групу крові. Реакція аглютинації еритроцитів відбулася зі стандартними сироватками груп 0, альфа-, бета (I), В,альфа- (III), та не виникла - з сироваткою А,бета- (II). Дослуджувана кров належить до групи: A * А, бета (II) B В, альфа-(III) C О, альфа-, бета-(I) D АВ (IV) E -

94 У людини 40 років з масою тіла 80 кг під час стресу виявили, що загальний час зсідання крові становив 2 хв., що є наслідком дії на гемокоагуляцію, перш за все: A *Катехоламінів B Кортізолу C Альдостенору D Соматотропіну E Вазопресину

95 Недбалий студент раптово зустрівся з

Page 115: крок 1 база 2014р_11шр-240ст (для брошури)_35грн

http://vk.com/my.printing

115

деканом. Концентрація якого гормона найшвидше збільшиться в крові студента? A *Адреналіна B Тиреоліберина C Кортикотропіна D Кортизола E Соматотропіна

96 При обстеженні спортсмена після інтенсивного фізичного навантаження виявлено порушення координації рухів при збереженні сили скорочення м’язів. Причиною цього може бути зменшення швидкості проведення збудження: A *Через центральні синапси B Через нервово-м’язові синапси C Еферентними нервами D Аферентними нервами E Провідними шляхами

97 В експерименті подразнюють скелетний м’яз серією електричних імпульсів. Який вид м’язового скорочення буде виникати, якщо кожний наступний імпульс припадає на період скорочення поодинокого м’язового скорочення? A *Суцільний тетанус B Зубчастий тетанус C Асинзхронний тетанус D Серія поодиноких скорочень E Контрактура м’яза

98 На ізольованому серці кроля частково заблокували кальцієві канали кардіоміоцитів. Які зміни серцевої діяльності відбудуться внаслідок цього? A *Зменшення частоти і сили скорочень B Зменшення частоти скорочень C Зменшення сили скорочень D Зупинка серця в діастолі E Зупинка серця в систолі

99 Тварині через зонд у дванадцятипалу кишку ввели слабкий розчин хлористоводневої кислоти. Вміст якого гормону збільшиться внаслідок цього у тварини? A *Секретин

B Холецистокінін-панкреозимін C Гастрин D Глюкагон E Нейротензин

100 Під час фізичного навантаження людина менш чутлива до болю. Причиною цього є активація: A *Антиноцицептивної системи B Ноцицептивної системи C Функції щитовидних залоз D Симпатоадреналової системи E Функції наднирників

101 У хворого виявляється повна демієлінізація волокон провідних висхідних шляхів. Формування яких відчуттів при цьому погіршиться найменше? A *Температурних. B Пропріоцептивних. C Зорових. D Дотикових. E Слухових.

102 У хворого хронічний неврит трійчастого нерва. Який з травних процесів буде порушений в найзначній мірі ? A *Жування. B Слиновиділення. C Формування відчуття смаку D Ковтання. E Слиноутворення

103 У здорової людини фізичне навантаження викликало помірне зниження діастолічного тиску. В чому причина цього явища ? A * Зниження тонусу судин у м'язах. B Посилення роботи серця C Зменшення еластичності судин. D Зменшення об"єму циркулюючої крові E Збільшення опору судин

104 У дитини від народження знижена функція щитовидної залози. Що є головним наслідком цього? A *Кретинізм. B Нанізм. C Гігантизм. D Гіпопітуітарізм. E Гіперпігментація шкіри.

Page 116: крок 1 база 2014р_11шр-240ст (для брошури)_35грн

116

105 У хворого виявлено різке зниження активності сурфактанту легень. Що буде наслідком цього? A *Схильність альвеол до спадання. B Зменшення опору дихальних шляхів. C Зменшення роботи дихальних м'язів. D Збільшення вентіляції легень. E Гіпероксемія.

106 Яка з зорових функцій порушується найбільше при пошкодженні паличок? A *Периферічний зір. B Кольоровий зір. C Бінокулярний зір. D Центральний зір. E Світлова адаптація.

107 В експериментіна тварині досліджують серцевий цикл . Закриті усі клапани серця. Якій фазі це відповідає? A *Ізометричного скорочення. B Асинхронного скорочення. C Протодіастолічний період. D Швидкого наповнення. E Повільного наповнення.

108 Внаслідокфізичної роботи знизилась працездатність людини. Зміни у яких структурах, перш за все, є причиною втоми? A *Нервові центри. B М’язи C Аферентні нерви D Еферентні нерви. E Нервово-м"язові синапси

109 Якщо температура повітря 38 градусів за Цельсієм, відносна вологість повітря 80%, швидкість вітру 0 м/с, то тепловіддача буде проходити за рахунок A *Випаровування поту B Радіації C Конвекції D Теплопроведення E Радіаційної конвекції

110 При переведенні погляду на близько розташований об'єкт, заломна сила оптичних серед

ока збільшилась на 10 діоптрій. Це є наслідком змін: A *Кришталика B Рогівки C Скловидного тіла D Вологи передньої камери ока E М'яза, розширює зіницю

111 Якщо в умовах високої освітленості спостерігається стійке розширенням зіниці, то це є наслідком: A *надмірноїактивності симпатичної нервової системи B нормального стану механіхмів регуляції C надмірної активності парасимпатичної нервової системи D паралічу м"яза, що розширює зіницю E паралічу ціліарного м"язу

112 У хворого високий артеріальний тиск внаслідок збільшеного тонусу судин. Дл я зниження тиску доцільно призначити блокатори: A *альфа-адренорецепторів B бета-адренорецепторів C альфа- та бета-адренорецепторів D М-холінорецепторів E Н1-рецепторів

113 Якщо дихальний об'єм ДО = 450 мл, а частота дихання ЧД = 20 за 1 хв. то альвеолярна вентиляція АВ дорівнює: A *6000 мл B 3000 мл C 4000 мл D 5000 мл E 8000 мл

114 Крива дисоціації оксигемоглобіну зміщена вправо. Які зміни в організмі людими можуть бути причиною цього? A *Гіпертермія B Збільшення концентрації 2,3-дифосфогліцерату в еритроцитах C Алкалоз D Гіпокапнія E Гіпоксемія

115 Зріст дорослої людини 100 см при пропорціональній будові тіла та нормальному

Page 117: крок 1 база 2014р_11шр-240ст (для брошури)_35грн

http://vk.com/my.printing

117

розумовому розвитку. Недостатнє вироблення якого гормону в дитячому віці є причиною цього? A *Соматотропіну B Гонадотропного C Адренокортікотропного D Тиреотропного E Пролактіну

116 У хворого після черепно-мозкової травми дихання стало рідким і глибоким. Де знаходиться пошкодження? A *Задній мозок B Гіпоталамус C Довгастий мозок D Кора великих півкуль E Мозочок

117 У хворого поперечний розрив спинного мозку нижче VІ грудного сегменту. Як внаслідок цього зміниться дихання? A Суттєво не зміниться B Зупиниться C Стане більш рідким D Стане більш глибоким E Стане більш частим

118 Після введення мікроелектродів у структури проміжного мозку тварина повністю втратила зір. Яка з підкоркових структур можливо при цьому була пошкоджена ? A *Латеральне колінчасте тіло B Медіальне колінчасте тіло C Асоціативні ядра таламуса D Супраоптичне ядро гіпоталамуса E Супрахіазматичне ядро гіпоталамуса

119 У приміщенні підвищений вміст вуглекислого газу. Як зміниться дихання (глибина і частота) у людини, що увійшла в це приміщення ? A *Збільшиться глибина і частота B Зменшиться глибина C Збільшиться глибина D Зменшиться частота E Збільшиться частота

120 У жінкив при дуоденальному зондуванні

після виведення до 12-ої палої кишки 30 мл рідкого масла не відбулося випорожнення жовчного міхура. Причиною цього може бути недостатнє виділення: A *Холецистокініну B Гастрину C Мотиліну D Бомбезину E Секретину

121 У пацієнта має місце зменшення швидкості проведення збудження по атріовентрикулярному вузлу. На ЕКГ при цьому буде реєструватися збільшення тривалості: A *Інтервалу Р-Q B Зубця Р C Іінтервалу R-R D Комплексу QRS E Сегмента S-T

122 Під час операції на головного мозку відмічено, що подразнення певних зони кори великих півкуль викликало у хворого і тактильні і температурні відчуття. На яку саме зону діяли подразники? A *Постцентральна звивина B Прецентральна звивина C Верхня латеральна звивина D Поясна звивина E Парагіпокампова звивина

123 У чоловіка 60 років крововилив у головний мозок спричинив тривалий сон. Пошкодження якої структури найвірогідніше призвело до цього стану? A *Ретикулярної формації B Гіпокампу C Чотиригорбикової структури D Кори великих півкуль E Чорної субстанції

124 Після введення жабі стрихніну вона на найменше подразнення відповідає генералізованими судомами. Причиною цього є блокада у ЦНС: A *Гальмівних синапсів B Збуджувальних синапсів C Клітин Реншоу D Адренорецепторів

Page 118: крок 1 база 2014р_11шр-240ст (для брошури)_35грн

118

E Холінорецепторів

125 Внаслідок блокади іонних каналів мембрани клітини її потенціал спокоюзменшився з -90 до -70 мВ. Які канали заблоковані? A *Калієві B Натрієві C Кальцієві D Магнієві E Хлорні

126 Людина стоїть у кімнаті в легкому одязі; температура повітря +14oС. Вікна і двері зачинені. Яким шляхом вона віддає найбільше тепла? A *Теплорадіація B Теплопроведення C Конвекція D Випаровування E Перспірація

127 У здорової дорослої людини проводять зондування порожнин серця і великих судин. Де знаходиться зонд, якщо протягом серцевого циклу зареєстровані зміни тиску від 0 до 120 мм рт.ст.? A *Лівий шлуночок B Правий шлуночок C Аорта D Легенева артерія E Передсердя

128 Піддослідному собаці через зонд у порожнину шлунку ввели 150 мл м’ясного бульону. Вміст якої з наведених речовин швидко збільшиться у крові тварин? A *Гастрин B Соматостатин C Інсулін D Нейротензин E Вазоінтестинальний поліпептид

129 У людини зменшений діурез, гіпернатріємія, гіпокаліємія. Гіперсекреція якого гормона може бути причиною таких змін? A *Альдостерон B Вазопресин C Передсердний натрійуретичний фактор D Адреналін

E Паратгормон

130 В експерименті на кролі через 2 тижні після звуження ниркової артерії виявлено збільшення кількості еритроцитів та гемоглобіну в крові внаслідок стимуляції еритропоезу еритропоетинами. Що посилює утворення еритропоетинів? A *Гіпоксемія B Гіперкапнія C Гіперосмія D Гіпоосмяї E Гіповолюмія

131 Методом непрямої калориметрії встановлено, що основний обмін досліджуваного на 40% нижче належного. Порушення діяльності якої ендокринної залози є причиною ? A *Щитовидна залоза. B Тіиус. C Підшлункова залоза. D Епіфіз. E Наднирники.

132 У хворого виник спазм гладенької мускулатури бронхів. Фізіологічно обгрунтованим буде використання для зняття нападу активаторів: A *бета-адренорецепторів B альфа-адренорецепторів C альфа- та бета-адренорецепторів D Н-холінорецепторів E М-холінорецепторів

133 У собаки в досліді подразнювали на шиї периферичний відрізок блукаючого нерва;при цьому спостерігали такі зміни серцевоїдіяльності: A *Зменшення частоти скорочень B Збільшення сили скорочень C Збільшення атріовентрикулярного проведення D Збільшення частоти та сили скорочень E Збільшення збудливості міикарда

134 У тварини в експерименті реєструють електричну активність нейронів спирального вузла, що дозволяє аналізувати аферентну

Page 119: крок 1 база 2014р_11шр-240ст (для брошури)_35грн

http://vk.com/my.printing

119

імпульсацію від рецепторів: A *кортієвого органа B присінкових C напівкружних каналів D вестибулярних E вестибулярних і кортієвого органа

135 При визначенні групи крові за системою АВО аглютинацію еритроцитів досліджуваної крові викликали стандартні сироватки I та II груп і не викликала - III групи. Які аглютиногени містяться в цих еритроцитах ? A *В B А C А та В D С E D та C

136 Чоловіку 35 років з виразковою хворобою зроблено резекцію антрального відділу шлунку. Секреція якого гастроінтестинального гормону внаслідок операції буде порушена найбільше ? A *Гастрин. B Гістамін. C Секретин D Холецистокінін E Нейротензин

137 У хворого на ЕКГ виявили збільшення тривалості зубця Т. Це є наслідком зменшення в шлуночках швидкості: A *реполярізаці B деполярізації та реполярізациї C деполярізації D скорочення E розслаблення

138 Жінка 25 років через місяць після пологів звернулась до лікаря зі скаргою на зниження утворення молока. Дефіцит якого гормону призвів до такого стану? A *Пролактину. B Соматостатіну. C Адренокортикотропного гормону. D Інсуліну. E Глюкагону.

139 Зріст дитини 10 років сягає 178 см, маса - 64 кг. З порушенням діяльності якої ендокринної залози це пов'язано? A *Гіпофізу B Щитовидної залози C Статевих залоз D Надниркових залоз E Паращитовидної залози

140 Під час хірургічної операції виникла необхідність масивного переливання крові. Група крові потерпілого - III (В) Rh+. Якого донора треба вибрати? A *III (В) Rh+ B I (О) Rh- C II (А) Rh+ D IV (АВ) Rh- E III (В) Rh-

141 В эксперименте на животном, которое удерживали на весу спиной книзу, ногами кверху, наблюдали рефлекторный поворот головы, направленный на восстановление нормального положения головы в пространстве. С раздражением каких рецепторов связан указанный рефлекс? A *Вестибулорецепторов преддверия B Внутренних органов C Проприорецепторов конечностей D Тактильных рецепторов конечностей E Вестибулорецепторов полукружных каналов

142 Верхние конечности стоящего человека в состоянии покоя находятся в легком сгибании. Что является причиной указанного состояния конечностей? A *Рефлекс с мышечных веретен при растяжении двуглавой мышцы B Врожденная готовность к действию C Антагонистический рефлекс со стороны разогнутых нижних конечностей D Рефлекс с рецепторов преддверия вестибулярного аппарата E Тонизирующее влияние лимбических структур и новой коры

143 Рост взрослого человека составил 100 см

Page 120: крок 1 база 2014р_11шр-240ст (для брошури)_35грн

120

при пропорциональном телосложении и нормальном умственном развитии. Для недостаточности выработки какого гормона характерны указанные признаки? A *Соматотропного гормона B Тироксина C Гонадотропных гормонов D Минералокофноидов E Антидиуретического гормона

144 В эксперименте на животном были повреждены нервные пути, проходящие в ножке гипофиза, что нарушило поступление в кровь следующих гормонов: A *Вазопрессина и окситоцина B Гормонов гипофиза C Гормонов аденогипофиза D Тиреотропного гормона E Аденокортикотропного гормона

145 У літніх людей підвищується частота виникнення пухлин. Одна з основних причин цього: A *Зниження активності клітинного імунітету B Зростання порушення мітозів C Підвищення активності утворення антитіл D Зниження інтенсивності утворення антитіл E Підвищення активності клітинного імунітету

146 Після побутової травми у пацієнта 18 років з'явились постійні запаморочення, ністагм очей, скандована мова, невпевнена хода. Це свідчить про порушення функції: A *Мозочка B Рухової кори C Базальних гангліїв D Чорної субстанції E Вестибулярних ядер

147 При лабораторному дослідженні крові пацієнта 33 років виявлено реакцію аглютинації еритроцитів в стандартних сироватках I і II груп. Реакції аглютинації з сироваткою III групи і антирезусною сироваткою не відбулась. Кров якої групи, враховуючи систему СДЕ, можна переливати в разі потреби? A *III (B) Rh- B I (О) Rh+ C II (A) Rh- D IV (AB) Rh+

E IV (AB) Rh-

148 У пацієнта 36 років після дорожньої травми виникли параліч м'язів кінцівок справа, втрата больової і температурної чутливості зліва, часткове зниження тактильної чутливості з обох сторін. Для ураження якого відділу мозку вказані зміни є найбільш характерними? A *Правої половини спинного мозку B Рухової кори зліва C Лівої половини спинного мозку D Передніх стовбів спинного мозку E Задніх стовбів спинного мозку

149 Дефіцит якого ферменту найчастіше є причиною неповного перетравлення жирів в шлунково-кишковому тракті і збільшення кількості нейтрального жиру в калі? A *Панкреатичної ліпази B Шлункової ліпази C Печінкової ліпази D Кишкової ліпази E Ентерокінази

150 У пацієнта 18 років при лабораторному обстеженні виявлено наявність глюкози в сечі при нормальній концентрації її в плазмі крові. Найвірогіднішою причиною цього є порушення: A *Канальцевої реабсорбції B Клубочкової фільтрації C Канальцевої секреції D Секреції інсуліну E Секреції глюкокортикоїдів

151 При штовханні штанги спортсмен закидає голову назад для максимального підвищення тонусу м'язів-розгиначів верхніх кінцівок. Де розташовані центри рефлексів, які при цьому виникають? A *В ядрах Дейтерса B В руховій корі C В базальних гангліях D В червоних ядрах E В спинному мозку

152 В результате несчастного случая произошла обтурация трахеи легкого. Какой

Page 121: крок 1 база 2014р_11шр-240ст (для брошури)_35грн

http://vk.com/my.printing

121

этап дыхания нарушится первым? A *Вентиляция легких B Газообмен в легких C Транспорт кислорода и углекислого газа D Газообмен в тканях E Тканевое дыхание

153 Проводять експеримент на спінальній жабі. Після збільшення площі шкіри, на яку діє розчин кислоти, час захисного згинального рефлексу зменшився з 10 до 6 секунд. Який з зазначених механізмів лежить в основі скорочення часу рефлексу? A *Просторова сумація збудження. B Iррадіація збудження дивергентними нервовими ланцюгами. C Часова сумація збудження D Принцип домінанти E Рециркуляція збудження

154 У практиці невідкладної терапії та реанімації нерідко зустрічаються стани, що супроводжуються набряком клітин мозку. Для боротьби з цим в організм хворих доцільно вводити речовини, які: A *Підвищують колоїдно-осмотичний тиск крові B Змінюють кислотно-лужний баланс крові C Понижують системний артеріальний тиск D Понижують центральний венозний тиск E Зменшують ОЦК

155 У кішки в експерименті проводять подразнення однієї з рухових структур головного мозку, внаслідок чого спостерігається підвищення тонусу м'язів-розгиначів з боку стимуляції. У тварини проводили подразнення: A *Nucleus vestibularis lateralis B Nucleus caudatus C Nucleus ruber D Nucleus reticularis medialis E Nucleus intermedius lateralis

156 При копрологічному дослідженні встановлено, що кал знебарвлений, у ньому знайдено краплі нейтрального жиру. Найбільш імовірною причиною цього є порушення: A *Надходження жовчі в кишечник

B Кислотності шлункового соку C Секреції підшлункового соку D Секреції кишкового соку E Процесів всмоктування в кишечнику

157 У людини виділяється мало густої слини, знижена її ферментативна активність, збільшений вміст слизу. Найбільш імовірною причиною цього є порушення функції: A *Привушних залоз B Власних залоз слизової оболонки C Під'язикових залоз D Піднижньощелепних залоз E -

158 У чоловіка 45 років через 3 роки після операції видалення шлунка вміст еритроцитів в крові складає 2,0*1012/л, Hb - 85 г/л, кольоровий показник - 1,27. Порушення всмоктування якого вітаміну викликало зміни еритропоезу? A *B12 B С C А D Р E B6

159 Після накладання джгута у досліджуваного спостерігаються точкові крововиливи на поверхні передпліччя (15 штук). З порушенням функції яких клітин крові це пов'язано? A *Тромбоцитів B Еритроцитів C Базофілів D Нейтрофілів E Макрофагів

160 У чоловіка при ураженні одного з відділів ЦНС спостерігається астенія, м'язова дистонія, порушення рівноваги. Який з відділів ЦНС уражено? A *Мозочок B Чорна субстанція C Ретикулярна формація D Червоні ядра E Вестибулярні ядра

161 У кішки в експерименті спостерігається

Page 122: крок 1 база 2014р_11шр-240ст (для брошури)_35грн

122

підвищений тонус м'язів-розгиначів кінцівок та спини (децеребраційна ригідність). На якому рівні зроблено переріз головного мозку? A *Нижче червоних ядер B Між спинним та довгастим мозком C Нижче вестибулярних ядер D Вище червоних ядер E Спинного мозку

162 У тварини в експерименті перерізали спинний мозок вище 5-го шийного сегменту. Як зміниться характер дихання? A *Припиниться B Стане поверхневим і рідким C Стане глибоким і частим D Стане поверхневим і частим E Стане глибоким і рідким

163 Які зміни з боку ізольованого серця жаби можна очікувати після введення в перфузійний розчин надлишкової кількості хлористого кальцію? A *Збільшення частоти і сили скорочень B Зменшення сили скорочення C Збільшення частоти скорочень D Збільшення сили скорочень E Зупинка серця в діастолі

164 При визначенні групи крові по системі АВО за допомогою стандартних сироваток були отримані наступні результати: аглютинація відбулася в сироватках I та II груп і не відбулася в сироватці III групи. Яка група досліджуваної крові? A *III (B) B II (A) C IV (AB) D I (O) E Неможливо визначити

165 При визначенні групи крові по системі АВО за допомогою стандартних сироваток були отримані наступні результати: аглютинація відбулася в сироватках I, II та III груп. Яка група досліджуваної крові? A *IV (AB) B III (B) C II (A) D I (O) E Неможливо визначити

166 Проводять дослідження на ізольованій збуджуваній клітині. Встановлено, що поріг сили подразнення клітини суттєво зменшився. Що із зазначеного може бути причиною цього? A *Активація натрієвих каналів мембрани B Iнактивація натрієвих каналів мембрани C Iнактивація кальцієвих каналів мембрани D Активація калієвих каналів мембрани E Блокада енергоутворення у клітині

167 Який механізм тепловіддачі найбільш ефективно спрацьовує при перебуванні людини в умовах 80% вологості повітря та температурі навколишнього середовища +35oС? A *Потовиділення B Радіація C Теплопровідність D Конвекція E -

168 У хворого на ЕКГ виявлено збільшення тривалості інтервалу QT. Це може бути наслідком зменшення у шлуночках швидкості: A *Деполяризації та реполяризації B Деполяризації C Реполяризації D Скорочення E Розслаблення

169 При обстеженні чоловіка 45 років, що тривалий час перебував на рослинній дієті, виявлено негативний азотистий баланс. Яка особливість раціону стала причиною цього явища? A *Недостатня кількість білків B Надмірна кількість води C Надмірна кількість вуглеводів D Недостатня кількість жирів E Недостатня кількість жирів і білків

170 У чоловіка 60 років діагностований інсульт у ділянці латеральних ядер гіпоталамуса. Які зміни поведінки слід чекати при цьому? A *Відмова від їжі B Агресивність

Page 123: крок 1 база 2014р_11шр-240ст (для брошури)_35грн

http://vk.com/my.printing

123

C Депресія D Спрага E Ненаситність

171 У спортсмена на старті перед змаганнями відзначається підвищення артеріального тиску та частоти серцевих скорочень. Впливом яких відділів ЦНС можна пояснити вказані зміни? A *Кори великих півкуль B Довгастого мозку C Середнього мозку D Проміжного мозку E Гіпоталамуса

172 При профогляді у людини, що не має скарг на стан здоров'я, виявлено лейкоцитоз. Причиною цього може бути те, що кров для аналізу здана після: A *Фізичного навантаження B Розумової праці C Відпочинку на курорті D Значного вживання води E Вживання алкоголю

173 При обстеженні чоловіка виявлено зменшення моторно-евакуаторної функції шлунку. З дефіцитом якого з наведених факторів це може бути пов'язано? A *Гастрин B Секретин C Аденозін D Соматостатин E Шлунково-інгібуючий пептид

174 У процесі старіння людини спостерігається зменшення синтезу та секреції підшлункового соку, зменшення вмісту в ньому трипсину. Це призводить до порушення розщеплення: A *Білків B Фосфоліпідів C Полісахаридів D Нуклеїнових кислот E Ліпідів

175 При аналізі спірограми у обстежуваного встановлено зменшення частоти і глибини дихання. Це призведе до зменшення: A *Хвилинного об'єму дихання

B Життєвої ємності легень C Резервного об'єму вдиху D Резервного об'єму видиху E Залишкового об'єму

176 У передстартовому стані бігуну необхідно підвищити вміст О2 у м’язах. Яким чином це можна зробити? A *Дихати в режимі гіпервентиляції B Дихати в режимі гіповентиляції C Робити швидкий вдих та повільний видих D Дихати поверхнево E Дихати з низькою частотою

177 Після вдихання пилу у людини виник кашель, що обумовлено збудженням: A *Іритантних рецепторів B Юкстакапілярних рецепторів C Хеморецепторів легень D Терморецепторів легень E Больових рецепторів

178 Аналіз крові жінки виявив підвищення швидкості осідання еритроцитів (ШОЕ), що обумовлено: A *Вагітністю B Фізичною працею C Втратою крові D Стресом E Прийомом їжі

179 При аналізі крові виявлено незначне підвищення кількості лейкоцитів (лейкоцитоз), без змін інших показників. Причиною чього може бути, що перед дослідженням людина : A *Поснідала B Не снідала C Погано спала D Палила тютюн E Випила 200 мл води

180 При палінні тютюну у людини часто виникає кашель. Подразнення яких рецепторів запускає цей рефлекс? A *Іритантних B Цетральних хеморецепторів C Хеморецепторів дуги аорти D Хеморецепторів каротидних синусів E Механорецепторів легень

Page 124: крок 1 база 2014р_11шр-240ст (для брошури)_35грн

124

181 У людини, яка вийшла з теплого приміщення на холодне повітря часто виникає кашель. Подразнення яких рецепторів запускає рефлекс кашлю? A *Іритантних B Цетральних хеморецепторів C Хеморецепторів дуги аорти D Хеморецепторів каротидних синусів E Механорецепторів легень

182 У дитини 10 років видалено задню частку гіпофіза в зв’язку з пухлиною. Це призведе до : A *Збільшення діурезу B Зменшення діурезу C Затримки росту D Затримки розумового розвитку E Гіперглікемії

183 У хворого видалено 12-палу кишку. Це призведе до зменшення секреції, перш за все: A *Холецистокініну та секретину B Гастрину C Гістаміну D Гастрину та гістаміну E Нейротензину

184 При переході здорової людини із положення лежачи в положення стоячи виникають наступні компенсаторні механізми: A *Збільшення ЧСС B Зменшення ЧСС C Зниження діастолічного артеріального тиску D Зменшення тонусу судин E Зменшення загального периферичного опіру

185 У здорової дорослої людини швидкість проведення збудження через атріовентрикулярний вузол дорівнює 0,02-0,05 м за 1 секунду. Атріовентрикулярна затримка забезпечує : A *Послідовність скорочення передсердь та шлуночків B Одночасність скорочення обох передсердь C Одночасність скорочення обох шлуночків D Достатнью силу скорочення передсердь E Достатнью силу скорочення шлуночків

186 У дитини 2-х років виникли судоми внаслідок зниження концентрації іонів кальцію в плазмі крові. Це обумовлено зниженням функції: A *Прищитовидних залоз B Гіпофізу C Кори наднирників D Шишковидної залози E Тимусу

187 В експерименті на собаці електростимуляція барорецепторів каротидного синусу призвела до: A *Розширення судин B Звуження судин C Збільшення частоти скорочень серця D Збільшення хвилинного об'єму крові E Збільшення систолічного об'єму

188 У людей, що проживають в гірській місцевості, має місце підвищення вмісту еритроцитів, що може бути обумовлено підвищенням продукції в нирках: A *Еритропоетину B Реніну C Урокінази D Простагландинів E Вітаміну Д3

189 У хворих з вадами серця часто виявляють підвищений вміст в крові гормона, який збільшує реабсорбцію натрія і води і виробляється в серці. Який з гормонів має таку дію? A *Натрійуретичний гормон B Ренин C Альдостерон D Вазопресин E Адреналін

190 Секреція яких гормонів гіпофізу гальмується після прийому оральних контрацептивів, які містять статеві гормони? A *гонадотропних B вазопресину C тиреотропного D соматотропного E окситоцину

Page 125: крок 1 база 2014р_11шр-240ст (для брошури)_35грн

http://vk.com/my.printing

125

191 У хворого 35 років спостерігається збільшення маси тіла, зниження температури тіла, сухість шкіри, пригнічення функції центральної нервової системи, брадикардія. Фукція якої залози знижена? A *Щитовидної B Прищивидних C Підшлункової D Мозкової речовини наднирників E Статевих залоз

192 Під час підготовки пацієнта до операції на серці проведено вимірювання тиску в камерах серця. В одній з них тиск протягом серцевого циклу змінювався від 0 мм рт. ст до 120 мм рт. ст. Назвіть цю камеру серця. A *Лівий шлуночок. B Правий шлуночок. C Праве передсердя. D Ліве передсердя. E -

193 У людини частота серцевих скорочень постійно утримується на рівні 40 разів за хвилину. Що є водієм ритму? A *Атріовентрикулярний вузол. B Синоатріальний вузол C Пучок Гіса. D Ніжки пучка Гіса. E Волокна Пуркін'є.

194 Під час хірургічного втручання на органах черевної порожнини сталася рефлекторна зупинка серця. Де знаходиться центр рефлексу? A *Довгастий мозок. B Спинний мозок. C Середній мозок. D Проміжний мозок. E Кора великих півкуль.

195 Лікар швидкої допомоги констатував у потерпілого прояви отруєння чадним газом. Яка сполука стала причиною цього? A *Карбоксигемоглобін.

B Карбгемоглобін. C Метгемоглобін. D Дезоксигемоглобін. E Оксигемоглобін.

196 Після гіпервентиляції у спортсмена спостерігається короткочасна зупина дихання. Які зміни в крові це зумовлюють? A *Зменшення напруги СО2 B Зменшення рН C Збільшення напруги СО2 D Зменшення напруги О2 E Збільшення напруги СО2 і О2

197 Після перерізки мозку у кішки виникає децеребраційна ригідність – різке підвищення тонусу м'язів-розгиначів. На якому рівні мозку зробили переріз? A *Між середнім і заднім мозком. B Між проміжним і середнім мозком. C Між довгастим і спинним мозком. D Між проміжним і кінцевим E Між довгастим мозком і мостом.

198 Після тривалого тренування у спортсмена розвинулося втомлення з різким зниженням працездатності. У якій ланці рефлекторної дуги втомлення виникло в першу чергу? A *У нервових центрах. B В аферентному провіднику. C У рецепторах. D В еферентному провіднику. E У м'язах.

199 Внаслідок дії електричного струму на збудливу клітину виникла деполяризація її мембрани. Рух яких іонів через мембрану відіграє основну роль в розвитку деполяризації? A * Na+ B НСО3- C Са2+ D Сl- E К+

200 У виробничому приміщенні температура повітря - 36 0С, відносна вологість повітря – 80\%, Переважно яким шляхом віддається тепло організмом людини за цих умов? A * Випаровування поту

Page 126: крок 1 база 2014р_11шр-240ст (для брошури)_35грн

126

B Теплопроведення C Радіація D Конвекція E -

201 Піддослідному собаці через зонд у 12-палу кишку ввели слабкий розчин соляної кислоти. Це, перш за все, призведе до підсилення секреції: A *Секретину B Гастрину C Гістаміну D Холецистокініну E Нейротензину

202 Піддослідному змастили кінчик язика місцевим анестетиком. Це призведе до відсутності сприйняття смаку: A *солодкого B солоного C кислого D гіркого E кислого та солоного

203 На останньому місяці вагітності вміст фібриногену в плазмі крові в 2 рази вище норми. Які величини швидкості осідання еритроцитів слід при цьому очікувати ? A *40 – 50 мм / годину B 0 – 5 мм / годину C 10 – 15 мм / годину D 5 – 10 мм / годину E 3 - 12 мм / годину

204 У дитини ознаки затримки психічного і фізичного розвитку (кретинізм). З дефіцитом якого гормону це пов'язано? A *Тироксину B Соматотропного. C Кальцитоніну. D Інсуліну. E Тестостерону.

205 При загальному дослідження пацієнта звертає на себе увагу потовщення шиї, екзофтальм, підвищення температури тіла, пульс 110 уд/хв. Вміст яких гормонів доцільно визначити в крові ?

A *Тироксину. B Статевих. C Катехоламінів. D Інсуліну. E Кортизолу.

206 У жінки протягом останніх 6 місяців є ознаки макулінізації: підсилення росту волосся на обличчі, по білій лінії живота, ногах. Нерегулярний менструальний цикл. Причиною цього може бути підвищення секреції: A *Андрогенів. B Естрогенів. C Соматотропного гормону. D Тироксину. E Мінералокортикоїдів.

207 У немовляти розвився спазм голосової щілини, в анамнезі схильність до розвитку судом. Про порушення функції яких ендокринних залоз слід думати ? A *Паращитовидних. B Підшлункової C Тимусу. D Щитовидної E Наднирників.

208 У вертикальному положенні пацієнт, заплющуючи очі, втрачає рівновагу. Які структури мозку у нього, вірогідно, пошкоджені? A *Мозочок. B Базальні ганглії. C Лімбічна система. D Таламус. E Прецентральна звивина кори великих півкуль.

209 У людини збільшена вентиляція легень внаслідок фізичного навантаження. Який з наведених показників зовнішнього дихання у неї значно більший, ніж у стані спокою? A *Дихальний об'єм B Життєва ємкість легенів C Резервний об'єм вдиху D Резервний об'єм видиху E Загальна ємкість легенів

210 У людини внаслідок патологічного процесу збільшена товщина альвеолокапілярної

Page 127: крок 1 база 2014р_11шр-240ст (для брошури)_35грн

http://vk.com/my.printing

127

мембрани. Безпосереднім наслідком цього буде зменшення у людини: A *Дифузійної здатності легень B Кисневої ємкості крові C Хвилинного об'єму дихання D Альвеолярної вентиляції легень E Резервного об'єму видиху

211 У людини вміст гемоглобіну в крові становить 100 г/л. Чому у неї дорівнює киснева ємкість крові? A *134 мл/л B 100 мл/л C 150 мл/л D 168 мл/л E 180 мл/л

212 Людина знепритомніла у салоні автомобіля, де тривалий час очікувала приятеля при ввімкненому двигуні. У крові у неї знайдено сполуку гемоглобіну. Яку саме? A *Карбоксигемоглобін B Дезоксигемоглобін C Карбгемоглобін D Метгемоглобін E Оксигемоглобін

213 У тварини видалили каротидня тільця з обох сторін. На який з зазначених факторів у неї не буде розвиватися гіпервентиляція? A *Гіпоксемія B Фізичне навантаження C Гіперкапнія D Ацидоз E Збільшення темпетарури ядра тіла

214 Проводять реєстрацію електричної активності нейронів. Вони збуджуються перед вдихом та на його початку. Де розташовані ці нейрони ? A *Довгастий мозок B Проміжний мозок C Середній мозок D Спинний мозок E Кора головного мозку

215 Велика група людей тривалий час знаходиться в закритому приміщенні

невеликого об'єму. Це призвело до розвитку у них гіпервентиляції внаслідок таких змін повітря: A *Збільшення вмісту вуглекислого газу B Зменшення вмісту кисню C Збільшення вмісту водяної пари D Збільшення температури E -

216 При обстеженні людини необхідно визначити, яка частка альвеолярного повітря оновлюється при кожному вдосі. Який з наведених показників необхідно розрахувати для цього? A *Коефіцієнт легеневої вентиляції B Хвилинний об'єм дихання C Хвилинну альвеолярну вентиляцію D Життєву ємність легень E Функціональну залишкову ємність легень

217 Людина зробила спокійних видих. Як називається об'єм повітря, який міститься у неї в легенях при цьому? A *Функціональна залишкова ємкість легень B Залишковий об'єм C Резервний об'єм видиху D Дихальний об'єм E Життєва ємність легень

218 Людина зробила максимально глибокий видих. Як називається об'єм повітря, що знаходиться в її легенях? A *Залишковий об'єм B Функціональна залишкова ємність легень C Ємність вдиху D Резервний об'єм видиху E Альвеолярний об'єм

219 При дослідженні ізольованого кардіоміоциту (КМЦ) встеновлено, що він не генерує імпульси збудження автоматично. КМЦ отримано з : A *шлуночків B Сино-атріального вузла C Атріовентрикулярного вузла D Пучка Гіса E Волокон Пуркін”є

Page 128: крок 1 база 2014р_11шр-240ст (для брошури)_35грн

128

220 В експерименті на ссавці руйнуванням певної структури серця припинили проведення збудження від передсердь до шлуночків. Що саме зруйнували? A *Атріовентрикулярний вузол B Синоатріальний вузол C Пучок Гіса D Ніжки пучка Гіса E Волокна Пуркін'є

221 При обстеженні людини встановлено, що хвилинний об'єм серця дорівнює 3500 мл, систолічний об'єм - 50 мл. Якою є у людини частота серцевих скорочень? A *70 скорочень за хвилину B 60 скорочень за хвилину C 50 скорочень за хвилину D 80 скорочень за хвилину E 90 скорочень за хвилину

222 При аналізі електрокардіограми встановлено, що тривалчсть серцевого циклу у людини дорівнює 1 сек. Якою у неї є частота серцевих скорочень за хвилину? A *60 B 50 C 70 D 80 E 100

223 У людини необхідно оцінити еластичність великих артеріальних судин. Яким з інструментальних методів дослідження доцільно скористатися для цього? A *Сфігмографія B Електрокардіографія C Фонокардіографія D Флебографія E Векторкардіографія

224 У людини необхідно оцінити стан клапанів серця. Яким з інструментальних методів дослідження доцільно скористатися для цього? A * Фонокардіографія B Електрокардіографія C Сфігмографія D Флебографія E Зондування судин

225

У міокарді шлуночків досліджуваної людини порушені процеси реполяризації. Це призведе до порушення амплітуди, конфігурації, тривалості зубця: A *Т B Q C R D S E P

226 Які з наведених механізмів регуляції НЕ МОЖУТЬ реалізуватися на ізольовану серця ссавця? A *Центральні рефлекси B Місцеві рефлекси C Факон серця Франка-Старлінга D Ефект Анрепа E Драбина Боудича

227 Необхідно в експерименті оцінити рівень збудливості тканини. Для цього доцільно визначити величину: A *Порогу деполяризації B Потенціалу спокою C Тривалості потенціалу дії D Амплітуди потенціалу дії E -

228 Необхідно у хворого оцінити рівень збудливості нерва. Для цього доцільно визначити для нерва величину: A *Порогової сили подразника B Потенціалу спокою C Критичного рівня деполяризації D Амплітуди потенціалу дії E Тривалості потенціалу дії

229 Внаслідок активації іонних каналів зовнішньої мембрани збудливої клітини значно збільшився її потенціал спокою. Які канали були активовані? A *Калієві B Натрієві C Швидкі кальцієві D Повільні кальцієві E Натрієві та кальцієві

230 У збудливій клітині повністю заблокували процеси енергоутворення. Внаслідок цього мембранний потенціал спокою: A *Зникне

Page 129: крок 1 база 2014р_11шр-240ст (для брошури)_35грн

http://vk.com/my.printing

129

B Незначно зменшиться C Суттєво зменшиться D Незначно збільшиться E Суттєво збільшиться

231 Нервово-м'язовий препарат жаби обробили отрутою. Після цього зберігається здатність м'яза до скорочення у відповідь на пряму стимуліцію, але втрачається у відповідь на стимуляцію нерва. Що блокує отрута? A *Нервово-м'язовий синапс B Спряження збудження і скорочення у м'язі C Натрієві канали D Калієві канали E Процеси енергоутворення

232 Киснева ємість крові плода більша, ніж у матері через великий вміст: A *HbF B HbA C HbH D HbS E HbP

233 Внаслідок тривалого перебування людини у горах на висоті 3000 м над рівнем моря у неї збільшилась киснева ємкість крові. Безпосередньою причиною цього є посилене утворення в організмі : A *Еритропоетинів B Лейкопоетинів C Карбгемоглобіну D Катехоламінів E 2,3-дифосфогліцерату

234 У людини внаслідок фізичного навантаження збільшилась швидкість зсідання крові. Причиною цього є збільшена концентрація в крові A *Адреналіну B Тироксину C Соматотропіну D Кортизолу E Плазмінів

235 У людини внаслідок хронічного захворювання печінки суттєво порушена її білковосинтезуюча функція. До зменшення якого параметру гомеостазу це призведе? A *Онкотичний тиск плазми крові

B Осмотичний тиск C РН D Щільність крові E Гематокритний показник

236 Тварини в експерименті перерізали передні корінці п’яти сегментів спинного мозку. Які зміни відбудуться в зоні інервації? A *Втрата рухів B Втрата дотикової чутливості C Втрата температурної чутливості D Втрата пропріоцептивної чутливості E Гіперчутливість

237 Який з зазначених процесів буде активізуватися перш за все у голодної людини, яка бачить смачну їжу? A *Секреція шлункового соку B Секреція кишкового соку C Моторика товстої кишки D Скорочення сфінктера Одді E Моторика тонкої кишки

238 У людини зменшився діурез внаслідок посиленої секреції вазопресину. Секрецію вазопресину стимулює збільшення: A *Осмостичного тиску плазми B Концентрації натрію C Об'єму циркулюючої крові D Онкотичного тиску плазми E Концентрації калію

239 В експерименті заблокували процеси енергоутворення в епітелії ниркових канальців, внаслідок чого діурез збільшився у 4 рази. Найбільш ймовірною причиною поліурії є зменшення: A *Реабсорбції іонів натрію B Швидкості клубочкової фільтрації C Секреції іонів калію D Ниркового кровотоку E Секреції сечовини

240 У людини внаслідок тривалого голодування швидкість клубочкової фільтрації зросла на 20\%. Найбільш вірогідною причиною змін фільтрації в зазначених умовах є: A *Зменшення онкотичного тиску плазми крові B Збільшення системного артеріального

Page 130: крок 1 база 2014р_11шр-240ст (для брошури)_35грн

130

тиску C Збільшення проникності ниркового фільтру D Збільшення коефіцієнта фільтрації E Збільшення ниркового плазмотоку

241 У пацієнта різко знижений вміст альбумінів в плазмі крові й онкотичний тиск. Що буде наслідком цього? A *набряки B зменшення діурезу C збільшення об’єму крові D зменшення ШОЕ E збільшення щільності крові

242 В дитини виявлено гельмінти. Які зміни в периферичній крові будуть спостерігатися при цьому? A *еозинофілія B лейкоцитоз C нейтрофілія D базофілія E моноцитоз

243 Після накладання джгута у досліджуваного виявили точкові крововиливи. З порушенням функції яких клітин крові це пов’язано? A *тромбоцитів B еозинофілів C моноцитів D лімфоцитів E нейтрофілів

244 У пацієнта діагностовано синдром Паркінсона. З порушенням яких медіаторних систем головного мозку це пов’язано? A *дофамінергічних B гістамінергічних C серотонінергічних D холінергічних E опіоїдних

245 В досліді в тварини зруйновано середню частину завитка внутрішнього вуха справа. Це призведе до порушення сприйняття звуків: A *середньої частоти B низької частоти C високої частоти D високої та низької частоти E порушень не буде

246 В старості втрачається еластичність кришталика. Який основний симптом буде виявлено? A *пресбіопія B астигматизм C міопія D порушення кольорового зору E порушення бінокулярного зору

247 У людини, яка обертається на каруселі виникло посилене частоти серцевих скорочень, потовиділення, нудота. З подразненням яких рецепторів, перш за все, це пов'язано? A *вестибулярних B пропріоцепторів C дотикових D слухових E зорових

248 У хворого різко знизився вміст Са2+ в крові. Це призведе до збільшення секреції такого гормону: A *паратгормону B тирокальцитоніну C альдостерону D вазопресину E соматотропного

249 Внаслідок фізичного навантаження киснева ємність крові в людини збільшилася з 180 до 200 мл/л. Основною причиною цього є те, що при фізичному навантаженні збільшується: A *Вміст гемоглобіну в одиниці об'єму крові B Дифузійна здатность легень C Вміст кисню в альвеолах D Спорідненість гемоглобіну до кисню E Хвилиний об'єм дихання

250 На прохання лікаря хворий зробив максимально глибокий видих. Які з наведених м'язів приймають участь у розвитку такого видоху? A *Живота B Діафрагма C Драбинчасті D Грудинноключичнососкові E Трапецієвидні

Page 131: крок 1 база 2014р_11шр-240ст (для брошури)_35грн

http://vk.com/my.printing

131

251 У людини в стані спокою значно збільшена робота м'язів вдиху. Що з наведеного може бути причиною цього? A *Звуження дихальних шляхів B Поверхневе дихання C Рідке дихання D Негативний внутрішньоплевральний тиск E Зменшення хвилинного об'єму дихання

252 Хворий втратив багато рідини, зменшився об'єм циркулюючої крові. Безпосередньо це призведе до збільшення секреції: A *вазопресину B альдостерону C натрійуретичного гормону D паратгормону E тирокальцитоніну

253 З метою схуднення жінка обмежувала кількість продуктів в харчовому раціоні. Через 3 місяці в неї з'явилися набряки, збільшився діурез. Дефіцит яких компонентів їжі є причиною цього? A *Білків B Жирів C Вуглеводів D Вітамінів E Мінеральних речовин

254 У пацієнта тривале вживання препаратів калію призвело до гіперкаліємії. Це призведе до такої зміни секреції: A *збільшення альдостерону B зменшення альдостерону C збільшення вазопресину D зменшення вазопресину E зменшення реніну

255 У людини вимірють внутрішньоплевральний тиск. У якій фазі людина затримала дихання, якщо величина тиску дорівнює –7,5 см.вод.ст.? A *Спокійний вдих B Спокійний видих C Форсований вдих D Форсований видих E -

256

У людини вимірють внутрішньоплевральний тиск. У якій фазі людина затримала дихання, якщо величина тиску дорівнює –25 см.вод.ст.? A *Форсований вдих B Спокійний видих C Спокійний вдих D Форсований видих E -

257 У людини вимірють внутрішньоплевральний тиск. У якій фазі людина затримала дихання, якщо величина тиску дорівнює +3 см.вод.ст.? A *Форсований видих B Спокійний видих C Форсований вдих D Спокійний вдих E -

258 Вимірюють тиск в альвеолах легень здорової людини. Цей тиск дорівнює 0 см водн.ст. під час: A *Паузи між вдихом і видихом B Спокійного вдиху C Спокійного видиху D Форсованого вдиху E Форсованого видиху

259 У хворого порушена реабсорбція води в нирках. З порушенням секреції якого гормону це безпосередньо пов'язано ? A *Вазопресин B Альдостерон C Натрійуретичний D Паратгормон E Тиреокальціотонін

260 При обстеженні пацієнта встановлено збільшення основного обміну на 50%. Збільшення секреції якого гормону спричинило цю зміну? A *Тироксину B Інсуліну C Паратгормону D Соматотропного E Пролактину

Page 132: крок 1 база 2014р_11шр-240ст (для брошури)_35грн

132

261 В експерименті кролику ввели внутрішньовенно 300 мл ізотонічного розчину NaCl, що привело до значного зростання об’єму циркулюючої крові. Концентрація якого фактору підвищиться в крові за цих умов? A *Натрійуретичного гормону B Реніну C Альдостерону D Ангіотензину ІІ E -

262 В умовах гострого експерименту, кролику зробили перев’язку ниркової артерії. Внаслідок цього значно зріс рівень артеріального тиску, що є результатом збільшення секреції: A *Реніну B Адреналіну C Вазопресину D Норадреналіну E Натрійуретичного гормону

263 В досліді вимірювали лінійну швидкість руху крові: вона найменша в капілярах. Причина в тому, що капіляри мають: A *Найбільшу сумарну площу поперечного перерізу B Малу довжину C Малий діаметр D Малий гідростатичний тиск E Найтоншу стінку

264 При лабораторному обстеженні чоловіка віком 54 роки було встановлено, що його кліренс інуліну 120 мл/хв. Це означає, що в нього нормальна (-ий): A *Швидкість клубочкової фільтрації B Канальцева реабсорбція C Канальцева секреція D Нирковий кровотік E Нирковий плазмотік

265 В експерименті ізольований м'яз жаби ритмічно подразнюють електричними імпульсами. Кожний наступний імпульс припадає на період розслаблення попереднього скорочення. Яке скорочення виникне?

A *Зубчастий тетанус B Одиночне C Асинхронне D Суцільний тетанус E Тонічне

266 Потужність, що розвиває м'яз, недостатня для піднімання вантажу. Який вид скорочення м'яза у даному випадку? A *Ізометричний B Тетанічний C Ізотонічний D Ексцентричний E Концентричний

267 Які зміни функції ізольованого серця будуть після збільшення у перфузійному розчині концентрації хлориду кальцію? A *Збільшення частоти і сили скорочень B Зменшення сили скорочень C Зменшення частоти скорочень D Зупинка серця в діастолі E Зменшення частоти і сили скорочень

268 В експерименті збільшили проникність мембрани збудливої клітини для іонів калію. Які зміни електричного стану мембрани при цьому виникнуть? A *Гіперполяризація B Деполяризація C Потенціал дії D Локальна вiдповiдь E Змін не буде

269 В експерименті збудливу клітину внесли в сольовий розчин, що не містить іонів натрію. Як це позначиться на розвитку процесу збудження ? A *Потенціал дії не виникає B Амплітуда потенціалу дії зменшується C Амплітуда потенціалу дії збільшується D Тривалість потенціалу дії збільшується E Тривалість потенціалу дії зменшується

270 Які зміни процесів гемокоагуляції виникнуть, якщо у людини при підвищенні активності симпатичної нервової системи? A *Гемокогуляцiя підсилиться B Гемокоагуляцiя зменшиться C Гемокоагуляцiя не зміниться

Page 133: крок 1 база 2014р_11шр-240ст (для брошури)_35грн

http://vk.com/my.printing

133

D Антизсiдальна система активується E Фiбринолiз зменшиться

271 При захворюваннях печінки, що супроводжуються недостатнім надходженням жовчі в кишківник, спостерігається погіршення гемокоагуляції. Чим можна пояснити це явище ? A *Дефіцитом вітаміну К B Дефіцитом заліза C Тромбоцитопенiєю D Еритропенією E Лейкопенією

272 Що потрібно додати до донорської крові, законсервованої цитратом натрію, щоб викликати зсідання ? A *Іони кальцію. B Іони натрію C Протромбін D Вітамін К E Фібриноген

273 У студента перед екзаменом виникла тахікардія. Які зміни на ЕКГ свідчитимуть про її наявність ? A *Укорочення інтервалу R – R B Подовження інтервалу R – R C Розширення комплексу QRS D Подовження інтервалу P – Q E Подовження сегменту Q–T

274 Після введення людині курареподібної речовини виникає розслаблення всіх скелетних м’язів. Що є причиною цього явища? A *Блокада Н- холінорецепторів постсинаптичної мембрани B Порушення виділення ацетилхоліну C Блокада Са+2 – каналів пресинаптичної мембрани D Порушення синтезу холінестерази E Порушення синтезу ацетилхоліну

275 Перерізка стовбура мозку між мостом і довгастим мозком викликає подовження фази вдиху. Причиною цього є порушення зв’язку дихального центру довгастого мозку з: A *Пневмотаксичним центром

B Ретикулярною формацією C Мозочком D Корою великих півкуль E Червоними ядрами

276 У чоловіка 35 років, який перехворів на грип, ускладнений враженням ЦНС, значно збільшилася добова кількість сечі. Який з відділів мозку найбільш вірогідно був уражений? A *Проміжний B Середній C Кінцевий D Задній E Спинний

277 З віком у людини розвивається пресбіопія (далекозорість). Причиною цього є: A *Зменшення еластичності кришталика B Видовження очного яблука C Укорочення очного яблука D Помутніння кришталика E Атрофія сітківки

278 Експериментальне зруйнування супраоптичних ядер гіпоталамуса у тварин виклає значне збільшення добового діурезу. Який із механізмів сечоутворення при цьому порушується? A *Реабсорбція води в дистальному сегменті нефрона B Реабсорбція води в проксимальному відділі нефрона C Клубочкова фільтрація D Реабсорбція води в низхідному коліні петлі Генле E Канальцева секреція

279 У хворого виявлено в сечі високомолекулярні білки. Причиною цього може бути порушення: A *Проникності ниркового фільтру B Величини ефективного фільтраційного тиску C Процесів секреції D Реабсорбції білків E Поворотно-протипотокової системи

280 В експерименті на тварині переростягненням передсердь кров’ю

Page 134: крок 1 база 2014р_11шр-240ст (для брошури)_35грн

134

викликали зменшення реабсорбції Na+ і води в ниркових канальцях. Впливом на нирки якого фактора це можна пояснити? A *Натрійуретичного гормону B Альдостерону C Реніну D Ангіотензину E Вазопресину

281 У хворого виявлено порушення внутрішньониркового кровообігу і системного ОЦК. Секреція якого фактора нирок зміниться? A *Реніну B Еритропоетинів C Простагландинів D Кініногену E Лейкотрієнів

282 Перебування людини в умовах пониженого атмосфероного тиску приводить до розвитку гіпоксії. Як зреагують на це нирки? A *Збільшенням секреції еритропоетинів B Зменшенням секреції еритропоетинів C Збільшенням фільтрації D Зменшенням фільтрації E Порушенням реабсорбції

283 Після споживання солоної їжі в людини значно зменшилася кількість сечі. Який з указаних гормонів уплинув на функцію нирок? A *Антидіуретичний B Адреналін C Соматостатин D Окситоцин E АКТГ

284 У результаті досліджень встановлено, що в нормі вихід рідини в інтерстицій перевищує її зворотний притік через стінку капіляра. Куди потрапляє надлишок рідини? A *У лімфатичні судини B У венозні судини C У міжплевральний простір D У черевну порожнину E В артеріальні судини

285 Пасажир після кількагодинного сидіння у вимушеній позі в автобусі помітив набряк

ступнів і гомілок (щиколоток). Яка причина такого набряку? A *Венозний застій B Дилятація артеріол C Підвищена проникність капілярів D Зниження рівня білків плазми E Високий рівень гістаміну

286 Чоловік середнього віку виїхав в іншу країну на обіцяну йому роботу, але працевлаштуватися тривалий час не вдавалося. Які з ендокринних залоз, більш за все, виснажуються? A *Надниркові B Прищитоподібні C Сім’яники D Підгрудинна E Щитоподібна

287 У пацієнта обстержували сприйняття звуків за допомогою камертона. При розташуванні його біля зовнішнього вуха пацієнт не чув правим вухом звук камертона. При розташуванні ніжки камертона на соскоподібному відростку пацієнт відчув звук. З ураженням якої частини слухової сенсорної системи це пов‘язано ? A * Середнього вуха B Нижніх горбиків C Внутрішнього вуха D Слухового нерва E Медіального колінчастого тіла

288 У пацієнта тривалість інтервалу P-Q ЕКГ перевищує норму при нормальній тривалості зубця Р. Причиною цього є зниження швидкості проведення збудження: A * Атріо-вентрикулярним вузлом B Сино-атріальним вузлом C Пучком Гіса D Ніжками пучка Гіса E Волокнами Пуркін'є

289 Який ефективний шлях віддачі тепла тілом робітників парникового господарства при температурі повітря 36 0 С, відносній його вологості - 70%? A * Випаровування поту B Проведення C Радіація D Конвекція

Page 135: крок 1 база 2014р_11шр-240ст (для брошури)_35грн

http://vk.com/my.printing

135

E -

290 У хворого відсутня провідність у язико-глотковому нерві. Яке відчуття зникне у хворого? A * Гіркого B Кислого C Солодкого D Солоного E Кислого й солоного

291 Досліджують процеси тепловіддачі у роздягненої людини при кімнатній температурі. З’ясовано, що за таких умов найбільша кількість тепла віддається шляхом: A *Теплорадіації B Теплопроведення C Конвекції D Випаровування E -

292 У сечі знайдено велику кількість білка, еритроцитів. Причиною цього може бути збільшення: A *Проникності ниркового фільтру B Ефективного фільтраційного тиску C Гідростатичного тиску крові в капілярах клубочків D Гідростатичного тиску первинної сечі в капсулі E Онкотичного тиску плазми крові

293 Невпізнання хворим предметів при їх обмацюванні виникло після черепно-мозкової травми. Який відділ мозку ушкоджено? A *Постцентральна звивина B Потилична частка C Скронева частка D Прецентральна звивина E Мозочок

294 У людини суттєво порушено перетравлення білків, жирів та вуглеводів. Знижена секреція якого травного соку, найімовірніше, є причиною цього? A *Підшлункового B Слини C Шлункового D Жовчі

E Кишечного

295 У людини травматичне пошкодження грудинно-ключично-сосцевидного м'язу. Це призвело до зменшення величини: A *Резервного об’єму вдиху B Резервного об‘єму видиху C Дихального об‘єму D Залишкового об‘єму E Функціональної залишкової ємкості легенів

296 Досліджуються рецептори, інформація від яких прямує до кори без участі талямусу. Які це рецептори? A *Нюхові B Дотикові C Смакові D Зорові E Слухові

297 В експерименті подразнюють гілочки симпатичного нерва, які інервують серце. Це призвело до збільшення сили серцевих скорочень, тому що через мембрану типових кардіоміоцитів збільшився: A * Вхід іонів кальцію B Вихід іонів кальцію C Вихід іонів калію D Вхід іонів калію E Вхід іонів кальцію та калію

298 У кішки з децеребраційною ригідністю потрібно знизити тонус м’язів, що можна зробити шляхом: A *Руйнування вестибулярних ядер Дейтерса B Подразнення отолітових вестибулорецепторів C Подразнення вестибулярних ядер Дейтерса D Подразнення вестибулослухового нерва E Подразнення ампулярних вестибулорецепторів

299 При визначенні повітряної та кісткової провідності звуку було встановлено, що у пацієнта ліве вухо краще сприймає звук при кістковій провідності, що могло бути пов’язане з

Page 136: крок 1 база 2014р_11шр-240ст (для брошури)_35грн

136

захворюванням: A *Середнього вуха зліва B Середнього вуха зправа C Внутрішнього вуха зліва D Внутрішнього вуха зправа E Зовнішнього вуха зправа

300 Внаслідок автодорожньої аварії у хворої виникло неутримання сечі. Які сегменти спинного мозку пошкоджені? A * S2-S4 B Т1-Т5 C L1-L2 D Т2-Т5 E Т1-L1

Патологічна анатомія 1 При черевному тифі некротизовані пейєрові бляшки тонкої кишки, забарвлюються у жовто-коричневий колір. Який пігмент просякає некротизовану тканину? A * Білірубін B Гемоглобін C Ліпофусцин D Індол E Меланін

2 В шкірі виявлена щільна, рухома, чітко відмежована від оточуючих тканин пухлина. На розрізі вона білого кольору, представлена волокнистою тканиною. Мікроскопічно: хаотично переплетені колагенові волокна, клітин мало. Що це за пухлина? A * Фіброма B Міома C Гістіоцитома D Дерматофіброма E Десмоїд

3 На розтині померлої виявлено морфологічні прояви стеноз лівого атріовентрикулярного отвору, недостатність мітрального клапана. Гістологічно в міокарді – вогнищевий кардіосклероз, наявність квітучих гранулем Ашоф-Талалаєва. Який з перерахованих нижче діагнозів найбільш імовірний? A * Ревматизм. B Склеродермія. C Дерматоміозит. D Вузликовий периартериїт. E Системний червоний вовчак.

4 При мікроскопічному дослідженні шийного лімфатичного вузла виявлено скупчення епітеліоїдних клітин, лімфоцитів і гігантських клітин Пірогова-Ланганса.В центре Казеозний некроз. Вкажіть найбільш вірогідну патологію. A * Туберкулез B Риносклерома. C саркоидоз D Сап. E Сифіліс.

5 При гістологічному дослідженні новоутворення шкіри виявлено: паренхіма сформована з покривного епітелію із збільшеним числом шарів. Строма разом з розростаннями епітелію формує сосочки. Вкажіть вид атипізму. A * Тканинний B Клітинний. C Гістохімічний D Функціональний. E Метаболічний.

6 Чоловік, 19 років страждав з раннього дитинства бронхо-ектатичною хворобою. Помер від ниркової недостатності. На розтині окрім множинних бронхоектатичних каверн, заповнених гнійним ексудатом виявлено збільшені в розмірах нирки щільної консистенції, кірковий шар потовщений, білого кольору, щільний. Піраміди нирки анемічні, чіткі. Назвати процес, який розвився в нирках ? A *Вторинний амілоїдоз B Гломерулонефрит C Хронічний пієлонефрит D Вроджений кістоз нирок E Вторинний нефросклероз

7 Дитина 3 років з множинними порушеннями розвитку кісток лицевого відділу черепа. Причина смерті – сепсис, який розвився на фоні бронхопневмонії. В крові вміст імуноглобулінів в межах фізіологічної норми.На розтині встановлена відсутність тимусу. Назвати головну причину страждання дитини ? A *Синдром недостатності клітинного

Page 137: крок 1 база 2014р_11шр-240ст (для брошури)_35грн

http://vk.com/my.printing

137

імунітету B Синдром комбінованого імунодефіциту C Вторинний імунодефіцитний синдром D Гострий лімфолейкоз E Синдром хронічної інтоксикації

8 У померлого 67 років на розтині знайдені ознаки фібринозного запалення в товстому кишківнику . Ваш діагноз: A *Дизентерія B Амебіаз C Черевний тиф D Холера E Балантидіаз

9 Хвора 70 років прооперована з приводу “гострого живота”. Під час операції виявлено близько 80 см клубової кишки чорного кольору, очеревина тьмяна, просвіт верхньої брижової артерії обтурований тромбом. Який процес розвинувся в кишці? A *Гангрена B Пролежень C Білий інфаркт D Білий інфаркт із геморагічним вінчиком E Коагуляційний некроз

10 Хворий, що страждав хронічною обструктивною емфіземою легень, помер від легенево-серцевої недостатності. Які зміни можна виявити в серці? A *Гіпертрофію правого шлуночка серця B Гіпертрофію лівого шлуночка серця C Амілоїдоз D Великоосередковий кардіосклероз E Розрив серця

11 У хворого цукровим діабетом з'явився різкий біль у правій стопі. При огляді великий палець стопи чорного кольору, тканини стопи набряклі, осередки відшарування епідермісу, виділення з неприємним запахом. Яка клініко-морфологічна форма некрозу розвилася в хворого? A *Гангрена волога B Пролежень C Секвестр D Гангрена суха

E Інфаркт

12 У хворого з вадою мітрального клапану з'явився кашель, мокротиння ржавого кольору. Який пігмент обумовив такий колір мокротиння? A *Гемосидерин B Меланін C Гемоглобін D Гемомеланін E Сірчасте залізо

13 У хворого висока температура, задуха, біль у правій частині грудної клітини. Плевральна пункція дала 700 мл в’язкої рідини жовто-зеленого кольору. Який патологічний процес розвився у плевральній порожнині? A *Емпієма плеври B Бронхопневмонія C Серозний плеврит D Геморрагичний плеврит E Карциноматоз плеври

14 Слизова оболонка товстої кишки у померлого при дизентерії на розтині повнокровна, покрита плівкою сірого кольору, що відривається із зусиллям. Який вид запалення розвинувся в кишці у хворого? A *Дифтеритичне запалення B Крупозне запалення C Геморагічне запалення D Серозне запалення E Катаральне запалення

15 Для гістологічного дослідження доставлене очне яблуко, у судинній оболонці якого виявлене пухлиноподібне утворення 1 х 0,4 см чорного кольору. . У клітках – множинні патологічні мітози і у цитоплазмі багатьох з них визначається пігмент жовто-бурого кольору. Ваш діагноз? A *Меланома B Неврінома C Ангіосаркома D Нейробластома E Гангліонейробластома

16 При мікроскопічному дослідженні біоптату з товстої кишки виявлена пухлина з

Page 138: крок 1 база 2014р_11шр-240ст (для брошури)_35грн

138

призматичного епітелію, що формує атипові залозисті структури різної форми і розміру. Клітини поліморфні, ядра гіперхромні, є патологічні мітози. Базальна мембрана залоз зруйнована . Ваш діагноз. A *Аденокарцинома B Базально-клітинний рак C Солідний рак D Слизовий рак E Недиференційований рак

17 У чоловіка 30 років при гістологічному дослідженні біоптату з шийного лімфатичного вузла виявлені гранульоми, які складаються з епітеліоїдних, лімфоїдних, багатоядерних гігантських клітин типу Пірогова-Лангханса. У центрі гранульом визначається некроз. Який збудник потрібно виявити в зоні некрозу для підтвердження діагнозу туберкульоз? A *Мікобактерію Коха B Бліду трепонему C Стафілокока D Бацили Волковича-Фріша E Сальмонели

18 У результаті гістологічного дослідження біоптату із стінки бронха хворого хронічним бронхітом в слизовому шарі, , розростання грануляційної тканини, що виступають над поверхнею слизового шару та містять дифузний запальний інфільтрат. Діагностуйте вид бронхіту: A * Хронічний поліпозний бронхіт B Хронічний слизисто-гнійний бронхіт C Хронічний слизистий бронхіт D Хронічний гнійний бронхіт E Хронічний деформуючий бронхіт

19 В ході гістологічного дослідження вишкрібку стінок порожнини матки 45-річної жінки з порушеннями оваріально-менструального циклу виявлено збільшення кількості ендометріальних залоз, деякі пилообразно извитые, деякі залози - кистозно розширені. Діагностуйте захворювання. A * Залозисто-кистозна гіперплазія ендометрію B Плацентарный полип C Атипова гіперплазія ендометрію

D Залозистий поліп ендометрію E Аденокарцинома ендометрію

20 Підліток скаржиться на потоншення м’язів і зменшення об’єму гомілки, які виникли після тривало незагоювавшогося перелому стегнової кістки без пошкодження нервів. Як називається така атрофія м’язів. A *Дисфункціональна B Нейротична C Визвана недостатнім кровопостачанням D Визвана здавлюванням E Від дії фізичних факторів

21 На розтиніпомерлого, хворівшого вадою серця виявлена збільшена в розмірі печінка строкатого виду, з малюнком мускатного горіха на розрізі. Назвіть вид порушення кровообігу A *загальне венозне повнокрів'я B Загальне артеріальне повнокрів'я C Недокрів'я D Крововилив E Кровотеча

22 У хворого виразковою хворобою шлунка з кровотечею при ендоскопії в шлунку знайдена рідина кольору кавової гущавини. Який пігмент обумовив такий колір вмісту шлунка? A *Солянокислий гематин B Гемосидерин C Білірубін D Феритин E Порфірин

23 При розтині померлого хворого від хронічної ниркової недостатності в слизуватій оболонці товстої кишки виявлені сіро-жовті, щільно з'єднані плівки , які відокремлюються з утворенням виразок. Уточніть вид запалення. A *Дифтерітичне B Серозне C Катаральне D Крупозне E Гнійне

24 У хворого при рентгенологічному обстеженні в плоских кістках виявлені

Page 139: крок 1 база 2014р_11шр-240ст (для брошури)_35грн

http://vk.com/my.printing

139

множинні вогнища остеопорозу і остеолізісу. У трепанобіоптаті виявлено високий зміст пухлинних плазматичних клітин. Ваш діагноз. A *Мієломна хвороба B Гострий моноцитарний лейкоз C Хронічний мієлолейкоз D Лімфогрануломатоз E Гістіоцитоз

25 У хворої, що страждала вторинним сифілісом, з'явилися вогнища депігментації шкіри у верхніх відділах спини. Назвіть патологічний процес у шкірі. A *Лейкодерма B Метаплазія C Лейкоплакія D Дисплазія E Пара кератоз

26 У хворого з кривавою блювотою на операції в шлунку знайдена виразка, проникаюча в м’язовий шар шлунка. Краї виразки щільні, в дні – кровоточива судина. При цитобіопсії в краях і дні виразки виявлена рубцева тканина. Яка це виразка? A * Хронічна кровоточива виразка B Пенетруюча виразка C Гостра кровоточива виразка D Перфоративна виразка шлунка E Малігнізована виразка

27 На розтині у померлого хворого виявлена аденома передміхурової залози і великі нирки з різко збільшеними баліями і чашечками, заповненими прозорою рідиною. Назвіть процес у нирках. A *Гідронефроз B Гломерулонефрит C амілоїдоз D Туберкульоз E Пієлонефрит

28 При мікроскопічному дослідженні біоптата шийки матки виявлена клітинна і ядерна атипія багатошарового плоского епітелію, патологічні мітози, а також рогові перлини в глибині епітеліальних шарів. Ваш діагноз: A *Плоскоклітинний рак зі зроговінням

B Перехідноклітинний рак. C Плоскоклітинний рак без зроговіння D Залозистий рак. E Анапластичний рак.

29 У молодої людини виявлений надлишок соматотропного гормону, збільшені розміри носа, губ, ушей, нижньої щелепи, кистей і стоп. Ваш діагноз. A *Акромегалія B Гіпофізарний нанізм C Хвороба Іценко-Кушинга D Адісонова хвороба E Адіпозогенітальна дистрофія

30 У померлої жінки, 86 років, яка страждала на атеросклероз судин головного мозку, на розтині виявлена атрофія кори головного мозку. Як називається ця атрофія відносно причини? A * Від недостатнього кровопостачання. B Від тиску. C Від дії фізичних та хімічних факторів. D Нейротична. E Дисфункціональна.

31 У хворого М. 14 років, діагностована тріада Гетчинсона: зуби діжкоподібної форми, паренхіматозний кератит та глухота.. Для якої хвороби характерні виявлені зміни? A * Сифіліс. B Токсоплазмоз. C Проказа. D Туберкульоз. E Опісторхоз.

32 Смерть хворого К, 16 років, настала від розлитого (поширеного) фібринозно-гнійного перитониту. На розтині, в нижній ділянці тонкої кишки виявлена виразка, яка повторювала форму пейерової бляшки, з перфорацією стінки кишки. Мікроскопічне дослідження виявило стертість малюнка лімфоїдної тканини, витіснення її проліферуючими моноцитами, які формують гранульоми. Ускладненням якого захворювання є причина смерті? A * Черевного тифу. B Дизентерії. C Холери. D Бруцельоза.

Page 140: крок 1 база 2014р_11шр-240ст (для брошури)_35грн

140

E Неспецифічного виразкового коліту.

33 Чоловік віком 55 років тривалий час хворів на хронічний гломерулонефрит. Помер при явищах хронічної ниркової недостатності.. На поверхні епікарда і перикарда виявляються сірувато-білуваті ворсинчасті нашарування. Який процес патологічний процес мав місце в перикарді? A *фібринозне запалення B організація C проліферативне запалення D геморагічне запалення E артеріальне повнокрів’я

34 На розтині трупа чоловіка віком 57 років, який помер від висипного тифу, виявлено, що м’язи передньої черевної стінки і стегон щільні, білуваато-жовтого кольору, нагадують стеаринову свічу. Проявом якого патологічного процесу є описані зміни у м’язах: A *восковидного некрозу B апоптозу C фібриноїдного некрозу D колікваційного некрозу E казеозного некрозу

35 При гістологічному дослідженні біоптата перегородки носу хворого, який страждав утрудненим носовим диханням, в слизовій оболонці знайдено гранульоматозне запалення з наявністю в гранульомах клітин Микуліча і бактерій Волковича-Фріша. Ваш діагноз: A *Риносклерома B Сифіліс C Туберкульоз D Сап E Лепра

36 При розтині трупа чоловіка, який хворів на черевний тиф, знайдені зміни у тонкій кишці:групові лімфоїдні фолікули збільшені, виступають над поверхнею слизової оболонки, вони сіро-червоні, соковиті, їх поверхня має вигляд звилин та боріздок . При мікроскопічному дослідженні відмічається утворення черевнотифозних гранульом. .

Вкажіть, яка з перелічених стадій черевного тифу найбільш вірогідна? A *Мозкоподібного набухання. B Некрозу. C Утворення виразок. D Чистих виразок. E Загоювання.

37 Хворий 67 років мав важку форму грипу з летальним наслідком. На секції зміни в легенях були подібні змінам “великих строкатих легень”. При мікроскопічному дослідженні виявлено: різке повнокрів’я судин, крововиливи, набряк легеневої тканини, в просвіті бронхів і альвеол ексудат, який містить переважно еритроцити. Про який характер запалення легень свідчать ці морфологічні ознаки? A * Геморагічна бронхопневмонія. B Катаральна бронхопневмонія. C Гнійна бронхопневмонія. D Десквамативна бронхопневмонія. E Фібрінозна пневмонія.

38 Апендикс, надісланий до патоморфологічного відділення після апендиктомії, потовщений і збільшений у розмірах, серозна оболонка тмяна, судини повнокрівні, з просвіту відростка на розрізі виділяється рідина жовто-зеленого кольору. При якій формі апендициту розвиваються такі зміни? A * Флегмонозний апендицит. B Простий катаральний апендицит. C Поверхневий катаральний апендицит. D Гангренозний апендицит. E Апостематозний апендицит.

39 На секції виявлено: множинні геморагічні інфаркти легень, у деяких судинах легень буруватого кольору щільні маси, які не прикріплені до стінки судин, варикозне розширення вен нижніх кінцівок, в яких наявні тромби. Про який патологічний процес ідеться мова? A * Тромбемболія судин легеневої артерії. B Жирова емболія судин легеневої артерії. C Тканинна емболія судин легеневої артерії. D Застійний тромбоз судин легеневої артерії. E Геморагічна бронхопневмонія.

Page 141: крок 1 база 2014р_11шр-240ст (для брошури)_35грн

http://vk.com/my.printing

141

40 При гістологічному дослідженні зішкріба слизової оболонки матки у хворої 54 років з клінічним діагнозом порушення оваріально-менструального циклу виявлено – розростання атипових залозистих структур, що складаються з клітин з гіперхромними ядрами, фігурами мітозів, атипією. Атипові залозисті структури вростають в міометрій..Для якого патологічного процесу характерні виявлені мікроскопічно зміни? A * Аденокарцинома матки. B Залозиста гіперплазія ендометрію. C Гострий ендометрит. D Плацентарний поліп. E Хоріонепітеліома матки.

41 У померлого 58 років на розтині мітральний клапан деформований, потовщаний, змикається не до кінця. Мікроскопічно: вогнища колагенових волоконець еозінофільні, дають плюсову реакцію на фібрін. Найвірогідніше це: A *фібріноїдне набухання B фібрінозне запалення C мукоїдне набухання D гіаліноз E амілоїдоз

42 На розтині померлого після абдомінальної операції в венах малого тазу були знайдені численні тромби. Клінічно був зафіксований тромбоемболічний синдром. Де слід шукати тромбоемболи? A *Легеневі артерії B Портальна вена C Лівий шлуночок серця D Головний мозок E Вени нижніх кінцівок

43 При розтині чоловіка 49 років, який поступив у стаціонар з картиною гепатотропної інтоксикації і раптово помер, печінка збільшена, дрябла, жовто-коричневого кольору; на поверхні розрізу печінки і лезі ножа помітні краплини жиру. Мікроскопічно: гепатоцити периферії класичних печінкових часточок вміщують масу дрібних крапель, які виповнюють цитоплазму і відсовують ядро на периферію. Який процес найімовірніше має

місце в печінці ? A *Жирова дистрофія печінки B Цереброзидліпідоз (хвороба Гоше) C Сфінгомієлінліпідоз (хвороба Німанна-Піка) D Гангліозидліпідоз (хвороба Тея-Сакса) E Генералізований гангліозидоз(хвороба Нормана-Ландінга)

44 У хворого на гостру виразкову хворобу шлунка, що ускладнилась шлунковою кровотечею, блювотні маси забарвлені в темно-коричневий колір, що описується як блювота “кавовою гущею”. Наявність якого пігменту в блювотних масах визначає таке їх забарвлення ? A *Солянокислого гематину B Гемоглобіну C Білірубіну D Гемомеланіну E Сульфіду заліза

45 У жінки 46 років під час паліативної операції з приводу раку шлунку встановлена наявність крукенбергівських метастазів в яєчники (“крукенбергівський рак яєчників”). Який з наведених шляхів метастазування призвів до ураження яєчників? A *Лімфогенний ретроградний B Лімфогенний ортоградний C Гематогенний D Імплантаційний E Каналікулярний

46 У хворого, з(явилася охриплість голосу, під час лярінгоскопії виявлена пухлина гортані, сіро-білого кольору, з сосочковою поверхнею. Мікроскопійно: розростання сполучної тканини, що вкрите багатошаровим плоским епітелієм зі збільшеним ороговінням, без клітинного атипізму. Найвірогідніше це: A *папілома B фіброма C поліп D ангіома E ангіофіброма

47 На розтині померлого 34 років від ревматизму т - поверхня епікарду ворсиста,

Page 142: крок 1 база 2014р_11шр-240ст (для брошури)_35грн

142

покрита плівками сірого кольору, що легко відділяються. Після їх відділення визначається набрякла повнокровна поверхня епікарду . Найвірогідніший діагноз: A *фібрінозий перікардіт B гнійний перікардіт C геморагічний перікардіт D проліферативний перікардіт E катаральний перікардіт

48 Мікроскопійне дослідження коронарної артерії у померлого 53 років виявило звуження прозору судини за рахунок фіброзної бляшки з домішком ліпідів. Найвірогідніша форма атеросклерозу: A *ліпосклероз B ліпоїдоз C доліпідна D атероматоз E виразкування

49 На розтині померлого від легенево-серцевої недостатності чоловіка 47 років у лівій легені знайдено порожнину розміром 4х4см, заповнену гнієм, стінка фестончаста, нерівна, представлена легеневою тканиною. Найвірогідніше це : A *гострий абсцес B хронічний абсцес C кавернозний туберкульоз D ехінококоз E фіброзуючий альвеоліт

50 На розтині у хворого в черевній порожнині виявлено близько 2.0 л гнійної рідини. Очеревина тьмяна, з сіруватим відтінком, на серозній оболонці кишок сіруватого кольору нашарування, що легко знімаються. Найвірогідніше це: A * фібрінозно-гнійний перітоніт B геморагічний перітоніт C серозний перітоніт D туберкульозний перітоніт E -

51 У померлого під час розтину знайдено тромбоз лівої середньої мозкової артерії та велике

вогнище сірого пом’якшення тканин лівої півкулі мозку. Який патологічний процес розвився в головному мозку? A * Ішемичний інфаркт B Коагуляційний некроз C Абсцесс D Волога гангрена E Секвестр

52 При розтині померлого хворого виявлена гіперплазія кісткового мозку плоских і трубчастих кісток (піоїдний кістковий мозок), спленомегалія (6 кг), гепатомегалія (5 кг), збільшення всіх груп лімфатичних вузлів. Якому захворюванню відповідають виявлені зміни? A * Хронічний мієлолейкоз B Хронічний лімфолейкоз C Мієломна хвороба D Справжня поліцитемія E Лімфогрануломатоз

53 При розтині померлого від набряку легень в міокарді знайдено велике вогнище жовто-сірого кольору, а в коронарній артерії – свіжий тромб. Уточніть діагноз: A *Інфаркт міокарда B Кардіосклероз C Міокардит D Амілоїдоз E Кардіоміопатія

54 У молодої жінки в зв’язку з гострими болями в клубній області, вилучена маткова труба з локальним розширенням її середньої третини, заповненої кров’ю. При гістологічному дослідженні в отворі труби знайдені хоріальні ворсини, великі поля еритроцитів з домішком лейкоцитів. Ваш діагноз: A *Трубна вагітність B Гострий гнійний сальпінгіт C Крововилив в маткову трубу D Геморагічний сальпінгіт E Гнійний сальпінгіт

55 У лежачого хворого з недостатністю кровообігу, нерухомого після перенесеного інсульту, шкіра і м’які тканини над крижами почорніли і набрякли, після відторгнення епідермісу в

Page 143: крок 1 база 2014р_11шр-240ст (для брошури)_35грн

http://vk.com/my.printing

143

чорних тканинах відкрились виразки. Який процес розвився у хворого? A *Пролежні B Суха гангрена C Флегмона D Інфаркт E Абсцес

56 У хворого знайдено асцит, в два рази збільшена селезінка, варикозне розширення вен стравоходу і прямої кишки. При гістологічному дослідженні біоптата печінки виявлений мікронодулярний цироз. Який процес ускладнив цироз печінки? A * Синдром портальної гіпертензії B Серцева недостатність C Гепато-лієнальний синдром D Печінково-клітинна недостатність E -

57 У померлого від хронічної серцево-судинної недостатності на розтині виявлене "тигрове серце". З боку ендокарду помітне жовтувато-біле покреслення, міокард тьмяний, глинисто-жовтий. Який процес зумовив дану патологію? A * Жирова паренхіматозна дистрофія B Вуглеводна дистрофія C Гіаліново-краплинна дистрофія D Жирова судинно-стромальна дистрофія E Амілоїдоз

58 Чоловік, 70 років, зі скаргами на біль у дрібних суглобах рук і ніг. Суглоби деформовані, болючі. Виявлений підвищений рівень солей сечової кислоти в крові та сечі. Про порушений обмін яких речовин йде мова? A *Нуклеопротеїдів. B Кальція. C Хромопротеідів. D Ліпопротеїдів. E Калію.

59 Чоловік віком 42 роки помер при явищах вираженої інтоксикації і дихальної недостатності. На розрізі тканина легень у всіх відділах строката, з множинними дрібновогнищевими крововиливами та вогнищами емфіземи. Гістологічно у

легенях: геморагічна бронхопневмонія з абсцедуванням, у цитоплазмі клітин епітелію бронхів еозинофільні і базофільні включення. Діагностуйте виявлене на секції захворювання. A * Грип B Парагрип C Аденовірусна інфекція D Респіраторно-синцитіальна інфекція E Стафілококова бронхопневмонія

60 У чоловіка віком 62 роки видалено нирку, у якій при макроскопічному дослідженні виявлено пухлину у вигляді вузла діаметром до 8см. Тканина пухлини на розрізі строката, з множинними крововиливами, некрозами. Гістологічно: пухлина складається із світлих клітин, які утворюють альвеолярні і сосочкові структури, помірно виражений інвазивний ріст. У багатьох клітинах пухлини визначаються патологічні мітози, гіперхромні ядра. Діагностуйте виявлену пухлину нирки. A * Світлоклітинний рак B Світлоклітинна аденома C Аденокарцинома D Нефробластома E Ацидофільна аденома з малігнізацією

61 При мікроскопічному дослідженні біопсії нирки виявлено вогнища, в центрі яких знаходяться зернисті еозинофільні маси, оточені інфільтратом з лімфоцитів, епітеліоїдних клітин та поодиноких клітин Пирогова-Лангханса. Виберіть патологічний процес, що найбільш повно відповідає зазначеним змінам: A * Гранулематозне запалення B Коагуляційний некроз C Казеозний некроз D Альтеративне запалення E Проліферація та диференціювання макрофагів

62 При мікроскопічному дослідженні тканини печінки було виявлено, що деякі клітини розпалися на невеликі фрагменти з окремими органелами та залишками ядра, оточені мембраною. Запальна реакція відсутня.

Page 144: крок 1 база 2014р_11шр-240ст (для брошури)_35грн

144

Виберіть патологічний процес, для якого характерні описані зміни. A *Апоптоз B Некроз C Каріорексис D Плазмоліз E Плазморексис

63 У хворого 28 років мигдалики значно збільшені, повнокровні, болючі, на їх поверхні – щільні брудно-сірі плівки, які поширюються на тверде піднебіння, щільно зв’язані з підлеглими тканинами, при спробі відокремити їх – розвивається кровотеча. Який патологічний процес обумовлює дані морфологічні зміни? A *Дифтеритичне ексудативне запалення. B Крупозне ексудативне запалення. C Катаральне ексудативне запалення. D Гнійне ексудативне запалення. E Геморрагічне ексудативне запалення.

64 У хворого, померлого від серцевої недостатності, при патоморфологічному дослідженні виявлено: стулки мітрального клапана деформовані, потовщені, зрослися по краях; у сполучній тканині міокарда -- дифузно розкидані вузлики, які складаються з ділянок фібриноїдного некрозу, навколо яких скупчуються макрофагоцити, що нагадують гігантські багатоядерні клітини. Подібні вогнища оточені лімфоцитами та поодинокими плазматичними клітинами. . Яка з перерахованих гранульом має місце у даного хворого? A * Ревматична. B Туберкульозна. C Актиномікотична. D Сифілітична. E Лепрозна.

65 На розтині хворої 25-ти років, яка померла від уремії: нирки збільшені, пістряві, з осередками крововиливів. Патогістологічно виявляються гематоксилінові тільця, капілярні мембрани клубочків у вигляді дротяних петель, гіалінові тромби та осередки фібриноїдного некрозу, у судинах селезінки – “цибулинний” склероз. Який найбільш

вірогідний діагноз? A *Системний червоний вовчак. B Системна склеродермія. C Ревматоїдний артрит. D Ревматичний артрит. E Вузликовий періартеріїт.

66 У хворого, який тривалий час страждав на переміжну кульгавість, тканини пальців стопи сухі, чорного колоьру, нагадують мумію. На невеликій відстані від почорнілої ділянки розташована двоколірна лінія (червоний колір прилягає до практично незмінених тканин, а біло-жовтий колір - до змінених тканин). Який вид некрозу у даного хворого? A *Гангрена B Інфаркт C Секвестр D Пролежень E Мацерація

67 У хворої 77 років защемлена пахвинна кила. При лапаротомії: стінка кишки ціанотичного кольору, роздута, набрякла, вкрита нитками фібрину, перистальтика не виявляється. Який патологічний процес розвинувся в стінці кишки внаслідок пристінкового защемлення кили? A * Волога гангрена B Суха гангрена C Коагуляційний некроз D Колікваційний некроз E Пролежень

68 У тканині печінки виявлено округле утворення діаметром 0.5 см. Мікроскопічно воно має наступну будову: у центрі – некротичні маси, їх оточує грануляційна тканина з наявністю у її складі плазматичних, лімфоїдних клітин і кровоносних судин з явищами васкуліту. Який діагноз необхідно поставити на підставі даних мікроскопії? A *Солітарна гума печінки. B Солітарна аденома печінки. C Солітарна лепрома печінки. D Хронічний абсцес печінки. E Рак печінки.

69

Page 145: крок 1 база 2014р_11шр-240ст (для брошури)_35грн

http://vk.com/my.printing

145

У хворого – глибока рвана рана із нерівними краями, вкрита гноєм. У крайових відділах – сочна грануляційна тканина, яка не здіймається над рівнем рани. Назвіть вид загоєння рани. A *Загоювання вторинним натягом. B Загоювання первинним натягом. C Загоювання під струпом. D Безпосереднє закриття дефекту епітеліальної тканини. E Організація рани.

70 На секції виявлено значне збільшення об’єму правої нирки. На розрізі в ній міститься камінь. Просвіт ниркової миски розтягнутий сечею, яка накопичується. Паренхіма нирки різко потоншана. Який з діагнозів є найбільш вірним? A * Гідронефроз. B Пієлоектазія C Гідроуретронефроз. D Кіста нирки E -

71 При мікроскопічному дослідженні бронхобіопсії виявлена пухлина, яка побудована з гніздних скупчень атипових клітин багатошарового плоского епітелію, місцями із характерними “перлинами”. Ваш діагноз? A * Плоскоклітинний рак із ороговінням. B Плоскоклітинний рак без ороговіння. C Солідний рак. D Слизовий рак. E Скір.

72 У дитини діагностовано в клініці дифтерія зіву. Померла від гострої серцевої недостатності. На розтині виявлено, що порожнини серця розширені в поперечнику. М’яз серця тьмяний, в’ялий, на розрізі строкатий, з жовтуватими ділянками. У цитоплазмі деяких кардіоміоцитів зі збереженою цитоплазмою виявляються дрібні вакуолі. На заморожених зрізах вакуолі забарвлюються суданом 3 у померанчовий колір. Який вид дистрофії виявлений у кардіоміоцитах?

A *Жирова B Вуглеводна C Балонна D Гіаліново-крапельна E Гідропічна

73 Під час емоційно напруженої роботи раптово помер молодий мужчина. На розтині виявлено нерівномірне кровонаповнення міокарда. Гістохімічно – зниження вмісту глікогену. Електронно мікроскопічно –деструкція мітохондрій, контрактури міофібрил. Вкажіть ймовірний розлад кровообігу? A *Гостра ішемія B Хронічна ішемія C Вакатна артеріальна гіперемія D Гостра венозна гіперемія E Ангіоневротична артеріальна гіперемія

74 Під час дорожно-транспортної пригоди водій отримав поранення в шию розбитим склом. Кровотеча була невелика, але через декілька хвилин потерпілий помер при явищах гострої ядухи. При розтині серця померлого у заповненій водою порожнині перикарду виділяються пухирці. Вкажіть ймовіринй патологічний процес. A *Повітряна емболія B Газова емболія C Жирова емболія D Тромбоемболія E Емболія чужерідними тілами

75 На розтині чоловіка, який помер від хроніосепсису, виявлено атрофію скелетних м’язів, буру атрофію міокарда, печінки. Порушення обміну якого пігменту виявлено у померлого? A *Ліпофусцину B Ліпохрому C Гемосидерину D Гемомеланіну E Меланіну

76 Чоловік тривалий час хворів гемобластозом. На розтині виявлено, що кістковий мозок,

Page 146: крок 1 база 2014р_11шр-240ст (для брошури)_35грн

146

селезінка, печінка, лімфатичні вузли коричневого кольору. Проведено гістохімічну реакцію Перлса. Встановлено, що ретикулярні, ендотеліальні і гістіоцитарні елементи цих органів містять гранули синього кольору. Який пігмент виявлено при застосуванні зазначеної реакції? A *Гемосидерин B Білірубін C Гематоїдин D Гематоїдин E Гематопорфірин

77 Хворий хронічним пієлонефритом помер від хронічної ниркової недостатності. При житті аускультативно відмічено феномен “шум тертя перикарду”. На розтині виявлено, що епікард тьм’яний, шорсткий, ніби покритий волосяним покривом. Який перикардит за характером запалення має місце? A *Крупозний B Дифтеритичний C Гнійний D Гнильний E Серозний

78 При розтині тіла дівчинки, яка померла від асфіксії, виявлено, що слизова трахеї і бронхів покрита біло-сірою плівкою, яка рихло з’єднана з підлеглими тканинами і легко знімається пінцетом. Просвіт сегментарних бронхів виповнений рихлими масами сіро-білого кольору. Який трахеобронхіт за характером ексудату відмічений при розтині? A *Крупозний B Катаральний C Дифтеритичний D Гнійний E Гнильний

79 У хворого видалено новоутворення шкіри, яке має вигляд вузла з сосочковою поверхнею, що нагадує цвітну капусту, щільної консистенції. Мікроскопічно пухлина складається з багатьох сосочків. Паренхіма сформована з покривного епітелію із збільшеним числом шарів. В епітелії збережена полярність клітин, стратифікація, цілісність власної

мембрани. Строма пухлини розташована в центрі сосочків. Ваш діагноз? A *Папілома B Фіброма C Аденома D Фіброаденома E Цистоаденома

80 У хворого 50 років на протязі багатьох років була ревматична вада серця. При загостренні захворювання розвинулась геміплегія і наступила смерть. Гістологічно в мітральному клапані виявлено виражений склероз, вогнищеві клітинні інфільтрати, фібринозні накладання. Для якої форми ендокардиту характерні виявлені зміни? A *Зворотньо-бородавчастого B Гострого бородавчастого C Дифузного D Фібропластичного E Виразково-поліпозного

81 При гістологічному дослідженні збільшеного шийного лімфатичного вузла відмічено, що загальний рисунок вузла нечіткий за рахунок розростання атипових гістіоцитарних клітин, з наявністю гігантських клітин Березовського-Штернберга, маються ділянки некрозу, склерозу, гіалінозу. Для якого захворювання характерні виявлені морфологічні зміни в лімфатичному вузлі? A *Лімфогранулематозу B Гострого мієлолейкозу C Хронічного мієлолейкозу D Грибовидного мікозу E Туберкульозу

82 Чоловік 38 років раптово помер, на розтині у задній стінці лівого шлуночку серця знайдено інфаркт міокарду. Які найбільш вірогідні зміни у будові міокардіоцитів можна побачити у вогнищі інфаркту мікроскопійно? A *Каріолізіс B Жирова дистрофія C Вуглеводна дистрофія D Обвапнення E Білкова дистрофія

83 На розтині хворого 65 років, що помер від хронічної серцевої недостатності внаслідок

Page 147: крок 1 база 2014р_11шр-240ст (для брошури)_35грн

http://vk.com/my.printing

147

ревматичної вади серця легені бурого кольору, збільшені у розмірах, ущільнені. Як звуться такі зміни у легенях? A *бура індурація легень B мускатні легені C сотові легені D хронічний бронхіт E хронічна емфізема

84 На розтині померлого 48 років виявлена обтурація просвіту середньої мозкової артерії тромбом. У тім`яно-скроневій області лівої півкулі головного мозку -- вогнище кашеподібної консистенції сірого кольору. Найвірогідніше це: A *інфаркт B секвестр C гангрена D казеозний некроз E фібріноїдний некроз

85 Хворий 66 років за 10 років до смерті переніс перітоніт. На розтині капсула печінки та селезінки місцями різко потовщана, ущільнена, напівпрозора. Найвірогідніше це: A *гіаліноз B некроз C мукоїдне набухання D фібріноїдне набуханняя E амілоїдоз

86 Слизова оболонка трахеї тьмяна, повнокровна, з нашаруванням сіро-білих плівок. Найвірогідніша форма запалення: A *фібринозне B гнійне C серозне D проліферативне E змішане

87 Губчастий та кортикальний шари гомілкової кістки на окремих ділянках у стані розпаду, порожниниі, що створилися, заповнені вершкоподібними масами зеленувато-жовтого кольору. Найвірогідніша форма запалення: A *гнійне B катаральне C серозне

D проліферативне E змішане

88 У хворого висока температура, задуха, біль у правій половині грудної клітини. Під час плевральної пункціїї видалено 700 мл вершкоподібної рідини жовтувато-зеленого кольору. Найвірогідніший діагноз: A *емпієма плеври B карціноматоз плеври C серозний плеврит D фібринозний плеврит E геморагічний плеврит

89 У розтині у печінці померлого 62 років виявлено вогнище розпаду тканини діаметром 4 см, заповнене рідиною жовтувато-зеленого кольору. Найвірогідніший діагноз: A *абсцес B флегмона C карбункул D емпієма E гранульома

90 У харкотинні хворого з мітральною вадою серця виявлені клітини, які вміщують бурий пігмент. Реакція Перлса позитивна. Який це пігмент ? A *Гемосидерин B Гематоідин C Меланін D Порфірин E Білірубін

91 В біоптаті слизової носа знайдені епітеліоїдні клітини, плазмоцити, клітини Мікуліча, еозинофільні тільця Русселя. Ваш діагноз? A *Ріносклерома B Сифіліс C Туберкульоз D Респіраторносинцитіальна інфекція E Алергічний риніт

92 Гістологічно в апендиксі в усіх шарах знайдені в значній кількості поліморфноядерні лейкоцити, повнокрів’я, стази. Така картина є характерною для: A *Флегмонозного апендициту B Гангренозного апендициту

Page 148: крок 1 база 2014р_11шр-240ст (для брошури)_35грн

148

C Поверхневого апендициту D Простого апендициту E Хронічного апендициту

93 При розтині трупа жінки, яка померла з явищами серцевої недостатності, серце збільшене в об’ємі, дрябле; міокард – глинисто-жовтий, тьмяний; з боку ендокарду видно жовто-білу посмугованість (тигрове серце). Мікроскопічно: в групах кардіоміоцитів відсутня поперечна посмугованість, цитоплазма кардіоміоцитів вміщує дрібні краплини, що фарбуються суданом-IV в чорний колір. Ваш діагноз? A *Жирова дистрофія міокарда B Кардіосклероз C Ревматичний міокардит D Ожиріння серця E Міомаляція

94 На розтині у чоловіка 73 років виявлено збільшену, м’яку, еластичну, злегка горбкувату передміхурову залозу, яка на розрізі складається з окремих вузлів, розділених прошарками сполучної тканини. При мікроскопії відмічено збільшення кількості залозистих елементів. Величина часточок і кількість залозистих елементів в них - різні. Який процес має місце у передміхуровій залозі? A *Залозиста нодулярна гіперплазія B М’язово-фіброзна (стромальна) нодулярна гіперплазія C Змішана нодулярна гіперплазія D Аденокарцинома E Недиференційований рак

95 У чоловіка 62 років, що помер при наростаючих явищах серцевої недостатності, на розтині знайдено збільшене в об’ємі серце. Серце дряблої консистенції, камери розтягнуті, міокард на розрізі тьмяний, глинисто-жовтий. З боку ендокарда видно жовто-білу посмугованість, яка особливо виражена в сосочкових м’язах. Який патологічний процес найбільш вірогідний? A *Жирова дистрофія міокарда B Ожиріння серця C Дилятаційна кардіоміопатія D Міомаляція E Кардіосклероз

96 У хворого 60 років при торакотомії в передньому середостінні знайдені збільшені в розмірах та спаяні між собою лімфатичні вузли, з яких взято біоптат. Мікроскопічно виявлено атипові клітини, серед яких переважають клітини Ходжкіна і гігантські клітини Рід-Березовського-Штернберга, присутні лімфоцити, єозінофіли; склероз відсутній. Яке захворювання найбільш відповідає цим ознакам ? A *Змішано-клітинний варіант лімфогранульоматозу B Лімфогранулематоз з переважанням нодулярного склерозу C Лімфогранулематоз з переважанням лімфоїдної тканини D Лімфосаркома E Лімфогранулематоз з пригніченим розвитком лімфоїдної тканини

97 Під час розтину в верхній долі правої легені виявлений крупний клиноподібний осередок темно-червоної, щільної тканини. При гістологічному дослідженні в ній виявлений некроз стінок альвеол, просвіт альвеол щільно заповнений еритроцитами. Який процес розвинувся в легенях? A *Геморагічний інфаркт легень B Карніфікація легень C Гангрена легень D Крововилив в легені E Ателектаз легень

98 При розтині померлої в комі молодої людини виявлено поширений тромбоемболічний інфаркт лівої півкулі мозку, а також велика септична селезінка, імунокомплексний гломерулонефрит, виразки в стулках аортального клапану, прикриті поліпоподібними тромбами з колоніями стафілококів. Яке захворювання привело до церебральної тромбоемболії? A *Септичний бактеріальний ендокардит B Септицемія C Гострий ревматичний вальвулит D Септикопіємія E Ревматичний тромбендокардит

Page 149: крок 1 база 2014р_11шр-240ст (для брошури)_35грн

http://vk.com/my.printing

149

99 У хворої бронхіальною астмою вірусне інфікування спровокувало астматичний статус зі смертельним наслідком. При гістологічному дослідженні легень виявлено спазм і набрякання бронхіол, в їх стінках виражена інфільтрація лімфоцитами, еозинофілами і іншими лейкоцитами, а також дегрануляція лаброцитів. Який механізм гіперчутливості лежить в основі описаних змін? A *Реагінова реакція гіперчутливості B Запальний C Аутоімунний D Імунокомплексний E Імуннозумовлений клітинний цитоліз

100 При томографії у хворого в середостінні знайдені збільшені лімфатичні вузли. При гістологічному дослідженні в лімфовузлі виявлені циркулярні розростання сполучної тканини, яка оточувала гранульомоподібні утворення з лімфоцитів, плазмоцитів і великих двоядерних клітин Березовського-Штернберга. Ваш діагноз? A *Лімфогрануломатоз B Лімфосаркома C Туберкульоз D Саркоїдоз E Лімфолейкоз

101 Для гістологічного дослідження доставлена видалена на операції матка. Під слизовою оболонкою визначені численні округлої форми вузли, які чітко відмежовані від оточуючої тканини. Мікроскопічно пухлина побудована з пучків гладкої мускулатури із явищами тканинного атипізму. Ваш діагноз? A *Лейоміома. B Рак матки. C Фіброміоми. D Хоріонепітеліома E Лейоміосаркома

102 При мікроскопічному дослідженні пухлини верхньої губи, виявлено, що вона побудована з числених щілиноподібних порожнин, стінка яких вистелена сплощеним ендотелієм, заповнених рідкою кров`ю і згортками. Поставити діагноз.

A *Капілярна гемангіома. B Венозна гемангіома. C Кавернозна гемангіома. D Гемангіоперицитома. E Гломус-ангіома.

103 У дитини після перенесеної ангіни різко збільшилися лімфатичні вузли: паратрахеальні, біфуркаційні, шийні. При мікроскопічному дослідженні шийного лімфатичного вузла виявлені вогнища некрозу, обмежовані лімфоцитами, епітеліоїдними клітинами, та клітинами Пірогова-Лангханса. Вкажіть найбільш вірогідну патологію. A *Туберкульоз B Саркоїдоз C Риносклерома D Сап E Сифіліс

104 На аутопсії померлої, яка тривалий час хворіла цистітом і дискінезією сечоводів, виявлено морфологічні ознаки уремії. Нирка була нерівномірно рубцево-зморщена. У просвіті мисок містилися дрібні уратні камені і пісок. Гістологічно виявлено “щитовидну нирку”, вогнища інтерстиційного запалення. Який із нижчеперерахованих діагнозів є найбільш вірогідний? A *Хронічний пієлонефрит B Гострий пієлонефрит C Атеросклеротично зморщена нирка D Первинно зморщена нирка E Амілоїдно зморщена нирка

105 Під час розтину померлого хворого від розповсюдженого перитоніту в дистальних відділах тонкої кишки виявлено численні виразки овальної форми, які розташовані вздовж кишки. Дно виразок чисте, гладеньке, утворене м'язовою або серозною оболонкою, краї виразок рівні, закруглені. У двох виразках є перфоративні отвори діаметром до 0,5 см. Яке захворювання треба запідозрити? A *Черевний тиф B Дизентерія C Холера D Туберкульоз E Висипний тиф

Page 150: крок 1 база 2014р_11шр-240ст (для брошури)_35грн

150

106 При розтині померлого хворого на атеросклероз в головному мозку виявлено тромбоз гілки внутрішньої сонної артерії та сірого кольору вогнище вологого роз'якшення тканини. Який патологічний процес виявлен в головному мозку? A *Ішемічний інфаркт B Геморагічна інфільтрація C Гематома D Енцефаліт E Пухлина мозку

107 У хворого виникло збільшення щитовидної залози в 2 рази. При пальпації залоза щільна, поверхня нерівномірно горбиста. При гістологічному дослідженні - дифузна інфільтрація тканини залози лімфоцитами, плазматичними клітинами з утвореннм фолікулів та посилене розростання сполучної тканини. Яке захворювання має місце у хворого? A *Зоб Хасімото. B Ендемічний зоб. C Спорадичний зоб. D Дифузний токсичний зоб. E Зоб Ріделя.

108 У хворого 17 років інтраопераційно на нижній поверхні печінки виявлена пухлина розмірами 4,5х5,0х3,5см. з субсерозною локалізацією, темночервоного кольору, на розрізі представлена порожнинами значним вмістом крові. Поставити попередній діагноз. A *Кавернозна гамангiома. B Капiлярна гемангіома. C Гемангiоперицитома. D Гемангiоендотелiома. E Лiмфангiома.

109 В біоптаті бронха хворого, зловживаючого палінням, в потовщеній слизовій оболонці виявлено хронічне запалення і трансформація одношарового війчастого епітелія в багатошаровий плоский епітелій. Який з процесів найбільш вірогідний? A *Метаплазія B Гіперплазія епітелія

C Плоскоклітинний рак D Лейкоплакія E Гіпертрофія епітелія

110 При розтині жінки 40 років, яка страждала ревматоїдним артритом знайдено збільшену щільну селезінку. На розрізі її тканина коричнево-червоного кольору зі збільшеними фолікулами, які мають вигляд напівпрозорих сірувато-білуватих зерен. Вкажіть, який з перелічених патологічних процесів найбільш вірогідний? A * Сагова селезінка B Глазурна селезінка C Сальна селезінка D Гіаліноз селезінки E Порфірна селезінка

111 У дитини 12 років, яка хвора на поліоміеліт, соматичні м'язи слабкі, обсяг їх зменшений, шкіра суха, бліда. При морфологічному дослідженні біоптату мяких тканин виявлені характерні морфологічні зміни. Визначити характер патологічного процесу м'яких тканин: A *Атрофiя B Гiпертрофiя C Гiперплазія D Метаплазія E Гiпоплазія

112 У хворого, який на протязі тривалого часу зловживав табакокурінням з'явився кашель з виділенням в'язкого слизу, слабкість після незначних фізичних навантажень, блідість шкіряних покровів, за останні дв місяці схуд на 12,0 кг. При ендоскопічному дослідженні біоптату діагноз: плоскоклітинний рак. Визначити характер патологічного процесу, який передував виникненню пухлини. A *Метаплазія. B Гiпоплазія. C Гiперплазія. D Некроз. E Склероз.

113 У хворої 18 років пахові лімфатичні вузли збільшені у розмірах, не болючі, ущільнені при пальпації. В ділянціслизової оболонки

Page 151: крок 1 база 2014р_11шр-240ст (для брошури)_35грн

http://vk.com/my.printing

151

геніталій невеликих розмірів виразка з ущільненими краями та "лакованим" дном сіруватого кольору. Який найбільш вірогідний діагноз? A *Сифiліс B Туберкульоз C Лепра D Трофiчна виразка E Гонорея

114 Чоловіка з кесонною хворобою омер з ознаками гострих порушень мозкового кровообігу в басейні a. meningea media лівої півкулі головного мозку. На розтині виявлено вогнище сірого розм'якшення мозгу зазначеній області розмірами 6х7х3,4 см. Визначити характер процесу, що визвав смерть людини. A *Газова емболiя. B Жирова емболiя. C Тромбоз. D Тромб емболiя. E Атеросклероз судин.

115 Чоловік 38-ми років загинув при спробі підйому ваги. Розвинувся колаптоїдний стан. На аутопсії виявлений розрив обширної аневрізми грудного відділу аорти. Протягом життя страждав вісцеральним сифілісом. Який патологічний процес в даному випадку обумовив зменшення міцності стінки аорти, її розширення і розрив. A *Зникнення еластичних волокон. B Зникнення колагенових волокон. C Атрофія м'язового шару. D Зміни інтіми по типу "шагреневї шкіри". E Новоутворення судин.

116 Після перенесеного геморагічного інсульту у хворого розвинулася кіста головного мозку. Через 2 роки помер від післягрипозної пневмонії. На секції трупа виявлено в мозку кісту із стінками біло-ржавого відтінку,реакція Перлса позитивна. Який з процессів найбільш вірогідний у стінці кісти? A *Місцевий гемосидероз

B Загальний гемосидероз C Місцевий гемомеланоз D Інфільтрація білірубіну E Первинний гемохроматоз

117 При розтині 56-річного чоловіка, який страждав фіброзно-кавернозним туберкульозом легень, знайдено збільшену у розмірах щільну селезінку. На розрізі тканина селезінки коричнево-рожевого кольору, гладка, з вісковидною поверхнею. Який з перелічених патологічних процесів найбільш вірогідний у селезінці? A * Сальна селезінка. B Глазурна селезінка. C Порфірна селезінка. D Сагова селезінка. E Цианотична індурація

118 При розтині тила померлого чоловіка 73-х років, який довго страждав на ішемічну хворобу серця з серцевою недостатністью, знайдено: "мускатна" печінка, бура індурація легень, цианотична індурація нирок та селезінки. Вкажіть, який з видів порушення кровообігу найбільш вірогідний? A * Хронічне загальне венозне повнокрів’я B Артеріальна гіперемія C Гостре загальнне венозне повнокрів’я D Гостре малокрів’я E Хронічне малокрів’я

119 Чоловік 44-х років, хворий на інфаркт міокарду, помер від лівошлуночкової недостатності. При розтині знайдено: набряк легень, дрібнокраплинні крововиливи у серозних та слизових оболонках. Мікроскопічно: дистрофічні та некробіотичні зміни епітелію проксимальних канальців нирок, у печінці – центролобулярні крововиливи та осередки некрозу. Який з видів порушення кровообігу найбільш вірогідний? A * Гостре загальнне венозне повнокрів’я. B Артеріальна гіперемія. C Хронічне загальне венозне повнокрів’я. D Гостре малокрів’я. E Хронічне малокрів’я.

120 У товщі шкіри макроскопічно знайдена щільна пухлина, рухлива. При мікроскопії

Page 152: крок 1 база 2014р_11шр-240ст (для брошури)_35грн

152

вона представлена хаотично розташованими пучками колагенових волокон з невеликою кількістю веретеноподібних клітин. Яка пухлина видалена? A *Щільна фіброма. B Лейоміома. C Меланома. D Ліпома. E Гломус-ангіома.

121 При огляді порожнини рота на слизовій оболонці щоки визначається щільна пляма білуватого кольору діаметром біля 1 см, яка трохи підвищується над рівнем слизової оболонки. Як називається цей патологічний процес? A *Лейкоплакія. B Еритроплакія C Карніфікація D Організація E Мукоїдне набухання

122 При гістологічному дослідженні стінки аорти виявлено велику кількість ксантомних клітин, які розташовані переважно в інтимі. Для якого захворювання можлива така картина? A *Атеросклероз B Гіпертонічна хвороба C Сифілітичний мезаортит D Неспецифічний аортоартеріїт E Нодозний періартеріїт

123 У мужчины 28 лет при гистологическом исследовании шейного лимфоузла обнаружено нарушение рисунка вследствие разрастания эпителиоидных, лимфоидных клеток и макрофагов с ядрами в виде подковы, в центре некоторых скоплений клеток - бесструктурные участки бледно-розового цвета с обломками ядер. Для какого заболевания характерны данные изменения? A *Туберкулез B Лимфогранулематоз C Актиномикоз D Метастаз опухоли E Сифилис

124 При вскрытии умершего патологоанатом обнаружил, что мягкие мозговые оболочки

полнокровные, утолщены, непрозрачные, желтовато-зеленоватого цвета. Для какой формы экссудативного воспаления характерны такие изменения в мягких мозговых оболочках? A *Гнойное B Серозное C Геморрагическое D Фибринозное E Катаральное

125 У хворого після термічного опіку в шкірі сформувалися болючі пухирі, заповнені каламутноватою рідиною. Яка морфологічна форма запалення виникла у хворого? A *Серозне B Продуктивне C Крупозне D Гранульоматозне E Дифтеритичне

126 У хворого на шкірі обличчя поступово розвилась бляшка з некрозом і виразкою в центрі. При патогістологічному дослідженні біоптату виявлено розростання атипових епітеліальних клітин з великою кількістю патологічних мітозів. Який найбільш вірогідний діагноз? A *Рак шкіри B Саркома C Папілома D Трофічна виразка E Фіброма

127 Хворий на фіброзно-кавернозний туберкульоз помер при зростаючих явищах ниркової недостатності. На розтині – запах сечі, гіпертрофія лівого шлуночка, фібринозний перикардит, фібринозно-геморагічний ентероколіт. Нирки дещо зменшені в розмірах, дуже щільні, з множинними втягненнями. Гістологічно на препаратах, забарвлених Конто-рот - рожеві маси у клубочках та стінках судин, загибель і атрофія більшості нефронів, нефросклероз. Дайте характеристику нирки при даній патології. A *Амілоїдно зморщені нирки B Первинно зморщені нирки C Вторинно зморщені нирки

Page 153: крок 1 база 2014р_11шр-240ст (для брошури)_35грн

http://vk.com/my.printing

153

D Атеросклеротично зморщені нирки E Пієлонефритично зморщені нирки

128 У хворого 21 року видалена пухлина лобної частки правої півкулі головного мозку діаметром 5 см, яка була нечітко відмежована від довколишньої тканини. На розрізі - однорідного вигляду, гістологічно - складається із зіркоподібних клітин, численні відростки яких утворюють густі сплетіння. Яка пухлина мала місце у хворого? A *Астроцитома B Олігодендрогліома C Гангліоневрома D Епендімома E Хоріоїдпапілома

129 При гістологічному дослідженні біоптату шкіри у хворого 24 років виявлено казеозний некроз, оточений клітинним інфільтратом з лімфоцитів, серед яких зустрічаються окремі велетенські клітини, має місце розростання сполучної тканини, ендоваскуліти. Який характер патологічного процесу? A *Продуктивне гранульоматозне запалення. B Продуктивне iнтерстицiальне запалення. C Абсцес. D Катаральне запалення. E Iхорозне запалення.

130 У шматочку шкіри 1х2 см, який доставлений для гістологічного дослідження, знайдено новоутворення бурого кольору діаметром 0,5 см. Мікроскопічно пухлина складається з невусних клітин у вигляді тяжів та гнізд, розташованих у дермі, з бурим пігментом у цитоплазмі, який дає негативну реакцію Перлса. Який пігмент найбільш вірогідний? A *Меланін B Гематоїдин C Гемосидерин D Білірубін E Гемомеланін

131 У померлого від раптової зупинки серця чоловіка 45 років знайдено симетричний тип ожиріння III ступеню, розрив стінки правого шлуночка з гемоперікардом; під епікардом надлишкові відкладення жиру. Мікроскопічно: жирова тканина з епікарду

розповсюджується у міокард з атрофією м'язових волокон. Який процес найбільш вірогідний? A *Ожиріння серця B Жирова дистрофія міокарда C Гострий інфаркт міокарда D Iшемічна хвороба серця E Гіпертонічна хвороба

132 При розтині чоловіка 70 років, померлого від серцево-судинної недостатності, знайдено хронічне венозне повнокрів'я органів, гіпертрофія лівого шлуночка серця з дрібноосередковим кардіосклерозом, об'ємні жовтувато-білуваті бляшки в інтимі аорти з кашоподібними масами у центрі, які заглиблюються у товщу стінки. Який патологічний процес найбільш вірогідний у аорті? A *Атероматоз B Ліпоїдоз C Ліпосклероз D Артеріолосклероз E Кальциноз

133 При розтині тіла жінки 40 років, померлої від уремії, знайдено: збільшені пістряві нирки, у нирках – потовщення капілярних мембран клубочків у вигляді “дротяних петель”, осередки фібриноїдного некрозу їх стінок та “гіалінові” тромби у просвітах, у ядрах – “гематоксилінові тільця”; у серці – ендокардит Лібмана-Сакса. Яке ураження нирок найбільш вірогідне? A *Вовчаковий нефрит B Ревматичний гломерулонефрит C Холерний гломерулонефрит D Склеротична нирка E Термінальний гломерулонефрит

134 При розтині тіла шахтаря та гістологічному дослідженні у легенях було знайдено численні тонкі тяжі сполучної тканини з осередками склерозу навколо бронхів та судин легень; розростання сполучної тканини у альвеолярних перегородках з розвитком бронхіоліту та бронхоектазів. Яка форма силікозу найбільш вірогідна?

Page 154: крок 1 база 2014р_11шр-240ст (для брошури)_35грн

154

A *Дифузно-склеротична B Вузликова C Змішана D Силікотична кавернозна E Силікотуберкульоз

135 При гістологічному дослідженні передміхурової залози, оперативно видаленої у чоловіка 72 років, який скаржився на утруднення сечовипускання, виявлено: збільшення кількості залозистих та м'язових елементів. Часточкова будова залози порушена. Який процес у передміхуровій залозі найбільш вірогідний? A *Змішана форма простатопатії B Залозиста гіперплазія C М'язово-фіброзна гіперплазія D Простатит E Аденокарцинома

136 Хворий інфекційного відділення скаржився на слабкість, відсутність апетиту, підвищення температури до 38oС. На 7 добу – різкий біль у правому підребер'ї та пожовтіння шкіри. При мікроскопії біоптату печінки: порушення балкової будови, у гепатоцитах – гідропічна та балонна дистрофія, в деяких гепатоцитах – некроз, тільця Каунсільмена, на периферії часточок – збільшена кількість багатоядерних гепатоцитів. Яка форма вірусного гепатиту найбільш вірогідна? A *Циклічна жовтянична B Злоякісна C Хронічна D Холестатична E Безжовтянична

137 Ребенок 7 лет заболел остро с повышения to до 38oC, появления насморка, кашля, слезотечения и крупнопятнистой сыпи на коже. Слизистая оболочка зева отечна, гиперемирована, с белесоватыми пятнами в области щек. Воспаление какого характера лежит в основе изменений слизистой оболочки щек? A *Катаральное B Гнойное C Фибринозное D Геморрагическое

E Серозное

138 Ребенок доставлен в санпропускник в состоянии асфиксии. При осмотре в гортани обнаружены беловатые обтурирующие просвет и легко извлекаемые пленки. Врач заподозрил дифтерию. О какой форме воспаления гортани идет речь? A *Крупозное воспаление B Катаральное воспаление C Дифтеритическое воспаление D Серозное воспаление E Гнойное воспаление

139 У ребёнка 6 месяцев на коже шеи обнаружен плоский узел красного цвета, при надавливании стеклом узел бледнеет. Каков наиболее вероятный диагноз? A *Гемангиома B Пигментный невус C Меланома D Лейомиома E Лимфангиома

140 У хворого з субфебрильною температурою в біоптаті збільшеного лімфатичного вузла виявлені численні гранульоми, які містять у центрі казеозний некроз, оточений епітеліоїдними клітинами, велетенськими багатоядерними клітинами Пирогова-Лангханса і лімфоцитами. Для якого захворювання характерні такі патогістологічні зміни? A *Туберкульоз B Лімфогранульоматоз C Лімфосаркома D Лімфолейкоз E Банальний лімфаденіт

141 У дівчини 18 років з'вились різкий біль при ковтанні, збільшення лімфатичних вузлів шиї, підвищення to до 39oС. На слизовій оболонці мигдаликів – біло-жовті плівки, які важко відділяються з утворенням дефекту. Стан прогресивно погіршувався. Хвора померла на 8-й день захворювання при наростаючих явищах серцевої недостатності. Які гістологічні зміни в кардіоміоцитах найімовірніше будуть виявлені? A *Жирова дистрофія B Гідропічна дистрофія C Гіаліново-крапельна дистрофія

Page 155: крок 1 база 2014р_11шр-240ст (для брошури)_35грн

http://vk.com/my.printing

155

D Балонна дистрофія E Слизова дистрофія

142 У дитини 7 років на шкірі розгинальних поверхонь ліктьових і колінних суглобів з'явились щільні, безболісні вузлики розміром 1-2 мм. В біоптаті вузликів - велике вогнище фібриноїдного некрозу сполучної тканини з лімфоцитами і макрофагами по периферії. При якому захворюванні спостерігаються такі вузлики? A *Ревматизм B Ревматоїдний артрит C Склеродермія D Вузликовий періартеріїт E Системний червоний вовчак

143 При огляді зіву у хворого на ангіну визначається гіперемія слизової оболонки піднебіння, мигдалики збільшені в розмірах, червоні, на їх поверхні помітні дрібні біло-жовті осередки. Який клініко-морфологічний варіант ангіни найбільш імовірний у даному випадку? A *Лакунарна B Катаральна C Гнійна D Фібринозна E Некротична

144 При розтині жінки 33 років знайдено потовщення стінки шлунка в пілоричному відділі (на розрізі шари стінки розрізняються) з розростанням щільної білуватої тканини в підслизовому шарі і дрібними тяжами її в м'язовому шарі. Рельєф слизової оболонки збережений, складки ригідні, нерухомі. Яка макроскопічна форма пухлини в даному випадку? A *Інфільтрат B Вузол C Виразка D Кіста E Інфільтративно-виразкова форма

145 Хворий 65 років, що страждав на атеросклероз, госпіталізований до хірургічного відділення з приводу розлитого гнійного перитоніту. Під час операції діагностовано

тромбоз брижових артерій. Яка найбільш імовірна причина перитоніту? A *Геморагічний інфаркт B Ішемія ангіоспастична C Ішемічний інфаркт D Стаз E Ішемія компресійна

146 При пункційній біопсії в трансплантованій нирці виявлена дифузна інфільтрація строми лімфоцитами, плазмоцитами, лімфобластами, плазмобластами, а також некротичний артеріїт. Який патологічний процес розвинувся у трансплантаті? A *Імунне відторгнення B Гломерулонефрит C Iшемічне пошкодження нирки D Тубулонекроз E Пієлонефрит

147 У чоловіка, що за життя страждав на бронхоектатичну хворобу, пневмосклероз з вираженими явищами кахексії, на розтині серце зменшене в розмірах, стінки стоншені, в'ялої консистенції, на розрізі тканина бурого кольору. Відкладання якого пігменту спостерігається в міокарді? A *Ліпофусцин B Гемосидерин C Гематоїдин D Цероїд E Ліпохроми

148 У больного 43 лет ожоги пламенем правой кисти. На ладони и тыльной поверхности кисти возникло отслоение эпидермиса и образование пузырей, заполненных слегка мутной жидкостью. Какое наиболее вероятное воспаление возникло у больного? A *Серозное B Гнойное C Катаральное D Гнильное E Фибринозное

149 При гистологческом исследовании щитовидной железы, удаленной при операции, обнаружена деструкция и атрофия фолликулов, диффузная лимфоцитарная

Page 156: крок 1 база 2014р_11шр-240ст (для брошури)_35грн

156

инфильтрация с формированием лимфоидных фолликулов в строме. К какой группе заболеваний относится этот тиреоидит? A *Аутоиммунных B Бактериальных C Вызванных физическими факторами D Инфекционно-аллергических E Вирусных

150 У больного хроническим циститом в биоптате со слизистой оболочки мочевого пузыря вместе с переходным эпителием выявлены очаги многослойного плоского неороговевающего эпителия. Какой процесс лежит в основе описанных изменений в эпителии? A *Метаплазии B Дистрофии C Гиперплазии D Дисплазии E Гиперкератоза

151 Больная 22-х лет, постоянно проживающая на Западной Украине, жалуется на затрудненное носовое дыхание. При морфологпическом исследовании биоптата слизистой оболочки носовой полости обнаружены лимфоидные, эпителиоидные, плазматические клетки и клетки Микулича. Какой диагноз наиболее вероятен? A *Риносклерома B Сап C Туберкулез D Лепра E Сифилис

152 У больной 65 лет, страдающей тромбофлебитом глубоких вен голени, в поликлинике, в очереди на прием к врачу, внезапно наступила смерть. На вскрытии трупа в общем стволе и бифуркации легочной артерии найдены свободно лежащие красные рыхлые массы с тусклой гофрированой поверхностью. Какой патологический процесс в легочной артерии нашел патологоанатом? A *Тромбоэмболию B Тромбоз C Тканевую эмболию

D Эмболию инородными телами E Жировую эмболию

153 На вскрытии трупа мужчины 60 лет в миокарде передней стенки левого желудочка сердца выявлено серый неправильной формы плотный очаг 5 х 4 см. с четкими границами волокнистой структуры. Какой диагноз наиболее вероятен? A *Постинфарктный миокардиосклероз B Диффузный мелкоочаговый миокардиосклероз C Инфаркт D Миокардит E Кардиомиопатия

154 Дівчинка 4 років на 3 добу від початку захворювання на дифтерію померла від справжнього крупу. На аутопсії слизова оболонка гортані, трахеї та бронхів потовщена, набрякла, тьмяна, вкрита сіруватою плівкою, що легко відокремлюється. Визначити вид ексудативного запалення гортані A *Фібринозне B Серозне C Гнійне D Змішане E Катаральне

155 На розтині у померлого в сигмоподібній та прямій кишці множинні червоні виразки неправильної форми, між якими слизова вкрита брудно-сірою плівкою. Етіологія захворювання: A *Шигелла B Амеба C Мікобактерія туберкульозу D Сальмонела E Стафілокок

156 Хворий 46р. скаржився на утрудненя дихання носом. В біоптаті потовщеної слизової носа знайдені клітини Мікуліча, скупчення епітеліоїдних клітин, плазмоцити, лімфоцити, гіалінові кулі. Ваш діагноз? A *Склерома B Аденовірусний риніт C Алергічний риніт D Риновірусна інфекція

Page 157: крок 1 база 2014р_11шр-240ст (для брошури)_35грн

http://vk.com/my.printing

157

E Менінгококовий назофарингіт

157 У померлого, який прижиттєво отримував чисельні ін’єкції наркотиків, в печінці гістологічно знайдена гідропічна дистрофія гепатоцитів, “матовосклоподібні гепатоцити”, ацидофільні тільця Каунсільмена, лімфоцитарно-макрофагальні скопичення в портальних трактах. Найбільш вірогідна етіологія захворювання: A *Вірусна B Бактеріальна C Токсична D Паразитарна E Грибкова

158 Макроскопічно печінка збільшена в розмірах, ущільнена, тканина сірувато-жовтого кольору, з сальним блиском. Який патологічний процес лежить в основі писаних змін? A *Амілоїдоз B Гіаліноз C Жирова дистрофія D Мукоїдне набухання E Гемахроматоз

159 Після травматичного пошкодження печінки згодом відбулося повне відновлення будови та функції печінки. Як називається такий вид регенерації? A *Реституція B Субституція C Патологічна регенерація D Звичайна фізіологічна регенерація E

160 На розтині виявлено, що нирки збільшені в розмірах, поверхня крупно-горбиста за рахунок наявності числених порожнин з гладкою стінкою, заповнених прозорою рідиною. Про яке захворювання йде мова? A *Полікістоз B Некротичний нефроз C Пієлонефрит D Гломерулонефрит E Інфаркт

161 На розтині виявлено, що вся права легеня

збільшена, щільна, на плеврі нашарування фібрину, на розрізі тканина сірого кольору, з якої стікає каламутна рідина. Для якого захворювання легенів характерна така картина? A *Крупозна пневмонія B Вогнищева пневмонія C Інрерстиціальна пневмонія D Гангрена легені E Фіброзуючий альвеоліт

162 У хворої людини 42 років виявлено значне збільшення в розмірах носа, вух, нижньої щелепи та стоп. Яке захворювання можна запідозрити? A *Акромегалія B Гігантизм C Нанізм D Церебральна кахексія E Адіпозогенітальна дистрофія

163 На розтині хворого виявлено: м’яка мозкова оболонка верхніх відділів півкуль головного мозку різко повнокровна, жовто-зеленого кольору, просочена гнійним та фібринозним ексудатом, що нагадує чіпець. Для якого захворювання характерна така картина? A *Менінгококового менінгіту B Туберкульозного менінгіту C Грипозного менінгіту D Менінгіту при сибірці E Менінгіту при висипному тифі

164 У чоловіка, що за життя страждав на митральний стеноз, під час розтину виявлені ущільненіі легені, бурого кольору. Про який патологічний процес в легенях йде мова? A *Гемосидероз B Гемохроматоз C Жовтяниця D Гемомеланоз E Ліпофусциноз

165 При дослідженні стегнової кістки виявлено хронічне гнійне запалення компактної речовини та кісткового мозку, утворення секвестрів. При якому захворюванні розвиваються такі зміни? A *Остеомієліт B Ретикулосаркома C Мієломна хвороба D Остеобластокластома

Page 158: крок 1 база 2014р_11шр-240ст (для брошури)_35грн

158

E Периостит

166 На розтині хворого, який багато років працював на шахті і помер від хронічної легенево-серцевої недостатності, виявлено, що легені малоповітряні, значно ущільнені, склерозовані, верхівки емфізематозно змінені, поверхня сіро-чорного кольору, на розрізі тканина легенів аспідно-чорного кольору. Від якої хвороби настала смерть? A *Антракоз B Силікоз C Талькоз D Асбесто з E Алюміноз

167 Хворий 70 років, який страждав цукровим діабетом та переніс інфаркт міокарда, помер при явищах прогресуючої серцево-судинної недостатності. На розтині – цианотична індурація селезінки та нирок, бура індурація легень та мускатна печінка. Який вид порушення кровообігу обумовив зміни внутрішніх органів? A *Загальна хронічна венозна гіперемія B Загальна гостра венозна гіперемія C Загальна артеріальна гіперемія після анемії D Артеріальна ішемія в резульаті перерозподілу крові E Місцева хронічна венозна гіперемія

168 При розтині чоловіка 28 років, який хворів підгострим гломерулонефритом і помер при явищах прогресуючої уремії, на поверхні перикарду, вісцеральному та пристінковому листках плеври, очеревини спостерігалися нашарування у вигляді білуватих ниток, які легко відокремлювалися від підлеглої тканини. Вказати різновид запалення в серозних оболонках. A * Крупозне фібринозне запалення B Дифтеритичне фібринозне запалення C Гнійне запалення D Дифузне проліферативне запалення E Серозне запалення

169 На слизовій оболонці мигдаликів та м’якого піднебіння виявляються білувато-сірого кольору плівки, які щільно з’єднані з

підлеглою тканиною, при спробі зняти плівку на її місці виникає глибокий дефект тканини. Визначити патологічний процес, який виник на слизовій оболонці мигдаликів та м’якого піднебіння? A *Дифтеритичне запалення B Серозне запаленя C Крупозне запалення D Гнійне запалення E Змішане запалення

170 Хворий, який страждав ішемічною хворобою серця і переніс повторний інфаркт міокарда, помер при явищах прогресуючої серцево-судинної недостатності. На розтині виявлена збільшена щільна селезінка, темно-вишневого кольору на розрізі. При мікроскопічному дослідженні органа встановлені склероз пульпи і атрофія фолікулів. Яким терміном визначають зміни селезінки? A *Цианотична індурація селезінки B “Сагова” селезінка C “Сальна” селезінка D Порфирова селезінка E Септична селезінка

171 У хворого 29 років з діагнозом: багатооскольчастий перелом правого стегна на 3 добу від одержання травми з’явилися скарги на біль у грудній порожнині зправа, утруднене дихання. Через добу на фоні прогресуючої серцево-дихальної недостатності настала смерть. При гістологічному дослідженні у кровоносних судинах легень та головного мозку виявлені суданофільні краплини оранжевого кольору, які повністю перекривали просвіти судин мікроциркуляторного руслу. З яким ускладненням пов’язана смерть хворого? A * Жировою емболією B Газовою емболією C Медикаментозною емболією D Мікробною емболією E Тромбоемболією

172 У померлого від гострого трансмурального інфаркту міокарда на розтині на поверхні перикарда виявлено: ниткоподібні

Page 159: крок 1 база 2014р_11шр-240ст (для брошури)_35грн

http://vk.com/my.printing

159

відкладення білувато-коричневого кольору, які з’єднували парієтальний та вісцеральний листки перикарда між собою. Вказати різновид запалення в перикарді? A *Крупозне запалення B Дифтеритичне запалення C Серозне запалення D Гнійне запалення E Гранульоматозне запалення

173 Чоловік 63-х років, який протягом 15 років страждав хронічною дифузною обструктивною емфіземою легень, помер від прогресуючої серцевої недостатності. На розтині виявлен мускатний цироз печінки, цианотична індурація нирок та селезінки, асцит, набряки нижніх кінцівок. Для якого типу серцевої недостатності характерні дані зміни у внутрішніх органах? A *Хронічна правошлуночкова недостатність B Гостра правошлуночкова недостатність C Хронічна лівошлуночкова недостатність D Гостра лівошлуночкова недостатність E Загальна серцева недостатність

174 У льотчика, який загинув внаслідок розгерметизації кабіни літака. При гістологічному дослідженні внутрішніх органів у судинах виявлено велику кількість пухирців, у печінці – жирова дистрофія. В головному і спинному мозку – множинні дрібні ішемічні осередки сірого розм’якшення. Вкажіть найбільш ймовірну причину таких змін. A *Газова емболія B Повітряна емболія C Жирова емболія D Тромбоемболія E Тканинна емболія

175 У жінки віком 45 років, яка померла від хронічної алкогольної інтоксикації, на аутопсії виявлена різко збільшена печінка, тістоподібної консистенції, жовтоватого кольору. Мікроскопічно в цитоплазмі гепатоцитів при фарбуванні гаматоксилино та еозином

виявляються різних розмірів оптично порожні вакуолі. Який вид дистрофії має місце? A *Паренхіматозна жирова B Вуглеводна паренхіматозна C Гіаліново-крапельна D Мезенхімальна жирова E Гідропічна

176 При ультраструктурному вивченні біоптату тканини встановлено, що в цитоплазмі макрофага, який входить до складу запального інфільтрату, виявляється значне збільшення кількості лізосом. Це свідчить про? A *Активацію фагоцитозу B Проліферативну активность клітини C Сповільнення фагоцитозу: D Активацію апоптозу E Сповільнення апоптозу

177 У жінки з важкою інтоксикацією, зумовленою сепсисом, який і послужив безпосередньою причиною смерті, на розтині виявлене “тигрове серце”. Мікроскопічно в цитоплазмі кардіоміоцитів виявлені ліпіди. Який морфогенетичний механізм розвитку переважно лежить в основі даної дистрофії? A *Декомпозиція B Інфільтрація C Трансформація D Спотворений синтез E -

178 У жінки віком 34 років після наобережного поводження з праскою на правому вказівному пальці з’явився різкий біль, почервоніння, припухлість. Через кілька хвилин виник міхур, заповнений прозорою рідиною солом’яно-жовтого кольору. Проявом якого патологічного процесу є описані зміни? A *Ексудативного запалення B Травматичного набряку C Альтеративного запалення D Проліферативного запалення E Вакуольної дистрофії

179 Хворий віком 34 роки, звернувся до лікаря

Page 160: крок 1 база 2014р_11шр-240ст (для брошури)_35грн

160

зі скаргами на локальний біль в області потилиці, підвищення температури тіла у цій ділянці. Макроскопічно визначається інфільтрат конусоподібної форми багряно-синюшного кольору з жовтувато-зеленуватою верхівкою, яка піднімається над поверхнею шкіри. Поставте діагноз. A *Фурункул B Флегмона C Абсцес D Карбункул E Емпієма

180 Дівчинка віком 5 років захворіла на дифтерію. На третю добу померла від асфіксії внаслідок справжнього крупу. На розтині встановлено, що слизова оболонка гортані, трахеї та бронхів стовщена, набрякла, тьмяна, покрита сіруватими плівками, які легко відокремлюються. Про який патологічний процес свідчать морфологічні зміни у гортані: A *Крупозне запалення B Серозне запалення C Гнійне запалення D Дифтеритичне запалення E Катаральне запалення

181 Робітник тваринницької ферми гостро захворів і при наростаючих явищах інтоксикації помер. На розтині встановлено, селезінка збільшена, в’яла, на розрізі темно-вишневого кольору, зішкріб пульпи обільний. М’які мозкові оболонки на склепінні та основі мозку набряклі, просякнуті кров’ю мають темно-червоний колір (“шапочка кардинала”). Мікроскопічно: серозно-геморагічне запалення оболонок і тканин головного мозку з руйнуванням стінок дрібних судин. Поставте діагноз. A *Сибірка B Туляремія C Бруцельоз D Чума E Холера

182 При гістологічному дослідженні біоптату шкіри виявлені гранульоми, які складаються

з макрофагальних вузликів з наявністю лімфоцитів та плазматичних клітин. Крім того, зустрічаються великі макрофаги з жировими вакуолями, які містять запакованих у вигляди куль збудників захворювання (клітини Вірхова). Грануляційна тканина добре васкуляризована. Для якого захворювання характерна описана гранульома? A *Лепри B Туберкульозу C Сифілісу D Риносклероми E Сапу

183 При мікроскопічному дослідженні біоптату шкіри виявляються гранульоми, які складаються з епітеліоїдних клітин, оточених в основному Т-лімфоцитами. Серед епітеліоїдних клітин розташовуються поодинокі гігантські багатоядерні клітини типу Пирогова-Лангхаінса. В центрі деяких гранульом виявляються ділянки казеозного некрозу. Кровоносні судини відсутні. Для якого захворювання характерні описані гранульому? A *Туберкульозу B Сифілісу C Лепри D Риносклероми E Сапу

184 Дитина віком 8 років захворіла гостро. Через дві доби від початку захворювання наступила смерть. На аутопсії виявлено, що м’які мозкові оболонки різко повнокровні, просякнуті густим каламутним жовтувато-зеленуватим ексудатом на базальній поверхні головного мозку. Тканина мозку набрякла. Поставте діагноз? A *Менінгококова інфекція B Скарлатина C Коклюш D Дифтерія E Кір

185 На розтині трупа чоловіка віком 56 років у термінальному відділі тонкої кишки виявлено

Page 161: крок 1 база 2014р_11шр-240ст (для брошури)_35грн

http://vk.com/my.printing

161

декілька виразок розмірами від 4 до 5 см. Краї виразок піднімаються над поверхнею слизової оболонки, стінки виразок покриті сірувато-жовтуватими масами, які кришаться. Реакція Відаля позитивна. Поставте діагноз. A *Черевний тиф B Паратиф C Поворотний тиф D Дизентерія E Хвороба Крона

186 Жінки віком 49 років, тривалий час хворіла хронічним гломерулонефритом, який привів до смерті. На розтині встановлено, що нирки мають розміри 7х3х2.5 см, масу 65,0 г, щільні, дрібнозернисті. Мікроскопічно: фібринозне запалення серозних і слизових оболонок, дистрофічні зміни паренхіматозних органів, набряк головного мозку. Яке ускладнення привело до вказаних змін серозних оболонок і внурішніх органів? A *Уремія B Анемія C Сепсис D ДВЗ-снндром E Тромбоцитопенія

187 Чоловік віком 63 роки хворіє на хронічний фіброзно-кавернозний туберкульоз легень протягом 24 років, поступив у нефрологічне відділення з явищами уремії. Прижиттєва проба на наявність у нирках амілоїду виявилася позитивною. Яка форма амілоїдозу в даному випадку має місце? A *Вторинний системний B Первинний системний C Обмежений (місцевий) D Сімейний вроджений E Сенильний (старечий)

188 У дитини, після перенесеного кору, при огляді виявлено у м’яких тканинах щік і промежини нечітко відмежовані, набряклі, червоно-чорного кольору ділянки, які злегка флуктують. Яке ускладнення розвинулося у дитини? A *Волога гангрена (нома) B Суха гангрена C Газова гангрена

D Пролежень E Трофічна виразка

189 У новонародженої дитин при огляді виявлено дефект в ділянці твердого піднебіння у вигляді порожнини. Внаслідок якого процесу він утворився? A *Порушення апоптозу B Атрофії C Некрозу D Дистрофії E Запалення

190 При електронномікроскопічному дослідженні біоптату печінки виявлено, що між чисельними мітохондріями знаходиться велика кількість плоских цистерн і міхурців із секреторними гранулами, стінки яких утворені мембранами. Про гіперплазію складових якої ультраструктури йде мова? A *Апарату Гольджі B Піноцитозних міхурців C Ендоплазматичного ретикулуму D Лізосом E Мікротрубочок

191 Під час розтину тіла дівчики 9 років у верхівці правої легені субплеврально було зайдене вогнище казеозного некрозу діаметром 15 мм, біфуркаційні лімфатичні вузли були збільшені, містили дрібні вогнища некрозу коагуляційного типу. Мікроскопічно - у легеневому вогнищі та в лімфатичних вузлах навколо некротичних мас були розташовані епітеліоїдні клітини, лімфоцити та поодинокі багатоядерні гігантські клітини. Діагностуйте захворювання. A *Первинний туберкульоз B Гематогенний туберкульоз з переважним ураженням легень C Гематогенний генералізований туберкульоз D Вторинний вогнищевий туберкульоз E Вторинний фіброзно-вогнищевий туберкульоз

192 Чоловік 59 років мав ознаки

Page 162: крок 1 база 2014р_11шр-240ст (для брошури)_35грн

162

паренхіматозної жовтяниці та портальної гіпертензії. Під час гістологічного дослідження пункційного біоптату печінки було знайдено: балково-часточкова будова порушена, частина гепатоцитів має ознаки жирової дистрофії, утворюються порто-портальні сполучнотканинні септи з формуванням псевдочасточок, з наявністю періпортальних лімфо-макрофагальних інфільтратів. Діагностуйте захворювання печінки. A * Цироз печінки B Алкогольний гепатит C Хронічний гепатоз D Вірусний гепатит E Токсичная дисрофія

193 У чоловіка 72 років на секції виявлено збільшення та деформація правого колінного суглобу. При гістологічному дослідженні тканин суглобу та прилягаючих ділянок знайдено: масивні вогнища казеозного некрозу, які оточені валоми епітеліоїдних, лимфоідних клітин з наявністю гігантських макрофагів. Діагностуйте захворювання. A *Туберкульозний артрит B Сифілітичний артрит C Ревматоїдний артрит D Гонорейний артрит E Деформуючий остеоартроз

194 Під час гістологічного дослідження стулок мітрального клапана серця жінки 30-ти років було встановлено, що ендотеліальні клітини вогнищево десквамовані, в цих ділянках на поверхні стулки розташовані дрібні тромботичні нашарування, сполучна тканина стулки з явищами мукоідного набрякання з ділянками склерозу та васкулярізації. Діагностуйте вид клапанного ендокардиту. A *Поворотньо-бородавчастий B Дифузний C Гострий бородавчастий D Фібропластичний E Поліпозно-виразковий

195 У збільшеному шийному лімфатичному вузлі дівчинки 14 років, мікроскопічно були знайдено, що тканинна будова вузла

порушена, лімфоїдні фолікули відсутні, є ділянки склерозу та вогнища некрозу, клітинний склад вузла поліморфний, присутні лімфоцити, еозинофіли, атипові клітини великих розмірів з багаточасточковими ядрами (клітини Березовського-Штернберга) та одноядерні клітини також великих розмірів. Діагностуйте захворювання. A *Лімфогранулематоз B Гострий лімфолейкоз C Хронічний лімфолейкоз D Лімфома Беркіта E Грибовидний микоз

196 Жінка 54 років мала значну деформацію суглобів пальців рук і стоп. Гістологічно: навколосуглобова сполучна тканина - мукоїдне набухання, ділянки фібриноїдного некрозу, скупчення макрофагів, ділянки склерозу; в синовіальній оболонці - набрякі ворсини, з ознаками мукоїдного та фібриноїдного набухання, в синовіальній порожнині зустрічаються поодинокі "рисові тільця". Діагностуйте захворювання. A *Ревматоїдний артрит B Ревматизм C Хвороба Бехтерева D Гематогенний туберкульоз E Подагра

197 Під час розтину тіла жінки, яка померла внаслідок пухлинної дисемінації муцинозної цистаденокарциноми і тривалий час мала вимушене положення в ліжку, були знайдені велики ділянки некрозу шкіри та підлеглих м’яких тканин крижової ділянці. Діагностуйте форму некрозу. A *Пролежень B Інфаркт C Секвестр D Сирнистий некроз E Воскоподібний (ценкеровський) некроз

198 У біоптаті слизової оболонки бронха в хворого на хронічний бронхіт були знайдені ділянки багатошарового плоского незроговілого епітелію. Діагностуйте патологічний процес в слизовій оболонці бронха. A *Метаплазія

Page 163: крок 1 база 2014р_11шр-240ст (для брошури)_35грн

http://vk.com/my.printing

163

B Атрофія C Гіперплазія D Дисплазія E -

199 Жінка 32 років померла в післяпологовому періоді. На розтині були знайдені ознаки гнійного ендометриту, гнійний тромбофлебіт вен матки, множинні абсцеси легень, абсцеси нирок і селезінки, апостематозний міокардит та гнійний менінгіт. Діагностуйте вид сепсису. A *Септикопіємія B Септицемія C Хроніосепсис D Затяжний септичний ендокардит E -

200 При мікроскопічному дослідженні міокарду дівчинки, яка померла від дифтерії внаслідок серцевої недостатності зайдені: жирова дистрофія та множинні осередки некрозу кардіоміоцитів, незначні вогнищеві клітинні інфільтрати інтерстицію. Про який міокардит йде мова? A *Альтеративній міокардит B Дифузний ексудативний міокардит C Осередковий ексудативний міокардит D Інтерстиціальний міокардит E Гранульоматозний міокардит

201 Під час розтину тіла чоловіка 29 років, який тривалий час хворів на виразкову хворобу дванадцятипалої кишки були знайдені ознаки перитоніту, множинні стеатонекрози позаочеревинної жирової тканини та підшлункової залози, а в діляці тіла ії знайдений виразкоподібний дефект діаметром 5 мм і глибиною до 10 мм, краї якого містили некротичні маси. Діагностуйте ускладнення виразкової хвороби дванадцятипалої кишки. A *Пенетрація B Кровотеча C Стеноз D Перфорація E Малігнізація

202 У чоловіка віком 50 років, який гостро захворів, діагностовано дизентерію. Смерть наступила на 8-й день захворювання. Під

час розтину виявлено потовщену стінку сигмовидної та початкових відділів прямої кишки, фібринозну плівку на поверхні слизової. Гістологічно: глибокий некроз слизової оболонки з просяканням некротичних мас фібрином. Який вид коліту мав місце у хворого? A *Дифтеритичний коліт B Катаральний коліт C Виразковий коліт D Гангренозний коліт E Фолікулярний коліт

203 У чоловіка 40 років в ділянці шиї виникло почервоніння та набряк шкіри і з часом розвинувся невеликий гнійник. На розрізі осередок щільний, жовто-зеленого забарвлення. В гнійних масах видно білі крупинки. Гістологічно виявлено друзи грибка, плазматичні та ксантомні клітини, макрофаги. Вкажіть найбільш ймовірний вид мікозу. A *Актиномікоз B Аспергільоз C Кандидоз D Споротрихоз E Кокцидіоїдомікоз

204 На розтині чоловіка, який помер на 5-у добу захворювання черевним тифом, виявлені наступні зміни: групові фолікули клубової кишки збільшені, повнокровні і виступають над слизовою оболонкою, на їх поверхні видно борозди та звивини. Гістологічно: повнокрів’я і набряк тканини, наявність гранульом, які складаються з великих клітин зі світлою цитоплазмою і містять черевнотифозні палички. Про який період місцевих змін при черевному тифі можна думати? A *Стадія мозкоподібного набухання B Стадія некрозу C Стадія загоювання виразок D Стадія чистих виразок E Стадія утворення виразок

205 При розтину трупа чоловіка 40 років виявлено різко розширений просвіт тонкої кишки, переповнений рідиною, яка нагадує “рисовий відвар”. Стінка кишки набрякла, на

Page 164: крок 1 база 2014р_11шр-240ст (для брошури)_35грн

164

слизовій велика кількість дрібнокрапкових крововиливів. Для якого інфекційного захворювання характерний описаний ентерит? A *Холера B Дизентерія C Сальмонельоз D Амебіаз E Черевний тиф

206 Під час розтину трупа чоловіка 55 років, який протягом останніх восьми років хворів на хронічну форму тропічної малярії, виявлено, що сіра речовина головного мозку та селезінка аспідно-сірого кольору. Який пігмент зумовив таке забарвлення? A *Гемомеланін B Ліпофусцин C Гематопорфірин D Меланін E Гемосидерин

207 При розтині трупу жінки 48 років, яка раптово померла, в інтимі аорти визначаються ділянки жовтуватого кольору, які не підвищуються над її поверхнею. При гістологічному дослідженні цих ділянок виявляються клітини з пінистою цитоплазмою, а при фарбуванні суданом ІІІ жовтого кольору. Для якої стадії атеросклерозу характерні такі зміни в аорті A *Ліпоїдозу B Ліпосклерозу C Атероматозу D Виразки E Атерокальцинозу

208 Чоловік хворів на цукровий діабет 15 років, помер від крововиливу у мозок. На розтині нирки зменшені у розмірах, поверхня дрібнозерниста. Епіпелій канальців дистального сегменту високий, зі світлою пінявою цитоплазмою, при забарвленні карміном Беста та при ШИК - реакції - яскраво-червоного кольору. Зміни в епітелії зумовлені наявністю A *Глікогену B Ліпідів

C Гіаліну D Протеїнів E Амілоїду

209 Жінка 45 років звернулася до гінеколога зі скаргами на ацикличні, надмірні, маткові кровотечі. При досліджені біоптату виявлено збільшення кількості залоз, кістозне її розширення. Для якого процесу характерні ці зміни A *Гіперплазії ендометрію B Атрофія ендометрію C Гіпертрофії ендометрію D Метаплазії E Організації

210 При розтині трупа жінки, 69 років яка тривалий час страждала на гіпертрофічну хворобу, паталогоанатом виявив, що обидві нирки щільної консистенції, значно зменшені у розмірах, поверхня їх дрібнозерниста. Ці зміни свідчать про A *Атрофію від недостатнього кровопостачання B Атрофію від тиску C Старечу атрофію D Атрофію дисфункціональну E Гіпоплазію

211 При розтині трупу чоловіка 62 років, який хворів на атеросклероз і помер від гострої серцевої недостатності, у передній стінці лівого шлуночка серця виявлена щільна ділянка біло-жовтого кольору розміром 6х5 см, неправильної форми, чітко відмежована від навколишніх тканин гемарагічним вінчиком. Встановить діагноз. A *Інфаркт міокарду B Постінфарктний кардіосклероз C Дрібно-осередковий кардіосклероз D Міокардит E -

212 Жінка 36 років, яка хворіла на дифтерію, померла від гострої серцевої недостатності. На розтині порожнини серця розширені, м’яз серця тьмяний, строкатий, на розрізі з жовтуватими ділянками. Який процес виявлений у кардіоміоцитах A *Жирова дистрофія

Page 165: крок 1 база 2014р_11шр-240ст (для брошури)_35грн

http://vk.com/my.printing

165

B Вуглеводна дистрофія C Балонна дистрофія D Гіаліново-крапельна дистрофія E Гідропічна дистрофія

213 У чоловіка 43 років, який тривалий час хворів на туберкульоз, розвинулась кровотеча з легень, що призвела до смерті. На розтині в легенях виявлено декілька порожнин овальної або круглої форми, стінка яких створена некротичними масами та тканиною легень. Для якої форми туберкульозу характерні такі зміни A *Гострий кавернозний B Фіброзно-кавернозний C Туберкулома D Казеозна пневмонія E Гострий вогнищевий

214 При мікроскопії біоптату печінки виявлені гранульоми, які складаються з плазматичних, лімфоїдних, гігантських багатоядерних клітин типу Пирогова-Лангхарса, дрібних судин з ознаками ендо- і периваскуліту, зустрічаються осередки казеозного некрозу. Такі гранулеми характерні для A *Сифілісу B Туберкульозу C Лепри D Риносклероми E Сапу

215 У дівчинки 8 років раптово підвищилась температура, з’явились катаральні явища дихальних шляхів. На 5-й день хвороби виник параліч м’язів нижніх кінцівок і приєднались дихальні розлади. В передніх рогах спинного мозку виявлена проліферація глії навколо загиблих нейронів. Про яке захворювання йде мова? A *Поліомієліт B Кір C Дифтерію D Менінгококцемію E Скарлатину

216 При розтині чоловіка, померлого від

опікової хвороби, знайдено набряк головного мозку, збільшення печінки, а також нирок, корковий шар яких широкий, блідо-сірий, мозковий – повнокровний. Мікроскопічно: некроз канальців головних відділів з деструкцією базальних мембран, набряк інтерстицію з лейкоцитарною інфільтацією та крововиливами. Який з перелічених діагнозів найбільш вірогідний? A *Некротичний нефроз. B Тубулоінтерстиціаальний нефрит. C Пієлонефрит. D Подагрична нирка. E Мієломна нирка.

217 При розтині чоловіка, який тривалий час зловживав алкоголем, печінка малих розмірів, щільна, дрібнобугриста. Мікроскопічно: псевдочасточки дрібні, розподілені вузькими прошарками сполучної тканини з лімфомакрофагальними інфільтратами; гепатоцити у стані великокрапельної жирової дистрофії. Який з перелічених діагнозів найбільш вірогідний? A *Алкогольний цироз B Хронічний активний алкогольний гепатит C Хронічний персистуючий алкогольний гепатит D Токсична дистрофія печінки E Жировой гепатоз

218 У 30-ти річного чоловіка, який хворів на гостре респіраторне захворювання та загинув при явищах гострої легенево-серцевої недостатності, під час розтину знайдено фібринозно-геморагічне запалення в слизовій оболонці гортані та трахеї, деструктивний панбронхит, збільшені легені, яки мають пістрявий вигляд за рахунок абсцесів, крововиливів, некрозу. Який з перелічених діагнозів найбільш вірогідний? A *Грип. B Парагрип. C Респіраторно-синцитіальна інфекція. D Кір. E Аденовірусна інфекція.

219

Page 166: крок 1 база 2014р_11шр-240ст (для брошури)_35грн

166

Під час розтину чоловіка, у якого після поранення кінцівки виникло тривале нагноєння рани, і який помер при явищах інтоксікації, знайдено загальне виснаженя, зневоднення, бура атрофія печінки, міокарду, селезінки, поперечно-смугастої мускулатури та амілоїдоз нирок. Який з перелічених діагнозів найбільш вірогідний? A *Хронісепсіс. B Септикопіємія. C Септицемія. D Хвороба Чорногубова. E Бруцельоз.

220 При розтині померлого, який хворів на туберкульоз, у верхній долі правої легені знайдено порожнину розмірами 3х2 см, яка сполучається з бронхом. Стінка порожнини щільна, має три шари: внутрішній – піогенний, середній – шар туберкульозної грануляційної тканини, зовнішній – сполучнотканинний. Який з перелічених діагнозів найбільш вірогідний? A *Фіброзно-кавернозний туберкульоз. B Фіброзно-осередковий туберкульоз. C Туберкульома. D Гострий осередковий туберкульоз. E Гострий кавернозний тубеокульоз.

221 У больного 40 лет при прогрессировании стафиллококкового гнойного периодонтита возникло гнойное воспаление костно-мозговых пространств альвеолярного отростка, а затем тела нижней челюсти. Микроскопически костные балки истончены, очаги некроза, костные секвестры, окруженные соединительнотканной капсулой. Назовите заболевание. A *Хронический остеомиелит B Острый остеомиелит C Пародонтома D Хронический фиброзный периостит E Гнойный периостит

222 У хворого на туберкульоз в біоптаті нирки при гістологічному дослідженні у вогнищі казеозного некрозу виявлені безпорядно розсипані дрібні зерна хроматину. Наслідком

чого є виявлені зміни? A * Каріорексису B Каріолізису C Пікнозу ядер D Мітотичної активності ядер E Апоптозу

223 У хворого о 8 годині ранку з’явилися болі за грудиною, о 9 годині ранку в приймальному відділенні за даними ЕКГ встановлений інфаркт міокарда. Через 10 хвилин хворий помер. Яку найбільш достовірну морфологічну ознаку інфаркту міокарда знайдуть при гістологічному дослідженні після розтину трупа? A * Зникнення глікогену в кардіоміоцитах B Вакуольну дистрофію кардіоміоцитів субендо_кар_ді_аль_ної зони C Жирову інфільтрацію кардіоміоцитів D Некроз міокардіоцитів E Розслаблення міофібрил кардіоміоцитів

224 В біоптаті нирки: склероз, лімфо-плазмоцитарна інфільтрація стінок мисок та чашок; дистрофія та атрофія канальців. Збережені канальці розширені, розтягнені колоїдоподібними масами, епітелій сплюснутий (“щитовидна” нирка). Який діагноз найбільш вірогідній? A * Хронічний пієлонефрит B Гострий пієлонефрит C Гломерулонефрит D Нефросклероз E Тубуло-інтерстиціальний нефрит

225 У померлого 44 років, громадянина України, за результатами розтину встановлено поєднання пневмоцистної пневмонії, саркоми Капоші та В-клітинної лімфоми. В анамнезі невпорядковані статеві контакти. Що є найбільш вірогідним? A *Інфекція вірусом імунодефіциту людини, стадія СНІДу B Інфекція вірусом імунодефіциту людини, стадія преСНІДу C Вторинний імунодефіцит внаслідок первинної В-клітинної лімфоми D Вторинний імунодефіцит внаслідок саркоми Капоші E -

Page 167: крок 1 база 2014р_11шр-240ст (для брошури)_35грн

http://vk.com/my.printing

167

226 Патологоанатом в біоптаті шкіри побачив гостре серозно-геморагічне запалення і ділянку некрозу. З анамнезу: захворювання почалося з появи невеликої червоної плями, в центрі якої утворився міхур із серозно-геморагічною рідиною. Згодом центральна частина стала чорною. Що є більш вірогідним? A *Карбункул при сибірці B Актиномікоз шкіри C Алергічний дерматит D Карбункул стрептококовий E Хімічний дерматит

227 На розтині померлого 18 років селезінка вагою 580 грамів, темно-червоного кольору на розрізі, дає рясний зскрібок пульпи. Гістологічно: виражена проліферація ретикулярних клітин, наявність великої кількості зрілих нейрофілів в сінусоїдних капілярах. Як правильно називати таку селезінку? A *“Септична” B “Сагова” C “Порфірова” D “Ціанотична” E “Лейкозна”

228 У юнака 18 років, який захворів гостро і помер від інфекційно-токсичного шоку, на автопсії виявлено збільшені мигдалики, вкриті сіро-білими плівками, що поширюються на піднебінні дужки, набряк м’яких тканин шиї. При гістологічному дослідженні: некроз епітелію мигдаликів та дужок, підлеглі тканини просякнуті фібринозним ексудатом, який утворює масивні нашарування на поверхні. Діагностуйте захворювання A *Дифтерія B Скарлатина C Аденовірусна інфекція D Інфекційний мононуклеоз E Стафілококова інфекція

229 Жінці 40 років проведено операцію тироїдектомії. При гістологічному дослідженні щитоподібної залози виявлено, що її

фолікули різновеликі, містять пінистий колоїд, епітелій фолікулів високий, місцями формує сосочки, в стромі - осередкова лімфоцитарна інфільтрація. Діагностуйте захворювання щитоподібної залози. A *Зоб Базедов B Тироїдит Хашімото C Тироїдит Ріделя D Тироїдит де Кервена E Вузловий зоб

230 У чоловіка 60 років, який тривалий час хворів на гіпертонічну хворобу і помер від хронічної ниркової недостатності, при автопсії виявлено, що обидві нирки значно зменшені у розмірах, поверхня їх дрібно-зерниста; гістологічно - більшість клубочків гіалінізована, частина склерозована, окремі гіперплазовані; в стромі поля склерозу, артеріоло- та артеріосклероз, еластофіброз великих гілок ниркових артерії. Назвіть виявлені зміни нирок. A *Артеріолосклеротичний нефросклероз B Атеросклеротичний нефросклероз C Хронічний гломерулонефрит D Хронічний пієлонефрит E Вторинно зморщена нирка

231 У чоловіка 42 років, що хворів на важку форму черевного тифу, розвинулась гостра ниркова недостатність, від якої він помер. При автопсії нирки збільшені у розмірах, набряклі, фіброзна капсула знімається легко; на розрізі кора – блідо-сіра, піраміди темно-червоні. При гістологічному дослідженні - у більшості канальців просвіт звужений, епітеліальні клітини збільшені у розмірах без ядер; клубочки колабовані; у стромі набряк, невелика лейкоцитарна інфільтрація, дрібні геморагії. Про яку патологію нирок йде мова? A *Некронефроз B Гострий пієлонефрит C Гострий гломерулонефрит D Піонефроз E Гідронефроз

232

Page 168: крок 1 база 2014р_11шр-240ст (для брошури)_35грн

168

У хворого 45 років поступово почали пропорційно збільшуватись в розмірах стопи, права кисть, ніс і губи. Виявлена аденома гипофізу. Яке захворювання можна запідозрити? A * Акромегалія B Нанізм C Хвороба Базєдова D Аддисонова хвороба E Цукровий діабет

233 При мікроскопічному дослідженні нирок спостерігається проліферація нефротелія капсули Боумена, подоцитів і макрофагів з утворенням в її просвіті “півмісяцевих” структур, які стискують клубочок. Капіляри клубочків зазнають некрозу, в їх просвіті спостерігаються фібринові тромби. Деякі клубочки склерозуються або гіалінізуються. Спостерігається виражена дистрофія нефроцитів, набряк та інфільтрація строми нирок. Назвіть патологію нирок. A * Швидкопрогресуючий гломерулонефрит B Післяінфекційний гломерулонефрит C Хронічний гломерулонефрит D Хронічний пієлонефрит E Амілоїдоз нирок

234 У хворого, який помер від хронічної ниркової недостатності на грунті хронічного гломерулонефрита нирки зменшені в розмірах, щільної консистенції, капсула знімається важко, оголяючи зернисту поверхню. На розрізі кірковий та мозковий шари тонкі, тканина нирок суха, недокрівна, сірого кольору. Як називається така нирка? A * Вторинно-зморщені нирки B Первинно-зморщені нирки C Атеросклеротично-зморщені нирки D Амілоїдно-зморщені нирки E Пієлонефротично-зморщені нирки

235 При гістологічному дослідженні щитовидної залози визначається значна інфільтрація лімфоцитами зутворенням лімфоїдних фолікулів, руйнування паренхіматозних елементів, розростання волокон сполучної тканини. Для якого захворювання характерна така картина? A * Зоб Хосімото

B Колоїдний зоб C Ендемічний зоб D Дифузний токсичний зоб E Паренхіматозний зоб

236 При розтині трупа чоловіка 47 років померлого раптово, в інтимі черевного відділу аорти знайдені осередки жовтого кольору у вигляді плям та смуг, що не вибухають над поверхнею інтими. При фарбуванні суданом ІІІ спостерігається померанчове забарвлення. Для якої сдадії атеросклерозу характерні такі зміни? A *Ліпоїдозу. B Ліпосклерозу. C Атероматозу. D Атерокальцинозу. E Стадія утворення атероматозної виразки.

237 У 30-річного померлого наркомана, який страждав на ВІЛ-інфекцію при патоморфологічному дослідженні виявлено, що обидві легені ущільнені, темно-бордово-сірі, мало повітряні, міжальвеолярні перетинки густо інфільтровані лімфоцитами, частина альвеолоцитів трансформовані у великі клітини, з центрально розташованим круглим ядром зі світлим обідком, і які нагадують “совине око”. Яка опортуністична інфекція викликала пневмонію у даного хворого. A *Цитомегаловірус B Пневмоциста карінії C Атипова мікобактерія D Герпес-вірус E Токсоплазма

238 У померлої дитини 4 років при житті: менінгіальна симптоматика, На розтині в м’якій мозковій оболонці макроскопічно виявлені просоподібні вузлики, які мікроскопічно представлені осередком казеозного некрозу з валами епітеліоїдних, лімфоїдних клітин, між якими зустрічаються великі клітини з ядрами на периферії у вигляді півмісяця. Який найбільш вірогідний менінгіт у дитини? A *Туберкульозний B Сифілітичний C Бруцельозний

Page 169: крок 1 база 2014р_11шр-240ст (для брошури)_35грн

http://vk.com/my.printing

169

D Грипозний E Менінгококовий

239 При гістологічному дослідженні біоптатів, взятих з потовщених країв виразки шлунка, виявлені невеликі гніздові скупчення різко атипових гіперхромних невеликих епітеліальних клітин, які розташовані серед дуже розвиненої строми. Визначте пухлину. A *Скіррозний недиференційований рак B Медулярний рак C Аденокарцинома D Недиференційована саркома E Аденома

240 Хворому хірург видалив порожнинне утворення печінки діаметром 2 см. Встановлено, що стінка порожнини утворена щільною волокнистою сполучною тканиною, вміст являє собою каламутну, густу, жовтувато-зеленуватого кольору рідину з неприємним запахом, яка мікроскопічно складається переважно з поліморфноядерних лейкоцитів. Якому патологічному процесу відповідають такі морфологічні зміни? A *Хронічному абсцесу B Гострому абсцесу C - D Емпіємі E Флегмоні

Патологічна фізіологія 1 Хворому, у якого підвищена кислотність шлункового соку, лікар порекомендував їсти варене, а не смажене м'ясо. Це пов'язано з тим, що механізм дії екстрактивних речовин полягає у: A *Стимуляції вироблення гастрина G клітинами B Подразненні смакових рецепторів C Подразненні механорецепторів ротової порожнини D Подразненні механорецепторів шлунка E Стимуляції вироблення секретіна в 12-типалій кишці

2 Після перенесеного сепсису у хворої 27 років з’явився бронзовий колір шкіри, характерний для аддісонової хвороби.

Механізм гіперпігментації полягає в підвищенні секреції гормону: A *Меланоцитстимулюючого B Соматотропного C Гонадотропного D B-ліпотропного E Тиреотропного

3 Хворому з ревматоїдним артритом тривалий час вводили гідрокортизон. У нього з’явилися гіперглікемія, поліурія, глюкозурія, спрага. Ці ускладнення лікування є наслідком активації процесу A *Гліконеогенезу B Глікогенолізу C Глікогенезу D Гліколізу E Ліполізу

4 У жінки 46 років після операції на щитовидній залозі в невдовзі з’явилися фібрилярні посмикування м’язів рук, ніг, обличчя. Ці порушення можна усунути шляхом введення A *Паратгормону B Трийодтироніну C Тиреотропіну D Тироксину E Тіреотропного гормону

5 У пацієнта, який півтора місяця тому переніс інфаркт міокарда, діагностовано синдром Дреслера з характерною тріадою: перикардит, плеврит, пневмонія. Який головний механізм цього ускладнення? A *Сенсибілізація організму антигенами міокарда B Зниження резистентності до інфекційних агентів C Активація сапрофітної мікрофлори D Інтоксикація організму продуктами некрозу E Викидання у кров міокардіальних ферментів

6 У хворого з гіпертонічною хворобою виявлено значне збільшення маси міокарда лівого

Page 170: крок 1 база 2014р_11шр-240ст (для брошури)_35грн

170

шлуночка. Це сталося внаслідок : A *Збільшення об’єму кардіоміоцитів B Збільшення кількості кардіоміоцитів C Розростання сполучної тканини D Затримки води в міокарді E Жирової інфільтрації міокарда

7 У дитини з геморагічним синдромом діагностована гемофілія В. Вона зумовлена дефіцитом фактора A *ІХ (Крістмаса) B ІІ (протромбіну) C VІІІ (антигемофільного глобуліну) D ХІ (протромбопластину) E ХІІ (Хагемана)

8 У хворого після оперативного удалення кисти підшлункової залози виник геморагічний синдром з вираженим порушенням зсідання крові. Розвиток цього ускладнення пояснюється A *Активацією фібринолітичної системи B Недостатнім утворенням фібрину C Зменшенням кількості тромбоцитів D Активацією протизгортальної системи E Активацією фактору Крисмаса

9 У хворого після травматичної перерізки сідничного нерва виникли трофічні зміни шкіри. Основним механізмом їх появи є : A *Припинення аксоплазматичного току B Втрата нервом збудливості C Фагоцитоз нервових закінчень D Руйнування мієлінової оболонки E Пошкодження перехватів Ранв’є

10 У лікарню доставили хворого на цукровий діабет у стані непритомності. Дихання типу Кусмауля, артеріальний тиск 80/50 мм рт.ст., з запахом ацетону з рота. Накопиченням в організмі яких речовин можна пояснити виникнення даних розладів? A *Кетонових тіл B Модифікованих ліпопротеїдів C Молочної кислоти D Вугільної кислоти E Складних вуглеводів

11 У хворої з феохромоцитомою після психічного навантаження виникає тахікардія,

підвищується артеріальний тиск, з’являється різкий біль у надчеревній ділянці. Ці приступи можна пояснити A *Масивним викиданням катехоламінів наднирниками B Звільненням норадреналіну симпатичними нервами C Активацією вегетативних ядер гіпоталамуса D Збільшенням секреції тиреоїдних гормонів E Підвищеним синтезом адренокортикотропного гормону

12 Електрокардіографічне дослідження пацієнта з гіпертонічною хворобою показало такі результати: ритм синусовий, правильний, частота серцевих скорочень 92/хв, тривалість РQ – 0,2 с, QRS – не змінений. У хворого є порушення A *Атоматизму B Провідності C Збудливості D Рефрактерності E Скоротливості

13 У пацієнтки, яка перехворіла на грип, шляхом електрокардіографічного дослідження виявлено таке: частота серцевих скорочень 140/хв, ритм синусовий, коливання величини R-R не перевищують 0,15 с, тривалість PQ – 0,2 c, QRS – не змінений. Ці показники свідчать про розвиток A *Синусової тахікардії B Синусової брадікардії C Фібриляція шлуночків D Пароксизмальної тахікардії E Мерехтіння шлуночків

14 У хворого виявлена аденома, що походить з клітин клубочкової зони кори наднирників. В результаті цього розвинувся первинний гіперальдостеронізм або хвороба Кона. На обмін якого іону впливає цей гормон? A * натрію B хлору C магнію D кальцію E заліза

Page 171: крок 1 база 2014р_11шр-240ст (для брошури)_35грн

http://vk.com/my.printing

171

15 У хворого має місце хронічна недостатність кіркової речовини надниркових залоз ( Аддісонова або бронзова хвороба). Недостатність якого гормону має місце при цьому патологічному процесі? A *Альдостерону B Інсуліну C Адреналіну D Тироксину E Вазопресину

16 Через 1 – 2 доби після видалення у собаки прищитовидних залоз спостерігались: млявість, спрага, різке підвищення нервово-м’язової збудливості з розвитком тетанії. Яке порушення обміну електролітів має місце при цьому? A *Гіпокальціемія B Гіперкальціемія C Гіпомагніемія D Гіпермагніемія E Гіпонатріемія

17 Хвора, 24 р., скаржиться на сухість в роті, зниження маси тіла, незважаючи на підвищений апетит, підвищення сечовиділення. Які дослідження для постановки діагнозу необхідно призначити в першу чергу? A *Визначення рівня цукру в добовій кількості сечі B Аналіз сечі по Зимницькому C Загальний аналіз сечі D Дослідження білкових фракцій сироватки крові E Коагулограма

18 Еритроцити – 3,0*1012/л; Hb – 90г/л; ретикулоцити – 0,5%. В мазку: пойкілоцити, гіпохромні еритроцити. Залізо сироватки крові – 80мкмоль/л. Для якої патології це характерно? A * Залізорефрактерна анемія. B Хвороба Мінковського-Шоффара. C Залізодефіцитна анемія. D В12-дефіцитна анемія. E Серпоподібноклітинна анемія.

19 Загальна кількість лейкоцитів-90(109/л. В

лейкоцитарній формулі: е-0%, б-0%, ю-0%, п-2%, с-20%, лімфобласти -1%, пролімфоцити-2%, лімфоцити-70%, м-5%, клітини Боткіна-Гумпрехта. У хворого збільшені шийні, підщелепні лімфатичні вузли. Для якої патології характерна така картина крові? A * Хронічний лімфолейкоз. B Гострий лімфолейкоз. C Лімфогранульоматоз. D Інфекційний мононуклеоз. E Хронічний мієлолейкоз.

20 Хворий скаржиться на часті нудоти, які нерідко завершуються блювотою. Порушення якої із функцій шлунку найвірогідніше запідозрити у даного хворого? A * Евакуаторної. B Єкскреторної C Всмоктувальної D Інкреторної. E Секреторної.

21 У хворого діагностовано асцит. На животі виступають крупні судини синюшного відтінку. Ознакою якої гіпертензії є дане явище? A * Портальної B Малого кола кровообігу. C Есенціальної D Церебро-ішемічної E Ниркової

22 У хворого М., 55р., виявили гіперплазію кори наднирників. АТ – 190/90 мм.рт.ст.; в крові - вміст глюкози – 20 ммоль/л, збільшення кортікотропину; в сечі - глюкозурія. Спостерігається ожиріння, гірсутизм. Для якої патології характерні виявлені зміни? A * Хвороби Іценка-Кушинга. B Хвороби Аддісона. C Синдрому Іценка-Кушинга. D Адипозогенітальної дистрофії. E Хвороби Барракера- Сіммондса.

23 Хвора, 28 р., скаржиться на в’ялість, швидку розумову та фізичну втомлюваність, диспептичні порушення. При обстеженні виявлено: позитивні туберкулінові проби, гіпоглікемія, АТ – 90/60

Page 172: крок 1 база 2014р_11шр-240ст (для брошури)_35грн

172

мм.рт.ст.,гіпонатріємію, пігментацію шкіри. При якій патології наднирників спостерігаються подібні явища? A * Хвороба Аддісона. B Синдром Іценка-Кушинга. C Гостра недостатність кори наднирників. D Гіпофункція мозкового шару наднирників. E Синдром Конна.

24 У фізично здорових молодих вояків після важкого фізичного навантаження при одноденному пішому переході на 50 км в сечі виявлено білок, рівень якого в середньому не перевищував 1 г/л. Який різновид протеїнурії, в першу чергу, мав місце ? A *Маршова протеїнурія B Дегідраційна протеїнурія C Аліментарна протеїнурія D Органічна протеїнурія E Несправжня протеїнурія

25 У хворого на ішемічну хворобу серця на грунті атеросклерозу коронарних артерій після коронарографії розвинувся тромбоз передньої міжшлуночкової вінцевої артерії. Який механізм в розвитку цього ускладнення є найбільш суттєвим? A *Пошкодження ендотелію судинної стінки B Сповільнення плину крові C Підвищення концентрації коагулянтів крові D Зменшення вмісту антикоагулянтів крові E Зниження активності фібринолітичної системи

26 У хворих на поворотний тиф виникає лихоманка, яка характеризується кількаденними періодами високої гарячки, що чергується з періодами нормальної температури. Така температурна крива називається: A *Febris recurrns B Febris hectica C Febris intermittens D Febris continua E Febris atypica

27 У чоловіка 49 років, який 12 років тому хворів ревматичним міокардитом та ендокардитом, є недостатність мітрального клапану. Дослідження показали, що

запального процесу зараз не має, хвилинний об’єм кровообігу достатній. Якому поняттю загальної нозології відповідає дана умова? A *Патологічний стан. B Патологічна реакція. C Патологічний процес. D Типовий патологічний процес. E Компенсаторна реакція.

28 У хворого 54 років, який на виробництві багато контактував зі свинцем, виявлена гіпохромна анемія. Лікування препаратами заліза протягом місяця ефекту не дало. Встановлено підвищений вміст заліза в сиворотці крові. Чим обумовлений розвиток анемії в цьому випадку? A *Порушенням синтезу порфіринів. B Дефіцитом вітаміна В12. C Дефіцитом фолієвої кислоти. D Гіпоплазією червоного кісткового мозку. E Дефіцитом белку

29 У хворого 20 років періодично з’являється жовтушність склер та шкіри, слабкість. Діагностована хвороба Мінковського-Шоффара. Що найбільш характено для картини крові при цьому захворювані? A *Мікросфероцитоз. B Анулоцитоз C Агранулоцитоз. D Макроцитоз. E Тромбоцитоз.

30 Хвора 45 років скаржиться на задишку при невеликому фізичному навантажені, набряки на ногах, в анамнезі часті ангини, хворіє на протязі двох років. Діагнастовано недостатність кровообігу. Який гемодинамічний показник декомпенсації серця спостерігається в даному випадку? A *Зменшення хвилинного об’єму серця. B Зменшення об’єму циркулюючої крові. C Зменшення венозного тиску. D Підвищення артеріального тиску. E Тахікардія.

31 У хворого на ішемічну хворобу серця раптово з’явився тяжкий приступ стенокардії. Про нього: обличча бліде, шкіра волога,

Page 173: крок 1 база 2014р_11шр-240ст (для брошури)_35грн

http://vk.com/my.printing

173

холодна, АД 70/50 мм рт. ст. екстрасистолія. Діагнастован інфартк міакарду та кардіогенний шок. Назвіть первинний ланцюг патогенезу. A *Зменшення хвилинного об’єму крові. B Екстрасистолія. C Токсемія. D Больовий синдром. E Гіпотензія.

32 Хвора 44 років скаржиться на загальну слабкість, збільшення маси тіла, ріст волосся на обличчі, зупинку менструацій, АТ 165/100 мм.рт.ст. Що допоможе диференціювати хворобу Іценко–Кушинга від синдрому Іценко–Кушинга? A *Рівень кортикотропіну в плазмі крові. B Рівень кортизолу в плазмі. C Вміст 17 –оксікетостероїдів в сечі. D Ренгенографія черепу. E Кількість еозинофілів в крові.

33 У чоловіка 25 років виявлена недостатність митрального клапану без порушення кровообігу. Який негайний механізм забеспечуе серцеву компенсацію? A *Гетерометричний B Гомеометричний C Міогенна дилятація D Зменшення маси серця. E Посилення синтезу катехоламінів

34 У дівчини 15 років виявлен стеноз аорти, проте розладів кровообігу у неї не спостерігалось. Який негайний механізм забеспечуе серцеву компенсацію? A *Гомеометричний B Підвищення АТ C Гетерометричний D Зменшення маси серця. E Міогенна дилятація

35 У дитини 5 років на ЕКГ спостерігалось порушення ритму серцевої діяльності.При затримці диханя ритм серцевої діяльності ставав правильним. Який вид порушень було виявлено у дитини на ЕКГ? A *Дихальна аритмія. B Миготлива аритмія. C Синусна екстросистолія.

D Предсердна екстросистолія. E Поперечна блокада серця.

36 У чоловіка 50 років раптово виникло сильне серцебиття, біль у серці, різка слабкість, підвищення АТ, пульс неправильний з дефіцітом. На ЕКГ виявлено відсутність зубця Р і різні інтервали R-R. Яке порушення серцевого ритму у хворого? A *Миготлива аритмія. B Дихальна аритмія. C Пароксизмальна тахікардія . D Поперечна блокада серця. E Синусова екстрасистолія.

37 Чоловік 57 років скаржиться на біль в серці, який виник після тривалих негативних емоцій. Лікар швидкої допомоги встановив ішемічну хворобу серця, що проявилась стенокардієюя. Який механізм ішемії найбільш вирогідний? A *Ангіоспастичний B Странгуляційний C Облітераційний. D Компресійний. E Обтураційний.

38 У хворого з алкогольним цирозом печінки скарги на загальну слабкість, задишку. Встановлено зниження артеріального тиску, асцит, розширення поверхневих вен передньої стінки живота, спленомегалію. Яке порушення гемодинаміки спостерігається у хворого? A *Синдром портальної гіпертензії. B Недостатність лівого шлуночка серця. C Недостатність правого шлуночка серця. D Колапс. E Тотальна серцева недостатність

39 У хворого виявлено порушення прохідності дихальних шляхів на рівні дрібних і середніх бронхів. Які зміни кислотно-основної рівноваги можуть розвинутись у пацієнта? A *Респіраторний ацидоз B Респіраторний алкалоз C Метаболічний ацидоз D Метаболічний алкалоз E КОР не зміниться

Page 174: крок 1 база 2014р_11шр-240ст (для брошури)_35грн

174

40 У дитини, хворої на дифтерію, розвинувся набряк гортані. Який розлад дихання спостерігається у даному випадку? A *Диспное (задишка) B Гаспінг-дихання C Апнейстичне дихання D Дихання Куссмауля E Дихання Біота

41 У хворої після пологів через декілька місяців почалося випадіння волосся, втрата ваги, млявість, випадіння зубів. АТ , температура тіла, рівень глюкози крові - знижені . При обстеженні рівень соматотропного і кортикотропного гормонів в крові знижений. Яке порушення функції гіпофізу у хворої? A *Пангіпопітуїтризм B Гіпофізарний нанізм C Акромегалія D Хвороба Іценко-Кушінга E Нецукровий діабет

42 Хвора 45 років поступила в лікарню з повною втратою свідомості, арефлексією, випадінням зіничного і рогівкового рефлексів, періодичним диханням типу Кусмауля. АТ, температура тіла - знижені. Аналіз крові: заг.білірубін - 16,0 мкмоль/л, сечовина - 3,6 ммоль/л, креатинін - 10,8 мкмоль/л, глюкоза - 22 ммоль,л. Для якого виду коми характерна дана картина? A *Гіперглікемічна B Гіпоглікемічна C Печінкова D Ниркова E При недостатності наднирників

43 Хворий 23 років поступив у лікарню із черепно-мозковою травмою у важкому стані. Дихання характеризується судомним тривалим вдохом який переривається коротким видихом. Для якого типу дихання це характерно? A *Апнейстичного B Гаспінг-дихання C Куссмауля D Чейн - Стокса E Біота

44 Зміни на ЕКГ проявлялись спочатку подовженням інтервалу P-Q, потім випадінням поодиноких комплексів QRS, пізніше – збільшенням кількості випадаючих шлуночкових комплексів, і, нарешті, передсердя скорочувались з частотою 70 уд./ хв., а шлуночки – 35 уд./ хв. Описані зміни характерні для: A * Атріовентрикулярної блокади B Внутрішньопередсердної блокади C Внутрішньошлуночкової блокади D Екстрасістолії E Брадікардії

45 Хворому з цирозом печінки разом з лікарськими препаратами довенно ввели 500,0 мл 5\% розчину глюкози. Яке порушення водно-електролітного балансу найбільш ймовірно може виникнути у хворого? A * Гіпоосмолярна гіпергідратація B Гіперосмолярна гіпергідратація C Ізоосмолярна гіпергідратація D Гіпоосмолярна дегідратація E Гіперосмолярна дегідратація

46 Тварині, сенсибілізованій туберкуліном, внутрішньоочеревенно введений туберкулін. Через 24 години при лапаратомії виявлено венозну гіперемію та набряк очеревини. У мазках-відбитках з очеревини велика кількість лімфоцитів та моноцитів. Який патологічний процес у тварини? A * Алергічне запалення B Серозне запалення C Гнійне запалення D Фібринозне запалення E Асептичне запалення

47 У хворого 70-ти років перебіг атеросклерозу ускладнився тромбозом судин нижніх кінцівок, виникла гангрена пальців лівої стопи. Початок тромбоутворення найбільш ймовірно пов’язаний з: A * Адгезією тромбоцитів B Активацією протромбінази C Перетворенням протромбіну в тромбін D Перетворення фібриногену в фібрин E Зниженням синтезу гепарину

Page 175: крок 1 база 2014р_11шр-240ст (для брошури)_35грн

http://vk.com/my.printing

175

48 У юнака 20 років травмоване праве яєчко. Яку небезпеку це може становити для лівого (здорового) яєчка на 2-3 тижні після травми? A * Демаскування антигену та виникнення ушкодження антитілами B Розвиток інфекційного процесу C Розвиток атрофії D Розвиток гіпертрофії E Не загрожує нічим

49 У хлопчика 5 міс., при дослідженні імунного статусу виявлено зменшення імуноглобулініві, особливо IgA та IgM. В крові та лімфатичних вузлах відсутні В-лімоцити та плазматичні клітини. Реакції Т-лімфоцитів збережені. Захворювання передається по спадковості як зчеплене із статтю. Яка патологія спостерігається у цієї дитини? A * Хвороба Бутона B Синдром Луі-Барр C Синдром Віскотта-Олдрича D Імунодефіцит швейцарського типу E Рання гіпогаммаглобулінемія

50 У хлопчика 3 років з вираженим геморагічним синдромом відсутній антигемофільний глобулін А (фактор VIII) в плазмі крові. Яка фаза гемостазу первинно порушена у цього хворого? A * Внутрішній механізм активації протромбінази B Зовнішній механізм активації протромбінази C Перетворення протромбіну в тромбін D Перетворення фібриногену в фібрин E Ретракція кров’яного згустку

51 У хворого в результаті вогнепального поранення стегна пошкоджений сідничний нерв. Будь-який вплив на хвору кінцівку спричиняє жорсткий, нестерпний біль. Який механізм формування болісних відчуттів найбільш ймовірний у цьому випадку? A * Каузалгічний B Рефлекторний

C Фантомний D Гіпофункція ендорфіну E Гіпофункція енкефаліну

52 У тварини в експерименті проведена перерізка обох блукаючих нервів. Як зміниться характер дихання у тварини? A * Рідке і глибоке B Часте та глибоке C Рідке та поверхове D Часте та поверхове E Періодичне

53 У хворого 44-х років на ЕКГ виявлені ознаки гіпертрофії обох шлуночків та правого передсердя. Діагностовано недостатність тристулкового клапана. Який патогенетичний варіант цієї недостатності? A * Перевантаження серця об’ємом B Перевантаження серця опором C Первинна міокардіальна недостатність D Коронарна недостатність E Тампонада серця

54 При обстеженні у юнака 16 років було виявлено прискорення серцебиття під час вдиху, сповільнене – під час видиху. На ЕКГ відмічалося: вкорочення інтервалу RR під час вдоху та подовження його під час видоху. Назвіть вид аритмії A * Синусова аритмія B Миготлива аритмія C Синусова тахікардія D Ідіовентрикулярний ритм E Синусова брадикардія

55 У хворого, 42 років, скарги на болі в епігастральній ділянці, блювоту; блювотні маси кольору “кофейної гущі”; мелена. В анамнезі виразкова хвороба шлунку. Аналіз крові: еритроцити – 2,8х1012/л , лейкоцити – 8х109/л, гемоглобін 90 г/л. Вкажіть найбільш ймовірне ускладнення, яке виникло у хворого? A * Кровотеча B Пенетрація C Перфорація

Page 176: крок 1 база 2014р_11шр-240ст (для брошури)_35грн

176

D Пререродження в рак E Пілоростеноз

56 У хворого після отруєння грибами з’явилося жовте забарвлення шкіри та склер, темний колір сечі. Який пігмент спричинює забарвлення сечі у хворого на гемолітичну жовтяницю? A * Стеркобілін B Моноглюкуронід білірубіну C Некон’югований білірубін D Вердоглобін E Білівердин

57 Для моделювання виразки шлунка тварині ввели в гастральні артерії атофан, який спричинює їх склерозування. Який механізм пошкодження слизової оболонки шлунку є провідним в даному експерименті? A * Гіпоксичний B Нейродистрофічний C Механічний D Дисрегуляторний E Нейрогуморальний

58 Хвора на хронічний гепатит скаржиться на підвищення чутливості до барбітуратів, які раніше вона переносила без симптомів інтоксикації. З порушенням якої функції печинки це пов’язане у найбільшій мірі? A * Метаболічної B Утворення жовчі C Гемодинамічної D Гемопоетичної E Фагоцитарної

59 У хворого на бронхіальну астму виникла гостра недостатність дихання. Який тип недостатності дихання виникає в даному випадку? A * Обструктивне порушення альолярної вентиляції B Рестриктивне порушення альолярної вентиляції C Перфузійний D Дифузійний E Дисрегуляторне порушення альолярної вентиляції

60 Щуру в плевральну порожнину введено 0,5 мл повітря. Який тип недостатності дихання

виникає в даному випадку? A * Рестриктивне порушення альолярної вентиляції B Обструктивне порушення альолярної вентиляції C Перфузійний D Дифузійний E Дисрегуляторне порушення альолярної вентиляції

61 У хворого на правець виникла гостра недостатність дихання. Який тип недостатності дихання виникає в даному випадку? A * Дисрегуляторне порушення альолярної вентиляції B Рестриктивне порушення альолярної вентиляції C Обструктивне порушення альолярної вентиляції D Перфузійний тип E Дифузійний тип

62 У ВІЧ-інфікованого хворого спостерігається пригнічення активності імунної системи. Ураження яких клітин найбільшою мірою обумовлює стан імунодефіциту у цього хворого? A * Т-хелперів B Т-супресорів C Макрофагів D В-лімфоцитів E Т-кілерів

63 При мікроскопічному дослідження пунктату з осередка запалення у хворого із абсцесом шкіри знайдено велику кількість різних клітин крові. Які з цих клітин першими надходять із судин до тканин при запаленні? A * Нейтрофіли B Моноцити C Базофіли D Еозинофіли E Лімфоцити

64 У хворого на атрофічний гастрит виник дефіцит вітаміу В12. Яка зміна лейкоцитарної формули є найбільш типовою для гіповітамінозу В12? A *Ядерний зсув вправо B Дегенеративний зсув вліво C Гіперрегенеративний зсув вліво

Page 177: крок 1 база 2014р_11шр-240ст (для брошури)_35грн

http://vk.com/my.printing

177

D Регенеративно-дегенеративний ядерний зсув вліво E Регенеративний ядерний зсув вліво

65 На п"яту добу післі гострої крововтрати у хворого діагностована гіпохромна анемія. Який головний механізм у розвитку гіпохромії? A *Надходження з кісткового мозку незрілих еритроцитів B Порушення всмоктування заліза у кишечнику C Посилене руйнування еритроцитів у селезінці D Порушення синтезу глобіну E Підвищення виділення заліза з організму

66 При дослідженні стану імунної системи хворого із хронічними грибковими ураженнями шкіри виявлено порушення клітинного імунітету. Зниження яких показників найбільш характерні при цьому? A * Т-лімфоцитів B Імуноглобулінів G C Імуноглобулінів E D В-лімфоцитів E Плазмоцитів

67 У хворого після відкритої травми хребта виявлено розрив правої половини спинного мозку. Зникнення якого виду чутливості слід очікувати тількі з боку розриву? A * Пропріоцептиної B Температурної C Больової D Тактильної E -

68 В експерименті кролю ввели нефроцитотоксичну сироватку морськоі свинки. Яке захворювання нирок моделювалося в цьому досліді? A * Гострий дифузний гломерулонефрит B Нефротичний синдром C Гострий пієлонефрит D Хронічна ниркова недостатність E Хронічний пієлонефрит

69 У хворого після важкої травми грудної

клитки розвинувся шок та з’явилися ознаки гострої ниркової недостатності [ГНН]. Що є провідним механізмом розвитку ГНН в даному випадку? A * Падіння артеріального тиску B Порушення відтоку сечі C Підвищення тиску в капсулі клубочка D Підвищення тиску в ниркових артеріях E Зменшення онкотичного тиску крові

70 У хворого із хронічною нирковою недостатністю встановлення зменшення кліренсу за інуліном до 60 мл/хв. З порушенням якої функції нирок це пов’язано? A * Клубочкової фільтрації B Канальцевої секреції C Реабсорбції в проксимальному відділі нефрону D Реабсорбції в дистальному відділі нефрону E Реабсорбції в збиральних ниркових трубочках

71 У хворого на хронічну ниркову недостатність з’явилися анорексія, диспепсія, порушення ритму серця, свербіння шкіри. Який механізм розвитку цих порушень є головним? A * Накопичення продуктів азотистого обміну в крові B Порушення ліпідного обміну C Зміни вуглеводного обміну D Нирковий ацидоз E Порушення водно-електролітного обміну

72 У хворого на цукровий діабет розвинулася діабетична кома внаслідок порушення кислотно-основного стану. Який вид порушення виник при цьому? A * Метаболічний ацидоз B Метаболічний алкалоз C Респіраторний ацидоз D Газовий алкалоз E Негазовий алкалоз

73 У дитини 6 років розвинулася гіперергічна форма запалення верхніх дихальних шляхів. З’явилася загорза серйозного порушення дихання, а тому виникла необхідність

Page 178: крок 1 база 2014р_11шр-240ст (для брошури)_35грн

178

застосувати протизапальні гормони. Серед гормонів протизапальний ефект проявляє A * Кортизол B Адреналін C Соматотропін D Тестостерон E Інсулін

74 До лікаря звернувся чоловік 27 років. При огляді було виявлено збільшення кистей, стоп та нижньої щелепи. Крім того спостерігалась деформація суглобів (kiphosis), гормональні порушення (імпотенція, атрофія яєчок). Функції якої залози порушені? A * Передньої частини гіпофізу B Надниркових залоз C Шишкоподібного тіла D Щитовидної залози E Прищитовидних залоз

75 У хворого Д., 32 роки, гнійна рана у нижній третині передпліччя. Хворому зроблено мазок із гнійного вмісту рани. Які клітини в основному виявлено при забарвленні мазку за Романовським-Гімзою? A *Нейтрофіли B Еозинофіли C Лімфоцити D Еритроцити E Базофіли

76 Під час бойових дій у госпіталь доставили солдата з тяжким осколочним пошкодженням хребта. У пораненого встановлено наявність перерізки правої половини спинного мозку (синдром Броун-Секара). Зникненням якого виду чутливости проявляеться цей синдром? A * пропріоцептивної - справа B температурної - справа C пропріоцептивної - зліва D больової - справа E тактильноі - справа

77 Хлопчик 1,5 років постійно хворіє на піодермію та тричі хворів на пневмонію. В крові знижена кількість имуноглобулінів G та A.

Який вид імунодефіциту виник у дитини? A * Гіпогаммаглобулінемія Брутона B Швейцарський тип C Гіпоплазія вилочкової залози D Синдром Віскотта –Олдрича E Синдром Луи-Барр

78 Жінці поставлено діагноз ерозія шійки матки, яка є передпухлинною патологією. Який захисний механізм може попередити розвиток пухлини? A * Збільшення природних кілерів (NK-клітин) B Високодозова імунологічна толерантність C Збільшення активності лізосомальних ферментів D Спрощення антигенного складу тканин E Низькодозова імунологічна толерантність

79 Хворого було доставлено до лікарні у стані діабетичної коми. Дихає хворий шумно, після глибокого вдоху йде посилений видих. Який тип дихання спостерігається у цього хворого? A * Куссмауля B Чейна-Стокса C Гаспінг D Апнейстичне E Біота

80 У хворого на дифтерію розвинувся набряк гортані. При цьому спостерігається рідке та глибоке дихання з затрудненням вдоху. Який тип дихання спостерігається при цьому? A * Стенотичне B Куссмауля C Чейна-Стокса D Апнейстичне E Гаспінг

81 При моделюванні запалення на брижі жаби спостерігали крайове стояння лейкоцитів та їх еміграцію крізь судинну стінку. Який із перелічених факторів обумовлює цєй процес? A * Вплив хемотаксичних речовин B Збільшення онкотичного тиску в осередку запалення C Зниження онкотичного тиску в судинах D Збільшення гідростатичного тиску в судинах

Page 179: крок 1 база 2014р_11шр-240ст (для брошури)_35грн

http://vk.com/my.printing

179

E Зменшення гідростатичного тиску в судинах

82 Під час огляду шкіри лікар помітив у хворого нагнійний процес у вигляді круглих підвищень червонуватого коліру, оточених зоною гіперемії. Які медиатори запалення зумовили явища судинної гіперемії? A *Гістамін B Інтерлейкін 1 C Фактор активації тромбоцитів . D Тромбоксан E Лізосомальні ферменти

83 У жінки 57 років після тривалого больового нападу у правому підребір’ї з’явилась жовтяниця, після чого хвора звернулась до лікаря. Виникла підозра на появність у хворої гострого калькульозного холециститу. Дослідження якого показника крові свідчить про непрохідність жовчних протоків? A *Вільного та зв’язаного білірубіну B Білкових фракцій C Загальних ліпідів D Сечової кислоти E Залишкового азоту

84 Хлопчику 15 років встановлено діагноз гострого вірусного гепатиту. Дослідження якого показника крові необхідно провести для підтвердження гострого ураження печінкових клітин? A *Активність амінотрансфераз [ЛЛТ і АСТ] B Вміст вільного та зв’язаного білірубіну C Швидкість осідання еритроцитів [ШОЕ]. D Рівень холестерину E Вміст білкових фракцій

85 У чоловіка 25 років діагностований гострий дифузний гломерулонефрит. З анамнезу хворого відомо, що за 18 днів до прояви цієї хвороби переніс ангіну. Який механізм ураження ниркових клубочків буде спостерігатися у цьому випадку? A *Імунний. B - C Нефротоксичний. D Ішемічний. E Медикаментозний.

86 У хворого внаслідок отруєння бертолетовою сіллю розвинулася гемічна гіпоксія. Утворення якої речовини грае роль в патогенезі цієї гіпоксиї? A *Метгемоглобіну B Оксиду азоту C Сульфгемоглобіну D Карбгемоглобіну E Карбоксігемоглобіну

87 Білому щуру ввели під шкіру сулему в дозі 5 мг/кг маси тіла. Через 24 години в плазмі крові концентрація креатиніну збільшилася в декілька разів. Який механизм ретенційної азотемії в даному випадку ? A *Зниження клубочкової фільтрації B Збільшення утворення креатиніну в м`язах C Зростання реабсорбції креатиніну D Зростання клубочкової фільтрації E Збільшення секреції креатиніну в канальцях нирок

88 Внаслідок травмування у хворого видалили прищитовидні залози, що супроводжувалося: млявістю, спрагою, різким підвищенням нервово-м’язової збудливості. З порушенням обміну якої речовини це пов’язано: A *Кальцію B Марганцю C Хлору D Молібдену E Цинку

89 У хворого з гломерулонефритом виявлено:анасарка, АТ – 185/105 мм рт.ст., анемія, лейоцитоз, гіперазотемія, гіпопротеінемія. Який показник свідчит про ускладнення гломерулонефриту нефротичним синдромом ? A *Гіпопротеїнемія B Лейкоцитоз C Гіперазотемія D Артеріальна гіпертензія E Анемія

90 Чоловік 63 років, страждає раком

Page 180: крок 1 база 2014р_11шр-240ст (для брошури)_35грн

180

стравоходу, метастази в лімфатичні вузли середостіння, ракова кахексія. Яка патогенетична стадія пухлинного процесу чоловіка? A *прогресії B промоції C трансформації D ініціації E -

91 Жінку 44 років вжалила оса внаслідок чого розвинувся шок. В анамнезі вже була тяжка алергічна реакція на жалення оси. Об"єктивно: пульс 179 уд /хв, слабкий, АТ-80/40 мм рт. ст., ЧД-26 за хв. Яка провідна ланка патогенезу анафілактичного шоку? A *зниження периферійного опору судин B тахікардія C біль D зменшення ударного об'єму серця E зменшення об"єму циркулюючої крові

92 Хворий скаржиться на періодичне ослаблення стула, яке пов’язує з прийомом багатої на жири їжі. При цьому він відмічає зменшення забарвленості калу. При лабораторному обстеженні встановлено нормальний вміст ліпідів в сироватці крові. Порушення якого із станів ліпідного обміну має місце у даного хворого? A *Всмоктування. B Транспорту кров’ю. C Проміжного обміну. D Депонування в жировій тканині. E Мобілізація із жирової тканини.

93 При повному (з водою) аліментарному голодуванні розвинулись генералізовані набряки. Який із патогенетичних факторів у цьому випадку є ведучим? A *Зниження онкотичного тиску плазми крові. B Зниження гідростатичного тиску міжклітинної рідини. C Зниження осмотичного тиску плазми крові. D Підвищення онкотичного тиску тканинної рідини. E Підвищення осмотичного тиску міжклітинної рідини.

94

У пілота на висоті 14000 м трапилася аварійна розгереметизація кабіни. Який із видів емболій у нього розвинувся? A *Газова. B Емболія стороннім тілом. C Тромбоемболія. D Повітряна. E Жирова

95 При работе с радиоактивными веществами сотрудник вследствие аварии получил дозу общего облучения 4 Гр. Жалуется на головную боль, тошноту, головокружение. Какие изменения в составе крови можно ожидать у больного через 10 часов после облучения? A *Нейтрофильный лейкоцитоз B Лимфоцитоз C Лейкопению D Агранулоцитоз E Нейтропению

96 У больной А., 18 лет, после перенесенной краснухи начала отмечаться потеря массы тела, постоянное ощущение сухости во рту, жажду, повышение аппетита, частое мочеотделение. Объективно: суточное количество мочи 6 л, глюкоза крови 17,8 ммоль/л, в моче выявлена глюкоза и ацетон. Каков наиболее вероятный патогенетический механизм вызвал повышение уровня глюкозы у больной? A *Уменьшение выработки инсулина B Увеличение глюконеогенеза C Повышенное разрушение инсулина D Повреждение инсулиновых рецепторов клеток E Увеличение выработки глюкокортикоидов

97 У больного С., выявлены такие изменения в периферической крови: Эр. 3,2x1012/л, Гем. 80 г/л, Лейк. 25x109/л. Лейкоцитарная формула: базофилы - 5%, эозинофилы - 9%, миелобласты - 3%, промиелоциты - 8%; миелоциты - 11%, метамиелоциты - 22%, палочкоядерные - 17%, сегментоядерные - 19%, лимфоциты - 3%, моноциты - 3%. Определите наиболее вероятную патологию соответствующую данному описанию картины крови:

Page 181: крок 1 база 2014р_11шр-240ст (для брошури)_35грн

http://vk.com/my.printing

181

A *хронический миелолейкоз B острый миелобластный лейкоз C эритромиелоз D лейкемоидная реакция E недеферинцируемый лейкоз

98 У больного П., выявлены такие изменения в периферической крови: Эр. 3,0x1012/л, Гем. 80 г/л, Лейк. 21x109/л. Лейкоцитарная формула: базофилы - 0%, эозинофилы - 0%, миелобласты - 54%, промиелоциты - 1%; миелоциты - 0%, метамиелоциты - 0%, палочкоядерные - 1%, сегментоядерные - 28%, лимфоциты - 13%, моноциты - 3%. Определите наиболее вероятную патологию соответствующую данному описанию картины крови: A *острый миелобластный лейкоз B хронический миелолейкоз C эритромиелоз D лейкемоидная реакция E недеферинцированный лейкоз

99 Больной Л., жалуется на отрыжку, изжогу, частые запоры. При титровании желудочного сока получили такие данные: общая кислотность - 88 млмоль/л., общая HCl - 83 млмоль/л, свободная HCl - 50 млмоль/л, связанная HCl - 33 млмоль/л, кислые фосфаты и органические кислоты - 5 млмоль/л. Оценить состояние кислотности желудка: A *Гиперацидное состояние B Гипоацидное состояние C Ахилия D Нормацидное состояние E Гипохлоргидрия

100 У больного Д., после длительного приступа сильных головных болей стали невозможны активные движения левой руки и ноги. На этих конечностях тонус мышц повышен, мышцы спазмированы, спинальные сухожильные рефлексы резко усилены, расширены зоны рефлексов. Какое расстройство нервной системы имеется у больного? A *Центральный паралич B Периферический паралич C Экстрацирапидный паралич

D Вялый паралич E Рефлекторный паралич

101 Больной В. 38 лет, доставлен в приемное отделение с признаками гипоксии развившейся после отравления угарным газом. Состояние средней тяжести, тахикардия, одышка, АД 160/100. Какой механизм токсического действия окиси углерода на организм? A *Образование карбоксигемоглобина B Образование метгемоглобина C Нарушение диссоциации оксигемоглобина D Образование карбгемоглобина E Блокада кальциевых каналов эритроцитов

102 У больного П., после травмы возникла необходимость введения противостолбнячной сыворотки, однако проба на чувствительность к сыворотке оказалась положительной. Как провести специфическую гипосенсибилизацию у больного? Введением: A *малых дробных доз специфического аллергена B физиологических доз глюкокортикоидов C разрешающей дозы специфического аллергена D лечебных доз антигистаминных препаратов E наркотических веществ снижающих чувствительность

103 У больной Л., 40 лет, через месяц после сочетанной автомобильной травмы отсутствуют активные движения во всех суставах правой ноги. Объем мышц в области голени правой ноги на 2 см меньше, чем на левой. Ахилов и коленный рефлексы справа отсутствуют. Определяется термгипестезия и гипальгезия на наружной поверхности правой ноги, потеря проприоцептивной чувствительности в области стопы. Нарушением какого отдела двигательного анализатора обусловлены имеющиеся расстройства. A *Периферических нервов B Спиномозговых проводящих путей C Пирамидных нейронов

Page 182: крок 1 база 2014р_11шр-240ст (для брошури)_35грн

182

D Нервно-мышечной синаптической передачи E Экстрапирамидных нейронов

104 Больному 25 лет установлен диагноз хронического гепатита. Больной жалуется на потерю массы тела на 10 кг на протяжении 2 месяцев. Объективно: кожа сухая, шелушащаяся, бледная с желтоватым оттенком, мелкоточечные кровоизлияния на коже, кровоточивость десен. Нарушение какой функции печени отражают мелкоточечные кровоизлияния и кровоточивость десен: A *Белковосинтетической B Пигментообразующей C Гликогенсинтетической D Детоксицирующей E Депонирующей

105 Больной И., 38 лет, поступил с жалобами на боль в боку, усиливающуюся при вдохе и кашле. Болевые ощущения уменьшаются в положении лежа на пораженном боку. При обследовании дыхание частое и поверхностное, заметно ограничение дыхательной подвижности соответствующей половины грудной клетки. Каков механизм изменения характера дыхания у больного? A *Ограничение корой мозга рефлекса Геринга-Брейера B Перераздражение легочных рецепторов блуждающих нервов C Интоксикация дыхательного центра продуктами воспаления D Повышение возбудимости дыхательного центра E Торможение коры мозга вследствие интоксикации

106 Больной Ж., 48 лет, находится в реанимации после отравлением четыреххлористым углеводородом. Состояние тяжелое, больной без сознания, пульс нитевидный, слабого наполнения, ЧСС 40 уд/мин АД 75/40 мм.рт.ст., дыхание периодическое типа Биота. В крови прямой билирубин - 155 мкмоль/л. В моче высокое содержание аммиака и билирубина. Что явилось основной причиной описанного состояния у больного?

A *Печеночная недостаточность, B Кардиогенный шок C Дыхательная недостаточность D Сердечная недостаточность E Почечная недостаточность

107 Больной А, 59 лет, директор частного предприятия. После проверки налоговой инспекцией вечером появились интенсивные жгучие боли, локализованные за грудиной, иррадиирующие в левую руку. Через 15 мин состояние больного нормализовалось. Какой из механизмов развития ишемии сердечной мышцы является ведущим у данного больного? A *Повышение в крови уровня катехоламинов B Атеросклероз коронарных артерий C Внутрисосудистая агрегация форменных элементов D Сдавление коронарных артерий при дилятации полостей сердца E Функциональная перегрузка сердца

108 43-летний больной поступил в нефрологическое отделение с массивными отеками. Два года лечился амбулаторно и при этом постоянно отмечалось повышенное АД. Дважды лечился преднизолоном, с положительным эффектом. В моче: относительная плотность 1017, белок 4,0 г/л, Эр – 15-18 в поле зрения (выщелоченные), Лей - 5-7 в поле зрения. Какая преимущественно функция почек нарушена у больного? A *Фильтрационная B Реабсорбционная C Секреторная D Инкреторная E Концентрационная

109 При підйомі в гори у альпініста розвинулась ейфорія, головний біль, запаморочення, серцебиття, задишка, яка чергувалася з апное. Яке порушення кислотно-основного стану розвинулося у альпініста? A *Газовий алкалоз B Метаболічний алкалоз C Негазовий алкалоз

Page 183: крок 1 база 2014р_11шр-240ст (для брошури)_35грн

http://vk.com/my.printing

183

D Газовий ацидоз E Негазовий ацидоз

110 Хворому поставили діагноз – синдром подразненої кишки. Виділення якого інкрету найбільш вірогідно збільшується при цьому? A *Мотиліну B Інтестинального пептиду C Глюкагону D Урогастрону E Секретину

111 Внаслідок поранення хворий втратив 25% об’єму циркулюючої крові. Назвіть терміновий механізм компенсації крововтрати. A * Находження міжтканинної рідини в судини B Відновлення білкового складу крові C Збільшення числа ретикулоцитів D Відновлення числа еритроцитів E Активація еритропоезу

112 У робітника, який працював літом у щільному костюмі, різко підвищилась температура тіла, з’явились задишка, тахікардія, нудота, судоми, втрата свідомості. Що явилось причиною тяжкого стану робітника? A * Зниження тепловіддачі B Підвищення теплопродукції C Підвищення тепловіддачі D Зниження теплопродукції E Тепловіддача дорівнює теплопродукції

113 В приймальне відділення поступила дитина 1,5 р. з ознаками отруєння нітратами: стійкий ціаноз, задишка, судоми. Утворення якої форми гемоглобину лежить в основі цих симптомів? A * метгемоглобіну. B карбгемоглобіну C карбоксигемоглобіну D редукованого гемоглобіну E оксигемоглобіну

114 Хвора 3 р., поступила в дитячу клініку у важкому стані з гемоглобінопатією (серпоподібноклітинна анемія). Заміна якою аминокислотою глутамінової кислоти в

бета–ланцюгу глобіну лежить в основї утворення патологічного гемоглобіну в даному випадку? A * валіном. B серином C тирозином D фенілаланіном E аргініном

115 Хвора поступила в клініку на обстеження. З дитинства відмічалось зниження гемоглобіну до 90-95 г/л. Лікування препаратами заліза було неефективне. Аналіз крові при поступленні: Е–3,2*1012/л, Hb–85 г/л, к.п.–0,78. В мазку анізоцитоз, пойкілоцитоз, мішеневидні еритроцити, ретикулоцити –16%. Поставлений діагноз – таласемія. До якого виду гемолітичних анемій можна віднести дане захворювання? A * Спадкова гемоглобінопатія. B Спадкова мембранопатія C Набута мембранопатія D Спадкова ферментопатія E Набута ферментопатія

116 Хворий на протязі останнього року став відмічати під_вищену втомлюваність, загальну слабість. Аналіз крові: Е–4.1*1012/л, Hb–119 г/л, к.п.– 0.87, лейкоцити – 57*109/л, лейкоформула: Ю–0, П–0, С–9%, Е–0, Б–0, лімфобласти–2%, пролімфоцити–5%, лімфоцити–81%, М–3%, тромбоцити–160*109/л. В мазку: нормохромія, велика кількість тіней Боткіна–Гумпрехта. Про яку патологію системи крові свідчить дана гемограма? A * Хронічний лімфолейкоз. B Хронічний мієлолейкоз C Гострий лімфобластний лейкоз D Гострий мієлобластний лейкоз E Хронічний монолейкоз

117 У хворого діагностована пелагра. Порушення обміну якої амінокислоти лежить в основі цього захворювання? A * Триптофану B Тирозину C Фенілаланіну D Цистеїну E Валіну

Page 184: крок 1 база 2014р_11шр-240ст (для брошури)_35грн

184

118 У хлопчика М., 2 років була діагностована хвороба Гірке, що супроводжується надмірним відкладанням глікогену в печінці і нирках, гіпоглікемією. При біохімічному дослідженні крові виявлено: A * Дефіцит глюкозо-6-фосфатази B Зниження активності аміло-1,6-глюкозидази C Дефіцит фруктозо-дифосфат альдолази D Зниження активності фосфорилази E Дефіцит кетогексокінази

119 Масугі викликав розвиток гломерулонефриту у щурів таким чином: гомогенат нирок щура вводив кролю. Через декілька тижнів сироватку сенсибілізованого кроля вводив щурам. Який тип алергічної реакциї за Джеллом та Кумбсом лежить в основі розвитку гломерулонефриту у щурів? A * Цитотоксичний B Анафілактичний C Імунокомплексний D Гіперчутливість сповільненого типу E Стимулюючий

120 У хворої 45-ти років при електрокардіографічному обстеженні на ЕКГ виявлено такі зміни: інтервал Р-Q подовжений, при цьому випадає кожен другий або третій комплекс QRST. Яке саме порушення провідності серця тут спостерігається? A *Атріовентрикулярна блокада Ш–го ступеня B Атріовентрикулярна блокада повна C Синоаурікулярна блокада D Внутрішлуночкова блокада E Атріовентрикулярна блокада І–го ступеня

121 З метою пригнічення аутоімунних реакцій після пересадки органів обов'язковим є проведення курсу гормонотерапії. Які гормони застосовують з цієї мети ? A *Глюкокортикоїди. B Мінералокортикоїди. C Статеві гормони. D Адреналін. E Соматотропний гормон.

122 У хворого, що прибув із Тунісу, виявлена альфа-таласемія з гемолізом еритроцитів і жовтяницею. Хвороба була діагностована на основі наявності в крові A * Мішенеподібних еритроцитів B Зернистих еритроцитів C Поліхроматофільних еритроцитів D Нормоцитів E Ретикулоцитів

123 Після тотальної резекції шлунка у хворого розвинулася тяжка В12-дефіцитна анемія. Свідченням її була наявність в крові A * Мегалобластів B Мікроцитів C Овалоцитів D Нормоцитів E Анулоцитів

124 На шостому місяці вагітності в жінки з’явилася виражена залізодефіцитна анемія. Діагностичною ознакою її була поява в крові A *Анулоцитів B Макроцитів C Пойкілоцитів D Ретикулоцитів E Нормоцитів

125 У хворого 38 років, який переніс гепатит і продовжував вживати алкоголь, розвинулися ознаки цирозу печінки з асцитом і набряками на нижніх кінцівках. Які зміни складу крові стали вирішальними в розвитку набряків? A * Гіпоальбумінемія B Гіпоглобулінемія C Гіпохолестеринемія D Гіпокаліємія E Гіпоглікемія

126 Після вимушеного швидкого підняття водолаза з глибини на поверхню у нього з’явилися ознаки кесонної хвороби – біль у суглобах, свербіння шкіри, мерехтіння в очах, затьмарення свідомості. Яким видом ємболії вони були зумовлені? A * Газовою B Повітряною C Жировою

Page 185: крок 1 база 2014р_11шр-240ст (для брошури)_35грн

http://vk.com/my.printing

185

D Тканинною E Тромбоємболією

127 У хворого Н. приступи гарячки виникають через день. Під час приступу температура різко підвищується і утримується на високому рівні до 2 год, а потім знижується до вихідного рівня. Цей тип гарячки характерний для A * Малярії B Поворотного тифу C Сепсису D Бруцельозу E Висипного тифу

128 В лікарню доставлений непритомний чоловік після отруєння чадним газом. Гіпоксія у нього зумовлена появою у крові A * Карбоксигемоглобіну B Метгемоглобіну C Карбгемоглобіну D Оксигемоглобіну E Дезоксигемоглобіну

129 Після занурення водолаза на глибину 60 м у нього з’явилися симптоми порушення функцій центральної нервової системи – збудження, ейфорія, ослаблення уваги, професійні помилки. Ці симптоми пов’язані з токсичною дією на нейрони A *Азоту B Кисню C Вуглекислого газу D Аміаку E Лактату

130 Електрик, працюючи з порушенням правил техніки безпеки, випадково торкнувся оголеного електропровода обома руками і загинув. Смерть настала внаслідок A * Фібриляції передсердь і шлуночків B Повної атріо-вентрикулярної блокади C Пригнічення автоматизму сино-атріального вузла D Зменшення скоротливої здатності міокарда E Порушення вагусної регуляції серця

131 До дерматолога звернулася пацієнтка із скаргами на екзематозне ураження шкіри

рук, що з’являється після контакту з миючим засобом “Лотос”. Використання гумових рукавичок запобігає цьому. Патологічна реакція шкіри зумовлена активацією A * Т-лімфоцитів B В-лімфоцитів C Моноцитів D Нейтрофілів E Базофілів

132 У хворого з синдромом Zollinger-Ellison [ пухлина підшлункової залози] відзначається збільшення секреції, перистальтики шлунково-кишкового тракту, а також діарея та пептичні виразки. Яка з перерахованих речовин, що секретується вказаною пухлиною, викликає цей комплекс симптомів? A *Гастрин B Вазоактивний інтестинальний пептид C Пепсин. D Трипсин E Секретин

133 У дитини двох років встановлено діагноз гіпоплазії тимуса. Який показник стану імунної системи є найбільш характерним для цього імунодефіцита? A * Зниження кількості Т-лімфоцитів B Зниження кількості В-лімфоцитів C Дефіцит Т и В-лімфоцитів D Відсутність плазматичних клітин E Зниження іммуноглобулінів М

134 Клінічне обстеження хворого дозволило встановити попередній діагноз рак печінки. Наявність якого білка в сироватці крові дозволить підтвердити діагноз? A *альфа-фетопротеїн B Пропердин C Парапротеїни D С-реактивний протеїн E гамма-глобуліни

135 У хворого на пневмонію виникла гарячка. Що безпосередньо спричинює зміну установочної точки температури в нейронах гіпоталамуса цього хворого? A * Простагландини Е1, Е2 B Ендотоксин

Page 186: крок 1 база 2014р_11шр-240ст (для брошури)_35грн

186

C Екзотоксин D Інтерлейкін-2 E Тромбоцитарний фактор росту

136 Через 1 годину після накладання кільця, що звужує аорту, в собаки різко зросла сила та частота серцевих скорочень, а об’єм циркулюючої крові та товщина стінки лівого шлуночка не відрізнялися від вихідних показників. Яка стадія гіпертрофії міокарда спостерігається у тварини? A *Аварійна B Декомпенсації C Прогресуючого кардіосклерозу D Відносно стійкої гіперфункції E Завершеної гіпертрофії

137 У хворого на артеріальну гіпертензію наслідком гіпертонічного кризу стала гостра серцева недостатність. Який механізм серцевої недостатності є головним в даному випадку? A * Перевантаження серця опором B Перевантаження серця припливом крові C Ушкодження міокарда D Абсолютна коронарна недостатність E Відносна коронарна недостатність

138 Хворий з Прикарпаття, що страждає на єндемічниї зоб, звернувся до лікаря із скаргами на гноєтечі з ясенних закутків та розхитування зубів. Що в даному випадку є основним фактором розвитку пародонтиту? A * Ендокринні порушення B Стресорні впливи C Гіперсалівація D Порушення ковтання E Неповноцінне харчування

139 У чоловіка 35 років через 30 хвилин після автомобільної аварії виявлена масивна травма нижніх кінцівок без значної зовнішньої крововтрати. Постраждалий знаходиться у збудженному стані . Який компонент патогенезу травматичного шоку є у пацієнта ведучим і потребує негайного корегування? A *Біль. B Внутрішня крововтрата. C Внутрішня плазмовтрата. D Інтоксікація.

E Порушення функції органів.

140 Жінка 53 років, ріст 163 см., вага тіла 92 кг, рівномірне відткладання жиру, лице одутле, малорухома, апатична. При натискуванні шкіри ноги залишається ямка. Порушенням функції якої залози обумовлений стан хвороби? A *Щитовидної. B Гіпофізу. C Надниркових. D Статевих. E Прищитовидних.

141 У хворої 36 років, яка лікувалася сульфаніламідами з приводу респіраторної вірусної інфекції, в крові гіпорегенераторна нормохромна анемія, лейкопенія, тромбоцитопенія. В кістковому мозку - зменьшення кількості мієлокаріоцитів. Яка це анемія? A *Гіпопластична B Гемолітична C Постгеморагічна D В12-фолієводефіцитна E Залізодефіцитна

142 В аналізі крові 35-річного хворого: Нв – 58 г/л, еритроцити – 1,3х1012/л, колірний показник – 1,3, лейкоцити – 2,8х109/л, тромбоцити – 1,1х109/л, ретикулоцити – 2o/oo, ШОЕ – 35 мм/час. Визначаються полісегментіровані нейтрофіли, а також тільця Жоллі, кільця Кебота, мегалоцити. Яка це анемія? A *В12 – фолієводефіцитна. B Гіпопластична. C Постгеморагічна. D Гемолітична. E Залізодефіцитна.

143 При обстеженні хворого встановлено, що кліренс ендогенного креатініну після збору 24-х годинного зразка сечі у нього становить 50 мл/хв (при нормі – 110-150 мл/хв]. Про зниження якої функції свідчить наявність такої ознаки? A *Клубочкової фільтрації B Канальцевої реабсорбції C Інкреторної функції нирок D Виведення з організму іонів. E Виведення з організму сечової кислоти.

Page 187: крок 1 база 2014р_11шр-240ст (для брошури)_35грн

http://vk.com/my.printing

187

144 Чоловік 30 років скаржиться на задуху, важкість в правій половині грудної клітки, загальну слабкість. Температура тіла 38,9оС. Об’єктивно: права половина грудної клітки відстає від лівої. Плевральна пункція дала ексудат. Що являється провідним чинником ексудації у хворого? A *Підвищення проникливості стінки судин. B Підвищення кров’яного тиску. C Гіпопротеїнемія. D Агрегація еритроцитів. E Зменшення резорбції плевральної рідини .

145 У чоловіка 48 років, хворого на хронічний бронхіт, діагностована емфізема легень. Що являється провідним чинником даного ускладнення? A *Зниження еластичних властивостей легень. B Зменшення розтяжності легень . C Зменшення загальної течії крові в легенях. D Порушення вентиляційно-перфузійного співвідношення E Зменшення альвеолярної вентиляції.

146 Хворий чоловік на протязі 17 років страждає хронічним гломерулонефритом. Пульс 82 за хвилину. АТ 190/120 мм рт.ст. Що являється первинним механізмом підвищення артеріального тиску у хворого? A *Підвищення загального периферичного опору. B Збільшення об’єму циркулюючої крові; C Підвищення тонусу венозних судин; D Збільшення ударного об’єму крові; E Збільшення хвилинного об’єму крові;

147 Піддослідній тварині ввели блокатор цитохромоксидази, що призвело до її миттєвої загибелі. Яка із сполук калію може викликати вказані зміни: A *Цианід B Нітрит C Сульфат

D Фосфат E Оксалат

148 При синдромі реперфузії активуються процеси вільнорадикального окиснення, що призводить до пошкодження клітинних мембран та порушення специфічних функцій клітин. Ці зміни пов’язані з надмірним накопиченням в цитоплазмі іонів: A *Кальцію B Магнію C Хлору D Натрію E Калію

149 У хворого в мазку крові виявлено: мікроанізоцитоз, пойкілоцитоз, анулоцитоз. Для якої анемії характерні ці зміни? A *Залізодефіцитної B В-12 дефіцитної C Гіпопластичної D Серповидноклітинної E Мікросфероцитарної

150 У хворого після резекції шлунка розвинулася В-12 фолієводефіцитна анемія. Який із перерахованих кольорових показниівк характерний для цієї патології? A *1,4 B 1,0 C 0,8 D 0,5 E 0,2

151 У хворого на ранній стадії цукрового діабету спостерігається поліурія. Чим вона виклікана? A *Гіперглікемією B Кетонемією C Гіпохолестеринемією D Гіперхолестеринемією E Гіперкалійемією

152 У повної жінки 52 років встановлено цироз печінки. Лабораторно: гіпоальбумінемія, гіперглобулінемія. Візуально: набряк рук, повік, ніг. Найбільш імовірною причиною набряків є зміна:

Page 188: крок 1 база 2014р_11шр-240ст (для брошури)_35грн

188

A *Онкотичного тиску крові B Буферної емності крові C Кислотно-лужної рівноваги D Дезінтоксикаційної функції печінки E Глікогенсинтезуючої функції печінки

153 У хворого 35 років розвинулася імунна гемолітична анемія. Який показник сироватки крові зросте в найбільшій мірі? A *Непрямий білірубін B Прямий білірубін C Стеркобіліноген D Мезобіліноген E Протопорфірин

154 У хворого 48 років спостерігається артеріальна гіпертензія, головний біль, м’язoва слабкість, судоми. В крові знижена концентрація К+ і підвищена концентрація Na+, що є наслідком гіперсекреції: A *Альдостерону B Адреналину C Паратгормону D Кортизолу E Дигідрохолестеролу

155 У хлопчика 15 років, хворого на алкаптонурію сеча набуває чорного кольору після відстоювання. Спадкове порушення обміну якої речовини має місце: A *Тирозину B Цистеїну C Аланіну D Сечовини E Сечової кислоти

156 При аналізі ЕКГ встановлено: ритм синусовий, правильний, інтервал RR 0,58 сек, розташування і тривалість інших інтервалів, зубців і сегментів не змінені. Назвіть вид аритмії. A *Синусова тахекардія. B Синусова брадикардія C Ідіовентрикулярний ритм D Синусова аритмія E Миготлива аритмія. Миготтлива – мерцательная

157

В медико-генетичній консультації при обстеженні хворого хлопчика в крові були виявлені нейтрофільні лейкоцити з 1 “барабанною паличкою”. Наявність якого синдрому можлива у хлопчика? A * Синдром Клайнфельтера. B Синдром Дауна C Синдром Шерешевського-Тернера D Синдром Едвардса E Синдром трисомії – Х

158 У хворого виявлено порушення прохідності дихальних шляхів на рівні дрібних та середніх бронхів. Які порушення кислотно-лужної рівноваги можна виявити в крові в даному випадку? A * Респіраторний ацидоз B Метаболічний ацидоз C Респіраторний алкалоз D Метаболічний алкалоз E -

159 У неврологічне відділення з приводу мозкового крововиливу поступив хворий, 62 р. Стан важкий. Спостерігається наростання глибини і частоти дихання, а потім його зменшення до апное, після чого цикл дихальних рухів відновлюється. Який тип дихання виник у хворого? A * Чейна–Стокса. B Кусмауля C Біота D Гаспінг–дихання E Апнеїстичне

160 У чоловіка 52 років через 3 роки після операції видалення шлунку вміст еритроцитів в крові складає 2,0х1012/л, Hb–85г/л, к.п.–1,27. Порушення засвоєння якого вітаміну викликало такї зміни? A *B12. B B6 C C D P E A

161 Хворому з закритим переломом плечевої кістки накладена гіпсова пов’язка.

Page 189: крок 1 база 2014р_11шр-240ст (для брошури)_35грн

http://vk.com/my.printing

189

Наступного дня з’явилася припухлість, синюшність і похолодання кисті травмованої руки. Про який розлад периферичного кровообігу свідчать ці ознаки? A *Венозна гіперемія B Артеріальна гіперемія C Ішемія D Тромбоз E Емболія

162 Хворий помер від інфаркту міокарда. Проведене патогістологічне дослідження міокарда виявило значні контрактурні зміни в кардіоміоцитах. Це зумовлено нагромадженням в кардіоміоцитах іонів A * Кальцію B Водню C Натрію D Магнію E Хлору

163 У дитини із розумовою відсталістю встановлено зелене забарвлення сечі після додавання 5% розчину FeCl3. Про порушення обміну якої амінокислоти свідчить позитивний результат цієї діагностичної проби? A * Фенілаланіну B Аргініну C Глутаміну D Тирозину E Триптофану

164 При моделюванні запалення нижньої кінцівки у тварини підвищилася температура тіла, збільшився вміст антитіл та лейкоцитів у крові. Які речовини обумовили розвиток цих загальних реакцій організму при запаленні? A * Інтерлейкіни B Глюкокортикоїди C Мінералокортикоїди D Лейкотриєни E Соматомедіни

165 При відтворенні артеріальної гіпертензії у собаки через 1 місяць товщина стінки лівого шлуночка зросла в 1.7 рази, а об’єм

циркулюючої крові не змінився порівняно з вихідними даними. Яка стадія гіпертрофії міокарда спостерігається у тварини? A * Завершеної гіпертрофії B Аварійна C Початкова D Прогресуючого кардіосклерозу E Декомпенсації

166 У хворого діагностовано тиротоксикоз. У крові знайдено антитиреоїдні антитіла. Який тип алергічної реакції за Кумбсом і Джелом спостерігається при розвитку цього захворювання? A * Стимулюючий B Анафілактичний C Цитотоксичний D Імунокомплексний E Гіперчутливість сповільненого типу

167 У жінки 68 років після інсульту відсутні рухи в верхній та нижній правій кінцівках. Тонус м’язів цих кінцівок і рефлекси в них підвищені. Є патологічні рефлекси. Яка це форма параліча? A *Геміплегія B Параплегія C Тетраплегія D Моноплегія E Дисоціація

168 У жінки з первинним гіперпаратиреоідизмом періодично повторюються напади ниркової коліки. Ультразвукове обстеження показало наявність дрібних каменів в нирках, найбільш імовірною причиною утворення яких є: A *Гіперкальциемія B Гіперфосфатемія C Гіперхолестеринемія D Гіперурікемія E Гіперкаліемія

169 Через кілька тижнів після народження у дитини почали відмічатися прояви ураження ЦНС, шкіра та волосся посвітлішали. При доливанні до свіжої сечі 5% розчину трихлороцтового заліза з'являється

Page 190: крок 1 база 2014р_11шр-240ст (для брошури)_35грн

190

оливково-зелене забарвлення. Який найбільш вірогідний діагноз? A *Фенілкетонурія B Алкаптонурія C Фруктозурія D Тирозиноз E Альбінізм

170 Підліток 15 років, скаржиться на недостачу повітря, загальну слабкість, серцебиття. ЧСС 130 уд/хв, АТ-100/60 мм рт. ст.на ЕКГ, комплекс QRS нормальної форми та тривалості. Число зубців Р та шлункових комплексів однакове, зубець Т злитий з зубцем Р.Яка аритмія серця спостерігається у підлітка ? A *синусова тахікардія B синусова екстрасистолія C мерехтіння передсердь D тремтіння передсердь E передсердна пароксизмальна тахікардія

171 В медико-генетичну консультацію звернулася жінка по рекомендації гінеколога з приводу відхилень фізичного і статевого розвитку. При мікроскопії клітин слизової ротової порожнини не знайдено статевого хроматину. Який буде найбільш вірогідний діагноз? A *Синдром Шерешевського-Тернера B Хвороба Дауна C Синдром Клаинфельтера D Хвороба Реклінгаузера E Трисомія по Х-хромосомі

172 У хворого через 9 діб після введення лікувальної сироватки зўявилася кропивўянка, зуд шкіри, набряк її та слизових оболонок, припухання лімфатичних вузлів. Яке захворювання розвинулось? A *Сироваткова хвороба. B Феномен Швартцмана. C Набряк Квінке. D Феномен Овері. E Поліноз.

173 У чоловіка 45-ти років після значного психоемоційного навантаження раптово з'явився

стискаючий біль в ділянці серця з ірадіацією в ліву руку, шию, під ліву лопатку. Обличчя стало блідим, вкрилос холодним потом. Нітрогліцерин зняв приступ болю. Який процес розвинувся у хворого? A *Стенокардія B Інфаркт-міокарда C Інсульт D Психогенний шок E Перфорація виразки шлунку

174 До клініки доставили пацієнта 32 років з масивною крововтратою внаслідок ДТП. Ps- 110 уд/хв., ЧДР- 22 за 2 хв., АТ- 100/60 мм рт.ст. Яка зміна крові із перелічених буде найбільш характерною чере 1 годину після крововтрати? A *Гіповолемія B Еритропенія C Гіпохромія еритроцитів D Лейкопенія E Гіпопротеінемія

175 У хворого 42 років при дослідженні периферичної крові виявлено: гемоглобін 80 г/л, еритроцитів 3,2 Т/л, лейкоцитів 25 Г/л; лейкоцитарна формула: базофілів - 5%, еозинофілів - 9%, міелобластів - 3%, проміелоцитів - 8%; нейтрофілів: міелоцитів - 11%, метаміелоцитів - 22%, паличкоядерних - 17%, сегментоядерних - 19%, лімфоцитів - 3%, моноцитів - 3%. Яка патологія крові найбільш вірогідна у хворого: A *Хронічний міелолейкоз B Міелобластний лейкоз C Еритроміелоз D Проміелоцитарний лейкоз E Панміелофтіз

176 У хворого М, 45 років, при аналізі ЕКГ встановлено: ритм синусовий, число передсердних комплексів більше числа шлуночкових комплексів; прогресуюче подовження інтервала P-Q від комплекса до комплекса; випадення окремих шлуночкових комплексів; зубці Р та

Page 191: крок 1 база 2014р_11шр-240ст (для брошури)_35грн

http://vk.com/my.printing

191

комплекси QRST без змін. Назвіть тип порушення серцевого ритму. A *Атріовентрикулярна блокада II ступеня B Сіноаурікулярна блокада C Атріовентрикулярна блокада I ступеня D Внутрішньопередсердна блокада E Повна атріовентрикулярна блокада

177 Людина отримала електротравму. При цьому струм перейшов через серцевий м'яз. Які небезпечні порушення в роботі серця можуть виникнути у цііі ситуаціі, що вимагають термінового втручання? A *Фібриляція шлуночків B Фібриляція передсердь C Екстрасистолія D Тахікардія E Брадікардія

178 До гастроентерологічного відділення потрапив хворий 57 років з підозрою на синдром Золінгера-Еллісона, про що свідчило різке збільшення рівню гастрину у сироватці крові. Яке порушення секреторної функції шлунку найбільш вірогідне? A * Гіперсекреція гіперацидна B Гіпосекреція гіперацидна C Ахілія D Гіпосекреція гіпоацидна E Гіперсекреція гіпоацидна

179 Жінка 38 років скаржиться на загальну слабкість, біль в області серця, підвищення апетиту, відсутність менструацій. Об'єктивно: зріст 166 см, вага тіла 108 кг, лице місяцеподібне, відкладення підшкірної клітковини переважно області верхнього плечового поясу, тулубу; на шкірі стегон, живота криваво-червоні смуги, пульс 62/хв., АТ-160/105 мм. рт. ст. Для якого із перелічених нижче станів найбільш характерний даний тип ожиріння? A *хвороба Іценко-Кушинга B аліментарне ожиріння C мікседема D інсулінома E синдром Бабінського-Фреліха

180 Чоловік 53 роки, доставлений у стаціонар у

непритомному стані. Об'єктивно: шкіра суха , дихання часте поверхневе, запах ацетону відсутній, пульс 126 уд/хв., АТ-70/40 мм рт. ст., вміст глюкози у крові 48 ммоль/л, реакція сечі на ацетон негативна. Для якого із перелічених станів найбільш характерні симптоми у хворого? A *гіперосмолярної коми B гіперкетонемічної коми C лактацидемічної коми D Токсичної коми E колапсу

181 Чоловік 28 років, поступив з скаргами на нудоту, блювоту, біль у правому підребер'ї. Об'єктивно: жовтушність шкіри, склер, температура тіла підвищена, печінка збільшена, сеча темна, кал гіпохолічний, гіпербілірубінемія (білірубін прямий та непрямий), білірубінурія, уробілінурія, гіпопротеінемія, зниження зсідання крові. Для якого з нижче перелічених станів найбільш характерні ці зміни? A *Клітинно-паренхіматозної жовтяниці B Підпечінкової жовтяниці C Надпечінкової гемолітичної жовтяниці D Гострого холециститу E Гострого панкреатиту

182 Жінку 50 років турбують часті головні болі, запаморочення, задишка при фізичному навантаженні. Останні три роки відзначались тривалі та значні менструації. При огляді- нормальної вгодованості, шкіра бліда суха. Аналіз крові: Hb-90г/л, еритроцити-3.7Т/л, кольоровий показник-0.7,ШОЕ-20 мм/год, значна гіпохромія еритроцитів, анізоцітоз, пойкілоцитоз. Яка анемія найбільш вірогідно має місце у хворого? A *геморагічна анемія B гемолітична анемія C метапластична анемія D В12 -фолієводефіцитна анемія E гіпопластична анемія

183 Чоловік 47 років скаржиться на слабкість , запаморочення . Півроку назад переніс операцію з приводу резекції шлунку Аналіз крові:Hb-80г/л, еритроцити-3.5Т/л,

Page 192: крок 1 база 2014р_11шр-240ст (для брошури)_35грн

192

кольоровий показник-0.69, ШОЕ-15 мм/год. Сироваткове залізо-5.4 мкмоль/л. Гіпохромія еритроцитів. Яка анемія найбільш вірогідно має місце у хворого? A *залізодефіцитна анемія B сидероахрестична анемія C геморагічна анемія D гемолітична анемія E В12 -фолієводефіцитна анемія

184 Після перенесеного важкого інфекційного процесу у хворого розвинувся міокардит з ушкодженням провідної системи серця, що супроводжувалось періодичною втратою свідомості внаслідок розвитку синдрому Моргані Едемса-Стокса, який виникає при: A *Переході неповної атріовентрикулярної блокади в повну B Блокаді лівої ножки пучка Гіса C Блокаді правої ножки пучка Гіса D Пароксизмальній тахікардії E Передчасному збудженні шлуночків

185 При загальному огляді хворого встановлена гіперемія всіх шкірних покровів з цианотичним відтінком. Звертає на себе увагу заторможеність хворого та сповільнення його рухів. Аналіз крові показав: еритроцити 9х1012/л гематокріт 60%. При якому патологічному стані має місце абсолютний еритроцитоз: A *Хвороба Вакеза B Мегалобластична анемія C Лімфома D Гемодилюція E Гемоконцентрація

186 В анамнезі: з дитинства відмічався знижений рівень гемоглобіна. Лікування препаратами заліза без эффекту При вступі в аналізі крові: кількість еритроцитів 3,1*1012/л, ретикулоцитів 16%, гемоглобіну 85 г/л, КП 0,75; в мазку крові анізоцити, пойкілоцити мішеневидні еритроцити, еритроцити з базофільною зернистістю; рівень заліза у сировотки 30 мкмоль/л. Для якої патології системи крові характерні дані? A *Талассемії B Залізодефіцитні анемії C В12-дефіцитні анемії D Фоліеводефіцитні анемії

E Гіпопластична анемія

187 Хворий Н. поступив зі скаргами на диспептичні розлади, мелену, гемороїдальні кровотечі. При дослідженні виявлено розширення сітки венозних судин на переднеій стінці животу в комплексі зі збільшенням його размірів. Як патологія ШКТ може виявитись такими симптомами? A *Портальная гіпертензія B Кишкова аутоінтоксикація C Виразкова хвороба D Коліт E Энтерит

188 У 40-річної вагітної проведено амніоцентез. При дослідженні картіотипу плоду одержано результат: 47,ХУ+21. Яку патологію плоду виявлено? A *Синдром Дауна B Синдром Клайнфельтера C Хвороба Шерешевського-Тернера D Фенілкетонурія E Хвороба Патау

189 Під час гри у волейбол спротсмен після стрибка приземлився на зовнішній край стопи. Виник гострий біль в гомілкостопному суглобі, активні рухи в ньому обмежені, пасивні - в повному обсязі, але болісні. Потім розвинулась припухлість в ділянці зовнішньої кісточки, шкіра почервоніла, стала теплішою на дотик. Який вид розладу периферичного кровообігу розвинувся в даному випадку? A *Артеріальна гіперемія. B Стаз C Емболія D Венозна гіперемія. E Тромбоз.

190 При роботі по ліквідації наслідків аварії на АЕС робітник одержав дозу опромінення 500 рентген. Скаржиться на головний біль, нудоту, запаморочення. Які зміни в кількості лейкоцитів можна чекати у хворого через 10 годин після опромінення? A *Нейтрофільний лейкоцитоз. B Лімфоцитоз.

Page 193: крок 1 база 2014р_11шр-240ст (для брошури)_35грн

http://vk.com/my.printing

193

C Лейкопенія. D Агранулоцитоз. E Лейкемія.

191 У хворого 42 років виявлені такі зміни в периферичній крові: гемоглобін 80 г/л, еритроцитів 3,2*1012/л, лейкоцитів 250*109/л; лейкоцитарна формула: базофілів - 5%, еозинофілів - 9%, мієлобластів - 3%, промієлоцитов - 8%, нейтрофілів: мієлоцитів - 11%, метамієлоцитів - 22%, паличкоядерних - 17%, сегментоядерних - 19%, лімфоцитів - 3%, моноцитів - 3%. Назвіть найбільш вірогідний діагноз. A *Хронічний мієлолейкоз. B Гострий мієлобластний лейкоз. C Еритромієлоз. D Нейтрофільний лейкоцитоз. E Еозинофільний лейкоцитоз.

192 Чоловік 26 років скаржиться на бесплідність.Об'єктивно: зріст 186 см, довгі кінцівки, гінекомастія, гіпоплазія яєчок, в соскобі слизової оболонки щоки знайдені тільця Барра. Діагностований синдром Клайнфельтера. Яки механізм хромосомної аномалії має місце при даному захворюванні? A *Нерозходження гетосом у мейозі B Нерозходження хроматид у мітозі C Транслокація D Інверсія хромосоми E Ділеція хромосоми

193 Жінка 23 років надійшла до стаціонару з діагнозом гостра пневмонія. Захворіла гостро, 2 дні тому, коли з'явився озноб з підвищенням температури тіла до 39oC, слабкість, сухий кашель. Який з перерахованих медіаторів запалення має властивості ендогенного пірогену? A *Інтерлейкін 1 B Тромбоксан А2 C Гістамін D Серотонін E Брадикінін

194 Жінка 68 років скаржиться на відсутність рухів в правих руці і нозі. Чотири місяці тому у неї був інсульт. Об'єктивно: рухи в правих руці і нозі відсутні, тонус м'язів цих кінцівок підвищений. Який з перелічених нижче стані спостерігається у хворої? A *геміплегія B моноплегія C параплегія D тетраплегія E -

195 У підлітка 12 років, який хворіє на бронхіальну астму, розвився тяжкий напад астми: виражена експіраторна задишка, блідість шкірних покровів. Який вид порушення альвеолярної вентиляції має місце ? A *обструктивний B рестриктивний C торако-діафрагмальний D центральний E нервово-м'язовий

196 Чоловік 32 років, чотири роки страждає хронічним гломерулонефритом, який супроводжується нефротичним синдромом. Відмічаються явні набряки на обличчі, в останній час з'явилися набряки на ногах та тулубі. Який із перелічених механізмів найбільш характерний для розвитку набряку у чоловіка? A *Зниження онкотичного тиску крові B Підвищення гідростатичного тиску крові у капілярах C Підвищення онкотичного тиску тканинної рідини D Утруднення лімфовідтоку E Підвищення проникливості капілярів

197 Чоловіку 46-ти років, що хворіє на дифузний токсичний зоб, була проведена операція резекції щитоподібної залози. Після операції відмічаються відсутність апетиту, диспепсія, підвищена нервно-м'язова збудливість. Маса тіла не збільшилася. Температура тіла у нормі. Чим із нижче переліченого обумовлений стан хворого? A *зниженням продукції паратгормону

Page 194: крок 1 база 2014р_11шр-240ст (для брошури)_35грн

194

B зниженням продукції тироксину C підвищенням продукції кальцітонину D підвищенням продукції тіреоліберіну E порушення продукції тироксину

198 З метою попередження відторгнення трансплантата після пересадки органів обов'язковим є проведення курсу гормонотерапії з метою імуносупресії. Які гормони застосовують з цією метою? A *Глюкокортикоїди B Мінералокортикоїди C Статеві гормони D Катехоламіни E Щитовидної залози

199 У хворого після перенесеного простудного захворювання виникло порушення міміки. Він не може закривати очі, наморщувати брови, вискалювати зуби. Який нерв пошкоджено? A *Лицевий B Блукаючий C Трійчастий D Язикоглотковий E Інфраорбітальний

200 Жінка 37 років скаржиться на загальну слабкість, часті запаморочення, утруднення ковтання їжі, бажання їсти крейду. Шкіра та видимі слизові оболонки бліді. В крові: eр.- 3,4*1012/л, Hb- 70 г/л, КП- 0,7, ретик.- 0,1%, лейк.- 4,7*109/л, е.- 2%, п.- 3%, с.- 64%, л.- 26%, м.- 5%. ШOЕ- 15 мм/г. Сироваткове залізо - 7,3 мкмоль/л. Дефіцит якої речовини обумовив виникнення захворювання? A *Заліза. B Білка. C Вітаміну В6. D Вітаміну В12. E Фолієвої кислоти.

201 Після геморагічного крововиливу в мозок у хворого стали неможливими активні рухи лівої руки і ноги. Тонус м(язів ціх кінцівок підвищено, їх спинні рефлекси різко підсилені, розширено зони рефлексів. Позитивний рефлекс Бабінського. Назвіть вид розладу центральної нервової системи у хворого: A *Центральний параліч B Периферичний параліч

C Спінальний шок D В(ялий параліч E Рефлекторний параліч

202 Альпініст на протязі кількох діб підіймався в гору. На висоті 5 000 метрів його стали непокоїти тахіпное, тахікардія, головний біль розпираючого характеру. Вкажіть можливі причини вказаних симптомів? A * Зниження парціального тиску кисню в повітрі B Зниження барометричного тиску повітря C Недостатня вентиляція легень D Газова емболія E Зниження температури повітря

203 Після 4-го підшкірного введення кінської сироватки у кролика на стегні розвиннулося різке запалення за типом феномена Артюса. До якого виду зміненої реактивності відноситься така патологія? A * Гіперергія B Позитивна гіпоергія C Негативна гіпоергія D Дізергія E Анергія

204 У молодої людини чоловічої статі віком 20 років високого росту та астенічної будови тіла з ознаками гіпогонадизму, гінекомастією та зменшеною продукцією сперми [азооспермія] виявлено каріотип 47 ХХY. Який спадкоий синдром супроводжується такою хромосомною аномалією? A *Клайнфельтера B Віскотта-Олдрича C Тернера D Луї-Барра E Дауна

205 Жінка 25-ти років скаржиться на постійний біль в області серця, задишку при русі, загальну слабість. Об'єктивно: шкіра бліда та холодна, акроціаноз. Пульс 96 за 1 хв., АТ - 105/70 мм рт.ст. Межа серця зміщена на 2 см вліво. Перший тон над верхівкою серця послаблений, систолічний шум над верхівкою. Діагностована недостатність

Page 195: крок 1 база 2014р_11шр-240ст (для брошури)_35грн

http://vk.com/my.printing

195

мітрального клапана серця. Чим обумовлене порушення кровообігу? A *Перевантаженням міокарда збільшеним об'ємом крові B Перевантаженням міокарда підвищеним опором відтоку крові C Пошкодженням міокарда D Зниженням об'єму циркулюючої крові E Збільшенням об'єму судинного русла

206 Одним із самих небезпечних моментів в патогенезі некрозу міокарда є подальше наростання зон некрозу, дистрофії та ішемії. Важлива роль в цьому належить підвищенню споживання міокардом кисню. Які речовини сприяють даному процесу? A *Катехоламіни B Ацетилхолін C Аденозин D Холестерин E Iони хлору

207 У больной с жалобами на боли в эпигастральной области опоясывающего характера при лабораторном обследовании обнаружено повышенное содержание диастазы в моче, а также содержание в кале большого количества непереваренного жира. Для какой формы патологии ЖКТ наиболее характерны описанные явления? A *Острый панкреатит B Язвенная болезнь желудка C Острый аппендицит D Воспаление толстого кишечника E Инфекционный гепатит

208 В эксперименте у животного в результате произведенной перерезки депрессорного нерва и разрушения каротидных клубочков развилась стойкая гипертензия. С нарушением какой функции нервной системы связано это явление? A *Вегетативной B Высшей нервной деятельности C Двигательной D Сенсорной E Трофической

209 У пациента, носителя наследственной серповидной аномалии эритроцитов,

заболевание пневмонией сопровождалось гемолитическим кризом и развитием анемии. Что является непосредственно причиной гемолитического криза в данном случае? A *Гипоксия, вызванная пневмонией B Изменение осмолярности крови C Гетерозиготность по Нb S D Мутация структурного гена E Гипероксия

210 При обследовании буккального эпителия мужчины был обнаружен половой хроматин. Для какой хромосомной болезни это характерно? A *Синдром Клайнфельтера B Болезнь Дауна C Синдром Шерешевского-Тернера D Трисомия по Х-хромосоме E Гипофосфатемический рахит

211 При обследовании больного определяется наличие гипергликемии, кетонурии, полиурии, гиперстенурии и глюкозурии. Какая форма нарушения кислотно-основного равновесия имеет место в данной ситуации? A *Метаболический ацидоз B Газовый ацидоз C Газовый алкалоз D Метаболический алкалоз E Негазовый алкалоз

212 У ребенка пяти лет, часто болеющего респираторными заболеваниями, отмечаются экзематозные явления после приема некоторых пищевых продуктов, склонность к затяжному течению воспалительных процессов. Какой вид диатеза можно предположить в данном случае? A *Экссудативно-катаральный B Геморрагический C Нервно-артритический D Лимфатико-гипопластический E Астенический

213 Хворий 59 років госпіталізований у кардіологічне відділення в тяжкому стані з діагнозом: гострий інфаркт міокарда в області задньої стінки лівого шлуночка та перегородки,

Page 196: крок 1 база 2014р_11шр-240ст (для брошури)_35грн

196

початковий набряк легень. Який первинний механізм, що викликає розвиток набряку легень у пацієнта? A *Лівошлуночкова недостатність B Легенева венозна гіпертензія C Легенева артеріальна гіпертензія D Гіпоксемія E Зниження альвеоло-капілярної дифузії кисню

214 При обстеженні юнака з розумовою відсталістю виявлено євнухоїдну будову тіла, недорозвиненість статевих органів. В клітинах порожнини рота - статевий хроматин. Який метод генетичного дослідження слід застосувати для уточнення діагнозу? A *Цитологічний B Біохімічний C Клініко-генеалогічний D Популяційно-статистичний E Дерматогліфіка

215 У ребенка, находящегося на искусственном вскармливании коровьим молоком, развилась тяжелая анемия: эритроциты -3,4*1012/л, Нb - 68 г/л, ретикулоциты - 0%. Какая анемия развилась у ребенка? A *Железодефицитная B B12-дефицитная C Врожденная гемолитическая D Гипопластическая E Серповидно-клеточная

216 У женщины на 7-м месяце беременности стала быстро нарастать анемия: эритроциты - 2,7*1012/л, Нb -90 г/л, анизоцитоз, пойкилоцитоз, единичные мегалобластные мегалоциты, ретикулоциты - 0%. Какой вид анемии развился в данном случае ? A *B12-дефицитная B Железодефицитная C Гемолитическая D Постгеморрагическая E Талассемия

217 У больного 24 лет через полторы недели после тяжелой стрептококковой ангины проявилась отечность лица, повысилось АД. Гематурия и протеинурия 1,2 г/л. В крови выявлены антистрептококковые антитела и

снижение компонентов комплемента. В микрососудах каких структур наиболее вероятно локализация скоплений иммунных комплексов, обусловивших развитие нефропатии? A *Клубочки B Пирамиды C Проксимальный отдел канальцев D Петля Генле E Нисходящий отдел канальцев

218 Больной сонлив, сознание помрачено, реакция на сильные раздражители заторможена. Кожа бледная, сухая, выраженные отеки. Мышечные фибриллярные подергивания. Мидриаз. Дыхание типа Чейна-Стокса с аммиачным запахом. При аускультации сердца - шум трения перикарда. Какой вид комы развился у больного? A *Почечная B Кетоацидотическая C Гиперосмолярная D Печеночная E Апоплексическая

219 Дитина, хвора на фенілкетонурію, страждає на розумову відсталість. Який механізм буде головним у розвитку пошкодження центральної нервової системи? A *Накопичення в крові фенілаланіну і фенілкетонів B Підвищення синтезу тирозину C Зниження синтезу меланіну D Зниження синтезу тиреоїдних гормонів E Збільшення екскреції з сечею фенілкетонових тіл

220 Після прийому амідопірину у хворого виникла лейкопенія. В крові знайдені антилейкоцитарні антитіла. Який тип алергічної реакції за Кумбсом і Джеллом виник у даному випадку? A *Цитотоксичний. B Стимулюючий. C Анафілактичний. D Гіперчутливість сповільненого типу. E Імунокомплексний.

221 Тромбоз коронарної артерії спричинив розвиток інфаркту міокарда. Які механізми ушкодження клітин є домінуючими при

Page 197: крок 1 база 2014р_11шр-240ст (для брошури)_35грн

http://vk.com/my.printing

197

цьому захворюванні? A *Кальцієві B Ліпідні C Ацидотичні D Електролітно-осмотичні E Протеїнові

222 Через 3 тижні після гострого інфаркту міокарда у хворого з'явилися болі в серці та суглобах, запалення легень. Який механізм є основним у розвитку постінфарктного синдрому Дресслера у цього хворого? A *Аутоімунне запалення B Вторинна інфекція C Iшемія міокарда D Тромбоз судин E Резорбція білків з некротизованої ділянки міокарда

223 Після ампутації лівої верхньої кінцівки хворий тривалий час відчував сильний біль у ній. Який механізм формування болісних відчуттів найбільш імовірний у цьому випадку? A *Фантомний B Каузалгічний C Рефлекторний D Гіпопродукція b-ендорфіну E Гіпопродукція енкефаліну

224 Хвора звернулась в клініку зі скаргами на слабкість, задишку, швидку стомлюваність, запаморочення. В крові: ер. - 1,8*1012/л; Hb - 80 г/л; к.п. - 1,5; лейк. - 3,2*109/л. У мазку: анізоцитоз, пойкілоцитоз, мегалобласти, мегалоцити. Який найбільш вірогідний діагноз? A *В12-дефіцитна анемія B Залізодефіцитна анемія C Постгеморагічна анемія D Iмуногемолітична анемія E Гострий лейкоз

225 У хворого після оперативного втручання на підшлунковій залозі розвинувся геморагічний синдром із порушенням третьої фази зсідання крові. Що буде найбільш вірогідним механізмом порушення гемостазу? A *Активація фібринолізу B Зниження синтезу протромбіну

C Зниження синтезу фібриногену D Якісні аномалії фібриногенезу E Дефіцит фібриностабілізуючого фактора

226 У собаки з ендокринною патологією було виявлено: зменшення споживання кисню у стані спокою, зниження толерантності до глюкози, зниження температури тіла. Недостатністю якого гормону можна пояснити виявлені зміни? A *Тироксину B Інсуліну C Соматотропного D Адренокортикотропного E Гонадотропного

227 У мишей з відсутнім волосяним покривом (тобто nude - голі) не було клітинних реакцій уповільненого типу. Для цієї патології найбільш вірогідним є: A *Відсутність вилочкової залози B Відсутність гамаглобулінів у крові C Порушення гемопоезу D Дефект фагоцитозу E Дефіцит компонентів системи комплементу

228 В медико-генетичну консультацію за рекомендацією андролога звернувся чоловік 35 років з приводу відхилень фізичного і психічного розвитку. Об'єктивно встановлено: високий зріст, астенічна будова тіла, гінекомастія, розумова відсталість. При мікроскопії клітин слизової оболонки ротової порожнини знайдено в 30% статевий хроматин (одне тільце Барра). Який найбільш вірогідний діагноз? A *Синдром Клайнфельтера. B Синдром Ді Джорджі. C Хвороба Дауна D Хвороба Реклінгаузена. E Хвороба Іценка-Кушинга.

229 При підйомі в гори у альпініста розвинулась ейфорія, яка замінилася головним болем, запамороченням, серцебиттям, задишкою, що перейшла в апное. Яке порушення

Page 198: крок 1 база 2014р_11шр-240ст (для брошури)_35грн

198

кислотно-лужної рівноваги розвинулось в даному випадку? A *Газовий алкалоз. B Негазовий ацидоз. C Газовий ацидоз. D Негазовий алкалоз. E Видільний алкалоз.

230 Хворий переніс операцію з приводу резекції пілоричного відділу шлунку. Через рік скаржиться на слабкість, періодичну появу темних кіл під очима, задишку. В крові: Hb - 70 г/л, ер. - 3,0*1012/л. Які зміни еритроцитів в мазках периферичної крові характерні для даного захворювання? A *Гіпохромні еритроцити B Еритроцити з тільцями Кебо C Еритроцити з тільцями Жоллі D Гіперхромні еритроцити E Макроцити

231 У хворого виявлені множинні синяки на тілі, тривалість кровотечі за Дуке 25 хвилин, число тромбоцитів крові 25*109/л. Для якого захворювання характерні такі ознаки? A *Спадковий дефект утворення тромбоцитів. B Гемофілія А. C Гемофілія В. D Хвороба Віллебранда. E Авітаміноз С.

232 Чоловік 67 років страждає на атеросклероз судин головного мозку. При обстеженні знайдена гіперліпідемія. Вміст якого класу ліпопротеїдів плазми крові найбільш вірогідно буде значно підвищений при біохімічному дослідженні? A *Ліпопротеїди низької щільності B Хіломікрони C Ліпопротеїди дуже низької щільності D Ліпопротеїди високої щільності E Комплекси жирних кислот з альбумінами

233 Чоловік 25 років скаржиться на часто виникаючі запальні захворювання різної локалізації. Встановлено, що він - ін'єкційний наркоман. Проба на ВІЛ-інфекцію виявилась позитивною. Який з перелічених типів клітин імунної системи найбільш суттєво вражається ВІЛ?

A *хелпери B кілери C нейтрофільні гранулоцити D плазматичні клітини E --

234 У дівчинки 6 років пастозність повік, губ, шиї, слизової оболонки язика виникла після того, як вона з'їла апельсин. Раніше на апельсини виникали висипання на шкірі, свербіння. Який патогенетичний механізм являється провідним у розвитку набряка у дівчинки? A *Підвищення проникливості капілярів B Порушення лімфовідтоку C Підвищення онкотичного тиску тканинної рідини D Зниження онкотичного тиску крові E Підвищення гідростатичного тиску крові в капілярах

235 Чоловік 32 років впродовж 4 років хворіє на хронічний гломерулонефрит. Госпіталізований з ознаками анасарки: АТ- 185/105 мм рт.ст. У крові: Hb- 110 г/л, ер.- 2,6*1012/л, лейк.- 9,5*109/л, залишковий азот - 32 ммоль/л, загальний білок - 50 г/л. Яка зміна з найбільшою вірогідністю вказує на гломерулонефрит з нефротичним синдромом? A *Гіпопротеїнемія B Анемія C Лейкоцитоз D Артеріальна гіпертензія E Гіперазотемія

236 На ЕКГ пацієнта мають місце такі зміни: зубець Р - нормальний, інтервал Р-Q - вкорочений, шлуночковий комплекс QRST - розширений, зубець R - двогорбий або двофазний. Яка із форм аритмії має місце у даного пацієнта? A *Синдром WPW (Вольфа-Паркінсона-Уайта) B Синдром Фредеріка (тремтіння передсердь) C Атріо-вентрикулярна блокада D Миготіння шлуночків E Миготлива аритмія

237 У хворого в коматозному стані відчувається запах яблук із рота. Вміст глюкози в плазмі крові - 18 ммоль/л. Яку із ком

Page 199: крок 1 база 2014р_11шр-240ст (для брошури)_35грн

http://vk.com/my.printing

199

найвірогідніше запідозрити в даному випадку? A *Кетоацидемічну B Токсичну C Гіперосмолярну D Лактатацидемічну E Гіпоглікемічну

238 Жінка 44 років скаржиться на загальну слабість, біль в ділянці серця, значне збільшення маси тіла. Об'єктивно: обличчя місяцеподібне, гірсутизм, АТ - 165/100 мм рт.ст., зріст - 164 см, вага - 103 кг, переважно накопичення жиру на шиї, верхньому плечовому поясі, животі. Що є основним патогенетичним механізмом ожиріння у жінки? A *Підвищення продукції глюкокортикоїдів B Зниження продукції тиреоїдних гормонів C Підвищення продукції інсуліну D Зниження продукції глюкагону E Підвищення продукції мінералокортикоїдів

239 У хворого з тромбофлебітом нижніх кінцівок раптово після навантаження виникла задишка, різкий біль у грудях, ціаноз, набухання шийних вен. Яке найбільш імовірне порушення кровообігу виникло у хворого? A *Тромбоемболія легеневої артерії B Тромбоемболія вінцевих судин C Тромбоемболія судин головного мозку D Тромбоемболія мезентеріальних судин E Тромбоемболія ворітної вени

240 Після інсульту з ураженням ядер гіпоталамусу у хворого виник нецукровий діабет. Що стало причиною посиленого сечовиділення у цього хворого? A *Зменшення реабсорбції води B Зменшення реабсорбції натрію C Зниження артеріального тиску D Гіперглікемія E Прискорення клубочкової фільтрації

241 У хворого із запаленням легень спостерігається підвищення температури тіла. Яка біологічно активна речовина відіграє провідну роль у виникненні цього прояву?

A *Iнтерлейкін-I B Гістамін C Брадикінін D Серотонін E Лейкотрієни

242 Жінку 32 років вжалила оса. На шкірі лівої щоки (на місці укусу) - набряк та гіперемія. Який механізм набряку є первинним у даному випадку? A *Підвищення проникливості капілярів B Підвищення гідростатичного тиску крові у капілярах C Зниження онкотичного тиску крові D Підвищення онкотичного тиску тканинної рідини E Утруднення лімфовідтоку

243 Чоловіку 27 років була проведена туберкулінова проба Манту. Через 24 години на місці ін'єкції відмічається інфільтрат 40x35 мм, шкіра над яким гіперемійована. Яка із груп біологічно активних речовин визначила в основному розвиток алергічного запалення у пацієнта? A *Лімфокіни B Кініни C Простагландини D Лейкотрієни E Біогенні аміни

244 У жінки 34 років діагностована спадкова мікросфероцитарна гемолітична анемія (хвороба Мінковського-Шоффара). Який механізм викликав гемоліз еритроцитів у хворої? A *Мембранопатія B Ензимопатія C Гемоглобінопатія D Аутоімунне ураження E Гіпоплазія кісткового мозку

245 Пацієнту з гострим інфарктом міокарда внутрішньовенно крапельно введено 1500 мл різних розчинів протягом 8 годин, кисень інтраназально. Смерть настала від набряку легень. Що спричинило набряк легень? A *Перевантаження лівого шлуночка об'ємом B Зменшення онкотичного тиску за рахунок гемодилюції

Page 200: крок 1 база 2014р_11шр-240ст (для брошури)_35грн

200

C Алергічна реакція D Нейрогенна реакція E Інгаляція кисню

246 Хворий 2 роки тому переніс операцію резекції пілоричного відділу шлунка. Спостерігається слабість, періодична поява темних кіл перед очима, задишка. В аналізі крові: Hb - 70 г/л, ер. - 3,0*1012/л, к. п. - 0,7. Які зміни еритроцитів у мазках крові найбільш характерні для даного стану? A *Мікроцити B Мегалоцити C Шизоцити D Овалоцити E Макроцити

247 У групі дітей, які їли солодкий соковитий кавун, у двох з'явились ознаки отруєння: різка слабість, запаморочення, головний біль, блювання, задишка, тахікардія, синюшність губів, вух, кінчиків пальців. Лабораторний аналіз кавуна показав високий вміст нітратів. Який провідний механізм у патогенезі отруєння тільки у двох дітей? A *Недостатність мет-Hb-редуктази B Недостатність супероксиддисмутази C Блокада цитохромоксидази D Недостатність глутатіон-піроксидази E Недостатність каталази

248 При обстеженні хворого виявлені наступні клінічні прояви: шкірні покриви рожеві, теплі на дотик, сухі, ЧСС - 92/хв.,ЧД - 22/хв., температура тіла - 39,2oC. Яке співвідношення процесів утворення і віддачі тепла в описаному періоді пропасниці? A *Теплопродукція дорівнює тепловіддачі B Теплопродукція перевищує тепловіддачу C Теплопродукція нижче за тепловіддачу D Зниження тепловіддачі на фоні незміненої теплопродукції E Посилення теплопродукції без зміни тепловіддачі

249 При гепатиті, інфаркті міокарда в плазмі крові хворих різко зростає активність аланін- і аспартамамінотрасфераз. Які причини зростання активності цих ферментів у крові?

A *Пошкодження мембран клітин і вихід ферментів у кров B Підвищення активності ферментів гормонами C Нестача піридоксину D Зростання швидкості синтезу амінокислот у тканинах E Збільшення швидкості розпаду амінокислот у тканинах

250 Хворий 50 років скаржиться на спрагу, п'є багато води, виражена поліурія. Глюкоза крові 4,8 ммоль/л, в сечі глюкози і ацетону немає, сеча безбарвна, питома вага 1,002-1,004. Яка причина поліурії? A *Нестача АДГ B Гіпотиреоз C Інсулінова недостатність D Альдостеронізм E Тиреотоксикоз

251 При диспансерному обстеженні хлопчику 7 років встановлено діагноз - дальтонізм. Батьки здорові, кольоровий зір нормальний. Але у дідуся по материнській лінії така ж аномалія. Який тип успадкування цієї аномалії? A *Рецесивний, зчеплений зі статтю B Домінантний, зчеплений зі статтю C Неповне домінування D Аутосомно-рецесивний E Аутосомно-домінантний

252 У жінки, яка протягом 15 років страждала вираженою гіпертензією, останній час з’явилась задишка, серцебиття, трохи знизився систолічний тиск. Який основний механізм виникнення у хворої серцевої недостатності? A *Перевантаження серця збільшеним опором викіду крові. B Перевантаження серця збільшеним об’ємом крові . C Ушкодження міокарду. D Порушення проведення імпульсу по міокарду. E Порушення регуляції серцевої діяльності.

253 У хворого з гіпохромною анемією січеться і випадає волосся, підвищена ламкість

Page 201: крок 1 база 2014р_11шр-240ст (для брошури)_35грн

http://vk.com/my.printing

201

нігтів, псування смаку. Який механізм указаних симптомів? A *Дефіцит залізовмісних ферментів. B Дефіцит вітаміна В12. C Зниження продукції паратирину. D Дефіцит вітаміна А. E Зниження продукції тиреоідних гормонів.

254 Хвора 27 років закапала в очі краплі, до складу яких входить пеніцилін. Через декілька хвилин з’явились свербіння та печія тіла, набряк губ, повік, кашель з свистом, став падати АТ. Які імуноглобуліни приймають участь в розвитку даної алергічної реакції? A *IgE і IgG . B IgM і IgG . C IgA і IgM D IgM i IgD E IgG i IgD

255 Хворий 12 років поступив в клініку з гемартрозом колінного суглобу, з раннього дитинства страждає кровоточивістю. Яка хвороба у хлопчика? A *Гемофілія. B Геморрагічний васкуліт. C Гемолітична анемія. D Вітамін В12 (фолієво) дефіцитна анемія. E Тромбоцитопенічна пурпура.

256 У чоловіка віком 50 років, який лікувався на виразкову хворобу шлунку, нормалізувалося травлення, зникли болі, поліпшився настрій. Але через кілька тижнів знов з’явились болі в епігастрії, печія, відрижка кислим. Як можна характеризувати такий перебіг хвороби? A *Рецидив хвороби. B Період ремісії. C Термінальний стан. D Продромальний період Продромальний період E Латентній період.

257 До клініки поступив чоловік віком 40 років якого укусила гадюка. Де переважно буде проходити гемоліз еритроцитів у цьому

випадку? A *У кровоносному руслі. B У клітинах печінки. C У клітинах селезінки. D У кістковому мозку. E У паренхімі нирок.

258 У жінки віком 45 років, яка тривалий час хворіє на бронхіальну астму, виник напад ядухи. Який патогенетичний механізм має це явище? A *Спазм дрібних бронхів. B Втрата еластичності легеневої тканини. C Зниження чутливості дихального центру. D Порушення рухомості грудної клітини. E Порушення перфузії легеневої тканини .

259 У жінки віком 67 років, яка тривалий час страждала на холецистит, після їжі раптово виникла різка біль у верхній частині живота, нудота, блювання. Встановлен діагноз – гострий панкреатит. Що є основною ланкою патогенеза цього захворювання? A *Передчасна активація ферментів підшлункової залози. B Зниження рівня ферментів у панкреатичному соку. C Підвищення активації ферментів у дванадцятипалій кишці. D Зниження секреції панкреатичного поліпептиду. E Підвищення рівня холецистокініну.

260 Чоловіку 57 років, після обстеження був поставлений діагноз - В12 дефіцитна анемія, назначене лікування. Через 3 доби був зроблений контрольний аналіз крові. Що буде найбільш адекватним критерієм підвищення ерітропоезу? A *Підвищення кількості ретикулоцитів. B Підвищення рівня гемоглобіну. C Зниження кольорового показника. D Підвищення кількості тромбоцитів. E Підвищення кількості лейкоцитів.

261 Хворий 21-го року поступив в стаціонар з загостренням хронічного тонзиліту. Скаржиться на слабкість, задуху при помірному фізичному навантаженні. Температура 37,5oС. ЧСС

Page 202: крок 1 база 2014р_11шр-240ст (для брошури)_35грн

202

110 за хв. ЕКГ: ритм синусів, інтервал PQ подовжений. Яка аритмія у хворого? A *Передсердньо-шлуночкова блокада І ст. B Передсердньо-шлуночкова блокада ІІ ст. C Внутрішньопередсердня блокада. D Порушення внутрішньошлуночкової провідності. E Предсердньо-шлуночкова екстросістолія.

262 Хворий на механічну жовтяницю поступив в лікарню з ознаками холемічного синдрому. На ЄКГ виявлена аритмія. Яке порушення ритму серця найбільш імовірне у хворого? A *Синусова брадикардія. B Синусова тахікардія. C Предсердна екстрасистола. D Шлуночкова екстрасистола. E Атріовентрикулярна блокада.

263 В експерименті при моделюванні ниркової патології у тварини отримали ознаки: набряки, висока протеїнурія, гіпопротеінемія, диспротеінемія, гіперліпідемія. Для якої патології нирок характерна така сукупність ознак? A *Нефротичний синдром. B Гострий дифузний гломерулонефрит. C Пієлонефрит. D Гостра ниркова недостатність. E Хронічна ниркова недостатність.

264 У хворого має місце підвищення опору відтоку крові з лівого шлуночка, що призвело до включення гомеометричного механізму компенсації. При якому із перерахованих патологічних процесів може мати місце цей механізм компенсації у лівому шлуночку серця A *Стеноз аортального клапана B Недостатність аортального клапана C Мітральний стеноз D Артеріальна гіпотензія E Емболія легеневої артерії

265 У хворого внаслідок отруєння сулемою розвинулася гостра ниркова недостатність, перебіг якої включав 4 стадії: перша-початкова, друга-оліго-, анурії, четверта-одуження. Як називається третя стадія гострої ниркової недостатності? A *Поліурична

B Метаболічна C Гемодинамічна D Ішемічна E Патохімічна

266 У хворого має місце мутація гена, що відповідає за синтез гемоглобіну. Це призвело до розвитку захворювання -серповидно-клітинної анемії. Як називається патологічний гемоглобін, що виявляється при цьому захворюванні? A *HbS B HbA C HbF D HbA1 E Bart-Hb

267 У хворого виявлено порушення реабсорбції глюкози в проксимальному відділі нефрону з розвитком глюкозурії, при цьому в плазмі крові має місце гіпоглікемія. Як називається це порушення? A *Ниркова глікозурія B Цукровий діабет C Фосфатний нирковий діабет D Позаниркова глікозурія E Галактоземія

268 У хворого має місце пошкодження нирок з розвитком масивної протеїнурії, гіпопротеїнемії, набряків та ретенційної гіперліпемії. Це порушення має назву: A *Нефротичний синдром B Гостра ниркова недостатність C Хронічна ниркова недостатність D Ниркова гіпертензія E Ізогіпостенурія

269 При запаленні ока у хворого відмічалося накопичення мутної рідини з високим вмістом білка на дні передньої камери, яке отримало назву - гіпопіон. Який процес лежить в основі спостерігаємих змін? A *Порушення мікроциркуляції B Первинна альтерація C Вторинна альтерація D Проліферація E -

Page 203: крок 1 база 2014р_11шр-240ст (для брошури)_35грн

http://vk.com/my.printing

203

270 У хворого з хронічним гіпоацидним гастритом має місце гіпохромна анемія. У мазку крові виявлені анулоцити, мікроанізоцитоз, пойкілоцитоз. Який вид анемії спостерігається у нього? A *Залізодефіцитна B Гостра посгеморагічна C Таласемія D Серповидноклітинна E Перніціозна

271 У хворого на гострий апендецит у крові виявлено зростання числа лейкоцитів. Який вид лейко_цитозу може мати місце за цих умов? A *Нейтрофільний B Базофільний C Еозинофільний D Лімфоцитоз E Моноцитоз

272 У хворого має місце передсердно-шлуночкова блокада І ступеня, що супроводжується подовженням інтервалу P-Q до 0,25 с. Порушення якої функції серця має місце за цих умов? A *Провідності B Автоматизму C Збудливості D Скоротливості E Засвоєння ритму

273 У хворого має місце зниження в крові кількості еритроцитів, гемоглобіну, кольорового показника, концентрації сироваткового заліза, мікроанізоцитоз, пойкілоцитоз. Ці зміни супроводжуються розвитком гіпоксії. Який вид гіпоксії спостерігається в даному випадку? A *Гемічна B Гіпоксична C Циркуляторна D Тканинна E Дихальна

274

У хворого має місце зміщення установочної точки терморегуляції на більш високий рівень регулювання температури тіла внаслідок пірогенного впливу інтерлейкіну 1. Як називається цей типовий патологічний процес? A *Гарячка B Перегрівання C Гіпотермія D Запалення E Гіпоксія

275 У хворого на крупозну пневмонію має місце гарячка з температурою тіла 39oС, при цьому добові коливання температури не перевищували 1 С впродовж 9 діб. До якого типу температурних кривих відноситься ця гарячка? A *Постійна B Гектична C Ремітуюча D Гіперпіретична E Поворотна

276 У хворого розвинулася гарячка, яка супроводжувалася зміщенням установчої точки терморегуляційного центру на більш високий рівень, з послідовним чергуванням наступних стадій: Incrementi, fastigii, decrementi. При якому захворюванні можуть спостерігатися подібні зміни( A *Гостра пневмонія B Акромегалія C Цукровий діабет D Ренальний діабет E Гіпертрофія міокарда

277 Хвора, 59 років, госпіталізована в хірургічне відділення з приводу загострення хронічного остеомієліта лівої гомілки. В аналізі крові: лейкоцитів - 15,0*109/л; лейкоцитарна формула: мієлоцити – 0%, метамієлоцити – 8%, паличкоядерні – 28%, сегментоядерні нейтрофіли – 32%, лімфоцити – 29%, моноцити – 3%. Яку назву має така картина крові? A * Регенеративний зсув формули вліво B Зсув формули вправо C Гіперрегенеративний зсув формули вліво D Дегенеративний зсув формули вліво

Page 204: крок 1 база 2014р_11шр-240ст (для брошури)_35грн

204

E Регенеративно-дегенеративний зсув формули вліво

278 Дівчина, 15 років, хворіє на бронхіальну астму. Весною, в період цвітіння трав, у неї розвинувся тяжкий напад експіраторної задишки. Яка з біологічно активних речовин вірогідно викликала спазм непосмугованої м’язової тканини бронхів у даному випадку? A * Повільно реагуюча субстанція. B Тромбоксан А2. C Простациклін. D Брадікінін. E Серотонін.

279 Хворий звернувся до лікаря із скаргами на те, що кожної весни, в період цвітіння рослин в нього відмічаються головний біль, нежить, слабість, підвищення температури. Який тип алергічної реакції за Gell і Coombs має місце у хворого? A *Анафілактичний B Цитотоксичний C Стимулюючий D Імунокомплексний E Клітинно-опосередкований

280 Хвора, 48 років, поступила в клініку із скаргами на слабість, дратівливість, порушення сну. Шкіра, склери жовтого кольору. В крові – прямий білірубін, холалемія; кал – ахолічний; сеча – темного кольору (білірубін). Яка жовтяниця має місце у хворого? A * Механічна. B Гемолітична. C Паренхіматозна. D Синдром Жільбера. E Синдром Кріглера-Найяра.

281 У хворого на дифтерію розвинувся міокардит з проявами недостатності кровообігу. Яке з порушень гемодінаміки є характерним для даного хворого? A * Зниження максимального артеріального тиску. B Зниження мінімального артеріального тиску. C Підвищення максимального артеріального тиску. D Зростання швидкості кровообігу.

E Зниження венозного тиску.

282 У больного отмечаются периодические приступы сердцебиений (пароксизмы), сильное потоотделение, приступы головной боли. При обследовании обнаружена гипертензия, гипергликемия, повышение основного обмена, тахикардия. При какой патологии надпочечников наблюдается подобная картина? A *Гиперфункции мозгового слоя B Гипофукции мозгового слоя C Гиперфункции коры надпочечников D Гипофункции коры надпочечников E Первичном альдостеронизме

283 У хворого 43 років артеріальна гіпертензія є наслідком помірного збільшення об’єму серця за хвилину і загального периферичного опору. Вкажіть гемодинамічний варіант розвитку артеріальної гіпертензії у даному випадку. A * Еукінетичний. B Гіперкінетичний. C Гіпокінетичний. D - E Змішаний.

284 У чоловіка 65 років на протязі 15 років була виражена артеріальна гіпертензія. Останнім часом систолічний тиск почав знижатися, а діастолічний залишився підвищеним. Який гемодинамічний тип артеріальної гіпертензії у хворого? A *Гіпокінетичний. B Нормокінетичний. C Гіперкінетичний. D Еукінетичний. E -

285 У чоловіка, віком 50 років при обстеженні було виявлено зниження кількості еритроцитів в крові і підвищення рівня вільного гемоглобіну в плазмі крові (гемоглобінемію). КП становив 0,85. Який вид анемії спостерігається у хворого? A *Набута гемолітична анемія. B Спадкова гемолітична анемія. C Гостра постгеморагічна анемія. D Хронічна постгеморагічна анемія.

Page 205: крок 1 база 2014р_11шр-240ст (для брошури)_35грн

http://vk.com/my.printing

205

E Анемія внаслідок порушення еритропоезу.

286 В основе развития иммунных и аллергических реакций организмом используются одинаковые механизмы ответа иммунной системы на антиген. Определите основное отличие аллергических реакций от иммунных? A *Развитие повреждения тканей B Количество попадающего антигена C Особенность строения антигенов D Пути попадания антигенов в организм E Наследственная предрасположенность

287 У хворого протягом дня підвищення температури тіла змінюється зниженням її до нормального рівня. Таке підвищення температури спостерігається періодично через 3 дні на четвертий. Який тип температурної кривої? A * Febris internuttens B Febris continua C Febris reccurens D Febris hectica E Febris remitens

288 У хворого на цукровий діабет розвинулась діабетична нефропатія з розвитком уремії. Швидкість клубочкової фільтрації 9 мл/хв. Який найбільш вірогідний механізм зниження швидкості клубочкової фільтрації і розвитку ХНН у пацієнта? A * Зменшення маси діючих нефронів B Зниження системного артеріального тиску C Закупорка просвіту канальців нефрона гіаліновими циліндрами D Розвиток ацидозу в тканинах E Спазм приносячих артеріол

289 У хворого діагностована спадкова форма коагулопатії, яка проявляється дефектом VIII фактору зсідання крові. Вкажіть в якій фазі зсідання крові виникають первинні порушення коагудяції в даному випадку? A *Утворення тромбопластину B Утворення тромбіну C Утворення фібрину

D Ретракція згустку E -

290 У хворого, який скаржився на біль у ділянці лівої лопатки, був діагностований інфаркт міокарду. Назвіть вид болю у хворого? A *Ірадіюючий (відбитий) B Вісцеральний C Фантомний D Перший (протопатичний) E Другий (епікритичний)

291 Для діагностування деяких хромосомних хвороб використовують визначення статевого хроматину. Назвіть хворобу, при якій потрібне це визначення: A * Синдром Шерешевського-Тернера B Хвороба Дауна C Гемофілія D Трисомія Е E Хвороба Брутона

292 Хвора К.,37 років,звернулася у клініку зі скаргами на головний біль,запаморочення,поганий сон,занеміння кінцівок.Останні 6 років працює на заводі газорозрядних ламп у свинцеврму цеху.При обстеженні в аналізі крові кількість еритроцитів та гемоглобіну знижена,вміст сироваточного заліза підвищений в декілька разів.Назвіть вид анемії? A *залізорефрактерна B залізодефіцитна C анемія Мінковського-Шоффара D гіпопластична E метапластична

293 При проведенні амніоцентезу в клітинах плода виявлено по 2 тільця статевого хроматину (тільця Барра). Для якого захворювання характерна дана ознака? A * Трисомія X B Синдром Клайнфельтера C Синдром Шерешевського-Тернера D Синдром Дауна E Синдром Патау

294 Відомо, що типові патологічні процеси розвиваються за однаковими закономірностями в

Page 206: крок 1 база 2014р_11шр-240ст (для брошури)_35грн

206

різних органах і тканинах та у різних видів тварин. Яке з перерахованих явищ можна віднести до типового патологічного процесу? A *Пухлину B Туберкульоз C Гіпертонічну хворобу D Непрохідність кишечника E Інфаркт міокарда

295 Пацієнт звернувся зі скаргами на гострий біль у правому підребер’ї. При огляді лікар звернув увагу на пожовтілі склери хворого. Лабораторні аналізи показали підвищену активність АЛТ та негативну реакцію на стеркобілін в калі. Для якого захворю-вання характерні такі симптоми? A Жовчнокам’яна хвороба; B Гемолітична жовтяниця; C Гепатит; D Хронічний коліт; E Хронічний гастрит.

296 У хворого діагностована залізодефіцитна сидероахрестична анемія, перебіг якої супроводжується гіперпігментацією шкіри. розвитком пігментного цирозу печінки, ушкодженням підшлункової залози та серця. Вміст заліза у сироватці крові підвищений. Яким порушенням обміну заліза спричиняється таке захворювання? A * Невикористання заліза і відкладання його в тканинах. B Надмірне поступлення заліза з їжею. C Порушення всмоктування заліза в кишечника. D Підвищене використання заліза організмом. E Поєднаний дефіцит вітаміну В-12 і заліза

297 При алкаптонурії відбувається надмірне виділення гомогентизинової кислоти із сечею. С порушенням метаболізму якої амінокислоти пов’язано виник-нення цього захворювання? A Тирозину; B Фенілаланіну; C Аланіну; D Метіоніну; E Аспарагіну.

298

При аналізі ЕКГ виявлено випадіння деяких серцевих циклів PQRCT. Наявні зубці і комплекси не змінені. Назвіть вид аритмії. A *Синоатріальна блокада B Миготлива аритмія C Атріовентрикулярна блокада D Передсердна екстрасистола E Внутрішньопередсердна блокада

299 При обследовании больного с сахарным диабетом 1-го типа выявлено нарушение белкового обмена, что при лабораторном исследовании крови проявляется аминоацидэмией, а клинически: замедлением заживления ран и уменьшением синтеза антител. Какой из перечисленных механизмов вызывает развитие аминоацидэмии? A *повышение протеолиза B гиперпротеинемия C уменьшение концентрации аминокислот в крови D повышение онкотического давления в плазме крови E увеличение липопротеидов низкой плотности

300 Рабочий коммунальной службы, пренебрегая правилами техники безопасности, спустился в канализационный колодец без средств защиты и через некоторое время потерял сознание. Врачами скорой помощи диагностировано отравление сероводородом. Какой вид гипоксии при этом развился? A Гемический B Перегрузочный C Тканевой D Циркуляторный E Респираторный

Фармакологія 1 При інфаркті міокарду була проведена нейролептанальгезія. Який препарат з групи нейролептиків найчастіше застосовується спільно з фентанілом? A *Дроперидол B Етаперазин C Левомепромазин D Клозапін E сульпірид

2

Page 207: крок 1 база 2014р_11шр-240ст (для брошури)_35грн

http://vk.com/my.printing

207

Лікар обговорював з колегами застосування нового протиепілептичного засобу – натрію вальпроату. Який можливій механізм дії препарату? A *Пригнічення активності ферменту ГАМК-трансферази B Стимуляція активності ферменту ГАМК-трансферази C Пригнічення активності Са2+залежної АТФ-ази D Стимуляція активності Са2+залежної АТФ-ази E Пригнічення моноамінооксидази

3 При гіпертонічному кризі хворому ввели магнію сульфат, в результаті чого настало різке зниження артеріального тиску. Введенням якого препарату можна запобігти побічні ефекти сульфату магнію? A *Кальцію хлорид B Калію хлорид C Трилон Б D Натрію бромід E Натрію сульфат

4 Хворому для ввідного наркозу внутрішньовенно ввели тиопентал натрію, після чого настав ларингоспазм, підсилилась салівація. Введення якого препарату можливо було б запобігти небажані ефекти? A *Атропіну сульфат B Адреналіну гідро хлорид C Прозерин D Аллоксим E Пірроксан

5 У хворого після короткочасного оперативного втручання проведеного з застосування дитиліну понад 30 хвилин відмічалось пригнічення дихання не відновився попередній тонус м’язів. Яку допомогу необхідно надати хворому? A *Перелівання крові B Гемодіаліз C Гемосорбція D Форсований діурез E Перитоніальний діаліз

6 До приймального відділення надійшов хворий у непритомному стані. Шкіра холодна, зіниці звужені, дихання з утрудненням, відзначається періодичність по типу Чейн-Стокса, артеріальний тиск знижений, сечовий міхур переповнений. Чим відбулося отруєння? A *Наркотичними анальгетиками B Транквілізаторами C Ненаркотичними анальгетиками D М-холіноблокаторами E -

7 Хворий з діагнозом вогнищевий туберкульоз верхньої долі правої легені в складі комбінованої терапії одержує ізоніазид. Через деякий час пацієнт почав пред’являти скарги на м’язову слабкість, зниження шкірної чутливості, порушення зору, координації рухів. Який вітамінний препарат доцільно використати для усунення даних явищ? A *Вітамін В6 B Вітамін А C Вітамін D D Вітамін В12 E Вітамін C

8 Хворому на гіпертонічну хворобу з супутнім обструктивним бронхітом призначили гіпотензивний засіб. Через деякий час у пацієнта почали з’являтися напади ядухи, розвинулась виражена брадикардія. На ЕКГ відмічались ознаки порушення атріовентрікулярного проведення. При призначенні якого препарату найбільше імовірне зявлення подібних ефектів? A *Анаприлін B Клофелін C Корданум D Верепаміл E Резерпін

9 Хворому на гіпертонічну хворобу з супутнім обструктивним бронхітом в складі комплексної терапії призначили анаприлін. Через деякий час у хворого розпочали з’являтися напади ядухи. З чим пов’язане виникнення даного побічного ефекту? A *Блокада β2 -адренорецепторів бронхів

Page 208: крок 1 база 2014р_11шр-240ст (для брошури)_35грн

208

B Блокада β1 -адренорецепторів бронхів C Стимуляція β2 -адренорецепторів бронхів D Блокада α2 -адренорецепторів бронхів E Стимуляція α1 -адренорецепторів бронхів

10 Хворий 60 років протягом 9 років хворіє на цукровий діабет, отримує для корекції гіперглікемії інсулін – семіленте. 10 днів тому почав лікування гіпертонічної хвороби. Через годину після прийому гіпотензивного препарату розвинулась гіпоглікемічна кома. Який з перелічених препаратів міг викликати подібне явище? A *Анаприлін B Празозин C Верапаміл D Каптоприл E Ніфедипін

11 Хворий 60 років протягом 9 років хворіє на цукровий діабет, отримує для корекції гіперглікемії інсулін – семіленте. 10 днів тому почав лікування гіпертонічної хвороби анаприліном. Через годину після прийому гіпотензивного препарату розвинулась гіпоглікемічна кома. Який механізм виникнення гіпоглікемії за умови призначення анаприліна? A *Пригнічення глікогенолізу B Зменшення виділення г періоду напіввиведення глюкагону C Збільшення періоду напіввиведення інсуліну–семіленте D Збільшення біодоступності інсуліну-семіленте E Зменшення всмоктування глюкози

12 Хворий 30-ти років звернувся до лікаря зі скаргами на пронос і біль у животі протягом 5-ти днів, підвищення температури тіла до 37,5oC з ознобами. На передодні хворий був у лісі, де випив води з відкритого водоймища. Встановлено бактеріологічно підтверджений діагноз: амебна дизентерія. Вкажіть препарат вибору для лікування цього захворювання: A *Метронідазол B Фуразолідон C Левоміцетин D Фталазол E Еметина гідрохлорид

13 У хворого 55 років на 4-й день лікування диклофенаком-натрію виникла шлункова кровотеча в результаті виразки слизової оболонки шлунка. З чим пов’язана ульцерогенна дія препарату? A *Зменшення утворення простагландіну Е1 B Зменшення утворення простацикліну C Зменшення утворення лейкотриєнів D Зменшення утворення циклічних ендоперекисів E Зменшення утворення тромбоксану

14 Хворий надійшов у хірургічне відділення з діагнозом: гострий панкреатит, розпочато консервативне лікування. Призначення якого препарату є патогенетично обгрунтоване?. A *Контрикал B Трипсин C Хімотрипсин D Панкреатин E Фібринолізин

15 У другій половині вагітності жінка приймала транквілізатори групи бензодиазепинів. Пологи наступили в строк, протікали нормально, але народилась дитина з численними аномаліями розвитку (заяча губа, полідактилія). Як називається описана дія препарату? A *Тератогенна B Мутагенна C Ємбріотоксична D Фетотоксична E -

16 У хворого при комбінованій терапії хронічної серцевої недостатності дігитоксіном та фуросемідом розвинулась різка м’язова слабкість. Які електролітні порушення можна виявити в крові? A *Гіпокаліємія B Гіперкаліємія C Гіпокальціємія D Гіперкальціємія E -

17 У хворої 45 років, котра протягом двох тижнів приймає неодикумарін з приводу

Page 209: крок 1 база 2014р_11шр-240ст (для брошури)_35грн

http://vk.com/my.printing

209

тромбофлебіту при черговому обстеженні в крові виявлено зниження протромбіну, в сечі спостерігається мікро гематурія. Який лікарській засіб необхідно застосувати в якості антагоністу неодикумаріну? A *Вікасол B Протаміну сульфат C Амбен D Натрію цитрат E Тромбін

18 До приймального відділення був доставлений хворий зі скаргами на запаморочення, зниження гостроти зору, нудоту, слинотечею та спастичні болі в животі. Був встановлений діагноз: отруєння фосфорорганічними з’єднаннями. Що доцільно включити в склад комплексної терапії? A *Атропіну сульфат та дипіроксим B Тіосульфат натрію та бемегрид C Тета цин-кальцій та унітіол D Налорфіну гідро хлорид та бемегрид E Глюкоза та бемегрид

19 У хворого, що страждає на хронічну серцево-судинну недостатність в процесі дігіталізації з’явились слідуючи симптоми: головний біль, втома, нудота, порушення кольорового зору (навколишні предмети сприймаються в зеленому кольорі). На ЕКГ з’явилась синусова брадикардія та ознаки порушення атріовентрикулярного проведення. Який засіб можна застосувати для зменшення симптомів інтоксикації? A *Унітіол B Налоксон C Бемегрид D Дипіроксим E Атропіну сульфат

20 У хворого після довготривалого застосування призначеного з приводу гострого респіраторного захворювання препарату виникли слідуючи симптоми: головний біль, запаморочення, шум у вухах, нудота, біль в епігастральній області. Який препарат міг викликати подібну клінічну картину? A *Ацетилсаліцилова кислота

B Вітамін С C Нафтизин D Бромгексин E Мідантан

21 До приймального відділення доставлений хворий зі скаргами на сухість в роті, світлобоязнь та порушення зору. Шкіра гіпіремійована, суха, зіниці розширені, тахікардія. При подальшому обстеженні був встановлений діагноз: отруєння алкалоїдами красавки. Який з лікарських засобів доцільно застосувати? A *Прозерин B Ацеклідин C Пілокарпін D Армін E Діпіроксим

22 У чоловіка, якому під час оперативного втручання застосували міорелаксант - дитилін, розслаблення скелетних м’язів і пригнічення дихання продовжувалось більше 2 годин. Відсутність якого ферменту в сироватці крові зумовлює цей стан? A *Бутирилхолінестерази B Каталази C Ацетилхолінестерази D Глюко-6-фосфатази E Глутатіонпероксидази

23 Жінка після перенесеного нервового потрясіння погано спить. Якому з перерахованих снодійних засобів слід віддати перевагу для лікування безсоння? A *Нітразепам B Фенобарбітал C Етамінал-натрій D Барбаміл E Барбітал

24 Хворому з гіпертонічною хворобою призначено лізиноприл. Який механізм дії даного препарату? A *Пригнічення активності ангіотензинперетворюючого ферменту B Блокада $\beta$-адренорецепторів

Page 210: крок 1 база 2014р_11шр-240ст (для брошури)_35грн

210

C Блокада А-адренорецепторів D Блокада рецепторів ангіотензину ІІ E Периферична вазодилятаційна дія

25 Хворому з виразковою хворобою призначено фамотидин. Кислотність шлункового соку значно знизилась. Який механізм лежить в основі дії даного препарату? A *Блокада Н2-гістамінових рецепторів B Блокада Н1-гістамінових рецепторів C Блокада М1-холінорецепторів D Пригнічення активності Н+К+АТФ-ази E Блокада Н-холінорецепторів симпатичних гангліїв

26 Для корекції артеріального тиску при колаптоїдному стані хворому було введено мезатон. Який механізм дії даного препарату? A *Стимулює α-адренорецептори B Стимулює β-адренорецептори C Блокує α-адренорецептори D Блокує β-адренорецептори E Стимулює α- β-адренорецептори

27 Для зняття марення і галюцинацій у хворої на шизофренію лікар використав аміназин. Який механізм антипсихотичної дії препарату? A *Блокада адренергічних і дофамінергічних процесів в ЦНС B Стимуляція адренергічних і дофамінергічних процесів в ЦНС C Стимуляція холінергічних процесів в ЦНС D Блокада холінергічних процесів в ЦНС E Інгібування зворотнього нейронального захоплення МАО.

28 У чоловіка 52 років діагностовано системний амебіаз з ураженням кишечника, печінки, легень. Який препарат слід призначити? A * Метронідазол B Хініофон C Тетрациклін D Хінгамін E Ентеросептол

29 Внаслідок інфаркту міокарда у хворого виникла шлуночкова аритмія. Серцевий ритм нормалізувався після введення

протиаритмічного засобу з місцевоанестезуючою активністю. Який препарат введено? A * Лідокаїн B Анестезин C Верапаміл D Панангін E Анаприлін

30 Після лікування високоефективним протитуберкульозним засобом у жінки 40 років виникли явища невриту зорового нерва, порушення пам’яті, судоми. Який препарат приймала хвора? A *Ізоніазид B Рифампіцин C ПАСК D Тіоацетазон E Канаміцин

31 Дитині 13 років хворій на ангіну призначено комбінований сульфаніламідний препарат бактерицидного типу дії. Який препарат призначено? A * Бісептол B Ентеросептол C Сульфален D Етазол E Уросульфан

32 У приймальне відділення поступив у важкому стані чоловік 38 років, який отруївся сулемою. Який антидот треба негайно ввести хворому? A * Унітіол B Дипіроксим C Атропін D Налорфін E Ізонітрозин

33 Жінка 33 років, яка тривалий час лікується з приводу хронічного поліартриту, скаржиться на підвищення артеріального тиску, зміни розподілу жирової тканини, порушення менструального циклу. Який препарат приймає хвора? A *Преднізолон B Індометацин C Бутадіон D Синафлан E Беклометазон

Page 211: крок 1 база 2014р_11шр-240ст (для брошури)_35грн

http://vk.com/my.printing

211

34 Хворому при безсонні, викликаному емоційними розладами, лікар призначив засіб, що викликає сон за рахунок транквілізуючої дії. Який снодійний препарат був призначений пацієнту? A * Нітразепам B Фенобарбітал C Хлоралгідрат D Етамінал-натрій E Бромізовал

35 Хворий 60 років з діагнозом ревматоїдний поліартрит тривалий час застосовує нестероїдний протизапальний засіб діклофенак-натрій. Який механізм дії цього препарату? A * Блокада ферменту циклооксигенази B Блокада ферменту ацетилхолінестерази C Блокада ферменту фосфоліпази D Блокада ферменту ліпооксигенази E Блокада ферменту фосфодіестерази

36 Для проведення оперативного втручання необхідно використати метод загального охолодження. При застосуванні якого препарату у комбінації з фізичним охолодженням спостерігається виразна гіпотермія? A * Аміназин B Ацетилсаліцилова кислота C Морфін D Дроперидол E Галоперидол

37 До лікаря звернувся пацієнт з проханням замінити відсутній на даний момент в аптеці бронхолітик ізадрин на найбільш близький за дією препарат. Який з наведених нижче засобів можна рекомендувати хворому? A *Сальбутамол B Еуфілін C Адреналіну гідрохлорид D Атропіну сульфат E Ефедрину гідрохлорид

38 Хворий знаходиться на стаціонарному лікуванні з приводу бронхіальної астми. Як супутнє захворювання діагностовано гіпертонічну хворобу. Який з наведених препаратів протипоказаний у даному випадку?

A *Анаприлін B Папаверину гідрохлорид C Атропіну сульфат D Сальбутамол E Ефедрину гідрохлорид

39 У інфікованого на СНІД виникла діарея бактеріального генезу. За результатами мікробіологічного дослідження йому був призначений ко-тримоксазол (бісептол, бактрим). Яким типом (характером) протимікробної дії володіє цей препарат? A *Бактерицидним B Бактеріостатичним C Фунгіцидним D Фунгістатичним E Вірулостатичним

40 У вагітної виник гострий трахеобронхіт. Для лікування цієї інфекції був призначений антибіотик з групи пеніцилінів. Визначте, який з наведених антибактеріальних засобів призначено вагітній хворій. A *Амоксицилін B Гентаміцин C Стрептоміцин D Тетрациклін E Фурацилін

41 Хворому перед операцією був ведений дитилін (лістенон) і проведена інтубація. Після закінчення операції і припинення наркозу самостійне дихання не відновилось. Недолік якого ферменту в організмі хворого подовжує дію м’язевого релаксанту? A *Псевдохолінестерази B Сукцинатдегідрогенази C Карбангідрази D N-ацетилтрансферази E K-Na-АТФ-ази

42 Хворий на пневмонію отримує бензилпеніцилліну натрієву сіль в дозі 500 000 ОД 6 разів на день. Після чергового введення лікарського засобу виникла пропасниця, втрата свідомості, судоми. Що розвинулось у хворого? A *Анафілактичний шок B Ідіосинкразія C Тахіфілаксія

Page 212: крок 1 база 2014р_11шр-240ст (для брошури)_35грн

212

D Кумуляція E Толерантність

43 Хворому з тромбоемболією вен нижніх кінцівок призначено медикаментозне лікування. Через 2 доби на шкірі з’явились геморагії. Який препарат із нижчезгаданих викликає подібне ускладнення? A *Гепарін B Курантіл C Ацетилсаліцилова кислота D Неодікумарін E Фенілін

44 В дитини спостерігаються залишкові явища перенесеного поліомієліту. Який лікарський засіб йому можна призначити? A *Галантаміна гідробромід B Пентамін C Дімеколін D Атропіна сульфат E Пірроксан

45 У хворого із сечокам'яною хворобою виникли нестерпні спастичні болі. Для попередження больового шоку йому ввели разом з атропіном наркотичний анальгетик, що не має спазмогенного ефекту. Який це був препарат? A *Промедол B Трамадол C Пірітрамід D Етилморфіну гідрохлорид E Морфіну гідрохлорид

46 До приймального відділення лікарні надійшла дитина 5 років у якої лікарем встановлені такі симптоми: сильне рухове збудження, марення, хриплий голос, розширені зіниці, які не реагують на світло, суху, гарячу, гіперемовану шкіру, тахікардію, тахіпное. Вказані симптоми виникли після споживання дитиною ягід красавки. Препарат якої з представлених фармакологічних груп ліків потрібно призначити? A *Антихолінестеразні засоби B N-холінолітики C N-холіноміметики

D М-холіноміметики E Реактиватори холінестерази

47 При комбінованому наркозі у хворого, якому проведено резек_цію шлунку, як міорелаксант застосовано тубокурарину хлорид. Який засіб-антагоніст слід ввести хворому для відновлення його самостійного дихання? A *Прозерин B Дитилін C Цитітон D Етимізол E Бензогексоній

48 До травматологічного пункту доставлено постраждалого після ДТП з діагнозом: закритий перелом середньої третини стегна зі зміщенням. З метою репозиції кісткових уламків хворому введено 10 мл 2\% розчину дитиліну в/в, внаслідок чого розвинулося тривале апное та міорелаксація. Дефіцитом якого ферменту зумовлена вказана фармакогенетична ферментопатія? A * Псевдохолінестерази B Уридіндифосфоглюкуронової трансферази C Глюкозо-6-фосфатдегідногенази D Метгемоглобінредуктази E N-ацетилтрансферази

49 Хворий 37-ми років, що страждає на облітеруючий ендартеріїт судин нижніх кінцівок, одержує фенілін в добовій дозі 60 мг/кг. В зв’язку з проявами судомного синдрому (в анамнезі ЧМТ) призначений фенобарбітал, після відміни якого у хворого виникла носова кровотеча. Дане ускладнення пов’язане з: A * Індукцією фенобарбіталом ферментів мікросомального окислення в печінці B Аліфатичним гідроксилюванням фенобарбіталу C Кон’югацією феніліну з глюкуроновою кислотою D Окислювальним дезамінуванням феніліну E Гальмуванням фенобарбіталом мікросомального окислення в печінці

50 Хворому з непереносимістю антибіотиків

Page 213: крок 1 база 2014р_11шр-240ст (для брошури)_35грн

http://vk.com/my.printing

213

для лікування пневмонії призначений ко-тримоксазол. Через декілька днів у хворого розвинувся гемоліз еритроцитів. Недостатність якого фермента в організмі хворого сприяла розвитку ціього побічного ефекта? A * Глюкозо-6-фосфатдегідрогенази B Ацетальдегіддегідрогенази C Холінестерази D Уридиндіфосфатглюкуронової трансферази E N-ацетилтрансферази

51 В результаті впливу на які елементи шкіри та слизових оболонок розвивається термінальна анестезія? A * Чутливих нервових закінчень B Епідерміс C Підшкірно-жирова клітковина D Стінки капілярів E Власне дерму

52 Вагітна жінка (20 тижнів вагітності) захворіла на пневмонію. Який препарат можна їй призначити без загрози для розвитку плода? A * Бензілпеніцилін B Гентаміцин C Сульфален D Левоміцетин E Офлоксацин

53 Хворий з діагнозом феохромоцитома страждає від підвищеного артеріального тиску, який переростає в гіпертонічні кризи. Яка група препаратів допоможе хворому в даній ситуації? A *Альфа-адреноблокатори B Бета-адреноблокатори C Гангліоблокатори D Симпатолітики E Блокатори кальцієвих каналів

54 40-річний хворий страждає на бронхіальну астму протягом тривалого часу, а також тахікардією. Вкажіть найдоцільніший в даній ситуації препарат для усунення бронхоспазму: A *Сальбутамол B Ефедрину гідрохлорид C Атропіна сульфат

D Адреналіну гідрохлорид E Ізадрин

55 Жінка 63 років одержувала ін’єкції галантаміну для відновлення функцій ЦНС після ішемічного інсульту мозку. Стан хворої значно покращився. Який механізм дії цього засобу? A *Блокада ацетилхолінестерази B Блокада холінорецепторів C Блокада катехол-орто-метилтрансферази D Блокада дофамін-и бета-гідроксилази E Блокада моноамінооксидази

56 Хворому, який скаржиться на хронічну серцеву недостатність, призначили дігітоксин. Однак, через тиждень після початку прийому препарату у хворого з’явились признаки інтоксикації препаратом (брадикардія, нудота, екстрасістолія), хоч одноразова доза його не привищувала терапевтичної і строк курсу лікування не вийшов. Як називається явище, що спостерігається. A * Матеріальна кумуляція B Функціональна кумуляція C Толерантність D Тахіфілаксія E Ідіосінкразія

57 Дитині 4 років треба призначити протимікробний засіб широкого спектру дії при кишковій інфекції. Який з наведених препаратів не призначається дітям у зв’язку із шкідливим впливом на розвиток кісткової тканини? A *Доксициклін B Амоксицилін C Ампіцилін D Левоміцетін E Ко-тримоксазол (бісептол)

58 Хворому на гіпертонічну хворобу був призначений препарат для зниження артеріального тиску з групи адренотропніх засобів. Через деякий час у хворого тиск нормалізувався, але розвинулася брадикардія до 50 ударів на хвилину та атріо-вентрикулярна блокада

Page 214: крок 1 база 2014р_11шр-240ст (для брошури)_35грн

214

11 ступеню. Який препарат було призначено? A *Анаприлін B Празозин C Клофелін D Мезатон E Верапаміл

59 В тубдиспансері у хворого інфільтративною формою туберкульозу легень, який лікувався ізоніазидом, проявилися симптоми В6-гіповітаминозу. Чому ізоніазид призводить до цього явища? A * Ізоніазид є антагоністом вітаміна В6 B Сповільнюється всмоктування вітаміна C Прискорюється елімінація D Утворюється міцний зв’язок з білками плазми крові E Прискорюється біотрансформація

60 Хворому 35 років для лікування хронічного бронхіту у фазі загострення лікар призначив бактрім. Який основний механізм дії препарату? A *Гальмує 2 послідовних етапи метаболізму фолієвої кислоти B Діє на синтез клітинної стінки мікробу C Гальмує синтез білка мікробної клітини D Пригнічує утворення цитолеми E Діє тільки на синтез фолієвої кислоти

61 Підлітку, що перебував у стані важкого алкогольного сп'яніння, лікар швидкої допомоги серед інших заходів здійснив внутрішньом'язове введення розчину кофеїну. Поясніть на основі якого принципу дії дана маніпуляція є доцільною: A *Фізіологічний антагонізм B Синергізм C Потенціація D Конкурентний антагонізм E Сумація ефектів

62 Хворий 60 років протягом 9 років хворіє на цукровий діабет, отримує для корекції гіперглікемії інсулін – семіленте. 10 днів тому почав лікування гіпертонічної хвороби. Через годину після прийому гіпотензивного препарату (бета-адреноблокатору), хворий відчув слабкість, запаморочення, знизився

артеріальний тиск і через кілька хвилин хворий знепритомнів. Виберіть необхідний засіб терапії даного патологічного стану. A * Глюкоза B Інсулін C Норадреналін D Натрію гідрокарбонат E Бемегрид

63 Хворій 65-ти років, що страждає на інсуліннезалежний цукровий діабет призначили всередину глібенкламід. Вкажіть механізм гіпоглікемічної дії цього препарату. A *Стимулює виділення ендогенного інсуліну бета-клітинами B Пригнічує глюконеогенез у печінці C Посилює утилізацію глюкози периферичними тканинами D Пригнічує всмоктування глюкози у кишківнику E Пригнічує альфа-глюкозидазу і розпад полісахаридів

64 Хворому проведено апендектомію. Для хіміопрофілактики анаеробних ускладнень в післяопераційний період першочергово доцільно призначити: A *Метронідазол B Цефатоксим C Тетрациклін D Ампіокс E Бісептол

65 Хворий з виразковою хворобою шлунку приймав антацидний препарат альмагель. Для лікування гострого бронхіту йому призначили антибіотик метициклін. Проте протягом 5-ти днів температура не знизилася, кашель і характер харкотиння не змінились. Лікар прийшов до висновку про несумісність ліків при їх взаємодії. Про який саме вид несумісності ліків йде мова? A *Фармакокінетична на етапі всмоктування B Фармакокінетична на етапі біотрансформації C Фармацевтична D Фармакодинамічна E Прямий антагонізм

66 Хворому бронхіальною астмою призначили інгаляції 0,5\% розчину ізадрину.

Page 215: крок 1 база 2014р_11шр-240ст (для брошури)_35грн

http://vk.com/my.printing

215

Бронхоспазм був усунутий, але пацієнт почав скаржитись на болі в серці та серцебиття. З чим це пов’язано? A * Стимуляцією бета-1-адренорецепторів B Стимуляцією бета-2-адренорецепторів C Активацією альфа-1-адренорецепторів D Активацією М-холінорецепторів E Пригніченням синтезу ацетилхоліну

67 Студент звернувся до лікаря з проханням допомогти йому перебороти страх перед стоматологічними маніпуляціями. Лікар порадив йому прийняти препарат: A *Діазепам B Аміназин C Дроперидол D Дімедрол E Пірацетам

68 Хворий перебував на лікуванні в кардіологічному відділенні з приводу декомпенсованої хронічної серцевої недостатності. Йому був призначений дигітоксин в дозі 0,0001 г з першого дня перебування в стаціонарі, але покращення він відмітив тільки через тиждень. Повільне настання ефекту препарату лікар пояснив: A *Стійким зв’язуванням дигітоксину з білками плазми крові B Недостатньою дозою дигітоксину C Недостатнім всмоктуванням препарату в кишечнику D Посиленням діурезу E Недостатньою кількістю вуглеводів у дієті

69 У пацієнтки 46 років на фоні миготливої аритмії почався набряк легень. Який січогінний препарат необхідно ввести в першу чергу для виведення хворої з важкого стану? A *Фуросемід B Тріамтерен C Верошпірон D Амілорид E Еуфілін

70 Пацієнту, що хворіє на цукровий діабет та алергічний дерматит, лікар призначив фторований гормональний препарат в мазі. На питання хворого, про переваги

призначеного препарату над маззю гідрокортизону лікар пояснив, що: A *Призначений препарат практично немає резорбтивної дії B Посилює синтез інсуліну C Діє короткочасно D Діє слабше E Коштує дешевше

71 Хворий 30 років звернувся до лікаря зі скаргами на пронос і болі в животі протягом 5 днів, підвищення температури тіла до 37,50С з ознобами. На передодні хворий був у лісі, де випив води з відкритого водоймища. Встановлено бактеріологічно підтверджений діагноз: амебна дизентерія. Вкажіть препарат вибору для лікування цього захворювання. A * Метронідазол B Левамізол C Левоміцетин D Фталазол E -

72 Хворому з неоперабельним раком шлунку призначили промедол для зняття вираженого больового синдрому. З часом хворий відмітив зменшення знеболюючого ефекту та тривалості дії препарату, різке посилення болей у всьому тілі. Лікар пояснив це тим, що: A * Розвинулось звикання B Виникла тахіфілаксія C Розвинулась психічна залежність D Промедол здатний до кумуляції E Зменшилась реабсорбція промедолу в канальцях нирок

73 Допоможіть медичній сестрі визначити, який з названих протимікробних препаратів не належить до антибіотиків групи цефалоспоринів: A *Ципрофлоксацин B Цефалорідин C Цефпіром D Цефалексин E Цефокситин

74

Page 216: крок 1 база 2014р_11шр-240ст (для брошури)_35грн

216

Хворому 60 років, який довгий час хворіє на стенокардію разом з коронаролітиками лікар призначив ацетилсаліцилову кислоту для зменьшення агрегації тромбоцитів. Який механизм антиагрегантної діі ацетилсаліцилової кислоти? A *Знижує активність циклооксигенази B Знижує активність фосфодиестерази C Підвищує активність аденілатциклази тромбоцитів D Підвищує синтез простоцикліну E Проявляє мембраностабілізуючу дію

75 Хвора К., 45 років, діагноз: Миготлива аритмія. Гіпертонічна хвороба II ст. Препарат вибору для припинення приступу? A *Анаприлін B Сустак-форте C Калію хлорід D Строфантин E Лідокаїн

76 Хворому з маніакально-депресивним синдромом в стадії де_пресії, який скаржився на відчуття тривоги, страху, був призначений антидепресант із супутнім психоседативним ефектом. Який це був препарат? A *Амітриптилін B Імізин (іміпрамін) C Ніаламід D Сіднофен E Інказан

77 Хворому на ангіну, лікар призначив бісептол. В чому приорітет бісептолу в порівнянні з іншими сульфаніламідами? A *Триметоприм блокує, черговий етап перетворення фолієвої кислоти B Триметоприм підвищує імітацію сульфаніламідом ПАБК. C Краще проникає в мікроорганізм D Знижує біотрансформацію сульфаніламіду E Зменшує зв’язок з білками крові

78 У хворого на сифіліс при лікуванні препаратами вісмуту з`явились сірі плями на слизовій оболонці ротової порожнини та симптоми нефропатії. Який засіб використовують як антидот при отруєнні препаратами

вісмуту? A * Унітіол B Налорфін C Бемегрид D Налоксон E Метиленовий синій

79 Здорова людина перебуває в небезпечному по захворюванню на малярію районі. Який із зазначених препаратів необхідно призначити з метою особистої хіміопрофілактики малярії ? A * Хінгамін B Сульфален C Тетрациклін D Метронідазол E Бісептол

80 Сорокалітня жінка звирнулась до лікаря зі скаргою на болі в колінних суглобах. При огляді виявлено: припухлість, почервоніння, гіпертермію в ділянціц цих суглобів. Лабораторне дослідження показало позитивні ревмопроби. Які препарати мають бути використані для лікуваня хворої ? A *Протизапальні засоби нестероїдної будови B Наркотичні анальгетики C Антидепресанти D Антибіотики E Сульфаніламіди

81 Дитина 10 років страждає на стафілококовий дерматит. Лікування бензилпеніциліном не дало результатів. Призначення комбинованого препарату пеніциліну з клавулановою кислотою дало швидке одуження. Яка причина позитивної дії цього препарату ? A *Інактивація бета - лактамази B Блокада транслокази C Гальмування аденозиндезамінази D Гальмування транспептидази E Активація фосфодіестерази

82 В лікарню потрапила дитина 6 років з різко вираженими симптомами рухового і мовного збудження, сухістю в роті,утрудненим

Page 217: крок 1 база 2014р_11шр-240ст (для брошури)_35грн

http://vk.com/my.printing

217

ковтанням, хриплим голосом. Шкіра суха, гаряча. Зіниці різко розширені, фотофобія, тахікардія. З анамнезу встановлено, що дитина з’їла якісь ягоди темно-фіолетового кольору. Вплив якої з токсичних речовин є причиною отруєння? A * Атропін B Пірензепін C Пілокарпін D Платифілін E Метацин

83 Для полегшення вправлення вивиху в плечевому суглобі з ціллю розслаблення м’язів хворому був введений міорелаксант дитилін. При цьому наступило “апное”. Що необхідно ввести хворому? A * Свіжу цитратну кров B Галантаміну гідробромід C Бемегрид D Дипіроксим E Ізонітрозин

84 Хворому виразковою хворобою шлунку на фоні підвищеної кислотності шлункового соку в стадії загострення [в анамнезі якого є глаукома] необхідно призначити М-холіноблокуючий засіб. Якому з наведених засобів надати перевагу, враховуючи супутні захворювання? A * Пірензепін B Спазмолітин C Атропін D Скополамін E Гоматропін

85 Хворий 18-ти років звернувся до лікаря зі скаргами на безсоння, що проявляється у важкому засипанні. В результаті цього він не висипається і на слідуючий день почуває втому, важко засвоює учбовий матеріал. Лікар встановив, що безсоння пов’язане з неврозоподібним станом. Зробіть раціональний вибір снодійного засобу: A * Нітразепам B Бромізовал C Хлоралгідрат D Фенобарбітал

E Етамінал-натрій

86 У стаціонар поступив хворий з діагнозом “Виразкова хвороба 12-палої кишки в фазі загострення.” Аналіз шлункового соку показав підвищення секреторної і кислотоутворюючої функції шлунку. Виберіть препарат, який знизить секреторну функцію залоз шлунку за рахунок блокади Н2-рецепторів. A * Фамотидин B Екстракт красавки сухий C Атропін D Метацин E Платифілін

87 У дівчинки, яка лікувалася антибіотиками з групи пеніцилінів з приводу гострого бронхіту, на 3-й день розвинувся алергічний дерматоз.Який препарат хворій треба призначити? A *Лоратадин B Кромолін-натрій C Беклометазон D Ефедрина гідрохлорид E Левамізол

88 Хворий з нейродермітом на протязі тривалого часу використовував преднізолон. При обстеженні в нього виявили підвищення рівня цукру в крові. Вплив глюкокортикостероїдів на яку ланку обміну вуглеводів призводить до виникнення цього ускладнення? A *Активація глюконеогенезу B Активація глікогеносинтезу C Посилення всмоктування глюкози в кишечнику D Пригнічення синтезу глікогену E Активація розщеплення інсуліну

89 У хворого з високою температурою, кашлем, задишкою після проведених обстежень встановлено діагноз стрептококова пневмонія. Який препарат залишається засобом вибору при цьому захворюванні ? A *Бензилпеніцилін B Гентаміцин

Page 218: крок 1 база 2014р_11шр-240ст (для брошури)_35грн

218

C Ампіцилін D Еритроміцин E Тетрациклін.

90 Хворому було призначено препарат А. Через декілька діб дія препарату значно знизилась і для отримання початкового ефекту потрібно збільшити дозу речовини. Укажіть назву цього типу зміни дії лікарської речовини. A * Звикання B Тахіфілаксія C Лікарська залежність D Кумуляція E Ідіосинкразія

91 Після введення атропину у хворого прискорився ритм серцевої діяльності, відмічається сухість слизових оболонок, мідріаз. Що необхідно призначити хворому для нормалізації функцій організму? A *Прозерин B Метацин C Салбутамол D Ефідрину гідрохлорид E Строфантин

92 Жінка 28 років звернулася у поліклініку із скаргами на головний біль. Лікар запропонував їй парацетамол, взявши до уваги наявність соматичного захворювання. Яка супутня хвороба визначила призначення саме парацетамолу? A *Виразкова хвороба шлунку B Ревматоїдний артрит C Холецистит D Атеросклероз E Нефрит

93 Юнак 23 років з діагнозом виразкова хвороба шлунку надійшов до гастроентерологічного відділення в зв(язку з частими рецидивами і незадовільною дією попереднього лікування. Хворий одержував в(яжучі, холіно-, гістаміно- та H+К+-АТФ-аза-блокатори. Яку групу препаратів найбільш доцільно ввести до комплексной терапії?

A *Антибіотики. B Транквілізатори. C Сульфаніламіди. D Гормони. E Вітаміни.

94 У хворого, що лікується з приводу ревматоїдного поліартриту, в анамнезі є виразкова хвороба шлунку. Який із зазначених препаратів найбільш небажано застосовувати? A * Аспірин B Дімедрол C Вольтарен D Уродан E Циклосерин

95 Жінка в період вагітності тривалий час без контролю лікаря приймала антибіотик. Через деякий час у неї погіршився апетит, з’явились нудота, пронос, зміни слизових оболонок ротової порожнини та шлунково-кишкового тракту. З часом виникла жовтяниця. У новонародженого відмічено порушення росту кісток. Який препарат приймала жінка? A * Доксациклін B Левоміцетин C Ампіцилін D Еритроміцин E Бісептол

96 Для проведення термінового оперативного втручання хворому в хірургічне відділення викликали анестезіолога, який вирішив провести нейролептаналгезію. Яка з перелічених комбінацій препаратів використовується для цього? A *Дроперідол + фентаніл B Промедол + сібазон C Аналгін + діазепам D Галоперидол + фентаніл E Дроперідол + сібазон

97 Хворому з частими приступами стенокардії був призначений сустак-форте по 1 табл. 2 рази на день. Спочатку відмічався позитивний ефект, однак на другу добу приступи стенокардії поновились. Чим можна

Page 219: крок 1 база 2014р_11шр-240ст (для брошури)_35грн

http://vk.com/my.printing

219

пояснит неефективність призначеного препарата? A *Тахіфілаксією B Кумуляцією C Сенсибілізацією D Ідіосинкразією E Залежність

98 Після закінчення операції співставлення перелому стегна з використанням тубокурарину не відновилось дихання. Що треба ввести хворому для зняття релаксації? A *Прозерін B Платифілін C Циклодол D Атропін E Ацеклідин

99 У хворого після операції резекції шлунка на 2-3 день не відновилась перистальтика кишківника. Що протрібно призначити хворому для стимуляції функції шлунково-кишкового тракта? A *Прозерін B Платифілін C Циклодол D Атропін E Дитилін

100 У хворого виявили туберкульоз легень. Який антибіотик слід йому призначити поряд з іншими протитуберкульозними засобами? A *Ріфампіцин B Азітроміцин C Тетрациклін D Левоміцетин E Кефзол

101 Після закапування в око крапель у хворого розвинувся мідріаз і параліч акомодації. Яка група препаратів здатна викликати такий ефект? A *М-холінолітики B М-холіноміметики C Антихолінестеразні D Бета-адреноміметики E Альфа-адреноміметики

102 В результаті тривалого застосування антибіотиків широкого спектру дії

розвинувся кандидомікоз кишечнику. Що призначити для його лікування? A *Ітраконазол B Ріфампіцин C Граміцидін D Інтерферон E Ремантадін

103 У хворого травма мозку. В післяопераційному періоді розвинулась загроза набряку мозку. Призначте препарат для усунення цього ускладнення A *Фуросемід B Папаверіну гідрохлорид C Спіронолактон D Діхлотіазид E Дигоксин

104 В кардіологічне відділення доставлений хворий з діагнозом: гострий інфаркт міокарду. Для купірування больового синдрому в складі нейролептанальгезії пацієнту необхідно ввести таку комбінацію лікарських препаратів: A *Дроперідол + фентаніл B Морфін + атропін C Аміназин + діазепам D Анальгін + піпольфен E Но-шпа + дімедрол

105 Хворому з серцевою недостатністю та набряками призначили дигоксин. З чим пов'язаний кардіотонічний ефект серцевих глікозидів? A *Блокада Na/K АТФ-ази B Стимулювання Na/K АТФ-ази C Рефлекторним вливом на сердце D Пригніченням провідності міокарду E Не прямою активацією адренорецепторів

106 У хворого по лабораторним даним визначили аскаридоз. Який засіб треба призначити? A *Мебендазол B Фенасал C Піперазін D Фуразолідон E Гентаміцин

Page 220: крок 1 база 2014р_11шр-240ст (для брошури)_35грн

220

107 Хворий довгий час вживав антибіотики широкого спектру, що викликали зниження апетиту, нудоту, проносну дію, випорожнення з гнилосним запахом, похудіння. Про яку побічну дію іде мова? A *Дисбактеріоз B Алергічна реакція C Пряма подразняюча дія D Гепатотоксична дія E Нефротоксична дія

108 У хворого з переломом стегнової кістки, для послаблення напруги поперечнополосатих м'язів при репозиціі відломків кістки, треба призначити курареподібний засіб короткочастної діі. Який препарат доцільно призначити хворому? A * Дітілін B Ардуан C Тубокурарин D Декаметоній E Меліктин

109 У хворого Н., 40 років, виразка шлунку у стадіі загострення із значним підвищенням кислотності шлункового соку, больовим та діспептичним синдромом. Виберіть відповідний засіб: A * Фамотидін B Аллохол C Папаверіну гідрохлорід D Но-шпа E Платифіліну гідротартрат

110 Хвора Е., 63 роки., діагноз:інсуліннезалежний цукровий діабет. Ендокринолог почав лікування з призначення глібенкламіду. Укажіть механізм діі цього засобу. A *Активує бета-клітини островків Лангерганса B Зменшує всмоктування глюкози в кишечнику C Гальмує транспорт глюкози в клітини D Підсилює руйнуванню білків E Стимулює гіпоталамічні центри

111 Хворий на епілепсію протягом довгого часу одержує фенобарбітал в добовій дозі 0,4. В останній час у нього стали частішими напади, спостерігається пригнічений

настрій. Чим викликане погіршення стану хворого? A * Індукція ферментів монооксигеназної системи печінки B Пригнічення ферментів монооксигеназної системи печінки C Активація липолізу D Активація глюконеогенезу E Пригнічення гліколізу

112 Хворій, що перенесла мастектомію у зв’язку з раком молочної залози, був призначений курс променевої терапії. Який з перерахованих вітамінних препаратів володіє вираженою радіопротекторною дією, зумовленою антиоксидантною активністю? A * Токоферола ацетат B Ергокальциферол C Тіаміну хлорид D Рибофлавін E Фолієва кислота

113 У хворого, що проходить у стаціонарі курс лікування з приводу бронхіальної астми супутнім захворюванням є глаукома. Препарати якої групи в зв’язку з цим не рекомендується включати в терапію бронхіальної астми? A * М-холіноблокатори B Адреноміметики C Міотропні спазмолітики D Глюкокортикоїди E Адреноблокатори

114 У хворого діагностовано діабетичну кому. Концентрація цукру в крові становить 18,44 ммоль/л. Який з цукрознижуючих препаратів необхідно призначити даному хворому? A *Інсулін короткої дії B Інсулін середньої тривалості дії C Інсулін тривалої дії D Препарат із групи бігуанідів E Препарат із групи похідних сульфонілсечовини

115 Хворий з рематоїдним артритом на протязі декількох тижнів приймав препарати глюкокортикостероїдів, потім раптово припинив їх прийом. Яке ускладнення може виникнути в цьому випадку? A *Сіндром відміни

Page 221: крок 1 база 2014р_11шр-240ст (для брошури)_35грн

http://vk.com/my.printing

221

B Підвищення артеріального тиску C Гіперглікемія D Загострення хронічних інфекційних процесів E Виразкування слизової оболонки шлунка і 12-ти палої кишки

116 У хворого з гострим гастроентероколітом після проведених бакттеріологічних досліджень встановлено діагноз: холера. Який антибіотик є засобом вибору при цьому захворюванні? A *Доксациклін B пеніцилін C Оксацилін D Цефалексин E Ампіцилін

117 Хворому, який лікувався доксацикліном, лікар порадив не вживати молочних продуктів. Чому лікар дав таку рекомендацію? A *Сповільнюється всмоктування антибіотика B Не засвоюються молочні продукти C Збільшується ризик дисбактеріозу D Зростає токсичність антибіотика E Порушується процес перетравлення їжі

118 У реанімаційне відділення поступив хворий з ознаками гострого отруєння морфіном. Який засіб необхідно використати у даному випадку для промивання шлунка? A *Калію перманганат B Натрію гідрокарбонат C Фурацилін D Розчин натрія хлоріду E Борну кислоту

119 Хворий звернувся до лікаря зі скаргами на забарвлення сечі та слізної рідини в червоний колір. З анамнезу відомо, що він лікується з приводу легеневого туберкульозу. З застосуванням якого протитуберкульозного засобу пов’язане дане явище? A *Рифампіцин B Ізоніазид C Етіонамід D Стрептоміцину сульфат E Етамбутол

120 Хворому призначено препарат дигоксин. Через декілька днів у хворого виявлено ознаки передозування цим препаратом, його вміст у крові значно перевищував верхню межу терапевтичної концентрації. Як називається такий варіант дії лікарських речовин? A *Кумуляція B Звикання C Тахікардія D Потенціювання E Антагонізм

121 У приймальне відділення лікарні поступив хворий зі скаргами на набряки, підвищення артеріального тиску. Що необхідно призанчити хворому для зменшення набряків? A *Дихлотіазид B Кордіамін C Резерпін D Салбутамол E Анаприлін

122 У приймальне відділення поступив хворих зі скаргами на набряки, частий пульс, задишку, синюшність слизових оболонок. Поставлений діагноз: хронічна серцева инедостатність. Що необхідно призначити хворому для покращення самопочуття? A *Дигоксин B Папаверину гідрохлорид C Мезатон D Кордіамін E Нітрогліцерин

123 У приймальне відділення поступив хворий з гіпертонічним кризом. Що необхідно ввести хворому для нормалізації артеріального тиску? A *Магнія сульфат внутрім’язево B Резерпін C Еналаприл D Магнію сульфат всередину E Анаприлін всередину

124 У хворого з пневмонією відмічається надмірний кашель, що спричинює різкі болі у грудній

Page 222: крок 1 база 2014р_11шр-240ст (для брошури)_35грн

222

порожнині. Що необхідно включити у комп-лексну терапію пневмонії для заспокоєння кашлю? A *Глауцин B Настій трави термопсису C Салбутамол D Камфору E Метацин

125 Чоловік 65-ти років надійшов до неврологічного відділення з діагнозом постінсультний синдром. Який препарат найбільш доцільно призначити хворому для прискорення одужання? A *Галантаміну гідрохлорид B Іпратропіум бромід C Дипіроксим. D Ізонітрозин. E Ацеклідин.

126 Хвора на глаукому звернулася до провізора аптеки з проханням видати їй очні краплі з атропіна сульфату, але їй пояснили, що цей препарат вживати не можна. Чому атропін протипоказаний при глаукомі? A *Підвищує внутрішньоочний тиск. B Викликає параліч акомодації. C Розширює зіниці. D Знижує відстань бачення. E Пригнічує очні рефлекси.

127 Хворому на міастенію було призначено препарат, що викликав покращення м(язевої діяльності. Але поступово виявився ряд недоліків: підвищене виділення слини, поту, діарея, тошнота. Який засіб було вжито для лікування? A *Прозерин. B Аналгін. C Стрихнін. D Кофеїн. E Армин

128 Жінці 36 років, що страждає на ішемічну хворобу серця, лікар спочатку призначив анаприлін. Яле з(ясувавши наявність супутнього захворювання він вирішив замінити цей засіб атенололом. Яке захворювання призвело до відмови від анаприліну? A *Бронхіальна астма. B Артеріальна гіпертензія.

C Виразкова хвороба 12-палої кишки. D Міастенія. E Холецистит.

129 В поліклініку звернулася за допомогою жінка із скаргами на поганий настрій, безсоння, болі в серці. Який засіб для лікування неврозу слід призначити хворій? A *Діазепам B Аміназин C Дроперидол D Трифтазін E Фенобарбітал

130 Чоловік 58 років звернувся до лікаря з приводу хвороби Паркінсона. Хворий страждає також на глаукому. Що треба йому призначити? A *Леводопу. B Циклодол. C Атропін. D Скополамін. E Метацин.

131 До прийомного відділення обласної лікарні надійшла жінка 40 років з діагнозом гострого панкреатиту. Що треба в першу чергу ввести хворій? A *Контрікал B Платифілін. C Атропін. D Метацин. E Пірензепін.

132 Хворий на серцеву недостатність ІІ стадії з великими набряками, тахікардією, розладом травлення та сну потребує лікарську терапію. Яке із приведених серцевих гликозидів короткої дії стане найбільш ефективним у цьому випадку? A *Дигоксин. B Дигітоксин. C Адонізід. D кардіовален. E Целанід.

133 Жінка 50 років звернулась до поліклініки із скаргами на головний біль, нудоту. Обстеження виявило початкову стадію гіпертонічної хвороби. Інтерн намагався

Page 223: крок 1 база 2014р_11шр-240ст (для брошури)_35грн

http://vk.com/my.printing

223

виписати їй резерпін, але лікар-консультант заперечив це, вказавши на наявність супутнього захворювання. Який патологічний процес є протипоказанням до призначення резерпіну? A *Виразкова хвороба шлунку B Холецистит C Нефрит D Міокардит E Панкреатит

134 Хвора на туляремію тривалий час приймала тетрациклін, що призвело до кандидозного генералізованого ураження вісцеральних органів. Який препарат треба ій призначити з метою найбільш ефективного лікування? A *Ітраконазол B Ністатин C Леворин D Гризеофульвін E Ципренол

135 При обстеженні чоловіка 40 років було встановлено діагноз: гіпохромна анемія. Який препарат треба призначити для лікування? A *Ферковен B Ціанокобаламін C Пентоксил D Гепарин E Вікасол

136 Хворому в післяопераційному періоді тривалий час уводили промедрол. Внаслідок відміни препарату у пацієнта виникли важкі психічні та соматичні порушення Як називається це явище? A * Абстинентний синдром B Ідіосинкразія C Тахіфілаксія D Синдром обкрадання E Синдром віддачі

137 Жінка 60 років, яка страждає на токсичний зоб скаржиться на збільшення частоти серцевих скорочень. Який з зазначених препаратів слід призначити для нормалізації серцевого ритму?

A * Анаприлін B Ізадрин C Сальбутамол D Адреналін E Пентамін

138 До лікаря звернулась жінка 48 років, що тривало та безконтрольно приймала натрію бромід, зі скаргами на загальмованість, порушення пам?яті, нежить, кашель. Яка причина цього стану? A * Кумуляція B Звикання C Лікарська залежність D Ідіосинкразія E Тахіфілаксія

139 Хворому з глаукомою для зниження внутрішньочного тиску лікар призначив прозерин у вигляді очних крапель. Який механізм дії даного препарату? A * Блокада ацетилхолінестерази B Блокада циклооксигенази C Блокада ліпоксигенази D Блокада фосфодіестерази E Блокада фосфоліпази

140 У хворого 55 років виявлено гіпертонічну хворобу з підвищеним рівнем реніну в крові. Якому гіпотензивному препарату слід надати перевагу при лікуванні хворого? A * Еналаприл B Клофелін C Магнію сульфат D Папаверин E Празозин

141 Через 2-3 години після парентерального введення препарату у пацієнта розвинувся коматозний стан, спостерігається дихання типу Чейн-Стокса, зіниці різко звужені, колінний рефлекс збережений. Який препарат міг спричинити отруєння? A * Морфін B Аміназин C Сібазон D Спирт етиловий E Фенобарбітал

142 Хворому із перенесеним гострим інфарктом

Page 224: крок 1 база 2014р_11шр-240ст (для брошури)_35грн

224

міокарду лікар рекомендував протягом 3-4 місяців приймати ацетилсаліцилову кислоту по 0,25 1 раз в 2-3 дні. На яку дію аспірину розраховував лікар? A * Антиагрегантна B Протизапальна C Жарознижуюча D Анальгезуюча E Судиннорозширююча

143 Хворому із гострою пневмонією було призначено препарат з протикашлевою дією, яка пов?язана із виразним пригніченням кашлевого центру. Який препарат призначив лікар? A * Глауцин B Лібексин C Бромгексин D Терпінгідрат E Калію йодид

144 Після 10-денного лікування препаратом із групи антибіотиків у пацієнта спостерігається явища дисбактеріозу: диспептичні явища, кандидомікоз; жовтяниця, фотосенсибілізація. Антибіотик якої групи приймав хворий? A * Групи тетрациклінів B Групи пеніцилінів C Групи цефалоспоринів D Групи рифампіцину E Групи аміноглікозидів

145 Для попередження пологів вагітній жінці ввели розчин препарату, який ще відомий як протисудомний, гіпотензивний, жовчогінний та проносний засіб. Скорочення матки та болі припинились, проявився заспокійливий ефект. Який із вказаних препаратів застосував лікар? A * Магнію сульфат B Окситоцин C Партусистен D Ерготаміну гідротартрат E Сальбутамол

146 У хворого, що звернувся в лікарню із скаргами на пронос діагностували амебну дизентерію. В комплексне лікування був включений доксациклін. Назвати вид дії призначеного препарату.

A * Етіотропна дія B Пряма дія C Рефлекторна дія D Основна дія E Незворотня дія

147 У хворого, який звернувся до лікаря діагностували анацидний гастрит. Для покращення роботи шлунку хворому призначили таблетки ацидин-пепсину. Який вид лікування був призначений хворому? A * Замісна терапія B Симптоматична терапія C Профілактичне застосування D Етіотропна терапія E Каузальна терапія

148 Хворий 37 років госпіталізований в венерологічне відділення з діагнозом сифіліс. Який із перелічених препаратів буде використовуватися для лікування хворого? A *Бензилпеніцилін B Бісептол C Нітроксолін D Левоміцетин E Тетрациклін

149 Хворому 45 років на гостру пневмонію був призначений антибіотик з групи пеніцилінів. При проведенні проби на індивідуальну переносимість до антибіотику виникла алергічна реакція. Яким із перелічених препаратів краще лікувати хворого? A * Ципрофлоксацин B Бензилпеніцилін C Феноксиметилпеніцилін D Біцилін – 5 E Еритроміцин

150 Перед проведенням інфільтраційної анестезії хворому проведено пробу на чутливість до новокаїну, яка виявилась позитивною. Який з перерахованих препаратів можно використати для проведення знечулення в даному випадку? A *Лідокаїн B - C Кокаїн D Анестезин

Page 225: крок 1 база 2014р_11шр-240ст (для брошури)_35грн

http://vk.com/my.printing

225

E Дикаїн

151 Для лікування хворого із запаленням легень призначено комбінований препарат з групи сульфаніламідів. Який препарат було призначено хворому? A * Бактрим B Етазол C Норсульфазол D Сульфален E Фталазол

152 В психіатричній клініці при тривалому лікуванні у хворого появилися симптоми паркінсонізму. Який препарат вводили хворому? A * Аміназин B Мезапам C Натрію бромід D Літію карбонат E Ніаламід

153 У хворого із значними периферійними набряками почергове використання дихлотиазиду, етакриновоі кислоти і фурасеміду не призвело до значного діуретичного ефекту. Аналіз крові вказав значне підвищення кількості альдостерону. Вкажить препарат вибору. A * Спіронолактон B Маніт C Клопамід D Сечовина E Амілорид

154 До приймального відділення надійшов хворий у непритомному стані. Шкіра холодна, зіниці звужені, дихання з утрудненням, відзначається періодичність по типу Чейн-Стокса, артеріальний тиск знижений, сечовий міхур переповнений. Ппоставлений діагноз: отруєння морфіном. Який препарат необхідно застосувати в якості антагоністу? A * Налоксон B Бемегрид C цититон D Унітіол E Тіосульфат натрію

155 До приймального відділення доставлений хворий зі скаргами на сухість в роті, світлобоязнь та порушення зору. Шкіра гіпіремійована, суха, зіниці розширені, тахікардія. При подальшому обстеженні був встановлений діагноз: отруєння алкалоїдами красавки. Який з лікарських засобів доцільно застосувати? A * Прозерин B Діазепам C Пілокарпін D Армін E Діпіроксим

156 Хворий на гіпертонічну хворобу, який лікувався гіпотіазидом, скаржиться на загальну слабість, втрату апетиту, серцебиття. Спостерігається гіпотонія м’язів, в’ялі паралічі, послаблення перистальтики кишечника. Що може бути причиною такого стану? A *Гіпокаліємія B Гіперурікемія C Гіпонатріємія D Гіперкаліємія E Гіперкальціємія

157 Хворий з інфекційним мононуклеозом на протязі 2-х тижнів приймав глюкокортикостероїдні препарати. Наступила ремісія, проте в нього виникло загострення хронічного тонзиліту. Результатом якої дії глюкокортикостероїдів є дане ускладнення? A *Імунодепресивної B Протизапальної C Протишокової D Антиалергічної E Антитоксичної

158 Хворий скаржиться на болі у шлунку, печію. При обстеженні шлункового соку виявлено підви-щення кислотності шлункового соку. Що необхідно призначити хворому для нейтралізації кислотності шлункового соку? A *Альмагель B Атропіну сульфат C Папаверину гідрохлорид D Прозерин E Бензогексоній

Page 226: крок 1 база 2014р_11шр-240ст (для брошури)_35грн

226

159 У хворого діагностовано гострий інфаркт міокарду, який супроводжується стійкими болями за грудиною. Неефективність попередньо введених препаратів дала підставу лікарю провести нейролептанальгезію. Який нейролептик використовується для цього виду знеболення? A * Дроперидол B Метаперазин C Галоперидол D Резерпин E Аміназин

160 Хворий на гіпотонію, відчувши погiршення загального стану: головокружіння, слабкість, шаткість при ході прийняв без контролю лікаря кілька таблеток ефедрину через короткі проміжки часу. Проте деяке покращання спостерігав тільки після використання першої таблетки. Яким явищем обумовлена така дія? A * Тахіфілаксія B Сенсибілізація C Ідіосинкразія D Звикання E Кумуляція

161 При черговому нападі бронхіальної астми хворий за допомогою інгалятора вдихнув препарат, який у вигляді інгаляцій використовується як бронхолітик, а в ін’єкціях - в акушерській практиці для попередження викиднів. Який препарат був використаний хворим? A * Фенотерол B Сальбутамол C Адреналін D Ефедрин E Еуфілін

162 В результаті безконтрольного прийому вітамінного препарату в дитини виникла анорексія, нудота, блювання, пронос, гіпертермія, з'явилися крововиливи на шкірі та слизових, а також явища менінгізму. Який препарат приймала дитина? A *Ретінолу ацетат

B Соматотропін C Нікотинамід D Рутин E Токоферолу ацетат

163 У чоловіка 36 років з черепно-мозковою травмою дихання послаблене, пульс ниткоподібний, рефлекси відсутні. Який шлях введення пірацетаму найбільше доцільний у данному випадку? A *Внутрішньовенний. B Підшкірний. C Пероральний. D Ректальний. E Інгаляційний.

164 У роділлі 35 років виявляється больовий синдром, зв'язаний з затримкою першого періоду родів. Який препарат краще за все слід застосувати для зменшення болі? A *Промедол. B Анальгін. C Морфін. D Кодеїн. E Парацетамол.

165 До приймального відділення доставлений чоловік у непритомному стані. Об'єктивно: на зовнішні подразники хворий не реагує, дихання періодичне по типу Чейн-Стокса, зіниці звужені, зіничний рефлекс відсутній. Бул встановлено, що дані симптоми обумовлені використанням морфіну. Назначити антидотну терапію. A *Налоксон. B Унітіол. C Апоморфіна гідрохлорід. D Кальція хлорід. E Протаміна сульфат.

166 До лікаря звернувся пацієнт зі скаргами на запаморочення, відчуття спраги, затруднення ковтання, погане бачення близьких предметів. Об’єктивно у нього часте дихання, розширені зіниці, загальне збудження, говірливість, однак мова малозрозуміла. АТ 110/70 мм рт ст, Пульс 110 в 1 хв. На передозування якого препарату можуть

Page 227: крок 1 база 2014р_11шр-240ст (для брошури)_35грн

http://vk.com/my.printing

227

вказувати наведені симптоми? A * Атропіну B Морфіну C Ефедріну D Аміназіну E Кофеїну

167 Чоловік 37 років надійшов до хірургічного відділення з явищами острого панкреатиту: блювота, понос, брадикардія, гіпотензія, слабкість, явища зневоднювання организму. Який препарат найбільш доцільно використати в першу чергу? A *Контрикал. B Но-шпу. C Платифілін. D Етаперазин. E Ефедрин.

168 Хворий на ішемічну хворобу серця не повідомив лікаря, що у нього бувають напади бронхоспазму. Лікар призначив препарат, після прийому якого напади стенокардії стали рідшими, але почастішали напади брогхоспазму. Укажіть, котрий препарат був презначений. A *Анаприлін B Атенолол C Верапаміл D Дилтіазем E Нітротросорбід

169 В дермато-венерологічний диспансер надійшла жінка 30 років з діагнозом первинний сифіліс. Який засіб найбільш доцільно їй призначити? A *Бензилпеніцилін. B Феноксиметилпеніцилін. C Тетрациклін. D Левоміцетин E Цефамізин.

170 До приймального вiддiлення лiкарнi доставлено хво-рого з вираженими явищами гостроi серцевоi недо-статностi. Яким iз зазначених препаратiв можно йому термiново допомоготи?

A *Корглiкон B Етазол C Дiтiлiн D Фiзостигмiн E Дигитоксiн

171 У хворого при дослідженні калу виявлено яйця аскорид. Який засіб слід призначити ? A *Мебендазол B Ністатин C Левоміцетин D Тетрациклін E Фуразолідон

172 У хворого на маніакально-депресивний психоз спостерігаються пригнічення психічної і рухової активності, тривожно-депресивний стан. Який препарат необхідно йому призначит для зняття патологічної депресії? A *Амітриптилін B Сиднокарб C Кофеїн-бензоат натрію D Пірацетам E Трифтазин

173 Хвора на бронхіальну астму приймала протягом 2-х місяців преднізолон у таблетках (по 1 табл. 3 рази на день). Внаслідок значного покращання стану раптово припинила його прийом Розвиток якого ускладнення високо ймовірний у цьому випадку? A *Синдрому відміни B Синдрому Іценко-Кушінга C Шлункової кровотечі D Ожиріння верхньої половини тулуба E Гіпотонії

174 Під час оперативного втручання на фоні використання гігронію різко знизився артеріальний тиск. Представники яких груп лікарських препаратів можуть нормалізувати артеріальний тиск? A *a-адреноміметики. B a-адреноблокатори. C Гангліоблокатори. D М-холіноміметики. E Н-холіноміметики.

175 У хворого на міастенію, після призначення

Page 228: крок 1 база 2014р_11шр-240ст (для брошури)_35грн

228

прозеріну, з'явилася нудота, діарея, посмикування м'язів язика і скелетних м'язів. Чим можна усунути інтоксикацію? A *Атропіну сульфат B Фізостигмін C Пірідостігміну бромід D Ізадрин E Мезатон

176 Хворий з хронічною серцево-судинною недостатністю приймав дігоксин. Після призначення додаткової терапії розвинулись явища інтоксикації серцевими глікозидами. Який препарат може викликати підсилення інтоксикації серцевими глікозидами? A *Кальцію хлорид. B Калію хлорид. C Магнію хлорид. D Аспаркам E Розчин глюкози.

177 У хворого після тривалого курсу лікування в’ялопротікаючої шизофренії виникли явища паркінсонізму. Який з нижчеперерахованих препаратів міг викликати дане ускладнення? A *Аміназин B Сибазон C Пірацетам D Літію карбонат E Галоперидол

178 Хворому, який переніс інфаркт міокарду, призначена ацетилсаліцилова кислота по 75 мг щоденно. З якою метою призначено препарат? A *Зменшення агрегації тромбоцитів B Зменшення запалення C Зменшення болю D Пониження температури E Розширення коронарних судин

179 З хімічного виробництва в токсикологічне відділення доставлений хворий з отруєнням ртуттю. Який препарат слід використати в даній ситуації? A *Унітіол B Ізонітрозин C Налоксон D Активоване вугілля E Ентеросорбент СКН

180

У хворої, яка хворіє на гіпертонічну хворобу, розвинувся приступ бронхіальної астми. Який засіб слід призначити для зняття приступу? A *Салбутамол B Ізадрин C Адреналін D Еуфілін E Ефедрин

181 Хвора гіпертонічною хворобою тривалий час лікувалася гіпотензивними засобами. В останній час стан хворої погіршився. З(явились нудота, набряк вік та губ, уртикарні висипи на шкірі, блювання, брадикардія, понос. Який препарат визиває ці явища? A *Резерпін B Клофелін C Бензогексоній D Анаприлін E Дибазол

182 У дитини 5-ти років гостра правостороння нижньодолева пневмонія. При сіянні харкотиння виявлено, що збудник захворювання стійкий до пеніциліну, але чутливий до макролідів. Який препарат найльш доцільно використати у даному випадку? A *Азитроміцин. B Тетрациклін. C Гентаміцин. D Стрептоміцин. E Ампіцилін.

183 В психіатричну клінику доставлений хворий 40 років у стані збудження, агресіі, марення. Який препарат слід ввести хворому? A *Аміназин. B Седуксен (діазепам). C Настойку валеріани. D Резерпін. E Натрію бромід.

184 Хворий із хронічним закрепом приймав щодня одне драже "Бісакоділ". Через деякий час для отримання ефекту він змушений був приймати по 2 драже. Укажіть, як називається такий тип зміни дії лікарських речовин: A * Звикання

Page 229: крок 1 база 2014р_11шр-240ст (для брошури)_35грн

http://vk.com/my.printing

229

B Лікарська залежність C Ідіосинкразія D Сенсибілізація E Кумуляція

185 Після введення лікарської речовини у піддослідної тварини зменшилося виділення слини, розширилися зіниці, а при наступному введенні у вену ацитилхоліну частота скорочень серця істотно не змінилася. Укажіть назву цієї речовини: A *Атропін B Адреналін C Анаприлін D Прозерин E Салбутамол

186 При тривалому використанні препарату у хворого можуть мати місце остеопороз, ерозії слизової шлунка, гіпокаліємія, затримка натрію і води, зменшення вмісту кортикотропіну у крові. Укажіть препарат. A * Преднізолон B Гіпотіазид C Дигоксин D Індометацин E Резерпін

187 Чоловіку 42 років, що страждає на виразкову хворобу 12-палої кишки призначені такі препарати: омепразол, амоксицилин, метронідазол. Терапія виявилась доцільною. Яка основна причина високої ефективності цього комплексу препаратів? A *Пригнічення Helycobacter pylori B Зменшення впливу патогенної флори кишечника C Підсилення резистентності слизової оболонки 12-палої кишки D Підсилення синтезу простагландинів E Покращення мікроциркуляції

188 Чоловіку 18 років з приводу флегмони плеча була зроблена внутрішньом'язова ін'єкція пеніциліну. Після цього у нього з'явилися тахікардія, ниткоподібний пульс, АТ

знизився до 80/60 мм рт.ст. Який вид фармакологічної реакції розвинувся? A *Анафілаксія B Центральна дія C Рефлекторна дія D Потенціювання E Периферична дія

189 Чоловік 52 років, що страждає на екзему, звернувся до дерматолога за консультацією з приводу застосування нового гормонального засобу із групи глюкокортикоїдів. Лікар звернув увагу на наявність фтору в молекулі цього препарату і вказав хворому приблизну різницю між новим засобом і преднізолоном. Чим відрізняється новий препарат? A *Діє сильніше B Діє слабше C Порушує обмін калію D Не викликає зниження рівня кортикотропіну E Не збільшує концентрацію глюкози в крові

190 До лікарні швидкої допомоги доставили чоловіка 63 років з явищами колапсу. Для боротьби з гіпотензією лікар вибрав норадреналін. Який механізм дії цього препарату? A *Активація альфа-1-адренорецепторів. B Активація серотонінових рецепторів. C Активація бета-адренорецепторів. D Активація дофамінових рецепторів. E Блокада м-холінорецепторів.

191 Чоловік 37 років госпіталізований до хірургічного відділення з явищами гострого панкреатиту (блювота, пронос, брадикардія, гіпотензія, слабість, явища зневоднювання організму). Який препарат найбільш доцільно використати в першу чергу? A *Контрикал B Но-шпа C Платифілін D Етаперазин E Ефедрин

192 Терапія анаприліном позитивно вплинула

Page 230: крок 1 база 2014р_11шр-240ст (для брошури)_35грн

230

на динаміку хвороби у жінки 44 років, яка страждає стенокардією. Який головний механізм дії цього препарату? A *Блокада бета-адренорецепторів і зниження потреби міокарда в кисні. B Зменшення окислювального обміну в міокарді внаслідок блокади ферментів циклу Кребса. C Зменшення енергозатрат міокарда внаслідок зниження навантаження. D Збільшення надходження кисню в міокард. E Зниження потреби і збільшення надходження кисню в міокард.

193 До приймального покою лікарні доставлено хворого з тяжким отруєнням невідомою речовиною в стані гострої судинної недостатності. Який із зазначених препаратів необхідно використати для термiнової допомоги? A *Мезатон B Iзадрiн C Анаприлін D Нафтизин E -

194 Больной 42-х лет страдает хроническим конъюнктивитом аллергического генеза. По профессии водитель. Какой антигистаминный препарат необходимо назначить больному с учетом специфики его работы? A *Диазолин. B Димедрол. C Супрастин. D Дипразин. E Кетотифен.

195 При сборе анамнеза у больного бронхиальной астмой врач выяснил, что приступы удушья возникают обычно ночью, им предшествует умеренно выраженная брадикардия. Какая группа холинэргических средств наиболее показана в данной ситуации? A *М-холиноблокаторы B М-холиномиметики C Антихолинэстеразные средства D Реактиваторы холинэстеразы E Н-холиномиметики

196

У хворого на сифіліс при лікуванні препаратами вісмуту з'явилися сірі плями на слизовій оболонці ротової порожнини та симптоми нефропатії. Який засіб використовують як антидот при отруєнні препаратами вісмуту? A *Унітіол B Метиленовий синій C Налоксон D Бемегрид E Налорфін

197 У хворого діагностували легеневу форму туберкульозу. Який лікарський засіб використовують для лікування туберкульозу? A *Ізоніазид B Пеніцилін C Норсульфазол D Тетрациклін E Фуразолідон

198 У хворого на бронхіальну астму виник гіпертонічний криз. Який антигіпертензивний препарат раціонально застосувати в комплексні терапії цього патологічного стану? A *Магнію сульфат B Анаприлін C Ніфедипін D Резерпін E Празозин

199 Хворому з бактеріальною інфекцією лікар призначив сульфадимезин в таблетках, порекомендувавши запивати його 1,5-2 л лужної мінеральної води щодобово. Чим обумовлена необхідність даної рекомендації? A *Для профілактики кристалізації ацетильних похідних препарату в ниркових канальцях B Для нейтралізації кислоти шлункового соку C Для зсуву рН крові в лужний бік D Для зменшення подразнюючого впливу на шлунок E Для пролонгування дії

200 Хворому з метою знеболювання при травматичному шоці ввели під шкіру розчин морфіну гідрохлориду. Який механізм анальгезуючої дії цього препарату?

Page 231: крок 1 база 2014р_11шр-240ст (для брошури)_35грн

http://vk.com/my.printing

231

A *Взаємодія з опіоїдними рецепторами B Блокада периферичних чутливих рецепторів C Зміна емоційного забарвлення болю D Порушення проведення імпульсів по аферентних нервах E Гальмування утворення медіаторів болю у периферичних тканинах

201 У хворого під час оперативного втручання з'явилися симптоми передозування тубокурарину. Препарат якої групи найдоцільніше використати для послаблення ефекту передозування? A *Антихолінестеразні B Гангліоблокатори C Адреноміметики D М-холіноблокатори E Н-холіноблокатори

202 У хворого на фоні прийому дігоксину з'явилась бігемінія, різка м'язова слабкість, діарея, блювання, порушення зору. Які препарати можуть ослабити явища отруєння? A *Препарати калію. B Препарати кальцію. C Препарати магнію. D Препарати заліза E Препарати натрію

203 До жінки, що намагалась покінчити з життям, був викликаний психіатр, який виявив стан ендогенної депресії. Який препарат найбільш доцільно призначити хворій для проведення курсу лікування? A *Амітриптилін B Сиднокарб C Етимізол D Ноотропіл E Кофеїн

204 Після введення анальгіну у хворого почалася гостра алергічна реакція, задуха, біль за грудиною. Пульс зник, припинилися серцеві скорочення. Які засоби фармакологічної допомоги слід застосувати в першу чергу? A *Внутрішньосерцево ввести адреналін B Внутрішньовенно ввести адреналін

C Внутрішньовенно ввести преднізолон D Внутрішньовенно ввести димедрол E Внутрішньовенно ввести строфантин

205 Жінку 59 років непокоять болі в правому колінному суглобі. Об'єктивно: шкіра в області коліна гіперемійована. Температура підвищена, тканини напружені, в ділянці колінного суглоба рентгенологічно виявлено ексудат. Який препарат найбільш ефективний для лікування цієї хворої? A *Диклофенак B Анальгін C Промедол D Парацетамол E Новокаїн

206 До гастроентерологічного відділення госпіталізовано чоловіка 25 років зі скаргами на голодні болі, відчуття важкості за грудиною, печію. Лікар призначив гастроцепін. Яка фармакодинамічна властивість гастроцепіну примусила лікаря зробити цей вибір? A *Зниження секреції соляної кислоти і гастрину B Розслаблення гладкої мускулатури шлунка C Розслаблення мускулатури жовчних протоків D Пригнічення Helycobacter pylori E Зниження секреції трипсину підшлунковою залозою

207 У человека с медикаментозным отравлением отсутствует сознание, наблюдается миоз и усиление спинальных моносинаптических рефлексов. Какое вещество могло быть причиной отравления? A *Морфин B Этаминал натрия C Димедрол D Диазепам E Аминазин

208 Больному с явлениями общих отеков, ацидоза и гиперкалиемии необходимо назначение мочегонного средства. Какое из

Page 232: крок 1 база 2014р_11шр-240ст (для брошури)_35грн

232

перечисленных средств желательно порекомендовать больному? A *Фуросемид B Спиронолактон C Дигоксин D Триамтерен E Маннит

209 Хворому призначено препарат з вираженими лiпофiльними властивостями. Яким буде головний механiзм його всмоктування? A *Пасивна дифузiя B Активний транспорт C Фiльтрацiя D Пiноцитоз E Зв'язування з транспортними бiлками

210 5-місячній дитині була призначена антибактеріальна терапія з приводу бронхопневмонії. Який засіб негативно впливає на розвиток зубів? A *Доксациклін B Левоміцетин C Нітроксолін D Пеніцилін E Бісептол

211 У жінки 42 років, яка перенесла операцію на нирці, після наркозу розвинулися явища рекураризації і припинилося дихання. Як міорелаксант був застосований дитилін. Який засіб найбільш доцільно застосовувати для відновлення тонусу м'язів? A *Плазму крові B Прозерин C Стрихніну нітрат D Кофеїн E Галантоміну гідробромід

212 Дитина 4-х років госпіталізована в ортопедичне відділення з переломом гомілки з зміщенням. Перед репозицією відламків необхідна аналгезія. Який препарат слід вибрати? A *Промедол B Анальгін C Морфіну гідрохлорид D Панадол E -

213 Хворий госпіталізований до лікарні після бджолиних укусів з вираженим набряком Квінке. Який протигістамінний препарат, що не виявляє одночасно седативний ефект, необхідно призначити хворому? A *Діазолін. B Супрастин. C Фенкарол. D Тавегіл. E Димедрол.

214 У дитини 9 років виявлено порушення навчання, зниження інтелектуального розвитку. Призначення якої групи психотропних засобів є виправданим в даному випадку? A *Ноотропів B Антидепресантів C Транквілізаторів D Нейролептиків E Адаптогенів

215 У хворої на хронічну серцеву недостатність з набряковим синдромом у крові виявлено підвищення вмісту альдостерону. Який діуретичний засіб найдоцільніше призначити? A * Спіронолактон B Тріамтерен C Діакарб D Гідрохлортіазид E Фуросемід

216 Хворій на гіпертензивну хворобу в комплексній терапії призначено гідрохлортіазид. Який механізм дії препарату сприяє зниженню артеріального тиску? A * Збільшення виділення іонів натрію та води B Збільшення утворення ангіотензину ІІ C Блокада кальцієвих каналів D Зменшення виділення іонів натрію та води E Блокада ферменту карбоангідрази

217 Хворому 42 років для лікування бактеріальної пневмонії призначено амоксицилін. Вкажіть, який механізм бактерицидної дії препарату?

Page 233: крок 1 база 2014р_11шр-240ст (для брошури)_35грн

http://vk.com/my.printing

233

A * Пригнічення синтезу клітинної стінки мікроорганізмів B Пригнічення внутрішньоклітинного синтезу білка C Порушення проникливості цитоплазматичної мембрани D Пригнічення SH-груп ферментів мікроорганізмів E Антагонізм із параамінобензойною кислотою

218 Для лікування системної склеродермії призначено імунодепресант, відомий ще як протималярійний засіб . Який препарат призначено хворому? A * Хингамин B Преднізолон C Дексаметазон D Азатіоприн E Циклосерин

219 Хворому 56 років із скаргами на спрагу, часте сечовиділення, після обстеження у ендокринолога було встановлено діагноз цукровий діабет та призначено глібенкламід. Вкажіть механізм дії цього препарату? A * Стимулює бета-клітини острівців Лангерганса B Сприяє засвоєнню глюкози клітинами тканин організму C Полегшує транспорт глюкози через клітинні мембрани D Пригнічує альфа-клітини острівців Лангерганса E Пригнічує всмоктування глюкози в кишківнику

220 У пацієнта після переохолодження у ділянці крил носа та верхньої губи з’явились герпетичні висипання. Для лікування була застосована мазь. Який противірусний засіб містить застосована мазь? A *Ацикловір B Азидотимідин C Дексаметазон D Індометацин E Інтерферон

221 Для ослаблення або припинення передачі збудження через синапс з нервового закінчення на м'язове волокно в клініці використовуються курареподібні речовини – міорелаксанти. Який механізм діїї цієї групи

лікарських засобів? A * Блокада Н-холінорецепторів постсинаптичної мембрани. B Блокада проходження Са2+ через канали пресинаптичної мембрани. C Пригнічення роботи Na+/K+-насосів. D Пригнічення ацетилхолінестрерази. E Зменшення виділення медіатора в синаптичну щілину.

222 У хворого на гіпертонічну хворобу виявлено високий рівень реніну в крові. Якому з гіпотензивних засобів слід віддати перевагу в цьому випадку? A * Лізиноприл B Анаприлін C Празозін D Ніфедіпін E Діхлотіазид

223 Хворий при проходженні курсу лікування метронідазолом вжив невелику кількість алкоголю, внаслідок чого розвинулось тяжке отруєння. Поясніть причину отруєння: A * Накопичення ацетальдегіду B Алергічна реакція C Невралгічні розлади D Серцево-судинна недостатність E Порушення функції нирок

224 У хворого гостра серцево-легенева недостатність, що супроводжується набряком легенів. Який препарат з групи діуретиків необхідно призначити? A * Фуросемід B Спіронолактон C Діхлотіазид D Діакарб E Триамтерен

225 Для лечения крапивницы с целью устранения зудящей сыпи на коже больному назначен димедрол. Какой механизм обеспечивает его эффективность в этом случае? A *Конкурентная блокада Н1-рецепторов B Ингибиция синтеза гистамина C Угнетение высвобождения гистамина D Ускорение разрушения гистамина E Независимый антагонизм с гистамином

226

Page 234: крок 1 база 2014р_11шр-240ст (для брошури)_35грн

234

Хворий страждає на слабкість, задишку, набряки нижніх кінцівок. Діагноз – хронічна серцева недостатність.Який засіб необхідно призначити хворому в першу чергу? A * Дігоксин B Кофеїн C Папаверін D Анаприлін E Раунатін

227 Хворий що страждає на нирково-кам’яну хворобу поступив в лікарню у зв,язку з нападом ниркової коліки. Вибрати засіб швидкої допомоги в цьому випадку. A * Промедол B Аналгін C Парацетамол D Фуросемід E Контрікал

228 Хворому встановлено діагноз - активний вогнищевий туберкульоз легень. Вкажіть, який із препаратів найбільш доцільно призначити в першу чергу? A * Ізоніазид B Сульфален C Циклосерін D Етіонамід E Етоксид

229 У хворого з ревматоїдним артритом, який лікувався диклофенак-натрієм, виникли ознаки гастропатії. З якою дією можна пов’язати виникнення цього ускладнення? A * Антициклооксигеназною B Антисеротоніновою C Антигістамінною D Антикініновою E Місцевоподразнюючою

230 В процессе лечения хронического отечного синдрома фуросемидом у больного возникло нарушение катионного состава плазмы крови. Какое средство следует использовать для его коррекции? A * Калия хлорид B Кальция хлорид C Натрия хлорид

D Натрия гидрокарбонат E Лития карбонат

231 Хворий поступив до офтальмологічного відділення з діагнозом: відкритокутова глаукома лівого ока. Після додаткового обстеження лікар призначив діакарб. Вкажіть механізм зниження внутришньоочного тиску під впливом діакарбу: A * Інгібування карбоангідрази циліарного тіла B Зменшення продукції спиномозкової рідини C Збільшення осмотичного тиску плазми крові D Блокування реабсорбції натрія в канальцях нефрону E Гальмування синтезу альдостерону в наднирниках

232 У хворого виявлена змішана глисна інвазія: аскарідоз кишківника та трематодоз печінки. Який з протигельмінтних препаратів доцільно призначити? A * Мебендазол B Левамізол C Пірантел D Хлоксил E Піперазину адипінат

233 В пологове відділення поступила вагітна жінка зі слабкою пологовою діяльністю. Призначте засіб гормональної природи для посилення пологової діяльності? A * Окситоцин B Прогестерон C Метандростенолон D Гідрокортизон E АКТГ

234 Хворому на цукровий діабет медсестра помилково ввела майже подвійну дозу інсулину, що призвело до гіпоглікемичної коми.Який лікарський засіб необхідно ввести хворому для виведення з коми? A * Глюкозу B Лідазу C Інсулін D Соматотропін E Норадреналін

235

Page 235: крок 1 база 2014р_11шр-240ст (для брошури)_35грн

http://vk.com/my.printing

235

У больного анафилактический шок. От какого из названных адреномиметиков можно ожидать наибольшего терапевтического эффекта в этом случае? A *Адреналина B Мезатона C Эфедрина D Фенотерола E Норадреналина

236 У больного гипертонической болезнью выраженный терапевтический эффект вызвал спиронолактон. Его терапевтическая активность обусловлена ослаблением действия: A *Альдостерона B Ренина C Ангиотензинпревращающего фермента D Ангиотензина ІІ E Брадикинина

237 Хворий похилого віку скаржиться на головний біль, запаморочення, швидку втому, погіршення пам’яті. В анамнезі черепно-мозкова травма. Яку групу препаратів необхідно призначити хворому? A * Ноотропні засоби B Снодійні C Нейролептики D Анальгетики E Транквілізатори

238 Виберіть сечогінний препарат, котрий доцільно призначити при хронічній серцевій недостатності з метою попередження розвитку гіпокаліємії A * Спіронолактон B Клопамід C Фуросемід D Діхлотіазид E Манніт

239 Який препарат доцільно призначити хворому на хронічний гастрит з підвищеною секреторною функцією? A *Гастроцепін B Панкреатин C Пепсин D Контрикал

E Дезопімон

240 Який з препаратів бета-адреноблокаторів не може бути застосований для лікування ішемічної хвороби серця у пацієнта, що одночасно страждає на бронхіальну астму? A *Анаприлін B Талінолол C Метопролол D Бісопролол E Ацебуталол

241 Унітіол є антидотом і застосовується, зокрема, при отруєннях солями важких металів. Як називається такий тип взаємодії лікарських речовин? A *Хімічний антагонізм B Фізичний антагонізм C Фізіологічний антагонізм D Синергоантагонізм E Неконкурентний антагонізм

242 Як зміниться фармакологічна активність препарату з високою спорідненістю до білків плазми крові при виникненні гіпоальбумінемії ? A *Підвищиться B Зменшиться C Не зміниться D Анулюється E Сповільниться

243 Хворому на туберкульоз в комплексній терапії призначено препарат – похідне гідразиду ізонікотинової кислоти. Визначити цей препарат? A * Ізоніазид B Стрептоміцину сульфат C Цефалоридін D Рифампіцин E Канаміцин

244 Хворому з дискінезією жовчовивідних шляхів та закрепами лікар призначив жовчогінний засіб, який має ще значну послаблюючу дію. Який засіб призначив лікар? A * Магнію сульфат B Алохол C Холосас

Page 236: крок 1 база 2014р_11шр-240ст (для брошури)_35грн

236

D Холензим E Нікодин

245 Хворому з кардіогенним шоком, гіпотензією, задухою, і набряками ввели неглікозидний кардіотонік. Вкажіть, який саме препарат був введений хворому. A * Добутамін B Кофеіну натрію бензоат C Кордіамін D Етимізол E Бемегрид

246 Хворому з гострою недостатністю надниркових залоз був призначений лікарський препарат після якого у нього з’явилися скарги на біль в кістках (двічі були переломи), часті простудні хвороби, набряки, повільне загоювання ран. Який препарат міг спричинити такі явища? A *преднізолон B ретаболіл C спіронолактон D естріол E тестостерон

247 Хворому з діагнозом цукровий діабет II типу ендокринолог призначив глібенкламід. Вкажіть основний механізм дії цього засобу. A *Стимулює секрецію інсуліну бета-клітинами острівців Лангерганса B Пригнічує глюконеогенез C Підсилює метаболізм глюкози D Підсилює захоплення глюкози периферичноми тканинами E Активує транспорт глюкози в клітину

248 Дитина народилася в стані асфіксії. Який препарат необхідно ввести новонародженому для стимуляції дихання? A * Етимізол B Лобелін C Празозин D Атропін E Прозерин

249 Хворий похилого віку страждає хронічним закрепом, в основі якого лежить гіпотонія товстого кишечника. Який препарат слід призначити хворому?

A * Бесакоділ B Натрію сульфат C Касторову олію D Ацеклідин E Прозерін

250 В кардіологічне відділення поступив хворий зі скаргами на частий пульс, задишку, синюшність слизових оболонок. При огляд: набряки на нижніх кінцівках, асцит. Який з перерахованих препаратів необхідно призначити хворому довенно для покращення самопочуття? A * Корглікон B Кордіамін C Адреналіну гідрохлорид D Дигітоксин E Но-шпа

251 Хворому з артеріальною гіпертензією було призначено один з антигіпертензивних засобів. Артеріальний тиск нормалізувався, однак хворого почав турбувати постійний сухий кашель. Який з перерахованих препаратів має таку побічну дію. A * Лізиноприл B Анаприлін C Клофелін D Резерпін E Ніфедипін

252 У дитини спостерігається сухий кашель. Який протикашльовий засіб ненаркотичної дії полегшить стан хворого? A Глауцину гідрохлорид B Кодеїну фосфат C Морфіну гідрохлорид D Калію йодид E Коріння алтеї лікарської

253 Чоловік 30 років, водій за професією, страждає на алергічний риніт із загостренням у весняний період. Лікар призначив хворому антигістамінний засіб з незначним седативним ефектом та тривалістю дії близько 24 год. Який із перерахованих засобів було призначено? A *Лоратадин B Димедрол C Гепарин

Page 237: крок 1 база 2014р_11шр-240ст (для брошури)_35грн

http://vk.com/my.printing

237

D Вікасол E Окситоцин

254 Хворому з безсонням на фоні висипки алергічного характеру і свербіжем призначили препарат, який має антигістамінну та снотворну дію. Назвіть цей засіб: A *Димедрол B Лоратадин C Преднізолон D Ацетилсаліцилова кислота E Анальгін

255 Під час проведення місцевого знеболення у хворого виник анафілактичний шок. Який із перерахованих препаратів необхідно ввести хворому? A *Адреналіну гідрохлорид B Діазепам C Атропіну сульфат D Анаприлін E Нітрогліцерин

256 До реанімаційного відділення поступив хворий без свідомості. Зі слів родичів стало відомо, що хворий помилково випив спиртовий розчин невідомого виробника. За даними проведеного обстеження діагностовано отруєння метиловим спиртом. Який антидот необхідно використати в даному випадку? A *Етанол B Тетурам C Налоксон D Протаміну сульфат E Ацетилцистеїн

257 Жінці 26 років, хворій на бронхіт, призначили засіб етіотропної терапії – антибіотик широкого спектру дії. Який це препарат ? A *Доксициклін

B Інтерферон C БЦЖ – вакцина D Амброксол E Дексаметазон

258 Хворому з атонією сечового міхура лікар призначив препарат, дозу котрого хворий самостійно збільшив. З'явилось потовиділення, салівація, діарея, м'язеві спазми. Препарат якої групи був призначений? A * Холіноміметики B Ре активатори холінестерази C Адреноблокатори D Токолітики E Гангліоблокатори

259 У больного кишечная колика на фоне гипертонической болезни. Средства какой из перечисленных групп наиболее целесообразно использовать для ее купирования в данной ситуации? A *Миотропные спазмолитики B Антихолинэстеразные средства C Симпатомиметики D М-холиномиметики E Адреномиметики

260 Призначення доксицикліну гідрохлориду викликало порушення симбіозу мікробної флори в кишечнику. Визначити тип порушень при антибіотикотерапії. A Дисбактеріоз B Сенсибілізація C Ідіосинкразія D Суперінфекція E Бактеріоз